You are on page 1of 572

Surgery Sixer

Worked Book Based on APP for 2021 NEET PG

By

Dr.R.Rajamahendran MS, MRCS, MCh( SGE), FMAS


• Author- Surgery Sixer for PGMEE ( CBS Publications)
• Author- Long Cases in Surgery ( Jaypee Publications)
• Author- Short Cases in Surgery ( Paras Publications)
• Author- Tumors in 7 days for PGMEE ( CBS Publications)

For all your doubts and queries and Controversies-


Join me on t.me/Surgerysixer Group…

SURGERY SIXER APP BASED WORK-BOOK 2020 1


Section A- General Surgery
Chapter 1- General Aspects
• 1a- Nutrition
• 1b- Shock and Blood Transfusion
• 1c- Sutures and Techniques
• 1d- Theatre and Theatre Protocols
• 1e- Surgical Site Infections

Chapter 2- Trauma
• 2a- Introduction
• 2b- Head Injury
• 2c- Thoracic Injury
• 2d- Abdominal injury
• 2e- Burns
• 2f- Miscellaneous Topics
• 2g- ATLS 10th edition Changes

Section B- Head and Neck, Thyroid, Breast and Hernia


( FINAL MBBS CLINICAL CASE TOPICS)
1. Head and Neck
• 1a Oral cavity lesions
• 1b. Salivary gland
• 1c. Neck Swellings

2. Thyroid Swellings
• 2a- Introduction
• 2b- Benign Disease
• 2c- Malignant Diseases
• 2d- Surgical aspects

3. Breast
• 3a- Triple Assessment of Cancer breast
• 3b- Management of Cancer Breast
• 3c- Miscellaneous Topics in cancer Breast

4. Hernia
• 4a- Anatomy of Inguinal Canal and Types of Hernia
• 4b- Named hernias
• 4c- Laparoscopic Anatomy and repair
• 4d- Peritoneum, Retroperitoneum and Mesentry

SURGERY SIXER APP BASED WORK-BOOK 2020 2


Section C- Gastrointestinal Tract Surgery
Chapter 1- Esophagus
• 1a- Motility Disorders, Diverticulum
• 1b- GERD, Hiatal hernias
• 1c- Miscellaneous topics in Esophagus
• 1d- Tumors in Esophagus

Chapter 2- Stomach and Bariatric Surgery


• 2a- Peptic ulcer disease
• 2b- Carcinoma stomach
• 2c- Miscellaneous Topics
• 2d- Upper GI Bleeding
• 2e- Bariatric Surgery

Chapter 3- Intestines
• 3a- Introduction
• 3b- Benign Topics intestine
• 3c- Small Intestine tumors
• 3d- Inflammatory Bowel Disease
• 3e- Polyps
• 3f- Carcinoma Colon
• 3g- Intestinal Obstruction
• 3h- Appendix

Chapter 4- Rectum and Anus


• 4a- Benign diseases
• 4b- Malignant diseases

Chapter 5- Miscellaneous
• Mesentry and Peritoneum

Section D- Hepatobiliary and Pancreatic Surgery


Chapter 1- Gallbladder and Bile duct
• 1a- Introduction
• 1b- Gall stone disease
• 1c- Bile duct injury
• 1d- CBD Stones
• 1e- Miscellaneous
• 1f- Cancer GB and Bile duct

Chapter 2- Pancreas
• 2a- Benign Disease
• 2b- Tumors in Pancreas

Chapter 3: Liver
• 3a- Introduction
• 3d- Portal Hypertension
• 3c- Liver surgical infections
• 3d- Tumors in Liver

Chapter 4: Spleen
SURGERY SIXER APP BASED WORK-BOOK 2020 3
Section E- Specialty Chapters
Chapter 1: Urology Chapter 2: Vascular Surgery
• 1a- Kidney, Ureter • 2a- Arterial System
• 1b- Bladder, Urethra • 2b- Venous system
• 1c- Prostate • 2c- Lymphatic System
• 1d- Penis
• 1e- Testis
Chapter 3: Chapter 4:
• Plastic Surgery and Cleft Lip • Paediatric Surgery
• Skin lesions

Chapter 5: Oncosurgery Chapter 6: Cardiothoracic Surgery


• 5a-Skin cancers • 6a-Thorax
• 5b- Sarcomas • 6b- Heart

Chapter 7 : Endocrine Surgery Chapter 8: Neurosurgery


• 7a- MEN Syndromes • 8a- Benign Topics
• 7b- Parathyroid • 8b- Brain tumors
• 7c- Adrenal
Chapter 9: Transplantation
• Introduction
• Individual Organ Transplantation

SURGERY SIXER APP BASED WORK-BOOK 2020 4


Section A: GENERAL SURGERY
Contents:
Chapter 1: General Aspects:
• Topic 1a- Nutrition
• Topic 1b- Shock and Blood transfusion:
• Topic 1c- Sutures , Suturing techniques, Knots and Anastomosis
• Topic 1d- Operation theatre, instruments and protocols
• Topic 1e- Surgical Site Infections:

Chapter 2: Trauma
• Topic 2a- Introduction
• Topic 2b- Head Injury, Face injury and neck injury
• Topic 2c- Thoracic Trauma
• Topic 2d- Abdominal Injuries
• Topic 2e- BURNS
• Topic 2f- Miscellaneous
• Topic 2g- ATLS latest protocols

SURGERY SIXER APP BASED WORK-BOOK 2020 5


Chapter : 1 : General Aspects in Surgery

Topic 1a: Nutrition

Assessment of Nutrition:
• Midarm Circumference
• Triceps Skin Fold thickness
• Body Mass Index
• Albumin ( Best of all methods in Surgical Patients)

Malnutrition Universal Screening Tool:


THE MUST TOOL- Clinical assessment tool of malnutrition: Based on 3 Factors:

1. BMI: 2. Weight loss in 3-6 3. Acute disease effect


months
0- >20 Add a score of 2 if there is
1- 18.5- 20 0- <5% reduced intake >5 days
2- <18.5 1- 5-10%
2- >10%

Indications for artificial nutrition


• Any patient who has sustained > 7 days of inadequate intake
• Any patient who is anticipated to have no intake for > 7 days.

Types of nutrition
Enteral Nutrition Parenteral Nutrition
Nutrition given via GI tract Nutrition given via Veins
1. Sip feeding 1. Peripheral Parenteral
2. Tube feeding 2. Central Parenteral
• Nasogastric tube ( Ryles tube) 3. Peripherally Inserted Central Catheter
• Naso jejunal tube
• Feeding Gastrostomy ( PEG)
• Feeding Jejunostomy .

Nasogastric Feeding:
Ryles Tube:
Length of Ryles Tube:
• Length- 110-130 cm in adults
• NEX rule ( Nose, Ear to Xiphisternum) in Adults,
• NEMU rule ( Nose, Ear, Midpoint of Epigastrium to Umbilicus)

SURGERY SIXER APP BASED WORK-BOOK 2020 6


Figure: Ryles Tube

Ideal method to insert Ryles Tube:


• Sitting with Neck flexed
• Fowler’s Position

Once desired length mark has reached, confirm its position by:
▪ Auscultating with stethoscope in epigastric region by simultaneously pushing air in
an empty syringe through the external port of tube
▪ Look for reflux of gastric contents in the tube
▪ Aspiration of secretion and test with litmus paper for pH ( Most ideal method**)

Nasojejunal Tube ( Freka Tube)


• Inserted with help of Flouroscopy or with help of Endoscopy ( Invasive Procedure)
• Indications for NG to NJ tube:
- Duodenal Fistula
- Acute Pancreatitis Not tolerating NG tube.

Maximum Time we can use Nasal Tubes is for 4 weeks only


If you want to give > 4 weeks- Go for Surgical Tubes like PEG, FG, FJ

PEG tube ( 4-6 weeks nutrition)


Techniques :
- Pull through
- Direct Stab

Figure: PEG tube

SURGERY SIXER APP BASED WORK-BOOK 2020 7


Feeding Jejunostomy/ Gastrostomy:
• Stamm Direct Stab Technique
• Witzel Tunneling Technique
• These methods are used for patients who need nutrition for long period more than >4
weeks.
• FJ is preferred in Coma Patients as FG will result in Aspiration

Complications of Enteral Nutrition:


1. Tube related Complications ( Most Common)- Malposition, Obstruction, Leakage,
Displacement
2. Osmotic Diarrhea
3. Electrolyte Imbalance
4. Refeeding Syndrome

Parenteral Nutrition:
Parenteral Nutrition types:
• Peripheral PN ( < 2 weeks)
• PICC ( Hickmann Lines)
• Central PN ( > 2 weeks)- IJV, SCV, FV are used

Best vein to give nutrition by central Vein:


• For Elective cases- Subclavian Vein
• For Trauma cases- IJV

TPN bag:
• Components of TPN:
- Dextran ( 60%)
- Fat ( 20%)
- Amino acids ( 20%)
- All essential nutrients, minerals and Vitamins
• Fat Free TPN- 75% Dextran+ 25%Amino acid ( Ratio 3:1)

SURGERY SIXER APP BASED WORK-BOOK 2020 8


Insertion of central Vein Catheter:
• IJV- Between two heads of SCM
• SCV- USG guided Below Midclavicular point of Clavicle
• Tip must be in SVC and not in Atrium
• Advice X ray chest- To look for Pneumothorax and Look for Tip in SVC

Why Should you Shift Enteral to Parenteral Nutrition?


When GIT is not good When BP is not good
• EC fistula • Shock
• Radiation enteritis • Severe Burns
• Short Bowel Syndrome • Severe Acute Pancreatitis
• Crohn’s Disease

Lab Values to be monitored in patient on TPN long term:


• Weekly Twice- Blood Sugar and Electrolytes
• Weekly once- RFT and LFT
• Change the catheter only if there is catheter related sepsis. (Both Tip C/S and Blood C/S
are showing same organism)- Put a new catheter on opposite side.

Complications of TPN
Catheter related Metabolic Electrolyte Disturbances Overfeeding
Complications
Infection Azotemia Hyper / hypo natremia Hyperglycemia
( M/C complication)** Essential fatty Hyper/ hypo kalemia
acid deficiency Hypophosphatemia* Hepatic steatosis –
Injuries Fluid overload Hyper/ hypo Jaundice+
• Pneumothorax, Metabolic bone magnesemia Hypercapnia
Hydrothorax disease Hyper/ hypo calcemia Fluid retention
• Cardiac Liver dysfunction High/ low serum zinc
tamponade Neutrophil High / low serum Excess fat:
Dysfunction* copper • Hypercholesterolemia
Central vein Thrombosis Hyper chloremic • Hyper
Glucose metabolic acidosis. triglyceridemia
Air embolism imbalance • Lipoprotein X
formation
Trace element • Hypersensitivity.
and vitamin
deficiency. Excess amino acid:
• Metabolic acidosis
• Uremia
• Hypercalcemia

SURGERY SIXER APP BASED WORK-BOOK 2020 9


Refeeding Syndrome:
• Characterized by severe fluid and electrolyte shifts in malnourished patients undergoing
refeeding*
• It can occur with Enteral and Parenteral nutrition ( MC with TPN)
• Lab values: Hypophosphatemia, Hypocalcemia, Hypomagnesemia* and also Hypokalemia
(Mnemonic- PCM*)
• Risk factors: Alcohol, Severe malnutrition, Anorexia, Prolonged fasting*
• Treatment: Avoid Overfeeding*
• Deliver calories slowly*
• Electrolyte imbalance needs to corrected*

Hepatic steatosis causing Jaundice:


• Stop TPN temporarily
• Restart with lipid Free TPN.

1st Sign of Sepsis- Elevated Blood Glucose**

Liver failure*- MC indication for Combined SB + Liver Transplant in Short Bowel Syndrome.
Other indication is Central vein thrombosis.

Image Based Questions:

Central vein catheter

PICC catheter ( Hickman Lines)

SURGERY SIXER APP BASED WORK-BOOK 2020 10


Space for added points:

SURGERY SIXER APP BASED WORK-BOOK 2020 11


Topic 1b: Shock and Blood transfusion:

• MC cause of death in Surgical Patients- Shock*

Types of Shock (Based on the Image in App)


Features Hypovolemic Shock Distributive Shock
(Haemorrhagic Shock) ( Septic Shock)
Cardiac Output Decreased Increased
Peripheral vascular Increased Decreased
resistance ( Cold Peripheries) ( Warm Peripheries)
Oxygen Consumption Increased Decreased
Venous Resistance Low Low
Mixed Venous Oxygen MVOS < 50%** MVOS>70%**
Saturation ( MVOS)
Normal- 50-70%

Monitoring of Shock:
Urine Output CVP Serum Lactate
• Best Clinical • Best method to • Best Lab value to
parameter calculate Amount of monitor Tissue
• Best clinical method Fluid to be given perfusion
to look for adequacy • Amount of Drug to be • Serum Lactate:
of resuscitation given • < 2 mmol/l- Good
• Best to Look for tissue resuscitation
perfusion • CVP is not accurate • >5 mmol/l- Bad
for cardiogenic shock resuscitation
and septic shock and • Best method to look
hence we use PCWP for Muscle and GIT
for those 2 shocks. perfusion

Haemorrhagic Shock:
• 4 classes of haemorrhagic shock: Class 1,2,3,4 based on amount of Blood Lost.
• Class 1 ( < 750 ml), Class 2 ( 750-1500 ml), Class 3 ( 1500-2000 ml) and Class 4 (
>2000 ml)
• Pulse rate, Respiratory rate increases and Pulse pressure decreases from Class 2 Shock
• BP falls from Class 3 shock

SURGERY SIXER APP BASED WORK-BOOK 2020 12


Management of Haemorrhagic Shock:
• Crash 2 Trial: Bp < 90 mmHg, PR >110/minute- Injection Tranexamic acid * given
immediately.
• 2 Green Venflons ( 18 Gauge is used)
• 1 Litre Crystalloid is used.
• O negative blood without cross matching ( For post reproductive females and males- O
positive can be given**)
• Balanced resuscitation: 1:1:1 unit of PRBC: Platelets: Plasma
• Permissible Hypotension: BP should not be raised too much or must not be too low. It
should be in such a way Brain also receives adequate blood and at the same time the
coagulated vessels must not reopen and rebleed.

Cannulas:
Colour codes of Cannulas:

SURGERY SIXER APP BASED WORK-BOOK 2020 13


• In dehydration and Diarrhea to infuse maximum Fluids the ideal cannula is GREY- 16G. (
From the Table note- 200 ml/ minute using Grey*** and 85 ml/ minute using Green)
• In Trauma we use short and wide bore 2 Cannulas of atleast 18 Gauge – 2 in numbers as
per ATLS guidelines*

Damage control surgery (Abbreviated Laparotomy)


The surgery is restricted to two goals only
• Stop any active bleed
• Control any contamination.

The aim is to resuscitate the patient and plan for definite surgeries after the patient becomes
stable.

Deadly triad**
Following a trauma, protracted surgery in physiologically unstable patient,the three factors that
carry high mortality are:
• Hypothermia
• Acidosis
• Coagulopathy
Hence originated a phenomenon- DAMAGE CONTROL SURGERY.

Phases of Damage Control Surgery:


• Phase -1: Initial Exploration
• Phase-2: Secondary Resuscitation
• Phase -3: Definitive operation

Phase -1 Phase -2 Phase- 3


Initial exploration Secondary Resuscitation Definitive treatment
• Control Active • Transfer to ICU • Planned re exploration and
haemorrhage and • Ventilatory support definite surgery*
Contamination* • Correct the deadly triad- • Done 48-72 hours after
• Midline incision- 4 Hypothermia, Acidosis, secondary phase*
quadrant packing done. Coagulopathy* • Complex reconstruction
• GIT perforations closed must be avoided*
with sutures or Staples* • Abdomen closure done in
• External drains kept for this phase only**
pancreatic/Bile duct
injuries
• Temporary closure of
abdomen using plastic
sheet known as OPSITE**

SURGERY SIXER APP BASED WORK-BOOK 2020 14


• We do Bagotta Bag
method.

Septic Shock:
• Defined as Sepsis+ Organ Dysfunction+ Hypotension
• SIRS is defined by:
Two or more of the following: (Mnemonic- Orthopaedics Love THR)
o L- Leucocyte Count ( >12000 or <4000)
o T- Temperature >38 or < 35
o H- Heart rate > 90 beats/ minute
o R- Respiratory rate > 20/ minute ( PaCo2 <32 mmHG)
• Quick Sequential Organ Failure Assessment Score ( qSOFA score)
(Mnemonic – Royal Challengers Bangalore will be in SOFA)
o R- Respiratory Rate- > 22/minute- 1 point
o C- Confused mental status- 1 point
o B- BP < 100 mmHg- 1 point
≥2 points indicates organ dysfunction. Score > 2 carries 10% mortality

Management of Septic Shock:


• Eradicate the septic Foci
• Antibiotics
• IV fluids- Crystalloids (no need of Blood transfusion)
• Vasopressor of Choice- Nor adrenaline**

Blood transfusion and Blood Products


Transfusion:
• The amount of blood withdrawn from donor- 450ml
• Maximum three times/year
• Storage of Blood- 45 days using Saline Adenine Glucose mannitol ( SAG- M)
• Old Storage Component- Citrate Phosphate Dextrose ( 3 weeks only)
• Each unit of blood is screened for- Hepatitis B, hepatitis C, HIV 1 and HIV 2 and syphilis
( Western countries – Creutzfeld Jacob Disease which is usually depleted by Leucocyte
Filters)

Blood Products:

Whole blood: Packed red cells:


• Rich in coagulation factors than • The cells are spun down and
packed cells and more metabolically concentrated.
active than stored blood. • Each unit is 330 ml*
• Available as 450 ml pack. • Hematocrit= 50-70%

SURGERY SIXER APP BASED WORK-BOOK 2020 15


• Contains RBC, WBC, Plasma, Platelet • Stored at 2-6 degrees C.
of which WBC and Platelet are non • I unit increases Hb by 1 gm/dl/
functional** • Hematocrit by 3%**
• Used in massive bleeding, open heart • Lacks coagulation factors.
surgery etc..
Fresh frozen plasma: Cryoprecipitate:
• Rich in coagulation factors • Supernatant precipitate of FFP and is
• Stored at -40 to -60 degrees C. (2 rich in factor VIII* and fibrinogen.
year shelf life)* • Stored at -30 degree C with a 2 year
• Rh D positive FFP can be given to Rh shelf life*
D negative women • Available in 15 ml**
• Available in 200-250 ml Packets**
Platelets: Prothrombin complex concentrates: (PCC)
• Mostly used in Dengue fever cases* • Highly purified concentrates prepared
• Platelets are stored on a special from pooled plasma.
agitator at 20-24 degrees c. ( Room • They contain factors II, IX, X. Factor
temperature) VII may be included or produced
• Half life only 5 days* as a Fresh separately.
component.
• Useful in patients with
thrombocytopenia

Practical Points:
• Cross matching is done in 45 minutes in blood bank
• Blood received must be transfused in 1- 4 hours
• Un crossmatched Blood used in Trauma is O negative ( males- O Positive)
• Un crossmatched Plasma used in Trauma is AB
• Autologous blood -For patients undergoing elective surgery they pre donate their blood
upto 3 weeks* before surgery for re transfusion during operation.
• Massive Blood Transfusion- Replacing whole body blood volume ( 10 units of Blood in
adults)
• 1st Sign in Un anaesthetised patients – Wrong matched blood shows Itching**
• 1st Sign in anesthetised patients- Wrong matched blood transfusion shows unusual
Bleeding from operative site –followed by fall in BP – Hematuria*

Complications of Massive transfusion:


• Coagulopathy
• Hypocalcemia* ( due to binding of ionized calcium by citrate used as anticoagulant)
• Hyperkalemia* ( due to RBC lysis)
• Hypomagnesemia
• Hypothermia

SURGERY SIXER APP BASED WORK-BOOK 2020 16


• Volume overload
• Dilutional thrombocytopenia
• Decreased oxygen delivery ( due to decrease in 2,3 DPG)
• Metabolic Alkalosis (even though the stored blood contains pH- 6.3, Because of massive
transfusion sodium citrate is metabolized in liver to sodium bicarbonate)
• Rare- Metabolic Acidosis*
Complications of regular blood transfusion:
• Febrile non Haemolytic Transfusion reaction* ( Most Common)- Due to WBC present –
Leucocyte reduction filters remove the WBC*
• Allergic reaction
• Infection:
o Bacterial infection ( as a result of faulty storage)
o Hepatitis- B, C, G
o HIV- 1,2
o HTLV- 1 and 2
o Malaria**
o West Nile virus, Parvo virus B-19, HHV-8, CMV
• Air embolism
• Thrombocytopenia
• Transfusion related acute lung injury – TRALI (usually MC from FFP)
• Fatal hemolysis
• GVHD
• HLA and RBC allo-sensitisation
• Patients who receive repeated transfusions over long periods may get iron overload. Each
transfused unit of RBCs contain approximately 250mg elemental iron**

Must Know Table:


Effects of storage of whole blood?
• Reduction in pH**
• Raise in Potassium concentration**
• Progressive reduction of red cell content of 2,3 di phosphoglycerate which results in
decrease in oxygen carrying capacity.
• Loss of platelet function in whole blood within 48 hours of donation
• Reduction of factor VIII to 10-20% of normal in 48 hours
• Coagulation factors VII and IX are stable in storage*

Blood component Temperature of storage Shelf life


Whole blood 1-6 Deg C 35 days
Packed cells 42 days

SURGERY SIXER APP BASED WORK-BOOK 2020 17


Platelets 20-24 deg C 5 days
FFP - 40 deg C 2 years
Cryoprecipitate -30 deg C 2 years

Perioperative blood transfusion: ( Bailey 27th Edition Update)


Haemoglobin level (gm/dl) Indication

< 6gm Benefit from transfusion


6-8 gm Transfusion unlikely to be beneficial in the absence of bleeding
or impending surgery
> 8gm No indication for transfusion

Images from APP:

Leukocyte reduction Filter to remove WBC and this prevents Non Hemolytic Febrile Reaction*

Foley’s Catheter:

SURGERY SIXER APP BASED WORK-BOOK 2020 18


Materials Used:
- Latex rubber
- Silicon

Silicon Catheter Latex Rubber Catheter


• Resistant to Chemical • Increased risk of Infection
• Insensitive to temperature • Hypersensitivity
(Chemical and Thermal Stability) • Irritation of Bladder mucosa
• Silicon catheter less chances of • Must be changed once in a month
Encrustation and Calcification and
hence less changing of catheter needed
(3 months once)

Color codes for Foley and Size:

• MC used is Orange ( 16 Fr)


• Women: 12 – 14 Fr, Men 16 – 18Fr catheter is used,
• Three Way Foley’s Catheter is used for – Bladder irrigation , Commonly used after
Prostate Surgery- TURP.
• In case of Retention of Foley’s inside Bladder and Unable to remove the Foley’s catheter-
we rupture the Foley’s Bulb using USG guidance.
• 1 mm= 3 French in units ( or) 1 French= 0.33 mm
• The Size mentioned in Foley’s catheter is based on Outer Diameter of the Catheter**
• Balloon is inflated with Sterile water only. Saline and Air must not be used. Saline may
cause blockage with crystals.
• In case of retained Foley’s how to remove it ideal- USG guided Puncture of Balloon

SURGERY SIXER APP BASED WORK-BOOK 2020 19


Topic 1c- Sutures , Suturing techniques, Knots and Anastomosis

Sutures:
• 1st Used Suture- ANTS

Non absorbable Natural materials:


Silk:
• Black color suture
• Mainly used for Skin Sutures
• Increased infection rate.

Nylon:
• Skin sutures

Steel Sutures:
• Bones and Ribs and Sternums

Non Absorbable Synthetic Suture materials:


Polypropylene:
• Blue color
• Monofilament materials (Advantage- Less infection . Disadvantage- Poor Knotting
property)
• Classical feature of Coiling back is known as MEMORY* ( Bad Knotting property)
• Prolene- 6-7 knots used.
• Applications of Prolene:
o Hernia repair
o Hernia mesh
o Vascular suturing ( Aorta, Femoral artery)
o Tendon repair ( like Tendo achilles)
o Fascia repair
o Diaphragmatic hernia

SURGERY SIXER APP BASED WORK-BOOK 2020 20


o Linea alba closure
o Pancreatic DUCT ** ( Prolene or PDS is used )
Poly tetra Floro Ethylene:
• Graft materials for Femoral artery, Popliteal artery
Dacron:
• Graft material for AORTA replacement
Absorbable Suture Materials
Natural Absorbable:
Catgut:
• Brown in color
• Plain – 10 days
• Chromic – 60-90 days
• Storage solution of Catgut- Isopropyl Alcohol**
• Applications:
o Subcutaneous
o Muscle approximations
o Plain catgut- Circumcision
• Highest enzymatic reaction* , hence Banned in UK
• Absorbed by Enzymatic degradation.

Polyglycolic Acid: ( Vicryl)- Workshop material in theatre


• Multifilament material ( Made up of Glycolic acid+ Lactic acid co polymers)- as it is
multifilament it has good knotting capacity but increased infection.
• Violet color
• Absorption period- 60 -90 days
• Absorbed by Hydrolysis
• Types of Vicryl:
o Barbed Vicryl- No knotting needed, Used in Facelift sutures, plastic surgery
o Vicryl plus- Antibiotic coated ( Trichlosan antibiotic)
o Vicryl Rapide- Absorbed in 10 days
o White Vicryl ( MONOCRYL) – Poly glycaprone ( Monofilament material)- Suture
of choice for Subcuticular sutures in Skin ( Cosmetic Skin Closure)
• Applications:
o Bowel Anastomosis ( 3’0 vicryl)
o Bile duct anastomosis (4’0 Vicryl)
o Muscle closure
o Subcutaneous closure
o Vicryl Meshes ( Inner side of Dual Mesh to prevent bowel Adhesions)
• Pancreatic duct- Vicryl must not be used ( use only prolene or PDS)

Poly Dioxanone:
• Longest absorption period- 180-240 days

SURGERY SIXER APP BASED WORK-BOOK 2020 21


• Used for Linea alba closures

Properties to be noted in suture materials:


Thickness size : Ascending Order: 12’0< 10’0< 8’0< 6’0< 4’0<3’0<2’O< 0 < 1 size sutures
• Aorta repairs- 3’0
• Femoral repairs- 4’0 or 5’0
• Ophthalmic surgeries- 6’0

Size is inversely proportional to tensile strength of sutures*

Needles:

Shapes of Needles:
Round Body Cutting Reverse Cutting
Less traumatic to tissues More traumatic Combines the advantage
Used in Bowel Used in Tendon, Fascia, Skin properties of both Round
Mainly in places where we body and Cutting like
need to penetrate easily - Less traumatic
Disadvantage: - Easy to penetrate
- Cannot be used on Disadvantage:
hard tissues like skin - Cannot be used on Used in vascular sutures
soft tissues like bowel

Blades:

SURGERY SIXER APP BASED WORK-BOOK 2020 22


Skin Suture Techniques:

X: 2X rule: Skin Sutures:


• The depth of wound is X- the suture is taken at X cm on each edges.
• The next suture must be 2X cm from the previous suture.

When to remove the skin sutures?


• Scalp- 6-8 days
• 3-5 days- Lids, Lips and face
• 8-10 days- Laparotomy, Thoracotomy, Ear lobe repair
• 10-14 days- Palms and soles, back

Bowel Anastomosis:
• Halstaed Mathieson Single layer – Extra mucosal suture- Best and MC done
• Kocher’s 2 layer full thickness technique
• Sero muscular suture over the previously done anastomosis- LAMBERT SUTURES**using
Silk or non absorbable material as a support to previous sutue.
• Cheatle Cut: Approximate Disproportionate bowel by end to end anastomosis*

Cheatle Cut

SURGERY SIXER APP BASED WORK-BOOK 2020 23


Stapler Anastomosis:
Linear stapler Circular Stapler
( Transverse Linear Cutter -TLC) ( EEA- End to End anastomosis staplers)
Side to side anastomosis End to End anastomosis

Vascular anastomosis:

Carrel triangulation is used in Vascular anastomosis

Image Based Questions:


1. What are the instruments you need for skin suturing (Video @49.07)
• Needle Holder
• Toothed Forceps
• Straight Scissors

2. TA ( Thoraco Abdominal Stapler)-Image

Only anastomosis and no cutting by TA stapler

SURGERY SIXER APP BASED WORK-BOOK 2020 24


3. Stapler Pin Color:
• Green Stapler-4 mm open pin- 2 mm Cclosed stapled Height ( Mnemonic- Gastric-
Green)
• Blue stapler- 3.5 mm open - 1.5 mm Closed Stapled Height ( Mnemonic- Blue- Bowel)
• Vascular stapler- 1 mm Closed Stapled height (Mnemonic- Vascular- Vellai( Tamil)color)

Small staple height prevents bleeding like vascular stapler (1mm )


If you use a smaller staple on a thicker bowel – anastomotic leak happens.

4. Langer’s lines- Cosmetically good looking lines on skin

5. Named Incisions in abdomen:


• Paramedian incisions- Less chances of hernia, but more painful and bleeding
• Transverse incision- for paediatric
• Right subcostal incision- Kocher’s incision for Gallbladder surgery
• Roof top incision- Chevron for Pancreatic surgery
• Lanz- Cosmetic incision for appendectomy
• Mc Arthur- Grid iron incision for appendectomy
• Muscle cutting incision in abdominal wall is Rutherford Morrison incision**
• Lanz, Mc Arthur, Pfannenstein all are muscle splitting incisions

SURGERY SIXER APP BASED WORK-BOOK 2020 25


6. Knots:

Granny knot ( Slip Knot) Square knot ( Reef Knot)


Right over the left Right over the left
Right over the left Left over the right
Not secure knot Secure knot and it will not get loosened
It will slip and fix in deeper pelvis or cavities
It will slip
Remember mnemonic- “Grandma will slip”

RRM’s method to remember:


Sir, very difficult to understand- any simple way?
• Yes- crude method- see in the crossing area of suture in centre:
• Granny knot will have colours crossing opposite ( Example in 1st image- Red is on
top in one and Blue is top on other)
• Square knot will have same colour on the top ( Example in 2nd image- Blue is on
top)
Just seeing the image you can say what knot it is by following my method.

Surgeons KNOT”- For added security- Two throw technique of knots done and is advisable
to prevent slippage.

SURGERY SIXER APP BASED WORK-BOOK 2020 26


Aberdeen Knot- is used when completing the continuous sutures. Free end is partially pulled
through the final loop before cutting.

Aberdeen knot

Space for additional points:

SURGERY SIXER APP BASED WORK-BOOK 2020 27


Topic 1d: Operation theatre, Instruments and Protocols
Surgical Safety List ( WHO UK Process)
• Sign in Check done before induction of anaesthesia*
• Time out Check done before Skin incision*
• Sign out Check done before patient leaves operating room**
• Please remember there is no column known as Time in**

Sign in Time Out Sign Out


• Name • Discussion between • Look for gauze count,
• Identity Surgeon and instrument count
• Confirm the disease anesthetist. • Any instrument
• Site marked or not • Anticipated time of problem is addressed
• Written consent surgery and blood loss here.
• H/o Drug Allergy • Prophylactic
• Associated diseases antibiotics given or
not is noted.

• Best Method to prevent operating wrong limb- Surgeon and Anaesthetist examining each
other separately.

Universal precaution kit: ( personal Protective Equipment kit)


• Boots
• Goggles ( Face shield)
• Plastic Gowns
• Double gloves ( Inside wear bigger gloves and outside your size gloves)

• Take high risk cases in 1st round


• Avoid unnecessary personnel in OT

SURGERY SIXER APP BASED WORK-BOOK 2020 28


• Transfer only in Kidney Tray

Sequence of removal of PPE after surgery ( Recent AIIMS)- Mnemonic- GFGF


• 1st- Gloves
• 2nd- Face shield/ Goggles
• 3rd- Plastic Gown/ Aprons
• 4th – Facemask/ respirator

MC Finger injured during surgery- Non Dominant hand index finger*


• Hollow needles have more risk of injury than surgical needles.

Inside OT:
Scrubbing- Hand Washing:
Sequence of washing ( Recent Washing)- Watch the procedure demonstration
• 1st- Palmar surface
• 2nd- Back of hand
• 3rd – Interdigits
• 4th- Finger tips
• 5th – Thumb

Wash upto elbow- 1st Normal water, 2nd Povidone scrub solution and finally Sterile water. Finally
apply sterilium antibacterial solution.

Instruments used in Operation Theatre:

Allis Tissue Forceps:

• Allis forceps for holding the tissues firmly*


• Used during laparotomy to hold the skin margins.
• Used to hold skin flaps during excision of Lipoma, Sebaceous cyst.
• It is helpful in creating flaps during thyroid surgeries*

SURGERY SIXER APP BASED WORK-BOOK 2020 29


Lane’s Tissue forceps:

• Used to hold structures very tight.


• It is used to hold submandibular gland and Parotid gland during dissection
from adjacent structures.
• Helps in holding breast during mastectomy.
• Also helpful like a towel clip to hold the suction tubes, draping sheet.

Babcock’s Forceps:

• Used to hold tissues very softly.


• The ideal instrument to hold and grasp the bowel and appendix during surgery*
• Hold the margins of stomach or small intestine during anastomosis*

Instruments used in Skin Suturing:


• Needle Holder
• Toothed forceps
• Straight scissors
Curved Scissors not used to cut sutures, used only to cut tissues.

Curved artery forceps- To catch the bleeding vessels


Straight artery Forceps- To hold tissues

SURGERY SIXER APP BASED WORK-BOOK 2020 30


Cheatle’s Forceps:

• It is used to pick instruments and Mopping pads/ gauzes from Bin in a sterile way.
• It is kept in a sterile Bottle Container.

Langenback retractor:

• Used to retract tissues during hernia surgery, Appendectomy etc,


• Available in various sizes.

SURGERY SIXER APP BASED WORK-BOOK 2020 31


Morris retractor:

• Used to retract the abdominal wall in a wide manner.


• It is the instrument of choice for retraction in left subcostal region because it avoids the
risk of splenic injury*
Czerny retractor:

• Serves the same purpose of a Langenback retractor, but has another limb with a double
hook with space in between.
• Sutures can be made in the tissues between the gap of hooks.

Deavers retractor

• Used to retract liver without any damage to liver parenchyma.


• Used during Cholecystectomy to retract liver.

SURGERY SIXER APP BASED WORK-BOOK 2020 32


• Used in places where a careful retraction without organ damage is needed.

Doyen’s retractor:
To retract the pelvis and bladder at pubic symphysis.

Thompson Self retaining Liver retractor:

• Thompson retractor has multiple arms which can be fixed to a rod which is attached to
the Operating table.
• Adjustable and Self retaining in nature.

Balfours Abdominal Self Retaining retractor:

SURGERY SIXER APP BASED WORK-BOOK 2020 33


• It’s used to retract the lateral abdominal walls.
• The third limb- used to retract the bladder downwards.

Joll’s self retaining Thyroid retractor:

• Used during thyroidectomy* to retract the platysma.


• It’s a self retaining retractor.

De Bakey Bull dog clamp:

• Used to clamp blood vessels without damaging the blood vessel.


• The clamp has opposite action compared to other forceps- When Pressed it opens and
When Clamp released it occludes the blood vessel**

De Bakey Satinsky Clamp:

• Used for vascular clamping during vascular repairs, injuries


• Used to hold the cut end of esophagus after Total gastrectomy.

SURGERY SIXER APP BASED WORK-BOOK 2020 34


• Used during shunt operations.

Towel Clips:
• Towel clips are used to hold the sterile dressing after drap in position*
• Some surgeons use this to hold and retract the tongue outside for intra oral surgeries.

Blade Handles: ( Bard Parker Handles)


• 3 size to load – 11 size blade and 15 size blade
• 4 size to load – 22 Size blade

Colour of venflons:
• Grey- 16 G
• Green- 18 G
• Pink- 20 G
• Blue- 22 G
• Yellow- 24 G

Demonstration of Foley’s catheter, Staplers are elsewhere in the Chapter 1

SURGERY SIXER APP BASED WORK-BOOK 2020 35


Drains:
Romo vac Suction Drain:

Energy devices in operation theatre:

1.Diathermy:
It is a machine which converts Electrical energy into heat energy.

Figure : Diathermy machine

• Yellow Button in the Pencil is for Cutting ( Continuous Wave forms)


• Blue Button in the Pencil is for Coagulation ( Intermittent Wave forms)
• Earth Plate prevents Electric Shock to patient.

Diathermy can be used for three purposes:


• Coagulation: the sealing of blood vessels.
• Cutting: used to divide tissues during bloodless surgery.

SURGERY SIXER APP BASED WORK-BOOK 2020 36


• Fulguration: the destructive coagulation of tissues with charring. It’s a blended current
which uses both the above forms of diathermy activity

Monopolar Diathermy:

• Monopolar diathermy – the electrical energy is passed via the diathermy probe and
returned to the machine via the patient earth plate as shown in image above.
• MC complication inside OT is diathermy burns**
• Pacemaker containing patients must be carefully watched during monopolar diathermy
and earth plate must be far away from pacemaker.
• Contraindications of Monopolar Diathermy:
o Finger tips
o Penis ( Circumcision)
o Thyroid and parotid surgery near nerves

Bipolar diathermy:

• Bipolar diathermy needs no earth plate.


• The current is passed between the limbs of diathermy probe.
• Safe in Surgeries near nerve and circumcision.
• Can be safely used in pacemakers.

SURGERY SIXER APP BASED WORK-BOOK 2020 37


Image Based Question:

Please note the Bipolar and Monopolar Probes

Harmonic Scalpel:

• The harmonic scalpel is an instrument that uses ultrasound technology to cut tissues while
simultaneously sealing them utilises a hand-held ultrasound transducer and scalpel at
frequency of 20 000–50 000 Hz
• Protein denaturation caused by vibration** rather than heat

Ligasure:
• Machine which involves same technology of Monopolar but uses the collagen and elastin of
the patient himself to seal and divide.
• 7mm Vessels can be ligated and cut

Drains in Surgery:

Open Drains Closed Drains

• Include Corrugated Rubber or Plastic Consists of Tubes draining into a bag or Bottle
Drains

• Drain fluid collects in a pad or Stoma Abdominal Drains, ICD drains, Suction drains
bag are examples

• Increased risk of infection seen Infection risk is less

SURGERY SIXER APP BASED WORK-BOOK 2020 38


Corrugated Rubber Open Drain

Jackson Pratt Closed Drain

Intercostal Drain Tube

Mallecot’s Self retaining Catheter drain

Double Pig tail Stent ( Double J stent)- Ureter, Bile duct

SURGERY SIXER APP BASED WORK-BOOK 2020 39


Place of Drain For Purpose of Removal on

Thyroid surgery To identify bleeding 24 Hours

Breast surgery To remove Serous Discharge 5 Days

ICD drain To drain Collections Collection < 20 ml/Day

T Tube Drain To Drain Bile 10- 14th POD

Pelvic drain after Colorectal To identify the leakage 7th POD


anastomosis

Inside the OT:


• Airflow/ hour- 15 to 25/ hour
• Laminar Air flow OT- Air exchange every 2 minutes ( 100-300/ hour may happen)-
Ortho and CTVS OT must have Laminar flow
• Temperature- 18-25 Degrees

Position of Patients in Operation theatre:

Trendelenburg:
Reverse Trendelenburg:
Varicose Vein Surgery
• Thyroid Surgeries

Llyod Davis position Jack knife position


• Trendelenburg + Leg Split • Prone Position
• Laparoscopy procedures • Pilonidal sinus operated
• Positional Asphyxia may happen

SURGERY SIXER APP BASED WORK-BOOK 2020 40


Lithotomy Position
Sitting position:
• Gynaecology procedures
- Neurosurgery
• Injury to Common Peroneal nerve
- Air embolism can happen
can happen

Laparoscopic Surgery:
• Insufflation of gas in peritoneal cavity and operating via small holes.
• Gases used : Carbon Dioxide( MC), Nitrogen, Air
• Carbon dioxide is rapidly absorbable and causes less post-operative pain and less incidence
of Fat embolism**
• CO2 is 200 times more diffusible and rapidly cleared from circulation*
• Intraperitoneal pressure maintained during surgery- 12-15 mmHg**
• Laparoscopy pneumo peritoneum is created by 2 methods: Open and Closed method

Open method Closed method


Using Hassan’s Cannula Using Verres needle
Risk of major vessel injury is less Risk of Bowel injury is less as it has safety
valve at tip.
Trocar penetrates- Skin, Superficial and
Deep fascia, Fascia Transversalis, parietal
peritoneum.

SURGERY SIXER APP BASED WORK-BOOK 2020 41


Complications of Laparoscopy:
• Primary cannula (1st Cannula Insertion)- Injury to Bowel, Blood Vessel, Stomach,
Bladder, Mesentry, Diaphragm, AORTA, IVC, Epigastric vessels are injury ( MC in
Secondary Cannula)
• Secondary cannulas (Under vision inserted)- Epigastric vessel injury MC

Gas embolism:
• Sudden drop in p0 2 immediately during induction of Pneumoperitoneum suggest Co2
gas entering the systemic circulation via the torn splenic vessels by Verres or Trocars.
• MC seen at the time of insufflation of gas by trocar or Verres.
• Less common when Co2 gas is used compared to air (as Co2 is more soluble)
• Initial rise of ETCo2 due to pulmonary excretion of absorbed Co2 is followed by a sudden
decrease due to fall in cardiac output**
RRM’s Extra Bite:
DURANT’S Position/ manuever:
• A position used in Air embolism in which the patient is immediately put in left lateral
decubitus with head low position( Trendelenburg) so that gas will remain in the apex of
ventricle and may be aspirated under Echo guidance.

Capacitance Coupling Injury:


• On sandwich of the insulator between two metal electrodes – A capacitor is created.
• This occurs when we pass a metal hook into a metal made Port. By electromagnetic
induction current is created and there is a damage to abdominal viscera.
• Patient Presents with delayed leak
• Plastic ports avoids this injury.

SURGERY SIXER APP BASED WORK-BOOK 2020 42


Delayed Complications:
• Unabsorbed gas can irritate the diaphragm and causes shoulder pain*. It is worst
at 24 hours after operation and settles in 2-3 days. Paracetamol is enough.

Other advanced Laparoscopic Surgery:


• SILS- Single Incision Laparoscopic Surgery ( Single Umbilical port with multiple
openings)- Increased incidence of hernia
• HALS- Hand Assisted Laparoscopic Surgery
• NOTES- Natural orifice Transluminal Endoscopic Surgery
• SPA- Single Port access ( Other name of SILS)

Robotic Surgery:
• Davinci Robot*

• Docking- Inserting the limbs of Robot inside


• Major Advantages:
▪ Eliminates tremors of the surgeon
▪ Multidirectional movement- 7 degree freedom
▪ Tele robotic Surgery can be done

VATS: Video Assisted Thoracoscopic Surgery:


• No need of Co2 Gas to create space.
• Space is created by collapsing the lungs.

Other instruments:
Desjardin Stone removal Forceps:
• To remove CBD stone

SURGERY SIXER APP BASED WORK-BOOK 2020 43


Vim’s Silverman Biopsy Needle:
• Liver Biopsy needle

Payr’s Soft Occlusion clamp of Bowel


• To clamp the bowel and prevent the spillage of the content

Space for Extra Points:

SURGERY SIXER APP BASED WORK-BOOK 2020 44


Topic 1e: Surgical Site Infections:

Type of Surgery Examples of Surgeries Infection rate Infection rate


with Prophylaxis without
prophylaxis
Clean Surgery: • Heart, Brain, Joint, Transplant 1% 1%
(No viscus opened) surgeries
• Herniorrhaphy*
• Swelling excision
Clean Contaminated • Wound of Bowel, Biliary and 3% 5-10%
Surgery: Pancreatic Surgery
(Viscus Opened, • Uncomplicated appendicitis*
Minimal Spillage) • Gastro Jejunostomy
Contaminated • Appendiceal abscess 6% 10-20%
surgery: • Perirectal abscess drainage
• Infected laceration
• Diverticulitis

Dirty Surgery • Worst wound 7% Upto 40%


• Acute cholecystitis with spillage of
pus from gall bladder.
• Traumatic wound
• Bowel obstruction with
enterotomy and spillage of
content.

Surgical Site infection is defined as :


• Wound Infection in < 30 days of Surgery
• Wound Infection in < 1 year of Implant cases
• Most common SSI pathogens are all gram-positive cocci—Staphylococcus
epidermidis**, S. aureus, and Enterococcus **
• For infra inguinal incisions and intra cavitary surgery, gram-negative bacilli such as
Escherichia coli and Klebsiella are potential pathogens.

Grading Systems for SSI:


Southampton Wound grading System: ( Mnemonic- EISPA)
Grade Appearance
0 Normal healing
1 Erythema
2 Inflammation
3 Serosanguinous discharge
Major Complications

SURGERY SIXER APP BASED WORK-BOOK 2020 45


4 Pus
5 Anatomical separation

ASEPSIS Scoring system


Criterion Points
A Additional treatment 0
• Antibiotics for Wound Infection 10
• Drainage of Pus under Local anesthesia 5
• Debridement of Wound under GA 10
S Serous Discharge Daily 0-5
E Erythema Daily 0-5
P Purulent Discharge Daily 0-10
S Separation of deep tissues Daily 0-10
I Isolation of bacteria from Wound 10
S Stay as inpatient prolonged over 14 days 5
(Author’s Excuse: Sorry for mentioning in video “this is not asked so far”- probably the
examiners noted it and asked it immediately in NEET PG 2020)

Causes of Post operative fever:


Day Cause ( Mnemonic- ABCDE)
1 Atelectasis of Lungs
2-3 Hospital Acquired Infection (MC UTI- Bladder infection)
Atelectasis- Pneumonia
Superficial Venous Thrombophlebitis ( Catheters)
4-5 MC-Surgical Site Infection
Deep Vein Thrombosis
6 Burst ( Exposed) Abdomen in Laparotomy cases
Salmon sign- Sudden gush of fluid is the 1st sign**
7 onwards Intra- abdominal Collection
(MC in Pelvis/ Morrison Pouch)

Incentive spirometer for Breathing Exercises

SURGERY SIXER APP BASED WORK-BOOK 2020 46


Pneumatic Compression Device to prevent DVT

Factors preventing wound healing:


• Hypoalbuminemia
• Obesity, Weight loss
• Diabetes
• Hypercholesterolemia
• Renal failure
• Cancer, AIDS, Chemotherapy, radiotherapy
• Jaundice
• Old age
• Poor surgical techniques

How to prevent SSI?


• 1 hour before surgery Prophylactic antibiotics ( before incision)
• Shaving not advised now ( If needed skin clippers or epilators used to remove the hair just
before shifting to OT)
• Avoid hyperglycemia during surgery ( Blood sugar < 200 mg/dl)
• Avoid Hypothermia
• Avoid contamination ( Sterile precautions)
• Avoid Hypoxia Intraop and post op also for 4 hours.
• Avoid Hypotension
• Surgery Extending > 4 hours ( Every 4 hours one more dose is given)- Prophylactic
antibiotics not given beyond 24 hours.

Burst abdomen closures:


• Tight abdominal closure ( not advised nowadays- may result in Abdominal Compartment
syndrome )
• Latest- Vacuum Assisted Closure Device ( Negative Suction of -120 mmHg of Excess fluid
is absorbed)

SURGERY SIXER APP BASED WORK-BOOK 2020 47


Space for Extra points:

SURGERY SIXER APP BASED WORK-BOOK 2020 48


Chapter 2: Trauma

Topic 2a- Introduction

Triage:
In cases of Mass Casualty, categorising the patients to be transferred to Hospital according to the
severity of injury is known as Triage.

Triage a french word means “to sort”


Four colour codes are given
• Red- first priority is a critical patient (Eg: Tension Pneumothorax, Pericardial
Tamponade)
• Yellow- urgent is second priority (Eg: Major Bone Fractures, Open Pneumothorax)
• Green is minor and third priority (Eg: Tibial Fractures, Wrist bone fractures)
• Black is dead or about to die patients.

• Multiple Casualty: Enough resources available to manage the patients


• Mass Casualty: Enough resources are not available to manage the patients coming to
casualty

Primary survey : cABCDE


• c- Exsanguinating External Haemorrhage
• A-Airway
• B-Breathing
• C-Circulation
• D-Disability and neurological status
• E-Exposure

C: Exsanguinating External Haemorrhage:

SURGERY SIXER APP BASED WORK-BOOK 2020 49


• Stop major vessel bleeding by pressure or Tourniquet first before ensuring airway

Airway with Cervical spine Control:


• Intubation when GCS< 8
• Put a Hard Cervical Collar and make the patient lie on a Hard Board
• Airway Manoeuvres to prevent Tongue Falling backward:
- Jaw Thrust
- Chin Lift
- Guedel Airway
- Definitive Airway: Oropharyngeal ,Nasopharyngeal airway insertion, Tracheostomy
intubation.

Orotracheal intubation Nasotracheal Intubation


• MC done method • Contraindicated in
• Contraindicated in Maxillo-facial - Base of Skull Fractures
injuries( more than one time we must - Cribriform Plate fractures
not do) - CSF Rhinorrhea+
- Maxillo facial fractures

• Maxillo facial injuries- Tracheostomy Intubation advised*

Tracheostomy:
Emergency Needle tracheostomy Elective Tracheostomy
• Also Known as Cricothyroidotomy • Done at 2/3rd
• 16 mm Needle Inserted Tracheal rings
• 3mm/ 4mm Catheter inserted
• High Flow Oxygen is infused
• Contraindicated in Children < 12 Years: Subglottic
stenosis will happen.
• Maximum for half an hour we can give High flow O2 ,
beyond that CO2 retention occurs

LEMON Assessment:
• L- Look Externally
• E- Evaluate 3-3-2 Rule ( 3 Fingers enter mouth, 3 Finger distance between hyoid bone
to chin, 2 finger distance between Thyroid notch to floor of mouth)
• M- Malampatti classification ( Class 4- Only Hard Palate is seen; difficult to intubate)
• O- Look for obstruction
• N- Neck Spine Tenderness

Breathing and ventilation


• Oxygen must be offered to all trauma patients via reservoir mask in high flow.

SURGERY SIXER APP BASED WORK-BOOK 2020 50


• Ventilation must be assessed by seeing chest walls, lungs
• Tension pneumothorax, flail chest, massive haemothorax and open pneumothorax are all
clinical diagnosis and not radiological diagnosis and need immediate treatment.

Circulation
• Rapid thready pulse is more reliable and warning sign than BP**
• It represents the Blood lost:
“ One on Floor and four more”
- Closed fist size Clot on floor ( 350 ml), Pad fully soaked ( 500ml)
- Abdominal Cavity
- Chest
- Pelvic
- Long Bones

Management of circulation:
• Two large bore, short, peripheral IV catheters. ( Green 16 G 2 Cannulas )
• Other options are Cut down into Saphenous veins or IJV
• Children < 6 years- Intraosseous transfusion
• 1 litre of 7.5% Hypertonic Saline
• Permissive Hypotension: Target Systemic pressure 70-90 mmHg; It must not go very
high to prevent rebleeding and must not go very low to prevent cerebral hypoxia.
• CRASH-2 Trial:
o Tranexamic Acid: 1 gm IV Stat over 10 minutes followed by 1gm for 8 hours.
o It should be given to all trauma patients suspected to have significant
haemorrhage with PR >110 /mt or BP < 90 mmHg systolic.
Disability and Exposure:
• Expose the patient fully and examine
• Log Roll is done in this heading ( Discussed separately)

Secondary Survey:
• Gold Standard Investigation: WBCT ( Whole Body CT Scan) from head to pelvis with IV
contrast for severely injured Adult Blunt trauma patient**
• One WBCT = 76 X ray Chest
• Provisional hot report issued in minutes for WBCT and definitive report obtained after
30-60 minutes,

Log Roll:
• Not done now as there is a delay in WBCT.
• Done by 4 people
• Primarily meant for Spine Examination
• Part of Primary Survey
• Along with Spin examination we can also do Anal Wink Reflex and PR examination.

SURGERY SIXER APP BASED WORK-BOOK 2020 51


• During PR- Vermooten sign to look for Floating Prostate in Membranous urethra injury
(contraindicated now)
• Log roll is absolutely -Contraindicated in Pelvic Fracture patients**

In secondary survey:
Remember Mnemonic AMPLE:
• Allergy H/o
• M- Medical H/o
• P- Pregnancy H/o/ Present Illness
• L- Last meal
• E- Explain the mechanism of injury

Xray Cervical Spine is taken in secondary survey:


• AP and Lateral View taken
• Based on NEXUS Criteria and Canadian C Spine Rule

Scoring Systems:
RTS score TRISS Score MESS Score
• R- Respiratory rate TRISS includes • M- Main energy that caused
• T- Tie and see BP (Blood • R- RTS injury
pressure) • I- Injury Severity score • E-Extremity Ischemia
• S- Scale (Glassgow Coma • S- Seen Age • S- Seen Age
scale) • S- Specific Mechanism • S- Shock
(Blunt or Penetrating)

ABBREVIATED INJURY SCALE:


• Has a Seven Digit Code and represented as 123456.7
1 Anatomic Body Region
2 Type of anatomic structure
3/4 Specific anatomic structure
5/6 Level of injury
Post dot 7 Grade of injury

• Pre dot Code for Body region (1st digit) is as follows


1 Head
2 Face
3 Neck
4 Thorax
5 Abdomen & Pelvis
6 Spine
7 Upper Extremities
8 Lower Extremities

SURGERY SIXER APP BASED WORK-BOOK 2020 52


9 Burns and other trauma

Damage control surgery (Abbreviated Laparotomy)


Indications for DCS:
Deadly triad**
• Hypothermia
• Acidosis
• Coagulopathy
Others:
• Serum Lactate > 5mmol/L
• ISS >36
• Systolic BP < 70 mmHg
• Transfusion > 15 units of Blood

Early Trauma Care ( ETC)- Definitive management in Trauma Patient:


- Within 36 hours ETC if there is none of the above features seen

Serum Lactate- Significance:


• <2 mmol/l- ETC
• 2-5 mmol/l- Management in ICU
• > 5mmol/l- DCS

Phases of Damage Control Surgery:


• Phase -1: Initial Exploration
• Phase-2: Secondary Resuscitation
• Phase -3: Definitive operation

Phase -1 Phase -2 Phase- 3


Initial exploration Secondary Resuscitation Definitive treatment
• In Emergency OT • Transfer to ICU • Planned re exploration
• Control Active haemorrhage • Ventilatory support and definite surgery*
and Contamination* • Correct the deadly • Done 48-72 hours after
• Midline incision- 4 quadrant triad- Hypothermia, secondary phase*
packing done. Acidosis, Coagulopathy* • Complex reconstruction
• GIT perforations closed with must be avoided*
sutures or Staples or as Stomas
• Temporary closure of abdomen
using plastic sheet known as
OPSITE**
• This technique of closure is
known as

SURGERY SIXER APP BASED WORK-BOOK 2020 53


“ VACPAC or OPSITE
SANDWICH”

Stages of Damage Control Surgery as Per Bailey and Love


Stage Procedure
I Select the patient
II Control haemorrhage, Contamination
III ICU resuscitation
IV Definitive repairs
V Closure of abdomen

Topic 2b : Head Injury, Face injury and neck injury

Values in Neurosurgery:
• Cerebral Blood flow= 55ml/minute
• Ischemia happens if < 20 ml/minute
• Maintain Cerebral perfusion pressure (CPP)– Normal= 75-105 mmHg to prevent
Ischemia
• CPP= MAP(90-110)- ICP ( 5-15)= 75-105 mmHg

Munro Kellie Doctrine:


• Skull is closed Compartment containing 3 Components in a Constant manner.
- Brain
- CSF
- Blood
• If one component is compressed or increased the other component will compensate.
• Hence you can realise- if there is a hematoma on one side of the brain it will do the
following: ( see the image below)
- Midline shift ( 1)
- Cingulate Gyrus herniation (2)
- Herniation of Uncus of Temporal lobe (3)
- Brain stem herniation ( 4)
- Cerebellum herniation ( 5)

SURGERY SIXER APP BASED WORK-BOOK 2020 54


Features of herniation:
• Uncus of temporal Lobe Herniation- Ipsilateral Fixed dilated pupil happen due to
Oculomotor nerve compression
• Cerebellum herniation- Cushing Triad- Hypertension, Brady cardia and respiratory
Irregularities.

How to prevent brain herniation?


• By reducing the CSF pressure inside: by reducing ICP:
• Head up position 15-30 degree
• Injections to reduce ICP: Mannitol( osmotic diuretics), Diuretics if BP is fine.
• Sedate them- Inj Barbiturates (Don’t give Diazepam as it can cause respiratory
depression)
• Seizure prophylaxis- Sodium Valproate
• Open Skull fractures- prophylactic antibiotics
• Don’t give Steroids in Trauma** ( Can be used in Malignancy in Brain causing Increased
ICP)
• Lumbar puncture is contraindicated.
• External ventricular drainage is used to remove the CSF**
• Nasal Oxygen

Neuro intensive care unit values:


• ICP < 20 mmHg
• CPP > 60-70mmHg
• MAP> 90 mmHg
• PaO2- 80-100 mmHg (10 -12 Kpa)
• PaCO2- 35- 45 mmHg (4-6 Kpa)

SURGERY SIXER APP BASED WORK-BOOK 2020 55


Glassgow Coma Scale:
Eye opening: Verbal Motor
• Spontaneous – 4 • Normal Oriented- 5 • Obeys commands- 6
• To loud voice- 3 • Confused- 4 • Localizes to pain- 5
• To pain stimuli- 2 • Inappropriate words- 3 • Withdrawal flexion- 4
• Do not open- 1 • Sounds only- 2 • Abnormal flexion- 3
• No sounds- 1 (decorticate)
• Extension- 2 (Decerebrate)
• No motor response-1

Mnemonic:
• Verbal- One Confused Word Sounds Nowhere* (54321)
• Motor- Obey Localities With Flexion and Extension Now. ( 654321)

Essential points in GCS


• Motor component – most predictive of future neurologic outcome
• Maximum Score= 15
• Minimum Score ( Dead Patient)=3
• Intubated Patient= Mentioned as VNT ( Not Testable)= Score -0
• Intubate when score is < 8
• At admission when score is <13 take CT scan
• Score represents the best performance elicited, Example: Flexion response to pain in left limb and
localising pain in right limb- take into consideration the best which is in right limb- M5**

Classification of Head injury based on GCS:


• Minor: GCS 15 with no LOC
• Mild: GCS 14 or 15 with LOC
• Moderate: GCS 9-13
• Severe: GCS 3-8

Salient points:
1. Hutchinson Pupil: Ipsilateral Fixed and Dilated Pupil due to Oculomotor nerve stretching. Hence
Blind Burrhole without CT scan is done in Side of Hutchinson pupil in olden Days.

2. Kernohan Notch Phenomenon: usually there will be contralateral hemiplegia if one side
hematoma is seen. But in this phenomenon, Huge hematoma will cause brain pushed to opposite
side and compresses Pyramidal tract on Same side itself.

SURGERY SIXER APP BASED WORK-BOOK 2020 56


3. Base of Skull fractures:
• Raccoon/ Panda eyes- bilateral peri orbital hematoma ( Anterior Cranial Fossa fractures)
• Battle’s sign- bruising over mastoid. Middle cranial fossa Fracture
• CSF otorrhea- Middle Cranial Fossa Fractue
• CSF rhinnorrhea – Anterior Cranial fossa fractures
• Haemotympanum or bleeding from ear.

Battle sign*

• Anterior cranial fossa fractures- NG tube and Nasotracheal intubation are contraindicated.
• CSF rhinorrhea testing:
- Put the CSF in a tissue paper
- Blood will occupy the centre and CSF will occupy the periphery.
- Beta -2 Transferrin will be elevated in fluid

• Posterior Cranial Fossa fractures:


- Visual disturbance
- VERNET Syndrome: 9-11 nerve paralysis due to Jugular foramen Compression
and fracture.

Indications of NCCT Brain- NICE Guidelines:


(National institute for health and clinical excellence)

Indications of CT in 1 hour:
1. GCS < 13 at any point*
2. GCS < 15 at 2 hours
3. Focal neurological deficit
4. Suspected open, depressed or base of skull fracture
5. Post trauma Seizure
6. Vomiting > one episode.

Indications of CT within 8 hours:


1. Age >65

SURGERY SIXER APP BASED WORK-BOOK 2020 57


2. Coagulopathy (on warfarin, aspirin or Rivaroxaban)
3. Dangerous mechanism of injury
4. Retrograde amnesia ( > 30 minutes)

EDH SDH SAH

Biconvex Appearance Concavo-Convex appearance MC type of haemorrhage-


Intracerebral haemorrhage
due to hypertension in
PUTAMEN.
The picture shown above is
Subarachnoid haemorrhage
with Blood in Subarachnoid
Cisterns
• M/c in young male • Accumulates in • MC cause of SAH
patients space between dura nowadays is Trauma
• Always associated and arachnoid • Other cause is Sudden
with skull fracture • Associated with rupture of Berry
• Injured vessel- middle Bridging vein Aneurysm
meningeal artery** disruption and brain
• M/c site of injury is laceration*
temporal bone at • Associated with
pterion (most thinnest primary brain injury
part of skull) which • C/F- Impaired
overlies the middle consciousness from
meningeal artery the impact time
• The hematoma is itself.
located between bone • Acute SDH- follows
and Durameter. trauma
• Lucid Interval is seen • Chronic SDH- people
on Anticoagulants*

SURGERY SIXER APP BASED WORK-BOOK 2020 58


Treatment: Treatment:
Burr hole is TOC Acute SDH- Decompressive
Mortality- 5% craniectomy and evacuation
Chronic SDH- Burr Hole
Mortality- 40%

Other injuries:
Diffuse Axonal injury:
• High Acceleration and deceleration injury
• No surgery done
• Wait and watch
• IOC- MRI

Skull fractures:
• Depressed Fracture- No need to worry
• Displaced Fracture ( open fracture)- Surgery needed.

AIS -2 Face injury:


• Mc injury – Nasal bone Fracture*
• Maxillofacial Fractures- Le fort Injuries:
Type 1: Fracture line at Hard Type 2: Pyramidal shaped Type 3: Cranio facial
Palate dislocation
-CSF rhinorrhea+

Mandible fractures: ( Sequence Based Question)


• Order of fracture in mandible- MC @ Neck of condyle> Angle of mandible > Canine
Tooth> middle of mandible.

Zygomatic Fractures
Orbit Fractures: Order of Injury is Floor> Medial wall> Lateral wall > Roof ( Sequence Based
Question).

SURGERY SIXER APP BASED WORK-BOOK 2020 59


Trapping of muscle happens in orbit fracture- Inferior rectus gets trapped in children Most
commonly.

IOC- CECT Facial Bones


TOC- ORIF ( open Reduction Internal fixation)

AIS 3: Neck Injury:


• It forms the highest mortality rate of all body regions with penetrating wounds are the
most common mechanism for this injury.
ZONES EXTENT REMARKS
Zone I From thoracic inlet to cricoid cartilage Large vascular structures, Trachea and
esophagus
Zone II Between Cricoid cartilage to angle of Most surgically accessible zone**
mandible Highest mortality **
Carotids, vertebral arteries, Jugular veins
Zone III Between angle of mandible and base of Difficult to access surgically
skull

Blunt cerebrovascular injury in Neck ( DENVER’S Grading):


Grade I Luminal irregularity with <25% narrowing
Grade II Dissection of hematoma with >25% luminal narrowing
Grade III Pseudoaneursym
Grade IV Occlusion
Grade V Transection with extravasation

• Grade 5: Needs Immediate Surgery


• Grade 1- 4: Anti thrombotic drugs ( Antiplatelets )

SURGERY SIXER APP BASED WORK-BOOK 2020 60


Topic 2C; Thoracic Trauma
AIS -4- Thoracic Injuries

Dangerous Dozen Injuries:


Immediately life threatening Potentially life threatening
• Obstruction of airway • Aortic injury
• Tension Pneumothorax • Trachea and Bronchus injury
• Pericardial tamponade • Myocardial contusion
• Open Pneumothorax • Diaphragm Rupture
• Massive Hemothorax • Esophageal Injuries
• Flail Chest • Pulmonary contusion

• MC injury in Thorax- Rib Fracture


• MC cause of Death in a Blunt Thoracic Injury- Tracheobronchial injury
• MC cause of Death in a Penetrating injury- Haemothorax

Intercostal Tube Placement: ( ICD Placement)


• Pass the ICD in upper border of the Rib ( in lower border intercostal vessels and nerve
are present)
• Triangle of Safety – Anteriorly by Lateral border of Pectoralis Major, Posteriorly by
Mid axillary line and Inferiorly by an imaginary horizontal line drawn at level of nipple
to meet mid axillary line .

Triangle of Safety

• Best space is 5th ICS in Midaxillary Line.


• 28Fr, 32 Fr tubes are used. Ensure all holes are inside the chest*
• Underwater drainage device providing 20 cm H2O suction.***

How to say ICD is functioning well?


• Moving of the Air liquid column in ICD tube in correspondence with respiration.

If the column is not moving: Implies:

SURGERY SIXER APP BASED WORK-BOOK 2020 61


• ICD is not in position
• ICD is blocked
• ICD is Displaced and hole lying outside
• Lung Has fully expanded

Other essential MCQs:


• ICD tube is removed when the fluid coming is less than 50 ml/day ( Some books 20
ml/day) or when lung is fully expanded.
• Remove the Tube in Full Inspiration.
• -5 to -20cm H2O Suction
• In hemothorax if there is excessive bleeding even if we clamp the tube is not useful ( it
doesn’t give tamponade effect)

Indications of thoracotomy in Thoracic Injury:


• Hemothorax more than 1500 ml( Blunt trauma)
• Hemothorax more than 1000 ml (Penetrating trauma)
• Hourly collection in ICD of > 200 ml/hour for 3 hours
• And all the Dangerous dozen Injuries ( Except pulmonary contusion, Flail Chest)

Pericardial Tamponade:
• MC mechanism – Penetrating trauma**
• Collection of Blood inside the pericardial cavity.
• Blood in pericardial cavity will compress the heart.
• Beck’s Triad: Raised JVP, Low BP and Muffled Heart sounds*
• X ray shows- Air Bag Appearance
• Differential Diagnosis: Tension Pneumothorax (Tachypnea with Absent Breath Sounds
seen)
• IOC – eFAST diagnosis ; Immediately under ECHO guidance at an angle of 45 degree you
must do Needle Pericardiocentesis*
• MC complication of Needle Insertion- Arrythmias*
• Definite Treatment- Thoracotomy and repair of Myocardial leak.

Tension Pneumothorax:
• Mechanisms- Penetrating trauma, Blunt Trauma with rib injuring, Central vein insertion.
• Collection of Air between the parenchyma and Pleura.
• Pleura is intact and hence during every breath air enters the pleural cavity and compress
the lung which is already compressed.
• This results in mediastinal shift, Absent Breath sounds**, low BP and raised JVP. ( DD:
Pericardial Tamponade)
• C/F- Tachypnea and Dyspnea

SURGERY SIXER APP BASED WORK-BOOK 2020 62


• Life saving Procedure – Midclavicular line along 2nd ICS – Put a Wide bore needle-
Emergency Thoracocentesis ( Latest ATLS manual- 5th ICS in Midaxillary Line in adults
and same old 2nd ICS in children)
• Definite procedure- ICD insertion.

Open Pneumothorax ( Simple)


• Both Lung and Pleura is injured.
• Chest wall is intact.
• Air will enter out of pleural cavity and air will go back to lungs and no tension.
• Not so dangerous and can be managed by putting an ICD tube*

Open Pneumothorax ( Sucking Chest Wound)


• Opening is seen in Lung ( Large defect >3cm) , Pleura and chest wall also.
• Air is entering from Atmosphere to lungs ( No proper Oxygenation)- Sucking the air.
• Immediate treatment- Three Way Flap valve type closure
• Definite Treatment- ICD insertion

Three way flap closure* in Sucking chest wound

Haemothorax:
• MC vessel injured- Intercostal artery
• MC vessel injured in massive hemothorax- Internal Mammary artery bleeding or Internal
thoracic vessels.
• 1st line treatment- ICD
• Immediately thoracotomy needed for >1500ml ( Blunt), >1000ml ( penetrating) and
>200ml/hour X 3 hours .

Rib fracture:
• 1st Rib fracture- Rare, but suspect Brachial plexus or Subclavian artery injury or apex of
lung injury.
• 10-12 ribs fractures- Think of Spleen or liver injuries
• CPR done- 4-6th Rib fractured*
• Management- Analgesics and No need of stripping or surgery needed for Simple fractures

SURGERY SIXER APP BASED WORK-BOOK 2020 63


Sternal Fracture:
• Very rare
• Suspect underneath Myocardial injury*

Flail chest:
• It’s a clinical diagnosis and not radiological diagnosis ( OLD CONCEPT)
• Three or more ribs fractured in two or more places**
• Even one rib fractured at Costochondral junction can cause Flail chest.
• Here a segment of chest wall does not have continuity with rest of thoracic cage.
Clinical features:
• On inspiration paradoxical movement of chest occurs. (segment moves inside on
inspiration and on expiration that flail segment will move outside)
• Due to pain there is hypoxia.
• Underlying pulmonary contusion is the most important prognostic factor*
Investigation of Choice:
• CT Scan with 3D reconstruction* to display vascular structures is the GOLD STANDARD
IOC . It also shows the underlying lung parenchymal injury
Management:
• Traditionally the treatment is mechanical ventilation.
• But current treatment is only oxygen administration, adequate analgesia and
physiotherapy.
• Mechanical ventilation is reserved for cases who develop respiratory failure, Pao2 <60%
and RR >18/minute- Intermittent positive Pressure ventilation(IPPV)
• Surgery and internal fixation of ribs are in use again

Paradoxical respiration in Flail chest.

Diaphragm Injury:
• Mc mechanism is penetrating trauma ( Below 5th ICS)- Wound will be bigger in
Diaphragm.

SURGERY SIXER APP BASED WORK-BOOK 2020 64


• Blunt Trauma – by rib causing perforation or Compression injury- MC on left side > right
side.
• Clinical feature: Most remain silent, if wound is big enough, the bowel , stomach and
abdominal viscera enters the chest.
• Ryles tube will enter the chest
• Don’t give bag and mask ventilation, immediately do IPPV.
• BERGVIST TRIAD: Fracture Rib+ Fracture Spine or Pelvis+ Diaphragm Rupture*
• MC organ to herniate – Stomach> Colon
Management:
• Most diagnostic investigation- VATS or Laparoscopy*- having added advantage of doing
repair simultaneously*
• Chest X ray and CECT are all used to diagnose*
• Surgery is advised for all cases whatever may be the size of tear**
• All injuries can be treated by Abdominal approach* and not via chest**
• Diaphragm repaired using 1’ size Polypropylene.

Aortic injury:
• Injury is seen distal to ligamentum arteriosum- tear or disruption happens
• False aneurysm can happen and blood can track along the false passage.
• Disproportionate BP between Upper limbs or between upper and lower limb is diagnostic .
• X ray chest: Widened mediastinum
• Unstable Chest IOC- Trans esophageal Echo
• Stable IOC- CT Scan
• Management- Keep BP less than 120 mmHg – By using Short acting Esmolol
• Endovascular Stenting or DACRON grafting for Aortic replacement.

Tracheo Bronchial injuries:


• Massive air leak will be seen
• Inflate the unaffected bronchus and shift to OT for Thoracotomy and repair*.

Topic 2d- Abdominal Injuries

Diagnostic Peritoneal Lavage:


• Insert a cannula below the umbilicus.
• About 1000 ml of warmed ringer lactate is instilled into the abdomen and is then
drained out.
• DPL has sensitivity about 97- 98% .
• DPL is especially useful in the hypotensive, unstable patient with multiple injuries as a
means of excluding intraabdominal bleeding.

SURGERY SIXER APP BASED WORK-BOOK 2020 65


Result: Positive
• > 1 lakh red cells/microlitre
• > 500 white cells/ micro litre
• Frank aspiration of > 10 ml on cannula insertion is also positive*
• >19 IU/L of Amylase
• Presence of Bile or Food particles

Disadvantages:
• Invasive
• Non-Repetitive
• So many false positive cases ( Non Therapeutic laparotomies happened)

Focussed Assessment with Sonography for Trauma ( FAST)


• Latest e-FAST is done at 6 regions, previously done by 4 FAST
• Done by Casualty doctors or Trauma surgeons trained for FAST.

Traditional four views


• Sub Xiphoid Transverse view – assess pericardial fluid
• Right Upper quadrant Longitudinal view – Collection in Morrison pouch or any liver/renal
injuries
• Left Upper quadrant Longitudinal view – Collection in perisplenic region or any splenic
injuries
• Suprapubic Longitudinal and transverse view – Collection in pelvis; assess bladder and
POD
• Added up now 2 extra areas- right and left thoracic views to rule out pneumothorax
(Stratosphere sign) or hemothorax.

Advantages of e-FAST:
• Repetitive
• Non Invasive
• Bedside investigation
• Short time study

Disadvantages:
• Detects Blood more than 100 ml only** ( Bailey value)
• Not detects bowel injury
• Not useful for penetrating injury ( NEET PG 2020)
• Unreliable for RP collections
• Operator Dependent.

IOC for Unstable patients- eFAST


IOC for Stable Patients- CECT abdomen

SURGERY SIXER APP BASED WORK-BOOK 2020 66


Sequence Order Question:
Blunt trauma Penetrating Trauma Gun shot Seat belt injuries
1. Spleen 1. Liver 1. Small bowel 1. MC is mesenteric
2. Liver 2. Stomach 2. Colon tear
3. Small Intestine 3. Small bowel 2. in GIT- DJ
4. Diaphragm flexure in injured
5. Colon
• Urban Bomb Blast- Ear drum > Lungs
• Underwater Bomb Blast- GIT ( MC in terminal Ileum )

Blunt Trauma Protocol:

Mechanism of injuries to each GI organ:


Stomach:
• MC mechanism- Penetrating trauma
• In Blunt Trauma- Greater curve at antrum is MC injured
• Penetrating Trauma- Don’t miss the posterior tear if present.
• Treatment- Surgery and repair of the rent

Small bowel Injury:


• MC mechanism- Blunt trauma> Penetrating Trauma
• Urgent Surgery is needed.

SURGERY SIXER APP BASED WORK-BOOK 2020 67


• Stable- Primary Repair
• Unstable cases- Clip and Drop technique**( Damage Control Surgery)- Proximal Bowel as
Stoma and keep the distal bowel in abdominal cavity itself by stapling.

Colon Injury:
• MC mechanism – Penetrating trauma> Blunt trauma
• Stable- Primary repair
• Unstable- Clip and Drop method

Rectal Injury:
• MC mechanism- Penetrating trauma > Blunt trauma
• Unstable and high contamination- Hartmann’s operation (Proximal colostomy and distal
closure)

Duodenal injury: ( Pancreatico Duodenal Injury)


• MC mechanism- Blunt trauma; MC site- Neck
• Penetrating Trauma: MC site= Equal in all sites
• IOC- CECT Abdomen
• DPL may show elevation of Amylase if duct disruption seen*
• Serum Amylase may be elevated in 50% cases.
• Stable Cases:
o Distal Pancreatic injury- Distal Pancreatectomy+ Splenectomy
o Duodenum normal with only Pancreatic head injury- Duodenum preserving
pancreatic head resection (BEGER Procedure)
o Combined Duodeno Pancreatic injuries- Trauma Whipple’s ( Pancreas head and
duodenum removed and no reconstruction done now- will be done later)
• Unstable cases of Duodeno Pancreatic injuries: Damage Control Surgery
• Triple Tube Ostomy is done – Decompressive Gastrostomy(A), Decompressive
duodenostomy (B) and Feeding Jejunostomy (C) is done after repair of 2 nd part
injury.

SURGERY SIXER APP BASED WORK-BOOK 2020 68


Solid organ injury-AAST Grading:

Management of Solid Organ injury:


Liver Injury
• Stable patients- 90% are managed by Non operative management ( NOM)
• Unstable Liver injury patient: 4P’s
o Push
o Plug
o Pack
o Pringle
• Pringle- Inflow occlusion manuever at Epiploic foramen- maximum done for 45 minutes.
If pringle done and bleeding stops means it is from PV or HA. If bleeding does not stop it
is from Hepatic vein**
• MC mechanism of Liver trauma- Penetrating Trauma > Blunt trauma**
• Even in penetrating trauma of Liver- Stable patients manage by Non operative
management.
• After 48 hours – while removing the pack- Hepatic artery can be ligated but Portal vein
needs to be repaired**

Spleen Injury:
• Stable patients- NOM
• Unstable patients- Grade 1,2,3- Splenorrhaphy
• Unstable Patients- Grade 4 and 5- Splenectomy
• NOM patients before discharge must undergo CECT abdomen to look for “SPLENIC
TUMOR BLUSH”- False aneurysm developed in Traumatic injury- Such cases need coil
embolization and this condition is known as failure of NOM**

SURGERY SIXER APP BASED WORK-BOOK 2020 69


Renal Injury:
• MC mechanism- Blunt trauma
• Omnious sign- Hematuria* ( hematuria is absent in 40% cases)
• CECT abdomen with oral contrast/ IV contrast- IOC
• 90% patients are managed by NOM.
• 10% patients need surgery.
• IVP is advised in NOM patients- to look for enhancement of both kidney.
• If one kidney is not visualised in an IVP- Treatment is exploration of renal hilum as there
can be hematoma in the renal hilar area compressing renal artery or renal artery injury.
If kidney is viable- Repair the renal artery.

Grades of renal laceration ( AIIMS November 2019- Sequence order question)


• Grade 1- Subcapsular Non Expanding Perirenal hematoma
• Grade 2- Laceration <1cm, Non Expanding Perirenal hematoma confined to
retroperitoneum and Gerota fascia
• Grade 3- Laceration > 1cm, No urine extravasation
• Grade 4- Laceration with Urine Extravasation, Injury to Main renal artery and Vein

Grade V:
• Shattered kidney.
• Avulsion of renal hilum: Devascularisation of a kidney due to hilar injury.
• Uretero pelvic avulsions**
• Complete laceration or thrombus of the main renal artery or vein.

Protocol for Penetrating injury:

SURGERY SIXER APP BASED WORK-BOOK 2020 70


Mesentric tear:

More dangerous is Transverse tear as this can result in small bowel gangrene and short bowel
syndrome.

Retroperitoneal hematoma:
In retroperitoneum 4 zones are explained and based on the hematoma in each zone management
differs.
ZONES CONTENTS MANAGEMENT
ZONE I Central vascular structures such as aorta Needs exploration
and IVC
ZONE II Kidneys and adrenal glands Observed
ZONE III Retroperitoneum associated with pelvic External pelvic compression and
vasculature fixation
ZONE IV Retro hepatic IVC and Hematoma behind Observation
portal Vein

Figure: Zones of retroperitoneal hematoma


• Mattox Manuever- Left side medial visceral rotation to reach aorta
• Cattel- Braasch Maneuver- Right side medial visceral rotation to reach IVC

SURGERY SIXER APP BASED WORK-BOOK 2020 71


Fullen zone classification of SMA injuries
• Zone I – located posterior to pancreas – exposed by Mattox maneuver
• Zone II – from pancreatic edge to middle colic branch, approached via lesser sac
• Zone III & IV – distal SMA injuries – approached directly within mesentery

Topic 2e- BURNS ( AIS-9)

MC cause of death in burns:


• At spot (Immediate death)- Asphyxia > Neurogenic shock
• 1-3 days (Early death)- Hypovolemic Shock
• Late death: Septic Shock
• MC cause of death in burns overall- Septic shock

If percentage of burns is more than 15% (Adults), 10%(Children) can develop hypovolemic shock.

Admission criteria for burns Patients:


• >20% partial thickness in > 10 years
• > 10% Partial thickness in < 10 years
• >5% Full thickness
• Eyes, face, genitalia, fingers burns
• Chemical and electrical burns admit
• Psychiatric/ No body to take care the patient

SURGERY SIXER APP BASED WORK-BOOK 2020 72


Pathophysiology of Burns:

• Zone of coagulation- Irreversible damage


• Zone of stasis- Can progress or regress
• Zone of Hyperemia- reversible zone ** - Vessels are vasodilated here.

Percentage of burns calculation:


Rule of nine ( Alexander Wallace rule)
• Each upper limb- 9% TBSA
• Each lower leg- 18% TBSA
• Anterior or Posterior Trunk- 18% TBSA
• Head and neck- 9% TBSA
• Perineum – 1% TBSA

Palm rule- Patient’s palm= 1%

Lund and Browder chart:

• Most accurate method for calculating the Burns percentage**


• Best method to calculate burns in children.

Berkow formula:
• Tabular column used to calculate the Burns Percentage.

SURGERY SIXER APP BASED WORK-BOOK 2020 73


• Just remember 0-1 year: head=19% and neck=2% from that formula
• Infants – head and neck percentage of burns= 21% as per this formula

Depth of burns
First degree of burns: Superficial burns:
• Erythema+ pain
• Heals without scar in 7 days

2 Degree Superficial (Superficial partial thickness


burns)
• Characterised by Blisters.
• Superficial touch ( Cotton)- Sensation+
• Pin Prick – Sensation+
• Heals without scar in 3 weeks

2 Degree deep ( Deep partial thickness burns)


• Characterised by Blisters
• Superficial touch ( Cotton)- Sensation
Absent**
• Pin Prick – Sensation+
• Heals with scar in 5 weeks

3 degree (Full thickness burns)


• Eschar formation seen
• Painless burns.
• Superficial touch ( Cotton)- Sensation
Absent**
• Pin Prick – Sensation absent**
• Can result in contracture and
compartment syndrome

4th Degree burns


• Involving deep muscles and tendons**
• Most of them die due to cardiac arrest.

Management:
• IVF of choice- Ringer lactate*
• Amount of fluid to be infused is calculated by Parkland formula:
• TBSA% × weight (kg) × 4 = volume (ml) needed
• Half this volume is given in the first 8 hours, and the second half is given in the
subsequent 16 hours

SURGERY SIXER APP BASED WORK-BOOK 2020 74


• This formula is calculated from time of BURNS** ( Not from the admission)
• < 24 hours we must give crystalloids only and not colloids as they may have leaking
capillaries , which will lead to subcutaneous edema
• > 24 hours Colloids like Dextran, hemacel, albumin are given based on “ MUIR and
BARCLAY Formula”
• Paediatric fluid formula: ( Both crystalloid and colloid)
- Galveston
- Brooke

• Curreri formula- For giving Nutrition to burns patients

Management of Burns Wound:


1st and 2nd Degree Superficial burns:
• Topical ointments
• MC used- Silver sulphadiazine ( Cannot penetrate deeper tissues like Eschar)
• Silver nitrate ( Black staining may happen)
• Mafenide Acetate- Can penetrate deeper into eschar( Can cause Metabolic acidosis**)
• Best of all agents- Cerium Bromide ( Immunomodulator)- Costly
• Collagen sheets are available in 2nd degree superficial burns

2nd Degree Deep burns:


• SSG ( Thiersch graft is used)- Taken by Humby knife.

3rd Degree Burns:


• SSG
• Escharotomy is done to avoid compartment syndrome

Topic 2f- Miscellaneous topics

Compartment syndrome Leg:


• MC after closed limb injuries
• There are 4 compartments in leg*
• Cause: Arterial hemorrhage, venous ligation or thrombosis, crush injuries and reperfusion
injury
• Prominent early symptom – Pain and pain aggravated by passive stretching of muscles
• Hallmark of early compartment syndrome – numbness between 1st and 2nd toes**
• Absent of distal pulse is a late sign.

Indications of Fasciotomy in leg:


• In lower limbs – two incisions needed to release four compartments :Indications are
o >30 mmHg (or)

SURGERY SIXER APP BASED WORK-BOOK 2020 75


o When clinical signs are present
o Ischemia > 6 hours
o Gradient < 35mm Hg (Gradient = Diastolic pressure – Compartment pressure)
from Bailey and Love

• Lateral incision decompresses- Anterior and lateral compartments*


• Medial incision decompresses- Superficial and deep posterior compartments*

Abdominal Compartment syndrome:


• Pathology is there in abdomen causing ACS is Primary
• Due to resuscitation if the fluid collects and ACS happens is Secondary

Values:
• Normal IAP= 5-8 mmHg
• IAH is if the pressure > 12 mmHg ( at a time gap of 4-6 hours for 3 times)
• ACS is if the pressure > 20 mmHg ( at a time gap of 1-3 hours for 3 times)

Clinical features of ACS:


• Renal blood flow compromise- Decrease urine output
• Decreased venous return- Decreased Cardiac output and decreased Stroke volume
• Diaphragm compression- Increased CVP and decreased Respiratory efforts
• Increased Intracranial pressure can happen.

Measurement of Intra abdominal pressure:


IAP is indirectly measured using three way Foley Catheter of Intra Bladder Pressure: ( Gold
Standard method)
o GI (IAP <10 to 15 cm H2O)
o GII (IAP <16 to 25 cm H2O)
o GIII (IAP <26 to 35 cm H2O)
o GIV (IAP >36 cm H2O).
Other methods:
• Gastric mucosal pH
• Near Infrared Spectroscopy
• CECT scan

Management:
• Grade 3- Suggest Laparostomy
• Grade 4- Definite Laparostomy
• Laparostomy- Leave the abdominal wall open and leave it ( no procedure inside)
• The bowel contents will protrude out and ACS resolves.
• Measure serial IAP after laparostomy also

SURGERY SIXER APP BASED WORK-BOOK 2020 76


Covering of laparostomy:
- Bagotta bag ( Plastic sheet)
- Opsite dressing
- VAC devices
Disaster Surgery:
• Bomb blast- MC organ injured Ear drum> lungs ; MC in underwater blast is GIT (
Terminal ileum)
• Volcanoes- MC cause of death -Suffocation
• Earth Quakes-MC cause of death- Crush Injury
• Tsunami- MC cause of death- Drowning

Topic 2g- Recent advances in Trauma


ATLS 10th edition Protocol

Topic Old Protocols ATLS- 10th Edition


1. Fluid infusion By two green ( 18 Gauge) By two green Venflon on both
Venflon on both upper limb wrist upper limb in wrist or Cubital
or cubital veins. veins.

2 Litres of crystalloid infused 1 Litre of Crystalloid


7.5% hypertonic saline*
Children= 20 ml/ kg/ BW
2. Massive Blood 10 Units of Whole blood replaced. 10 units of Packed Cells: 10 units
Transfusion of Plasma: 10 units of Platelets
(Ratio 1:1:1 is transfused to avoid
Coagulopathy)

In paediatrics: 10-20 ml/Kg in


1:1:1 ratio
3. Intubation Intubation by Conventional Video Laryngoscopy intubation
method advised in casualty.
4. Four Classes of Does not take into account Base Takes into account Base deficit
Haemorrhagic Shock Deficit* also:
• Class 1: 0 to -2
• Class 2: -2 to -6
• Class 3: -6 to -10
• Class 4: -10 onwards

5. Tranexamic acid Doubtfully used Crash 2 Trial:


usage for Bleeding • Trauma patients with
Trauma Patients PR>110/minute or

SURGERY SIXER APP BASED WORK-BOOK 2020 77


Systolic BP < 90mmHg
can get Injection
Tranexamic Acid: 1 gm in
10 minutes followed by
1gm in 8 hours.

6. Life threatening Immediate Life threatening: Of the above 6 immediate life


Injuries • Airway Obstruction threatening injuries,
• Tension pneumothorax Flail Chest is removed and they
• Open Pneumothorax added
• Pericardial Tamponade “ Tracheo Bronchial Injury”
• Massive Haemothorax
• Flail Chest
7. Tension Was a clinical diagnosis* e-FAST can be used to diagnose
Pneumothorax Tension Pneumothorax where we
Insert wide bore needle in 2nd ICS get- Barcode Sign or seashore
at Mid clavicular line for all. /Stratosphere Sign in M mode

In adults- Wide bore needle now


inserted in Mid axillary line at 5th
ICS.**
In children – same 2nd ICS at
MCL is followed.
8. Aortic Rupture No Betablocker used Short acting ESMOLOL with a
management goal to maintain
HR at < 80 beats/ minute and BP
60-70 mmHg is recommended.
9. Urethral Injury Per rectal examination done Don’t do PR for identifying
Examination by Per floating prostate hereafter.
rectal method ( No more Vermooten Sign in
( Vermooten sign- future)
Floating Prostate in
membranous urethral
injuries)
10. Post Trauma Sodium valproate or Phenytoin No benefit of using
Seizure prophylaxis was used. prophylactically these drugs. ATLS
never advised Seizure prophylaxis
hereafter.
11. Cervical Spine X Routinely done Not done routinely:
ray -Indications
Indicated based on Canadian C’
Spine Rule

SURGERY SIXER APP BASED WORK-BOOK 2020 78


Or NEXUS criteria*

NEXUS- National Emergency X-


ray Utilisation Study:

Indicated for following cases:


Mnemonic:
• N- Neuro Deficit
• E- Ethanol intoxication
• X- eXtreme mechanism of
injury
• U- Unable to give History
(LOC+)
• S- Spinal tenderness+

12. Burns Fluid Parkland Formula: Superficial Burns or Scalds:


Resuscitation Amount of Fluid= 4X wt. in kgX (1st Degree and 2nd Degree)
TBSA of Burns Above 14 years:
• Fluid input= 2ml Ringer
lactate X weight in Kg X
TBSA
• Maintain Urine output 0.5
ml/kg/hr

Below 14 years:
• Fluid Input= 3ml RLX
Weight in Kg X TBSA (
Less than 14 years)
• Maintain Urine output 1
ml/Kg/Hr

Electrical Injury ( all ages :


• Fluid input= 4 ml RL X
weight in Kg X TBSA..
• Maintain Urine Output 1-
1.5 ml/ Kg/Hr

SURGERY SIXER APP BASED WORK-BOOK 2020 79


Space for Extra Points:

SURGERY SIXER APP BASED WORK-BOOK 2020 80


Section : B: Head and Neck, Thyroid and Breast

Chapter 1: Head and Neck


• 1a- Oral cavity
• 1b- Salivary Gland
• 1c- Neck swellings

Chapter 2: Thyroid
• 2a- Introduction
• 2b- Benign diseases
• 2c- Malignant diseases
• 2d- Surgical aspects

Chapter 3: Breast
• 3a- Triple assessment
• 3b- Management of Cancer Breast
• 3c- Miscellaneous

Chapter 4:
• 4a- Anatomy of Hernia
• 4b- Types of Hernia
• 4c- Laparoscopic Anatomy and Hernia repair

SURGERY SIXER APP BASED WORK-BOOK 2020 81


Chapter 1. HEAD and NECK LESIONS

Chapter 1a. Oral cavity

Ulcers in oral cavity


Painful Ulcers Painless Ulcers

• Aphthous ulcer • Cancer – lateral margin of tongue


• Dental ulcer – lateral margin of • Gummatous ulcer – syphilis- anterior
tongue 2/3rd of tongue in midline
• TB – tip of tongue

Figure: Gummatous Ulcer

Syphilis oral lesions:


Primary Secondary Tertiary

• Chancre ( painless) • Hutchinson’s warts ( • Gumma


midline of tongue) • Glossitis (
• Snail tract ulcer premalignant)
• Mucous patches

Figure: snail track ulcers in Syphilis

SURGERY SIXER APP BASED WORK-BOOK 2020 82


Conditions associated with malignant transformation:
High risk Medium risk (3S) Low risk ( equivocal risk)
• Erythroplakia ( • oral Submucous • Discoid lupoid
speckled > fibrosis erythematosis
homogenous) • Syphilitic glossitis • Oral lichen planus
• Proliferative verrucous • Sideropenic dysphagia • Discoid keratosis
leukoplakia congenita
• Chronic hyperplastic
candidiasis

1. Leukoplakia

• White patch/ plaque

• M/c in smoking

• m/c premalignant lesion

• Specked variant – increased risk – 2.4% malignant

• management- stop smoking and laser

2. Erythroplakia

• 17 times malignant potential

• Highest risk

3. Chronic hyperplastic candidiasis

• Invasive

• Treat with oral antifungal drugs

4. Oral submucous fibrosis

• Betel nut produces – arecoline

• Causes submucous fibrosis

5. Sideropenic dysphagia (plummer Vinson or Paterson Kelly syndrome)

• Females

• 20-40 yrs

• Koilonychias + iron deficiency anemia

• D/t iron deficiency – they have epithelial atrophy – which is premalignant

• Iron supplement – heals epithelial atrophy

Etiological factors
• Smoking – m/c – 1.5-2% risk

• Tobacco

• Arecoline

• Pan masala

• Alcohol

SURGERY SIXER APP BASED WORK-BOOK 2020 83


• Dietary- deficiency in antioxidant ( Vit A,C,E)

• Environmental – UV light and sunlight ( for Ca lip)

• Immunocompromised patients

• Infections- HIV 1,2, HPV 16,18 and Syphilis

• Genetic mutation

o P53- m/c

o Plummer Vinson

o Fanconi anemia

o Li Fraumeni syndrome

Pathology
• 90% SCC
• Field cancerisation : entire oral cavity is exposed to carcinogen- therefore we can get
2nd Ca anywhere
o Synchronous - < 6 months
o Metachronous - > 6months ( Most common – 80%)
o 15% chances of 2nd cancer

Lymph node metastasis

o Systematic way – level I – II- then III


o Skip mets is seen in tongue Ca ( poorest prognosis)

Oral cancers- M/C is SCC


• Lips – M/C western – best prognosis
• Buccal mucosa – M/C India
• Tongue – M/C world – worst prognosis( skip mets)
• Alveolus
• Retromolar trigone

B/L lymph node enlargement is seen in CA

• Lower lip
• Soft palate
• Supra glottis

SURGERY SIXER APP BASED WORK-BOOK 2020 84


TNM staging – AJCC 8th edition
T1- < 2cm or DOI ( depth of invasion) <5mm N1 - <3cm ( single Node)
T2- 2-4 cm or DOI 5-10 mm N2 – 3-6 cm
T3- >4 cm or DOI >10 mm • Single
T4 • Multiple ipsilateral
a. Involvement of skin of face, inferior • Multiple bilateral
alveolar nerve, floor of mouth N3
b. Pterygoid plates, carotid sheath, • >6 cm size
masticator space, internal carotid • Extranodal extension +
artery M1 – distant mets

Stage grouping:
I- T1N0

II- T2N0

III- T3NO; any T N1 ( node starts in III)

IV-

a. T4a; any T N2

b. T4b; any T N3

c. Mets +

• Modifications in 8th edition AJCC:

1. DOI is added to T staging

2. Extra nodal extension is N3b

3. Extrinsic muscle invasion is removed from T4

Investigations
• Oral ulcer – to be evaluated if following are seen:

✓ > 3 weeks

✓ > 3 weeks swelling in oral cavity/ neck

✓ Swallowing difficulty

✓ Trismus +

✓ Sore tongue

• Wedge edge biopsy- with base towards lesion ( 2% toludene blue to enhace lesion)

• To do T& N staging –

❖ MRI of oral cavity ( short Tau Inversion Recovery) ( STIR – MRI)- NEET

SS question

SURGERY SIXER APP BASED WORK-BOOK 2020 85


❖ Gadolinium – suppression techniques and variation in image sequence

acquisition is noted

• M/C distant mets : Lungs – CT thorax before surgery

• PET – CT- used only for non-surgical patients – to plan where all we have to give

radiotherapy

• If neck node seen- USG guided FNAC

• CT facial Bones – to look for bony invasion

Tongue cancer:
• m/c presentation- painless ulcer

o Pain – if infiltrating nerve

• Trismus- infiltration of pterygoid plates

• Bony invasion( mandible) – bony pain

Investigation of choice to look for mandible or bone invasion : CT scan Mandible/ Facial bones
• T1- <2cm – 1cm margin excision and approximate cut edge

• T2 – 2-4cm – hemiglossectomy + reconstruction

• T3 - >4cm – hemiglossectomy + reconstruction

• T4 – cross midline – total glossectomy

Reconstruction of tongue
1. RFFF ( Radial Forearm Free Flap)

• M/c used

SURGERY SIXER APP BASED WORK-BOOK 2020 86


• Based on radial artery

2. PMMC flap ( Pectoralis Major Myocutaneous flap)

• Based on thoraco acromian artery

• Can’t do in females – can lead to Ca breast later

3. Delto pectoral flap ( DP flap)

• Can be used in females

• Based on internal mammary artery

Figure: Radial Forearm free Flap


• CT scan

o Abutting mandible – marginal mandibulectomy ( inner table of mandible is removed)

o Infiltrating mandible – segmental mandibulectomy and reconstruct with radius bone.

Cancer Lip
• 90 % are lower lip

o MC type is SCC

o Lower lip cancer – B/L lymph nodes+

• Upper lip

o MC is BCC ( along tear flow area)

o SCC

o Verrucous Ca

• Management:

o Remove tumor with 1cm margin

▪ If Defect <1/3rd gap – close it by primarily

▪ If Defect >1/3rd gap- reconstruction with flaps

• JOHANSSEN STEP LADDER FLAP

• KARPANDAZIC FLAP- Huge defects

• BERNARD FLAP

• ABBES FLAP- Centre of lip

• ESTLANDER FLAP – Angle of lip

SURGERY SIXER APP BASED WORK-BOOK 2020 87


CA BUCCAL MUCOSA
• Wide local excision with 1cm margin 3 dimensionally

• With reconstruction using flap

o PMMC in males

o DP flap in females

CA Retromolar Trigone / FLOOR OF MOUTH


• Split mandible approach – VISOR approach

• All cancers are SCC

o Surgical management

▪ Preferred

▪ 1cm margin 3 dimensionally

▪ Reconstruction

o Interstitial radiotherapy/ Brachytherapy

▪ Contraindicated in alveolar cancer ( since it can lead to osteo radio necrosis)

o Either of them can be used to treat

NECK NODES: Levels and Dissections

7 levels of Neck nodes:


1. Submental and Submandibular nodes

2. Upper jugular Extend from base of skull to hyoid bone

3. Middle Jugular Extend from hyoid bone to cricoid cartilage

4. Lower Jugular Extend from cricoid cartilage to suprasternal


notch

5. Supraclavicular or Posterior Group

6. Pre laryngeal/ Pre tracheal Extend from Hyoid bone to sternal notch

( Schwartz says- prelaryngeal as Delphian


nodes)

7. Upper mediastinal nodes

SURGERY SIXER APP BASED WORK-BOOK 2020 88


Figure: Neck Nodes

If nodes are positive – neck dissection operation is required.


Radical Neck Dissection (RND) Modified Radical Neck COMMANDO
Dissection (mRND)
CRILE’S OPERATION BOCCA’S OPERATION
Removed : Removed : • Com- combined
• Level I – V • Level I – V • Man – mandible
• Fat • Fat • Nd – Neck dissection
• Fascia • Fascia • O - operation
• Strap muscles • Strap muscles
SIS - removed All SIS preserved- functional Indication:
• S- Sternocleidomastoid neck dissection Oral cavity cancer
• I – Internal jugular infiltrating mandible also
vein
• S- Spinal accessory
nerve
Removed: Removed: Remove:
• Submandibular gland • Submandibular gland • Cancer + mandible +
• Part of parotid gland • Part of parotid gland nodes
NOT REMOVED:
• Carotid artery
• Vagus nerve
• Lingual nerve
• Hypoglossal nerve
• Phrenic nerve

SURGERY SIXER APP BASED WORK-BOOK 2020 89


• Cervical branch of
facial nerve
NOTE:
• while performing B/L
RND – preserve at
least one IJV.

Supra omohyoid Neck dissection:


• Removal of level I, II, III + submandibular gland = Supra omohyoid neck dissection

• Indicated in N0 cases and selected N1 cases

Complications:
• Carotid blow out- loss of blood vessel supplying tunica adventitia

- Prevented by PMMC/ DP flap

Named incisions in oral cavity::

o Weber ferguson incision


▪ Maxillary CA
▪ Hard palate Ca
o Shobinger incision – FOR RND
o Mc fee incision – FOR RND

BENIGN LESIONS
1. Submandibular space infection – LUDWIG’S ANGINA

• Group A streptococci

• c/f-

SURGERY SIXER APP BASED WORK-BOOK 2020 90


i. pain , swelling, tongue pushed backward

ii. odynophagia

iii. trismus

iv. airway obstruction

• spreads to parapharyngeal space

• treatment – broad spectrum antibiotics and I&D

2. EPULIS

• Benign tumor of gums

3. Odontogenic cyst

• Dental cyst

• Dentigerous cyst
Dental cyst Dentigerous cyst
Arises from erupted tooth Unerupted tooth
d/t infected root
Inflammatory etiology Developmental etiology
a/k/a radicular cyst** a/k/a Follicular odontoma**
Middle age 2/3rd decade
X- ray- SOAP BUBBLE APPEARANCE
• Both are Lined by stratified squamous non keratinized epithelium
• Both are Painless
• Both m/c in upper jaw
• REMOVE BOTH

• Odontogenic keratocyst/ keratocystic odontogenic tumor

o Only cyst which enlarges without causing bony expansion**

o D/t enzymatic erosion

o Grow anterio- posteriorly

Figure: Dentigerous Cyst

SURGERY SIXER APP BASED WORK-BOOK 2020 91


Chapter 1B. SALIVARY GLANDS

PAROTID GLAND
o Superficial lobe and deep lobe

o Separated by Patey’s facio venous plane= retromandibular vein+ facial nerve

o Drains via Stenson’s duct

▪ 5cm in length

▪ Opens opposite to upper 2nd molar

▪ Koplik’s spots in measles are also found here

Facial nerve trunk identification:


Anterograde method
• Conley’s pointer ( tragal pointer)**

o Junction of bony and cartilaginous EAC – points downward 1cm inferior

• Posterior belly of digastic insertion ( above this)

• Stylomastoid foramen

• Facial nerve monitors

Retrograde method:
• Tracked backwards ( eg. From buccal branch – trace backwards towards trunk)

Nerve supply of parotid:


1. Parasympathetic

• Salivary nucleus – via glossopharyngeal nerve ( crossing Tympanic membrane) –

relay in Otic ganglion – post ganglionic fibres Via auriculotemporal nerve ( branch

of trigeminal nerve) supply parotid

• Leads to increased salivary secretion

• Serous discharge

2. Sympathetic supply

• Increase blood flow to gland

SURGERY SIXER APP BASED WORK-BOOK 2020 92


Frey’s syndrome (Gustatory sweating syndrome)
• Injury to Auriculotemporal nerve ( post ganglionic parasympathetic nerve injury)

• Sympathetic fibres overgrow, join with auriculotemporal nerve and reach skin

• c/f- pain, erythema and sweating over face on just seeing food

• Presents – 2-3 months after parotidectomy operation

• Prevented by:

o SCM flap

o Temporalis muscle flap

o Membrane

• IOC: starch iodine test

• Treatment – latest Botox injection at areas of reinnervation appearing as Erythema on

Starch Iodine test

o Old – cut the glossopharyngeal nerve at Tympanic membrane – JACOBSON’S

NEURECTOMY

SURGERY SIXER APP BASED WORK-BOOK 2020 93


Figure: Starch Iodine test

Submandibular gland
• 2 lobes separated by mylohyoid muscle

• Wharton’s duct

o 5cm

o Opens under the surface of tongue on either side of frenulum

o Lingual nerve and hypoglossal nerve are closely related to duct

• Mucinous secretion

• Flow of saliva is non-dependent

• Rich in amylase and lipase

• m/c gland to develop stones- radio opaque on xray

• Management of stones:

o Old concept:

✓ stone distal to lingual nerve- lay open duct and remove stone

✓ stone proximal- submandibular gland excision

SURGERY SIXER APP BASED WORK-BOOK 2020 94


Extra Mile Points:

• 800 minor salivary glands


• Sublingual salivary gland
o Ducts of Rivinus
o Ducts of Bartholin
Most common site for ectopic salivary gland- angle of mandible
o Called as STAFNE BONE CYST (radioluscent bony expansion)
Most common ectopic salivary gland tumor- hard palate

ECTOPICS:

❖ Salivary : angle of mandible ( stafne bone cyst)


❖ Sebaceous gland : lips ( Fordyce)
❖ Stomach : meckel’s diverticulum
❖ Pancreas : duodenum > stomach
❖ Spleen : splenic hilum
❖ Thyroid : lingual ( foramen caecum)
❖ Testis : superficial inguinal puch ( d/t inguinal tail domination)
❖ Pregnancy : ampulla of fallopian tube
o If happens in ovary – SPIGELBERG CRITERIA
o If happens in cervix – RUBIN’S CRITERIA

SURGERY SIXER APP BASED WORK-BOOK 2020 95


TUMORS IN SALIVARY GLAND:

BENIGN MALIGNANT

PAROTID 80% 20%

SUBMANDIBULAR 50% 50%

MINOR GLANDS 10% 90%

As the gland becomes smaller malignancy increases.

Types of epithelial tumors:


Benign :
❖ Pleomorphic adenoma

❖ Warthin’s tumor

Malignant:
❖ Mucoepidermoid

❖ Adenoid cystic

❖ Acinic cell tumor

❖ Pleomorphic adenocarcinoma

❖ Adeno cancer

❖ Squamous cell cancer

Most commons:
Adults:
• m/c benign tumor- in parotid and submandibular- PLEOMORPHIC ADENOMA

• m/c malignant tumor

o Parotid- mucoepidermoid cancer

o Submandibular – adenoid cystic cancer

Children: -35 % are malignant


• m/c tumor: hemangioma

• m/c benign epithelial tumor- pleomorphic adenoma

• m/c malignant tumor – mucoepidermoid cancer

SURGERY SIXER APP BASED WORK-BOOK 2020 96


Explanation of benign tumors:
Pleomorphic adenoma Warthin’s tumor

a/k/a mixed tumor a/k/a adenolymphoma

(has epithelial and mesenchymal tissue) (congenital pre parotid lymphatics give rise to
this tumor)

m/c benign tumor of parotid, Submandibular Exclusive for Parotid gland**


gland and minor salivary glands

Seen in tail of parotid Lower pole of parotid**

3 classical features

• Lift the ear lobule -


• Curtain sign positive
• Obliteration of retromandibular groove

Bi digitally palpable+ -

Tonsil pushed medially (if deep lobe is -


enlarged)

Pathology:

• Rarely capsule+ - not well defined 1. well defined capsule


capsule
• Pseudopod extension into normal gland

Only Parotidectomy is TOC Enucleation or

parotidectomy

C/f :

All salivary gland tumors are most common in Old male ( 5th – 7th decade)
females.

High yield Points in Warthin’s:

• Multi centric and multifocal


• 10% bilateral
• h/o smoking – risk factor
• Tc99m scan – hot spot

SURGERY SIXER APP BASED WORK-BOOK 2020 97


GODWIN’S TUMOR- benign lymphoepithelioma of parotid

Figure: Huge Pleomorphic adenoma

Investigations:
• FNAC- IOC to confirm diagnosis

• Avoid incision biopsy- can lead to seeding of tumor

• CT scan – in deep lobe tumor

• Tc99m scan- warthin’s and oncocytoma

• MRI – best investigation for salivary gland tumor.

FEATURES SUGGESTIVE OF MALIGNANCY IN PAROTID SWELLING:


• Facial nerve paralysis

• Sudden enlargement of gland

• Severe pain ( usually painless otherwise)

• Satellite nodule/ ulcer in skin

• Cervical node palpable

Mucoepidermoid cancer Adenoid cystic cancer a/k/a cylindroma

High or low grade variants High or low grade variants

m/c malignant tumor of parotid m/c malignant tumor of submandibular gland

Perineural infiltration +

Base of skull extension

30% LN enlargement No LN mets

( Increased LN mets)

SURGERY SIXER APP BASED WORK-BOOK 2020 98


Acinic cell tumor
o Only low grade tumor

o 3% B/L

o 2nd most common tumor in children

SURGICAL ASPECTS:
❖ LAZY S INCISION/ MODIFIED BLAIR INCISION/ SISTRUNK INCISION

1. Simple parotidectomy

• Only Superficial lobe removed

• Deep lobe and Facial nerve preserved

• Done in benign tumors

2. Total conservative parotidectomy

• Superficial + deep lobe removed

• Facial nerve is preserved

• Done in deep lobe benign tumors

3. Radical parotidectomy:

• superficial + deep lobe + facial nerve removed

• done in malignant

o mucoepidermoid

o adenoid cystic tumor

o During surgery

o Use only bipolar cautery ( monopolar causes lateral spread)

o If nerve injured ( especially buccal branch) – reconstruct with greater auricular

nerve

o Buccal branch is very important and must be reconstructed if injured.

Post op complications:
1. Nerve injuries

• m/c nerve injured- any branch of facial nerve

✓ Bell’s phenomenon

✓ Deviation of mouth

✓ Loss of nasolabial fold

• m/c cutaneous nerve injured- greater auricular nerve**

✓ Paraesthesia in shaving area

• Auriculotemporal nerve( post ganglionic parasympathetic injury) – Frey’s

syndrome

SURGERY SIXER APP BASED WORK-BOOK 2020 99


2. Flap necrosis

3. Infection

4. keloid formation

5. Sialocele

6. Parotid fistula

• Stenson’s duct not identified property and tied

• Require –SEABROOKE OPERATION

SUBMANDIBULAR STONES
• Thick mucinous secretion

• Non-dependent secretion

• Mostly radio opaque stones

• Causes for obstruction of Submandibular duct**

o m/c stones

o 2nd - strictures

• If stone causes complete obstruction – MEAL TIME SYNDROME**

o Rapid swelling in submandibular region immediately

o Pain + ( d/t pressure)

o Pain disappear in 1-2 hours gradually

• If stone causes incomplete obstruction- only mild discomfort

• Treatment: new concept

o Stone <4mm and mobile – remove by scopy with DORMIA BASKET

o Stone >4cmm : INTRA DUCTAL LITHOTRIPTER – break stone and remove

Parotid Duct Stones:


• Parotid gland stones are rare

• If present they are radiolucent

• M/c site: duct crossing masseter

• Treatment: sialography ( since radiolucent on xray)

o <4mm- endoscopy

o >4mm – remove gland itself

RANULA

• Mucous extravasation cyst from sublingual salivary gland


• Develop form floor of mouth ( remember it’s not retention cyst)
• Surgery: excision of cyst and affected gland ( not marsupilisation)

SURGERY SIXER APP BASED WORK-BOOK 2020 100


Figure: Ranula

Plunging ranula:

❖ Retention cyst of submandibular and sublingual gland


❖ Presents as swelling in oral cavity and submandibular region
❖ Bidigitally palpable
❖ Fluctuation +
❖ Cross fluctuation +
❖ Treatment : intra oral resection of plunging ranula
▪ ( old concept: incision over neck )

Chapter 1C. NECK SWELLINGS

TRIANGLES OF NECK
Anterior Triangles Posterior Triangles

• Carotid 1. Sub occipital


o Posterior belly of digastric a. SCM
o Superior belly of omohyoid b. Inferior belly of omohyoid
o SCM c. Anterior border of trapezius
• Submental 2. Supraclavicular
o Between the 2 anterior belly a. SCM
of digastric b. Inferior belly of omohyoid
o Hyoid bone c. clavicle
• Muscular
o Hyoid bone
o Superior belly of omohyoid
o SCM
o Imaginary midline
• Submandibular

SURGERY SIXER APP BASED WORK-BOOK 2020 101


o Anterior belly of digastric
o Posterior belly of digastric
o mandible

Most common swelling in neck – lymph node enlargement**

Figure: Triangles of Neck

Carotid triangle Posterior triangle


Solid type Carotid body tumor Cervical rib
Liquid type Branchial cyst Cystic hygroma
Air Filled swelling Laryngocele Pharyngeal puch ( zenker’s
diverticulum)

Carotid body tumor

• Carotid body : chemo receptor – located posterior medial surface of bifurcation of

common carotid

• Hypoxia stimulates carotid body – results in tachypnes

• Persistent hypoxia – carotid body tumor

o High altitude

o Cyanotic congenital heart disease

• Other names-Chemodectoma, Potato tumor, Non chromaffin paraganglioma

• Clinical Features:

o m/c middle age female

SURGERY SIXER APP BASED WORK-BOOK 2020 102


o U/L

o Firm +

o Movement : “ fontaine sign” – only horizontal movement + , no vertical

movement

o Swelling in carotid triangle

o Transmitted pulsation- keep 2 fingers- fingers get lifted but not separated

o Incidence – 0.5%

o Malignancy in 5-10%

o Shamblin classification:

▪ Type I - <25% adherent

▪ Type II – 50% adherent

▪ Type III- encasing the vessel

• IOC to diagnose :

o Angiography

▪ Vessels are splayed out (LYRE SIGN)

o FNAC AND BIOPSY ARE CONTRAINDICATED – Bleeds torrentially

o Duplex scan

• For type I and II- Excise the tumor with vessel control

• For type III- Require resection and prosthesis

Complications of Surgery:
• m/c nerve injured: superior laryngeal nerve

• FIRST BITE SYNDROME:

✓ Sympathetic nerves are damaged

✓ Leads to parasympathetic reinnervation

✓ As we start chewing – pain in neck +

✓ Opposite to frey’s syndrome

• Preop radio therapy should not be given

• Post op RT can be given in malignant tumor, disease recurrence, incomplete resection

CERVICAL RIB and TOS

Types:
• Partial rib

• Fibrous cord

• Full rib

• Partial rib with fibrous extension

SURGERY SIXER APP BASED WORK-BOOK 2020 103


Clinical features
• Bilateral in 50 % cases

• Young to middle age females ( 70%)

• Scalene triangle:

o Between scalenus anterior and medius and 1st rib

o Contains brachial plexus (upper , middle and lower trunk ) and subclavian artery

o Subclavian vein in not in the triangle

• Cervical rib occupies scalene triangle- compression of structures leads to thoracic outlet

syndrome

Figure: Scalene Triangle and TOS

Thoracic outlet syndrome


Neurogenic Vascular

C8T1 (Lower trunk) Subclavian artery compression

Ulnar nerve

M/C in Cervical rib

Pain along medial side of arm + forearm Vascular- claudication

(as you start working – blood supply decrease


– pain develop – makes you stop work)

SURGERY SIXER APP BASED WORK-BOOK 2020 104


Ulcer in hand

Gangrene in finger

Tinel’s sign: Subclavian artery steal syndrome-

Tap on brachial plexus – shooting pain + Collateral between vertebral artery and
subclavian artery

Leads to giddiness on starting to use hand

Figure: Cervical rib with TOS

Tests for TOS:


These tests bring out occult cases to be diagnosed by manuevers

Adson Test Roos test** Halstead test

• Extend arm • Abduct arm • standing in military


• Look opposite side • Flex elbow position cause pain
• Take a deep breath • Open and close fist for 5
• Radial artery min
dynamicity decreases ( • Claudication pain develops
not pulse rate) • Patient puts hand down

Investigations:
• Xray – cervical rib

• CT thorax and neck- surrounding vessel compression

• Angiogram – to look for SCA compression

SURGERY SIXER APP BASED WORK-BOOK 2020 105


NOTE:
• Post stenotic aneurysm of subclavian artery may develop.

• Cervical rib is palpated in posterior triangle ( just above the clavicle)

OTHER CAUSES OF TOS:


CONGENITAL OSSEUS:
o Cervical rib (m/c)
o Long C7 transverse process
o Abnormal 1st rib
Soft tissue:
o Anomalous scalene insertion
o Scalene muscle hypertrophy
o Congenital bands and ligaments
Postural:
o Heavy breast
o Sagging shoulder
OSSEUS:
ACQUIRED o Fracture clavicle/ 1st rib
o Exostosis/ tumor
Soft tissue:
o Scalene muscle injury
o Previous surgery scar
o Soft tissue tumor
o Direct brachial plexus injury

Branchial cyst and Branchial fistula( Sinus)

• 6 pharyngeal arches – 5th degenerate on both sides

• 2nd arch fuses with 6th arch encompassing 2nd pouch between them

• Persistence of pouch between 2nd and 6th is known as branchial cyst

• Failure to fuse – cleft – brachial sinus

• External opening – lower 1/3rd of SCM anteriorly

• Presents at 15-25 years ( though congenital)

• 1st arch remnant – preauricular sinus

• 3rd arch remnant- clavicular / suprasternal sinus

• 2nd arch remnant – most common

SURGERY SIXER APP BASED WORK-BOOK 2020 106


Figure: Branchial Arches

BRANCHIAL CYST BRANCHIAL SINUS/ FISTULA


Upper 1/3rd of SCM Lower 1/3rd of SCM
Swelling + Opening seen
Transillumination negative -
Thick content)
(contain cholesterol crystals- also seen in
Hydrocele)
Lined by squamous epithelium** Ciliated columnar epithelium**
Persistence of 2nd pouch Congenital branchial sinus / fistula
• Inner opening- in tonsillar fossa
• Anterior to the posterior pillar**
Complication: Fistula passes through
• Acquired branchial sinus • Bifurcation of common carotid
• Branchiogenic carcinoma- Can have • Pierces superior constrictor of pharynx
squamous or columnar epithelium • Tonsillar fossa
Treatment: Multiple stepladder incision – follow fistula
Complete excision of cyst and remove it completely
Acquired branchial cyst:
• Complication of infected branchial cyst/ surgeon thinks it as abscess and does I&D

• But opening is at upper 1/3rd

IOC: branchial fistulogram


TOC: complete excision of cyst and sinus.

SURGERY SIXER APP BASED WORK-BOOK 2020 107


CYSTIC HYGROMA – on posterior triangle of neck

❖ M/c in new born

❖ Associated turner’s syndrome, trisomy 21, 18

❖ Occurs d/t congenital sequestration of lymphatics

❖ C/f- swelling in posterior triangle

o Transillumination +

❖ Earliest neck swelling in humans

o As early as manifesting as obstructed labor

❖ Complications:

o Respiratory difficulty

o Infection

o Spontaneous resolution is also seen

❖ Management:

o Conventional: conservative neck dissection ( unnecessary)

o Latest :

▪ Injection sclerosant – PICIBANIL (OK 432) or INJ. BLEOMYCIN

▪ No CHEMO / RADIO THERAPY REQUIRED.

Figure: Cystic Hygroma


LARYNGOCELE
❖ CAUSES:

o Weakness in thyrohyoid membrane ( Bilateral)

o And laryngeal mucosa comes out of it

❖ Seen in trumpet players/ nathaswaram players

❖ Laryngocele can be internal / external

❖ c/f:

o Presents as swelling in carotid triangle

o Resonant on percussion

SURGERY SIXER APP BASED WORK-BOOK 2020 108


o Cough impulse +

o Valsalva maneuver – more prominent

o Moves with deglutition

❖ Treatment :

o Resect and repair thyrohyoid membrane

CLINICAL PEARLS:
❖ Swelling that moves with deglutition:
o Thyroid swelling
o Thyoglossal cyst
o Laryngocele
o Subhyoid bursa
o Midline dermoid cyst
❖ Cricothyroid membrane : used for emergency needle tracheostomy
❖ Tracheostomy : between in 2nd and 3rd ring

Pharyngeal pouch
▪ Old men

▪ Left side

▪ Posterior triangle swelling

▪ Clinical features-Dysphagia, Halitosis,Compress- swelling reduces with gurgling sound

▪ False diverticulum

▪ IOC: Barium swallow

▪ TOC:

o Diverticulectomy or

o DOHLMAN’S Procedure ( endoscopic stapling operation)

SURGERY SIXER APP BASED WORK-BOOK 2020 109


Chapter :2 Thyroid Gland

2a. Introduction

Anatomy of thyroid gland


• Butterfly shaped organ

• Derived from thyroglossal tract (originating from foramen caecum) which comes down

and divides into thyroid bud.

• Ultimobranchial body give rise to parafollicular cells

Arterial supply:
• Superior Thyroid Artery arises from External carotid. Enters the gland and then divides

into branches.

• Inferior Thyroid artery arises from Thyrocervical trunk of Subclavian artery and divides

into multiple branches before entering the gland.

• Inferior Thyroid artery – Supplies all 4 parathyroid glands

• Rarely- One more artery-from arch of Aorta- Arteria Thyroidea ima** is seen

Ligation of STA and ITA:


• Old Concept- Ligate STA close to gland to prevent injury to ELN, ITA ligated away from

gland to prevent injury to RLN.

• New Concept- to prevent Hypocalcemia due to parathyroid loss of Blood supply- Now we

ligate the ITA very close the gland as individual vessels. ( Remember both STA and ITA are

ligated close to gland)

Most common site of ectopic thyroid: LINGUAL THYROID ( failure to descend from Foramen
Cecum of Tongue)

Venous Drainage:
• STV- drains to IJV

• ITV drains to Brachiocephalic vein

• There is MTV ( But MTA)- drains into IJV

• 4th vein- Kocher’s vein drains into IJV or Brachiocephalic vein

SURGERY SIXER APP BASED WORK-BOOK 2020 110


Recurrent Laryngeal Nerve course:
• RLN arises from Vagus nerve as it descends down it gives the branches- Right and Left

RLN

• RLN Hooks and comes back

• Right RLN hooks around Right Subclavian Artery (Errata in App**)

• Left RLN hooks around Ligamentum arteriosum

• RLN is non recurrent in 5% cases.

• RLN is both motor and sensory.

o Motor- all muscles of vocal cord including posterior crioco arytenoid

▪ Helps in abduction of vocal cord( opening)

▪ Also called safety muscle of larynx

o Sensory – below the mucosa of vocal cord

Injuries to nerves:
• ILN: paroxysmal nocturnal cough/ Aspiration ( d/t loss of sensation above vocal cords)

• ELN: huskiness of voice @ loss of timber of voice or loss of pitch

• U/L RLN : Hoarseness of voice

• B/L RLN: stridor

External Laryngeal nerve injury:


• Most common nerve injured : EXTERNAL LARYNGEAL NERVE

SURGERY SIXER APP BASED WORK-BOOK 2020 111


Classification of ELN course in relation to superior pole of thyroid. – CERNEA Classification
I- >1cm from the superior pole

II- < 1cm from the superior pole

a. Above the gland

b. On the gland – hence has high chance of damage.

Recurrent laryngeal nerve – concepts from Bailey 27th Edition:


• Most common site of injury BEAHR’S triangle.

o Laterally- common carotid artery

o Medically by trache oosophageal groove

o Superiorly inferior thyroid artery.

• Tubercle of Zuckerkandl= most posterior lateral portion ( not Posteromedial – errata in

app) part of thyroid, also known as Pointing tip towards RLN.

o Close to berry’s ligament- which is a condensation of pretracheal fascia

▪ Connecting thyroid to trachea

▪ Reason for movement of thyroid with deglutition.

Injury of RLN- Management:


• U/L Injury: hoarseness – recover within 3 months

• B/L injury: Stridor post op--- 1st sign due to B/L nerve going for Cadaveric position

Stridor Management:
• Step 1- Immediate Reintubation

• Wait for 24-48 hours (Many neuropraxias will recover with steroids)

• Extubate with entire team ready for Tracheotomy.

• If there is stridor again- and Both nerves not functioning Tracheostomy done.

SURGERY SIXER APP BASED WORK-BOOK 2020 112


• After few months if it is a permanent damage we must go for some special procedures:

o Advice Arytenoidectomy/ Lateralisation of cord ( Types of thyroplasty)

Nutshell: Anatomy

• Lingual thyroid – m/c site of ectopic thyroid—


o d/t undescended thyroid located in formen caecum
o c/f-deglutition problems.
• 3 arteries- STA, ITA, ARTERIA THRYOIDEA IMA – Ligate close to gland
• 4 veins : STV, MTV, ITV, KOCHER’S VEIN
• ELN- M/C nerve injury- CERNEA Classificaiton
• ILN- Paroxysomal nocturnal cough
• RLN- M/C INJURED IN BEAHR’S triangle ( Between TE groove, CCA and ITA Postero
Laterally

Physiology:
• RDA for I2 – 0.1 – 0.15 mg/ day

Steps in synthesis. ( Mnemonic- TOBC)


• Trapping

• Oxidation ( iodine to iodide via thyroid peroxidase)

• Binding iodide with thyroglobulin ( to form mono iodothyroglobulin (MIG) Or Di

iodothyroglobulin( DIG).

• Coupling

▪ MIG +DIG = T3 – t half= 24 hrs

▪ DIG +DIG= T4 – t half = 7 days.

▪ T3 is the active form.

▪ Peripherally T4 is converted to T3.

SURGERY SIXER APP BASED WORK-BOOK 2020 113


Diseases related to physiology:
• Deficiency of iodine: m/c cause of hypothyroidism in the world

• Congenital thyroid peroxidase deficiency – PENDRED SYNDROME

o Sensory Neural Hearing Loss

o Congenital Hypothyroidism

• Hashimoto thyroiditis: inhibitory antibody against

o TPO – MCC

o THYROGLOBULIN

o TSH-R

o MCC OF HYPOTHYROIDISM IN WESTERN COUNTRIES**

• GRAVE’S DISEASE:

o Stimulatory antibody to TPO, TG and TSH- R which results in increased T3 and

T4 production

Thyroid Function Tests:


Thyroid profile
▪ Free T3- 3-9 micro mol/l
▪ Free T4 – 8-24 n mol/l
▪ TSH - <3 mU/l
▪ TPO Value- >25 U/ml
▪ Anti TG >1:100

Hypothyroidism:
▪ Free T3- N/ decreased

▪ Free T4- N/ decreased

▪ TSH- INCREASED

SURGERY SIXER APP BASED WORK-BOOK 2020 114


Hyper thyroidism:
▪ TSH- DECREASED

▪ FT3 – N/ increased

▪ FT4- N/ increased

• Most sensitive IOC for hypo/hyper thyoidism- TSH value.

• Subclinical cases- Free T3

• Refetoff syndrome – increased T4 along with normal or increased TSH. Due to peripheral

T4 resistance syndrome

Other investigations:
1) USG – neck:

• Helps to identify if it is thyroid / not

• Consistency ( solid / cystic)

• Lymph node

TIRADS- grading system (Thyroid Imaging Reporting and Data System)


o Composition of gland and tumour
o Echogenecity
o Size
o Echogenic foci
o Margin

2) FNAC/ FNNAC( FINE NEEDLE- NON ASPIRATION CYTOLOGY)

o FNNAC- Tissue is collected in needle hub (no negative pressure created)

o After that take gas in syringe and push it on glass slide

o BETHESTDA classification - based on FNNAC/FNAC

o Thy1 – Non Diagnostic

o Thy 1c – cystic lesion

▪ Completely disappear

• Non recurrent/< 3 times recurrence

• Recurrent>3 times - surgery

▪ Incompletely disappear – surgery is treatment of choice

o Thy 2 – Benign

o Thy 3 – Follicular neoplasm

o Thy 4 – Suspicious of malignancy

o Thy 5- Definite malignancy

SURGERY SIXER APP BASED WORK-BOOK 2020 115


3) IDL scopy: 3% patients have occult vocal cord paralysis

o Document for medicolegal purpose

4) Radio Active Iodine Uptake Study: (RAIU)

▪ Mixed with milk and taken orally)

o I131 : t ½ - 8 days

o I123 : t ½ - 12-14 hrs

o I132 : t ½ - 2.5 hrs

o Tc99m : t ½- 6 hrs

• Gland traps RAI and Emits beta radiation.- used for diagnostic and therapeutic

purposes.

• Caught on Gamma cameras- to see for uptake- called as THYROID SCAN.

Diagnostic uses: (m/c used is low dose I123 ) Therapeutic uses:


( MC used is I131)

1. <16% uptake – hypothyroidism Treatment of choice for


>48% uptake – hyperthyroidism 1. Grave’s disease
1-48 % uptake- Normal 2. To destroy thyroid
2. Warm nodule- increased activity than the cancer mets
surrounding tissue
5% are malignant.
Cold nodule- Decreased activity than the surrounding
tissue
20% are malignant.
3. To locate ectopic thyroid
4. Thyroglossal cyst:
▪ IOC in adults: USG NECK
▪ IOC in children : RAIU study – to rule out if
this is the only functioning thyoid tissue
5. After total thyroidectomy for malignancy- look for
metastasis
▪ Not done before thyroidectomy as uptake will
be only in thyroid tissue.

SURGERY SIXER APP BASED WORK-BOOK 2020 116


Side effects of RAI
Immediate( usually recover) Long term tumours Long term damage
Neck pain Leukemia Infertility
Thyroiditis Papillary CA of thyroid Abortion
Sialadenitis Anaplastic CA of thyroid Pulmonary fibrosis
Cerebral oedema Breast CA Bone marrow suppression
Vocal cord paralysis Gastric CA Ovarian / testicular failure
Bone marrow supression Lung CA Hypo parathyroidism
Salivary tumours
Bladder Ca

After RAI I 131

• Isolate patient for 8 days

• Don’t spit/ urinate/ kiss in public

RAIU is also used to know the cause for hyperthyroidism

Excessive T3& T4 production : increased RAIU Normal production but increased release : N/
decreased RAIU
o Grave’s disease o Thyroiditis (in acute inflammatory
o Secondary toxicosis phase – increased release of hormone)
o Jod – basedow effect o Thyrotoxicosis factitia ( excessive
o Struma ovarii thyroxine tablet taken)
o Hydatidiform mole o Hamberger toxicosis ( south
o TSH secreting pituitary adenoma Americans eat thyroid of Cow in
o Drugs- Amiodarone burger)

Extra Edge Points:


Thyroid doesn’t extent superiorly because?

o Pretracheal fascia extends from hyoid bone to arch of aorta ( in superior

mediastinum).

o Therefore thyroid swelling goes only retrosternally.

Post RAI – What happens to Physiology?

o After RAI ablation :most grave’s disease patient becomes euthyroid in 2

months.

o After 6 months : 50 % remain euthyroid

o Remaining 50 % becomes hypo/ hyper thyroid.

o Contraindication for RAI

SURGERY SIXER APP BASED WORK-BOOK 2020 117


1. Severe ophthalmopathy

2. Pregnancy and lactation

3. Children

4. Nodules +

5. Smoking females

2b. Benign Thyroid Disorders

Diffuse Nodular Inflammatory Neoplastic


Physiological Colloid goitre Hashimoto’s Papillary Ca
Grave’s disese Solitary nodular Reidel’s Follicular Ca
Colloid goiter Multinodular Dequervain Medullary Ca
Anaplastic Ca
Clinical features:

Hypothyroidism Hyperthyoridism
Obese Slim
Less intake Increased intake
Cold intolerance Heat intolerance
Constipation Diarrhea( increased bowel movement)
Menorrhagia--- amenorrhoea( anemia) Oligomenorrhoea
Tendon reflex – decreased Tendon reflex – increased
Pseudo myotonic reflex/ hungup ankle reflex- Fine tremors
delayed ankle jerk
• Loss of eye brow Eye signs- exophthalmos ( early)
• Macroglossia • Pretibial Myxoedma – deposition of
• Mask like faces mucopolysaccharides in front of tibia
• Depression • Acrobachy – subperiosteal bone resoprtion
of fingers- look like clubbing
Both are late features
Cause: Cause:
Cretinism ( paediatrics) Grave’s disease( 1’ toxicosis)
Myxodema ( adults) Plummer’ s disease (2’ toxicosis) due to
MNG > SNT ( for exams)

Grave’s Plummer’s
1’ 2’
Diffuse enlargement Long standing Solitary
with simultaneous Nodular thyroid
toxic features

SURGERY SIXER APP BASED WORK-BOOK 2020 118


suddenly go for toxic
features.
Affect: Affect:
CNS+ Old guys
EYE SINGS+ CVS+:
1. Sinus atrial
tachycardia( mc)
2. Atrial extra systole
3. Atrial fibrillations
4. Congestive cardiac
failure
5. Thyroid storm
6. Lerman scratch
sound( systolic
scratch sound in
Left 2nd ICS on
expiration)- rub
between pleura
and pericardium

Treatment:
50-100 micro gram tab. Thyroxine
(supplement dose)
MCC:
▪ Iodine deficiency : India
▪ Hashimoto : western

Grave’s disease
• MCC of hyperthyroidism.

• Autoimmune disease : HLAB8, HLADR3, HLADQ1

• Diffuse enlargement , soft consistency, highly vascular

o Palpation: thrill in superior pole

o Auscultation:

▪ arterial bruit

▪ Venous hum+ ( d/t increased sympathetic activity causing hyperdynamic

circulation)

SURGERY SIXER APP BASED WORK-BOOK 2020 119


Figure: Pretibial Myxedema in Graves disease

• Clinical tests in thyroid: ( All clinical examination videos will be showed in Clinical

Examination Section)

▪ Examined from behind.

▪ Short neck- PIZZILO’S METHOD. (hand behind neck & hyperextension of neck)

▪ Retrosternal goiter – Pemberton sign( to look for svc compression- lift both arm with

arm touching ears- look for facial congestion)

▪ Lahey’s test: 4 finger examination ( push on one side and palpate with 4 fingers)

▪ Crile’s test- for doubtful nodule( using only thumb- ask patient to swallow- feel for

nodule)

▪ Trail’s sing- prominent SCM muscle ( thyroid swelling pushes trachea)

▪ Berry’s sign- non palpable carotid artery against vertebrae ( d/t engulfment of the

tumour)

Figure: Pizzilo’s Test


Figure: Pemberton Sign

SURGERY SIXER APP BASED WORK-BOOK 2020 120


Figure: Lahey’s test Figure: Crile Test

Tests for toxicosis:

CNS CVS Eye signs


▪ Increased reflexes Sleeping Pulse rate Exophthalmos
▪ Tremors- fine ( CRILE’S GRADING)
▪ Fasciculations in METHOD
tongue (with
▪ Give
tongue
tab. P Phenobarbitone night dose and measure.
inside mouth)

Figure: Eye Signs in Thyrotoxicosis

Eye signs:

Mild exophthalmos – due to only sympathetic activity

1. Von grafe’s lig lag sign- ask patient to follow vertical movement- lid lags behind

SURGERY SIXER APP BASED WORK-BOOK 2020 121


2. Dalrymptes sign/lid retraction sign- visible upper sclera

3. Stellwag’s sign- starring look

Moderate – due to retroorbital accumulation

4. Joffroy’s sign- absence of forehead wrinkle on looking up

Ask patient to look the ceiling (patient eye is already out)

Severe – due to intraocular accumulation and paralysis of muscle

5. Mobius sign- Inability to converge ( paralysis of intraocular muscle)

Malignant – though treatment given- exophthalmos increases

• Epiphora

• Congestion

• Redness

Other signs:

Jellwek’s sing- pigmentation in upper eyelid

Naffziger’s sign- see through supraorbital ridge – normally eyes not seen

o If seen it is exophthalmos or proptosis

Gifford test ( eversion of upper eye lid to differentiate exophthalmos and proptosis)

o Exophthalmos- muller muscle spasm + - inability to evert upper eyelid

o Proptosis – pathology behind eye ball - able to evert upper eye lid.

Diagnosis:

▪ T3,T4 increased, TSH decreased

▪ RAIU increased

▪ Autoimmune antibodies increased: Anti TG, Anti TPO – 75% cases

▪ LATS- Long Acting Thyroid Stimulator is elevated in 90 % cases. Also Known as Thyroid

stimulating Antibody or TSH-R stimulating Ab

▪ FNAC is not beneficial.

- Haemorrhagic aspirate.

- Therefore not advised in grave’s disease.

SURGERY SIXER APP BASED WORK-BOOK 2020 122


Management of Grave’s disease:

Preparation of the thyrotoxic patient to euthyroid state

Surgery
Radioactive iodine ablation
Therapy
Gold standard

Preparation of the thyrotoxic patient to euthyroid state

• If not done properly - thyroid storm ( MC cause is inadequate preparation to Surgery)

Anti thyroid drugs Quick preparation Lugol’s iodine


Inhibit • Decrease sympathetic Mechanism:
• Trapping of iodine overactivity of the • Decrease vascularity
• Oxidation of iodine gland. of the gland
• Peripheral conversion • Early preparation is • Makes the gland firm
T4 to T3 possible

Carbimazole – leads to Beta blockers 5% iodine


reversible agranulocytosis- • Propranolol +
Ask h/o sore throat (short acting) – 10% KI
1st sign of decreased wbc 40 mg TDS 10 drops TDS
count • Nadolol
( long acting)
160 mg - OD
Methimazole – 10-30 mg
TDS
Propylthiouracil- used in
pregnancy – don’t cross
placental barrier
Take for 3- months to LAST 10 DAYS BEFORE
become euthyroid. SURGERY
Block and Replacement
therapy: Antithyroid drugs –
high dose carbimazole +
thyroxine

SURGERY SIXER APP BASED WORK-BOOK 2020 123


Last dose of carbimazole given Note :Continue beta blocker 7
night before surgery days post op including Intraop
period**

After patient is prepared: RAIU

I 131: 8-12 mci

Mixed with milk


Isolate them for 8 days.

Contraindication for RAI


▪ Severe ophthalmopathy

▪ Pregnancy and lactation

▪ Children

▪ Nodules +

▪ Smoking females

Surgery is indicated in these cases.

After RAI ablation :most grave’s disease patient becomes euthyroid in 2 months.

After 6 months : 50 % remain euthyroid

Remaining 50 % becomes hypo/ hyper thyroid

Total thyroidectomy- advised in grave’s disease

Indications:

▪ RAI is contraindicated
▪ Suspicious of cancer
▪ Goiter >80 g
▪ Compressive symptoms
▪ Patient not responding to radioactive iodine
▪ Rapid control is needed

Thyroid storm:
MCC- Inadequate preparation of thyroid patient
OTHER CAUSES-
▪ Illness

▪ Stress

▪ Amiodarone

SURGERY SIXER APP BASED WORK-BOOK 2020 124


Characterised by:
CNS problems – agitation/ depression

CVS problems – dysfunction- CCF

10% mortality

Clinical features:
• Tachycardia

• Hyperpyrexia

• Dehydration

• CCF( symptoms+)

• Hyperexcitability

• Atrial fibrillation

Treatment of thyroid storm:


Basic life support Antithyroid drugs Special drugs
O2 IV propranolol Steroids – hydrocortisone
200mg- ( adrenal is
exhausted)
Tepid sponging Lugol’s iodine Diazepam ( they are
hyperexcitable)
IV fluids PTU Diuretics- ( they have CCF)
Glucose Carbimazole Digoxin – ( if in AF)
KI – in IV

Secondary Toxicosis
▪ Solitary nodule
▪ Multinodular- Plummer’s disease ( some books mention as SNT with toxicosis also)
▪ Old patient
▪ CVS manifestation are MC** than Eye signs and CNS manifests.
▪ Eye signs may be present- lig lag+ and lid retraction+

Treatment of choice:
▪ Prepare them to euthyroid state

▪ Surgery- subtotal/ total thyroidectomy (as nodule presence is a contraindication for RAI)

Thyroiditis
• Mc in females 30-40 yrs

• Inflammation of thyroid gland

SURGERY SIXER APP BASED WORK-BOOK 2020 125


Hashimoto Reidel( fibrosis) Dequervain Acute
Mc type Mc in children
a/k/a chronic lymphocytic/ Associated with • a/k/a post viral or • Bacterial
autoimmune thyroiditis • Invasive fibrosing subacute infection
thyroiditis ( after few weeks of viral • Mc-
• Dupuytren’ s infection) streptococcu
contracture • Associated with s
• Retroperitoneal fibrosis HLAB35 • NO HLA
association
Abs: (inhibitory) Persistent
• TPO- MC pyriform sinus
• TG fistula-
• TSH-R important cause
of recurrent
acute bacterial
thyroiditis
FNAC: FNAC: cannot be done-
• Hurthle cells/ woody hard- mimics cancer
Ashkenazy cells Do trucut biopsy ( done in
(Abdundant cytoplasm with anaplastic and reidel)
varying size of nucleus)
• Lymphocytes+
C/F: ▪ ALWAYS ▪ Predominantly ▪ Severe neck
▪ Pain + HYPOTHYROID**- hyperthyroid with pain pain+
▪ Thyrotoxicosis( d/t produce compressive for long period odynophagia
increased release not symptoms ▪ Later become ▪ Increased
production) hypothyroid ESR
▪ Burn out leading to ▪ Increased
hypothyroidism DD: Grave’s- increased RAIU WBC
▪ When patient Dequervain-
presents- painless ▪ Decreased RAIU
enlarged thyroid ▪ Increased ESR
with
hypothyroidism.
Complications:
▪ Hypothyroidism ▪ Laryngitis
▪ Compressive ▪ Perichondrit
symptoms is
▪ Premalignant- ▪ Sepsis
lymphoma and ▪ Tracheal
papillary Carcinoma rupture

SURGERY SIXER APP BASED WORK-BOOK 2020 126


▪ Esophageal
rupture
▪ IJV
thrombosis

Treatment:
▪ Tab thyroxine ▪ Tab . thyroxine ▪ Analgesics ▪ Antibiotics
▪ Painkiller for pain ▪ Steroids ▪ steroids ▪ Remove
Surgery is not advised ▪ Antiestrogen- pyriform
except in compressive tamoxifen sinus fistula
symptoms ▪ Mycophenolate
mofetil

Retro sternal goiter:


Why Goitre extends Downwards?
• Pretracheal fascia – attaches from hyoid to aortic arch (superior mediastinum)

• SCM prevent upwards movement.

Types :
Primary Mediastinal: Secondary Mediastinal: ( Retrosternal Goitre)
▪ Arising from ectopic ▪ Typical retrosternal extension
thyroid gland present – 99% patients
inside chest ▪ Blood supply from superior
▪ Blood supply from thoracic and inferior thyroid artery
vessels ▪ Approach through neck
▪ Sternotomy is required incision- put finger and hook it
= TOBOGANN maneuver
▪ Pemberton sign+ - ( hands on
ear- flushing of face)

Managament:
▪ Investigation of choice: ( COMPULSORY) CECT THORAX /NECK

Elderly: asymptomatic – wait and watch


Indications for sternotomy:
▪ Previous neck surgery

▪ Recurrent retrosternal goiter

▪ Posterior mediastinal extension

▪ 1’ mediastinal goiter (blood supply from mediastinal blood vessels)

SURGERY SIXER APP BASED WORK-BOOK 2020 127


▪ Adhesions in mediastinum

Thyroglossal cyst (congenital):


• Persistence of tract

• c/f:

o 5-25 yrs (though congenital)

o Females

o Moves with protrusion of tongue

o Mc site- sub hyoid

• Complications:

o Infection- m/c

o Rupture / I&D –leads to TG fistula= Acquired

o Malignancy- papillary Ca

• IOC: USG Neck

• Paediatric case: RAIU study

o If this is the only thyoid- wait till child becomes adult

o If normal thyroid gland is present : SISTRUNK OPERATION

▪ Remove whole tract + central hyoid bone

NOTE:
Sistrunk operation : lymphedema of limbs

Sistrunk incision : parotid surgery

Thyroid cyst
• Patient presents with nodule

• On FNAC – clear fluid aspirate- BETHESDA thy1c

Indications for hemithyroidectomy:


o Incomplete disappearance

o Haemorrhagic aspirate

o Solid component +

o Cyst is recurring >3times aspirated

o Cyst >4 cm ( very big)

o Malignant cells +

SURGERY SIXER APP BASED WORK-BOOK 2020 128


2c Thyroid Malignancies

DUNHILL CLASSIFICATION:

DIFFERENTIATED MEDULLARY UNDIFFERENTIATED

Papillary Ca – 80% 2.5% Anaplastic – 5%

Follicular Ca – 10% Lymphoma ( NHL) – 2.5%

Hurthle cell Ca

Sequence Based Question: Papillary > follicular> anaplastic > medullary=lymphoma

Predisposing factors:
▪ Long standing Multinodular goiter: follicular Ca

▪ Irradiation : papillary Ca

▪ Hashimoto’s : NHL and Papillary CA

▪ Thyroglossal cyst: papillary ca

▪ Genetic mutation and syndromes

o RET ongogene: Medullary Ca

o RAS oncogene: follicular > papillary

o PTEN : follicular ca

o P53: Anaplastic ca

• Syndromes:

o COWDEN SYNDROME

o FAMILIAL ADENOMATOUS POLYPOSIS

o MEN 2A AND MEN 2B

o CARNEY’S triad

o Mc Cune Albright Syndrome

MEN 2A (SIPPLE SYNDROME) MEN 2B


Medullary Ca thyroid Medullary Ca thyroid
Pheochromocytoma Pheochromocytoma
Hyperparathyroidism Marfanoid habitus
Hirschsprung disease Neuromas ( mucosal)
Prophylactic thyroidectomy done for Both Patients
< 5years <1year

Staging of thyroid CA:

SURGERY SIXER APP BASED WORK-BOOK 2020 129


T Staging:
o T1a: < 1cm

o T1b: 1-2 cm

o T 2: 2-4 cm

o T3: >4 cm , muscle /soft tissue+

o T4a: Removable Structures: RLN/ Trachea/ larynx/ esophagus

o T4b: Non Removable Structures: carotid vessels/ medistinum/ prevertebral

fascia

Nodes:
o N1a: Level VI ( Delphic nodes)

o N1b : More than Level VI

Papillary Carcinoma Follicular Carcinoma


MC thyroid cancer Mc in iodine deficiency areas
Pathology
• Multifocal- one lobe many foci • Single
• Multicentric – B/L • Well defined capsule
• May or may not have capsule
Spread:
Lymph node mets Hematogenous
“lateral aberrant thyroid” M/c site: skull- pulsatile osteolytic secondaries
Metastatic node from an occult
papillary ca.

Distant mets: lungs Distant mets: skull


FNAC: (3p)
• Psammoma bodies( concentric • Inconclusive ( can’t define benign /
calcification) malignant)
• Pale empty orphan annie eye nucleus • Capsular invasion cannot be detected with
• Papillary projections FNAC

Predisposing factors:
• Irradiation Long standing multinodular goitre
• Hashimoto’s
• Thyroglossal cyst
Prognosis:
Excellent: 95% 10 yr survival Good prognosis
Treatment: Management:
Total thyroidectomy+ modified radical neck • If >4cm – do total thyroidectomy directly
dissection ( if nodes +ve) • If Thy 3 ( <4cm)

SURGERY SIXER APP BASED WORK-BOOK 2020 130


Hemithyroidectomy

cut and send for HPE ( not frozen- miss


malignancy)

result in 2-3 days


Based on Result:
o if benign: discharge patient
o if malignant: on day 3- do
completion total thyroidectomy.

After Total thyroidectomy in differentiated Cancers- Post op protocol:


Post op:
1. Look for mets - RAIU study

o Mets+ - do RAI Ablation therapy with higher doses

o No mets – follow up with tumour marker thyroglobulin

o When TG rises- do RAIU Study and look for mets.

2. TSH dependent tumour – papillary Ca of thyroid-

o Give suppression dose of thyroxine (only for papillary Ca) – 0.3 mg

o Give substitution dose after total thyroidectomy in Follicular cancer– 0.2 mg

Scoring system for prognosis: AGES, AMES, MACIS


Low risk High risk
Age <40 yrs >40 yrs
sex Female Male
Mets Absent Present
Grade Well differentiated Poorly differentiated
Size <2cm >4cm
extent Confined Out of capsule +
Completion of surgery completed Not completed
R1- microscopic residual
tissue.
Hurthle cell Carcinoma:

SURGERY SIXER APP BASED WORK-BOOK 2020 131


Rare variant of follicular carcinoma

o Malignancy indicated by capsular invasion

o FNAC not useful

Differentiating features:

o From oxyphil cells of thyroid

o Do not take up RAIU

o Multifocal and multicentric

o Lymph node mets +

o Bad prognosis- 20% mortality in 10 years.

Treatment: Total thyroidectomy +m RND+ Central neck node dissection

Central neck node dissection: Removal of level 6 nodes- advised in medullay Ca of thyroid.
Central neck Nodes are seen between
o Hyoid bone superiorly

o Clavicle inferiorly

o SCM laterally

Post op management of Dunhill’s differentiated Ca:


1. Total thyroidectomy followed by RAIU study to look for mets. If mets are present RAI

ablation therapy govem

o RAI dose: 30-100 mci

Indication for RAI ablation therapy


▪ Distant mets+

▪ Extrathyroid spread+

▪ Size >4cm tumour

▪ Lymph node mets are+

▪ Papillary Ca- variants

o Tall cell variant

o Insular variant

o Columnar variant

2. External beam radiotherapy (EBRT) – for follicular Ca

3. Chemotherapy

o Doxorubicin

o Paclitaxel

4. Hormone therapy

Papillary Ca (TSH suppression with thyroxine tablets – 0.3mg)

Medullary Ca of thyroid
• Incidence -2.5%

SURGERY SIXER APP BASED WORK-BOOK 2020 132


• Parafollicular c cells- (ULTIMOBRANCHIAL BODY)

• RET oncogene mutation : MEN2A , MEN 2B

SPORADIC FAMILIAL
80% - 20%
M/C TYPE
Unilateral Bilateral- multicentric and multifocal
MEN2A, MEN2B, Familial medullary
carcinoma thyroid syndrome

Properties:
Neuroendocrine tumour (secretory)

o Calcitonin – tumour marker for follow up ( > 0.08ng/ml is significant)

o CEA – for prognosis

o ACTH- cushing syndrome

o Serotonin- diarrhea

o PGE2 AND PGF2 ALPHA

o HISTAMINIDASE

Rule out pheochromocytoma- 24 hr urinary VMA INCREASED. IF PRESENT- OPERATE

Pheochromocytoma first.

FNAC shows- amyloid stroma

Recent advances
EGFR antibodies : Vandetanib
Anti CEA antibodies : Labetuzumab

Treatment: total thyroidectomy + central neck node dissection ( irrespective of node -/+) +

mRND ( if node +)

MC site of distant metastasis – Liver ( like other neuroendocrine)

Prophylactic thyroidectomy :

o MEN2A : <5 yrs

o MEN2B : <1yr

Anaplastic carcinoma – p53 mutation


• Old women

• Aggressive

• Bad prognosis

• In 6 months – they die

• FNAC – cannot be done- stony hard

SURGERY SIXER APP BASED WORK-BOOK 2020 133


• Trucut biopsy has to be done

• MCC of death – respiratory obstruction

If a k/c/o anaplastic Ca coming with stridor- LET HIM DIE PEACEFULLY.

Not a k/c/o anaplastic ca with stridor- Isthmectomy / tracheostomy are done

Miscellaneous
1. Mets to thyroid – (post mortem dissection)

• m/c breast – overall

• m/c lungs – in men

2. Lymphoma :

• NHL – ‘B’ cell

• CHOP REGEIMEN

• CYCLOPHOSPHAMIDE

• HYRDROXYADRIAMYCIN

• ONCOVIN

• PREDNISOLONE

Follow up protocol: Thyroid Cancer


1. Thyroglobulin : <5 ng/ml
2. Calcitonin, CEA – for medullary Ca
3. USG Neck – 6th month, 12th month and then annually till 5 years
4. PET CT: when?
o TG is increased BUT
o RAI IS NORMAL
o USG Neck is normal
In above cases to know the source we do PET CT.

2d .THYROID SURGERY

TYPES OF THYROIDECTOMY

• Lobectomy/ hemithyroidectomy – remove isthmus + one lobe

• Subtotal thyroidectomy – leave behind 8 g in each TE groove

• Near total thyroidectomy/ Hartley – Dunhill procedure – 2-4 g is left behind on one side

• Total thyroidectomy – nothing is left behind

SURGERY SIXER APP BASED WORK-BOOK 2020 134


Steps in thyroid surgery:

1. Reverse trendlenberg position( head up)- decrease bleeding from neck

2. Neck extended position / dog barking position

3. Skin crease incision

4. Subplatysmal flap- raise above and below

5. Open deep fascia of neck

6. Strap muscle is cut / retracted

7. 1st ligated vessel- MIDDLE THYROID VEIN

8. Then ligate artery near the gland

- superior thyroid artery – prevent ELN injury

- inferior thyroid artery – preserve Parathyroid gland blood supply

9. Place drain for 24 hours- look for bleed

Surgeon finds a yellow tissue with the dissected thyroid. What to do?
o Saline float test – place tissue in normal saline
▪ If sinks – it is parathyroid gland
▪ If floats- it is fat / lymph node

▪ Intra operative – place parathyroid gland in SCM muscle pocket


▪ Post operative- place parathyroid gland in non dominant brachioradioalis
as implant.

Post op complications

1. Stridor – B/L RLN injury/ neuropraxia- reintubate -wait for 48-72 hours by giving

steroids and anti-inflammatory drugs

▪ If stridor disappear – extubate

SURGERY SIXER APP BASED WORK-BOOK 2020 135


▪ If persistent stridor –extubate and do tracheostomy

▪ Permanent B/L RLN injury- do lateralization of cord / Arytenoidectomy

2. Life threatening complication in post op-

▪ Hematoma – due to slippage of ligature of superior thyroid artery

▪ Management- remove suture in bed side- then prepare O.T

▪ Nowadays we don’t get it as we have drain-

▪ If bleed is seen in drain- shift patient to O.T and capture the bleed

3. Nerve injuries

▪ U/L RLN: - hoarseness of voice

▪ B/L RLN- Stridor

▪ ELN( MC) – Huskiness of voice ( timber is lost)

▪ ILN – Paraxysomal nocturnal cough/ aspiration

4. Laryngomalacia – after removal of thyroid- the larynx collapses

5. Thyroid storm

6. Respiratory dyspnea

▪ Hematoma

▪ B/L RLN injury

▪ Vocal cord oedema

▪ Laryngomalacia( tracheomalacia)

▪ Laryngismus stridulus (hypocalcemia)

7. Hypocalcemia

Hypocalcemia
• 2-5 days post op

• Due to loss of blood supply to parathyroid gland

• Manifestations: subclinical or overt

Subclinical overt
• On measuring BP- Carpopedal spasm • Tetany- opisthotonus
– trousseau sign • Laryngismus stridulous
• On tapping face- chvostek sign
Management – oral Calcium IV Calcium gluconate

▪ No need to monitor sr. calcium for Looking for Hypocalcemia

▪ Normal Sr. calcium- 8-12 mg/dl.

▪ For a pre op patient with 12 mg/dl – post op value of 8 is also hypo calcemic

though in normal range. So serum calcium is not reliable.

▪ For transient – oral calcium

▪ For severe- iv calcium

SURGERY SIXER APP BASED WORK-BOOK 2020 136


▪ Permanent( <10% cases where all 4 parathyroid glands are removed)- oral calcium + vit d3

for life time

Recent trends-
1. Monitoring of RLN injury

Continuous Intermittent
Electrode on vagus nerve Electrode on vocal cords at ET tube
When we give traction on RLN – it is detected Indicates after vocal cords lie in cadaveric
and we get signal position
Prevents injury Injury is detected ( after palsy)

2. Minimally invasive surgery or BY ROBOTIC surgery

BY various approaches
▪ Axillary approach

▪ Sub mammary approach

▪ Supraclavicular approach

▪ Breast Approach

SURGERY SIXER APP BASED WORK-BOOK 2020 137


Chapter : 3 Breast

3a. Triple Assessment of Cancer Breast

ANATOMY:
• 16-20 ducts

• 10-100 lobules

• Breast is held in place with suspensory ligament of Cooper

• Breast lies over pectoralis major muscle and pectoralis fascia

• Sappey’s plexus – sub areolar lymphatic plexus

• Lymphatics of breast are classified based on pectoralis minor

o Level I- ( lateral to pectoralis minor)

▪ Anterior

▪ Posterior(subscapular)

▪ Lateral ( Along Humerus)

o Level II – ( at the level of p minor a/k/a central nodes)

▪ Interpectoral nodes/ Rotter’s nodes

o Level III – ( medial to p minor a/k/a infraclavicular/ apical group)

o Other nodes

o Supraclavicular node= Regional node

o Internal mammary node= seen only in CECT chest

Figure: Axillary Nodes.

SURGERY SIXER APP BASED WORK-BOOK 2020 138


Case discussion of Breast Cancer:

Triple assessment of breast


• Clinical examination

• Radiological examination

• Pathological examination

Positive predictive value of diagnosis of CA breast by this method is 99.9%

Clinical examination:

o Position:

• Sitting

• Sitting and leaning forward- look for chest wall fixity( ribs)

• Arm on hip with alternate contraction and relaxation- look for pectoralis

major fixity

• Arms raised above head – look for Nipple retraction and paeu de orange

• Semi recumbent position- Best position - fat gets dispersed and tumor is

well elicited

o Methods to examine the breast: ( not to miss out areas)

• Circumferential method – palpate from centre to periphery with 4 fingers

• Vertical strip method – to top to bottom from lateral to medial

• Dial clock method – go outside to inside in each clock hour

SURGERY SIXER APP BASED WORK-BOOK 2020 139


c TNM (8th edition AJCC):

• C TNM : ( Clinical TNM)

• p TNM : pathological TNM

• y TNM : Post neo adjuvant CT/RT

• Tis: Tumor in situ – Ductal CA in situ, Paget’s disease

(Lobular Ca in situ is removed in 8th Edition as it is a physiological precursor for CA


breast)
• T staging:

• T1: <2cm

• T2 : 2 -5 cm

• T3: >5cm

• T4 :

a. Chest wall fixity – involvement of inter costal muscles, serratus anterior, ribs

b. Skin involvement –

o Paue de orange ( subdermal lymphatics infiltrated)

o Ulcer in skin

o Satellite nodules + on skin

c. Both a+ b

d. Inflammatory breast cancer (worst prognosis)

Things not included in skin involvement are:

SURGERY SIXER APP BASED WORK-BOOK 2020 140


o Fixity

o Tethering

o Nipple retraction

o Dimpling

o Puckering

N staging:
N1 : mobile axillary nodes
N2
a: fixed axillary nodes
b: Internal mammary nodes ( By CECT- but comes in c TNM)
N3
a. infraclavicular node( apical)

b. Both axillary and internal mammary

c. Supraclavicular nodes

Contralateral nodes : always metastatic ( eg. Opposite supraclavicular nodes)

M1: metastatic
o M/c site:

o Lumbar vertebrae ( batson plexus)

o Long bones ( both osteolytic and osteoblastic)

o Liver

o Lungs

NOTE:
• If patient presents with 2 tumors in breast – take the biggest one for T staging**

• IF patient presents with B/L tumors – do separate staging for both breast

STAGING
• Stage I, IIa , IIb – Early breast cancer – do MRM or BCS

• Stage IIIa,b,c – Locally advanced breast cancer – do neoadjuvant chemotherapy followed

by MRM + RT

• Stage IV – Metastatic cancer – palliative therapy – Hormone therapy+ chemotherapy

SURGERY SIXER APP BASED WORK-BOOK 2020 141


At the end of clinical examination of Mrs. Kamala- let us assume the stage T4b N2a M0 – stage
IIIb

Radiological examination

o USG breast – indicated for females <35 years( due to dense breast)

o Mammography - > 35 years ( less dense breast)

o MRI – In females with silicon implant (to look for capsular rupture – Linguine sign

– i.e floating of ruptured capsule)

Figure: Linguine Sign in MRI


Mammography:

SURGERY SIXER APP BASED WORK-BOOK 2020 142


• X ray

• Low voltage and high amplitude

• Bremstrahlung type of X ray ( Not a conventional X ray)

o Dose – 0.1 c Gy = 4 times chest x ray

o No increased risk of cancer

• Screening mammography - >40 YEARS

o Risk of cancer breast related deaths decreased by 30 %

• Views to be taken in mammography

o Cranio-caudal view

o Mediolateral oblique view

o Done always for both breast

Findings on mammography:
1. In cancer breast ;

o Micro calcification ( most common finding)

o Braching calcification

o Spiculations

o Mass effect

o Asymmetry

Figure: Microcalcification in cancer breast

2. In DCIS :

o Micro calcification

o Scattered calcification

o Clustered calcification

Radiological IOC for DCIS is Mammography.


3. LCIS : NO radiological finding.

4. LC invasive type:

SURGERY SIXER APP BASED WORK-BOOK 2020 143


o Neighboring calcification

5. Fibroadenoma breast

o Popcorn calcification / macrocalcification

BIRADS grading ( Breast Imaging Reporting And Data System)

0- Inconclusive
1- Benign - followup
2- Benign- followup
3- Benign - followup
4- Probably malignant – advice biopsy
5- Mostly malignant – take appropriate action
6- Biopsy proven malignancy

Biopsy is advised for BIRADS 4 and above

PIRADS – Prostate

TIRADS – thyroid

Mammography:
o Screening investigation of choice for >40 years** for general public

o IOC for DCIS

MRI Breast (for screening)


Indications:
o Those who have high risk (20-25%) – said by GAIL model / CLAUSS model

o BRCA 1 , BRCA 2 mutation

o 1st degree relative of BRCA 1 or 2 mutation

o Li Fraumeni syndrome

Age for screening - >30 years in high risk cases**

Other indications of MRI breast


o Scarred breast

o Previous h/o breast conservative surgery

o Silicon implant breast

SURGERY SIXER APP BASED WORK-BOOK 2020 144


Let us assume our case:
o On C/E – stage IIIb

o On R/E: BIRADS 4 ( Hence biopsy advised)

Pathological examination

1. FNAC : with 22/24 G needle

2. Corecut Biopsy : 16/18 G needle,

• IOC to diagnose cancer breast- Helps to diagnose Type of cancer and also

Hormone receptor status study

3. Excision biopsy- only with small and inconclusive lesion

Findings:
• Ducts giving cancer– Ductal cancer -90 % ( insitu or invasive)

• Lobules giving cancer – Lobular cancer – 10% ( insitu or invasive)

• Supporting tissue gives cancer – Phyllodes tumor

Ductal Ca – invasive Lobular Ca - invasive


80% 10%
m/c type: Bilateral
• NST( no specific type) or • Multicentric –in all quadrants
• NOS (not otherwise specified) or • Multifocal – multiple foci in same
• Scirrous quadrant

TYPES : • Always Advice FNAC of opposite


1. MEDULLARY CA breast
• Triple negative CA • Biopsy shows INDIAN FILE
• Soft in consistency APPEARANCE of cells
• Bad prognosis • Mammography shows Neighboring
2. Colloid CA Calcification
o Mucinous cancer • Treatment : B/L MRM advised
o Old age
o Best prognosis
3. Tubular CA –Good prognosis

SURGERY SIXER APP BASED WORK-BOOK 2020 145


Figure: Indian File Pattern in Lobular Cancer

DCIS LCIS
• India : m/c presentation : painless • m/c painless lump
lump • no radiological finding
• Western : mammography ( as
microcalcification)
• Converted to Ductal Ca invasice in • Converted to Ductal Ca invasive (
same place and same type m/c) Or any cancer and any where
• ANATOMICAL PRECURSOR OF • PHYSIOLOGICAL PRECURSOR OF
MALIGNANCY MALIGNANCY
Classified under Tis Removed form Tis
Types: • Pleomorphic type of lobular ca in situ
Low grade – can be seen on mammography with
• Papillary type calcification
• Cribriform type • Needs resection
High grade
• Solid type
• Comedo type
Management:
• Simple mastectomy ( with sentinel • High risk patients – advise regular
node biopsy) screening
• Breast conservative surgery • With T. tamoxifen
• Tamoxifen given
• RT to remaining breast after BCS/SM
VAN NUY’S grading system: based on
1. Age

SURGERY SIXER APP BASED WORK-BOOK 2020 146


2. Grade
3. Size
4. Margin clearance

Molecular classification

Immunohistochemistry done for


• ER +/-

• PR +/-

• HER 2 NEU+/-

o 0 ,+1 – HER 2 NEGATIVE

o +2 – EQUIVOCAL – do FISH to confirm (+/-)

o +3 – HER 2 POSTIVIE

Luminal A ( m/c Luminal B( ALL Her 2 Basal type


type) POSITIVE) (ONLY HER 2 ( ALL NEGATIVE)
POSTIVIE)
ER + ER+ ER- ER-
PR + PR+ PR- PR-
HER 2 - HER 2 + HER 2 + HER 2 -
BEST PROGNOSIS Poor prognosis Worst prognosis
a/w BRCA 2 MOST COMMON
CANCER IN BRCA
1**

Consolidation:

• MC presentation- Painless lump

• Nipple discharge- Bleeding

• Nipple retraction

o Circumferential – malignant

o Slit like – benign

• Most common quadrant

o Ca breast and fibro adenosis – upper and outer quadrant

o Fibroadenoma – lower quadrants

SURGERY SIXER APP BASED WORK-BOOK 2020 147


Risk factors causing Ca breast.
Non modifiable Modifiable Histological risk factor
Reproductive Reproductive • Atypical ductal
• Early menarche (<12yr) • Child birth (>30 yrs) hyperplasia
• Late menopause (>55yr) • Nulliparous • Atypical lobular
• Absent breast feeding hyperplasia
• Atypical epithelial
hyperplasia

Age (With increasing age) Tobacco LCIS (Physiological precursor


of malignancy)
Female sex alcohol
Family history obesity
BRCA 1,2 mutation Night shift work
Li Fraumeni syndrome Lethargic activity
H/o radiation exposure in Hormones :
childhood • OCP: Yes
• HRT (post
menopausal) : Yes
• Young female after
B/L salpingo
oopherectomy on
estrogen only pills :
NO

Classification of breast cancer:


1. Sporadic : 60-75%

2. Familial : 20-30 %

• 1st degree relative has 13 % risk

3. Hereditary 5%

• BRCA 1 -45%

• BRCA 2 - 35%

• Li Fraumeni syndrome -1%

• Ataxia telangiectasia -1%

• Muir torre ( lynch II)- 1%

• Cowden- 1%

SURGERY SIXER APP BASED WORK-BOOK 2020 148


BRCA 1 BRCA 2
Chromosome 17 q Chromosome 13 q
Poorly differentiated Well differentiated
Triple negative ( basal type) Triple + / ER +
Bad prognosis Good prognosis
m/c – m/c-
• Breast ca – 60% • Breast ca ( 80 %)
• Ovarian ca – 40 % • Ovarian ca – 20%
• Prostate ca • Prostate ca
• Colon ca • Colon ca
• Pancreatic ca
• Gastric ca
• Melanoma
• Male breast ca
Advise them to undergo prophylactic B/L simple mastectomy
And after completion of family – B/L salpingo oopherectomy

3 B. management of cancer breast

Multimodality treatment
SURGIGAL MANAGEMENT
• Simple mastectomy

• Modified radical mastectomy

• Radical mastectomy

• Sentinel node biopsy

• Breast conservation surgery

• Breast reconstruction

Simple mastectomy (SM)

• Remove entire breast + nipple + areola

• No nodal dissection

Indications:
o Prophylaxis : BRCA 1,2

o Toilet simple mastectomy- in ulcerative ca breast ( as a palliative measure)

o Extended simple mastectomy = SM + level I nodes removed

o Nipple sparing / skin sparing SM – BRCA 1,2 mutation

SURGERY SIXER APP BASED WORK-BOOK 2020 149


Modified radical mastectomy

Structures removed are:


• Nipple

• Areola

• Breast tissue (entire)

• Pectoralis fascia

• Axillary dissection procedure ( level I, II, III)

Boundaries:
• Laterally – latissimus dorsi

• Medial- sternum

• Superiorly – subclavius muscle

• Inferiorly – 2cm below the infra mammary fold

Figure: Boundaries of MRM

Structures preserved are: (ABCDM)


• Axillary vessels

• Bell's nerve

• Cephalic vein

• Nerve to lattismus Dorsi ( thoraco dorsal nerve)

• Pectoralis Major

Axillary dissection boundaries


• Superiorly – axillary artery and vein

SURGERY SIXER APP BASED WORK-BOOK 2020 150


• Inferiorly – angular vein

• Medially- bell’s nerve ( long thoracic nerve)

• Laterally- thoraco dorsal pedicle ( vessels + nerve)

❖ Minimum 10 nodes to be removed to call a complete axillary dissection

❖ Most common nerve injured – intercosto brachial nerve- only cutaneous supply**

o Other nerves

▪ Bell’s nerve- winging of scapula

▪ N. to LD – shoulder movements are affected (eg. Medial rotation, climbing

trees)

▪ Medial and lateral pectoral nerve- supply pectoralis major

Figure: Boundaries of Axillary Dissection.

STEPS OF MRM
❖ Position- supine with arm extended in 90’

❖ Skin incision – elliptical Stewart’s incision

❖ Elevate skin flaps till boundaries mentioned

❖ To deal with Pectoralis minor in axillary dissection:

o Patey’s method-

▪ cut and remove

▪ level I, II, III can be removed

o Auchincloss-

▪ Retract P. minor

▪ Level I, II only can be removed

o Scanlon

▪ Cut and resuture

SURGERY SIXER APP BASED WORK-BOOK 2020 151


▪ Level I, II, III can be removed

❖ Place 2 drains – one in axilla and another in bed of surgical site

❖ Closure of skin with subcuticular suture – using monocryl (polyglycaprone)

Complications:
• M/c complication – seroma –

o remove drain on 5th day post op

• Nerve related- intercosto brachial nerve

• Flap necrosis

• Lymphoedema of arm

• Lymphoedema of arm + radiotherapy = lymphangiosarcoma—called as STEWART

TREVES SYNDROME

Radical mastectomy of Halstead

• MRM +

• Removal of

o Pectoralis major

o Pectoralis minor

o Serratus anterior ( partly)

• 3 structures preserved ( ABC)

o Axillary vessels

o Bell’s nerve

o Cephalic vein

Sentinel node biopsy

• Sentinel node- 1st node from Ca breast (Axillary node)

• Axillary dissection is the main cause of morbidity.

• To prevent this – sentinel node biopsy is done

• Therefore test the 1st node and look for deposit in 1st node-

o if positive- dissect axilla

o if negative – don’t dissect axilla

❖ sentinel node of breast – GUILIANO sentinel node biopsy ( axillary node)

❖ sentinel node of penis – CABANA ( superficial inguinal node)

▪ Indications:

• Early breast cancer – stage I, IIa, IIb ( T1,T2)

SURGERY SIXER APP BASED WORK-BOOK 2020 152


• No node palpable ( on clinical examination or on USG )

▪ Contraindications:

• T3,T4 lesions ( locally advanced breast cancer)

• Palpable node +

• Previous h/o surgery in breast ( 1st level lymphatic is disturbed – it will have a

different pathway)

• Pregnancy

• Contraindications for radiotherapy

o Collagen vascular disease

o Previous h/o RT

o Pregnancy

▪ Procedure of sentinel node biopsy

o Indian method:

▪ Inject methylene/ isosulphan blue in peritumor area

▪ Open axilla dissection see for blue colour node

▪ Dissect and send for frozen section

o Western method:

▪ Inject radio labelled substance ( Tc 99)

▪ High uptake area is captured with gamma camera

▪ Dissect it

o Best method – combined method**

Frozen section biopsy:


o Positive – axillary dissection completely

o Negative – call the radiotherapist- give RT to axilla and send home

Figure: Sentinel Node Biopsy by Radiolabelled Colloid

SURGERY SIXER APP BASED WORK-BOOK 2020 153


BREAST CONSERVATIVE SURGERY

Indications:
o Early breast cancer

o DCIS

BCS has 4 components:


1. Wide local excision

• Clean negative margin from pathologist on all sides ( 1 mm negative )

2. Axillary status should be identified

• N0- Sentinel node biopsy

• N1 –Axillary dissection

3. Radio therapy to remaining breast

4. Keep patient on follow up

• Since only tumor was removed – it may recur

Contraindications of BCS:
1. If wide local excision not possible like in

• Locally advanced breast cancer (skin fixed, chest wall fixed)

• After 2 times wide local excision – margin is positive – go for MRM

• Multicentric and multifocal cancer ( can’t do WLE)

• Small breast but tumor is big – cosmetic deformity

2. Where RT is contraindicated

• Collagen vascular disease

• Previous h/o RT

• Pregnancy- 1st and 2nd trimester (in 3rd trimester – we can give RT after

delivery)

• Huge pendulous breast – we’ll have to expose abdominal contents while giving

radiotherapy to the remaining Breast.

3. Patients who tend to Loss to follow up

• Low Socio-economic patients

• Uneducated patients

• Psychiatry patients

Following are not contraindications:


• Family history

• Central/ subareolar tumor

• Node+

SURGERY SIXER APP BASED WORK-BOOK 2020 154


BREAST RECONSTRUCTION SURGERY

• Can be done immediately (if RT is not needed)

Autogenous tissues Implants (Silicon) – kept in sub pectoralis layer


• TRAM flap ( previous decade) Indications:
• LD flap If Patient wants
• Gluteus maximus muscle ▪ Augmentation of breast
• DIEP flap ( latest) ▪ B/L breast
reconstruction ( don’t
have enough tissue for
graft)
Contraindications:
▪ Silicon allergy
▪ If you are planning
radiotherapy ( since it
will rupture)

TRAM flap (Transverse Rectus Abdominis Muscle flap)


• Pedicle – superior epigastic artery based

• Free flap – inferior epigastric artery based

o Require microvascular anastomoses

• Disadvantage: weakness in abdominal wall

DIEP ( Deep Inferior Epigastric Perforator ) flap


• (Skin + subcutaneous + fat) Free flap – Inferior epigastric artery based ( from pubic

region)

• Require microvascular anastomoses

SURGERY SIXER APP BASED WORK-BOOK 2020 155


Figure: Flaps to reconstruct

Adjuvant Therapy

Chemotherapy Hormone Therapy Radiotherapy


To prevent micro mets For ER+ cases To prevent local recurrence (
and distant mets To prevent cancer in opposite since pectoralis major is left
breast behind)

Indicated in Indicated for: Indicated in


• >1cm • ER + premenopausal(ovary • Locally advanced
• Node + produces estrogen) breast cancer
• Anti oestrogen – • Chest wall+
Tamoxifen/ SERM • To axilla if
• ER+ Post-menopausal o <10 nodes are removed
(oestrogen form peripheral o Extracapsular spread
conversion of androgen to o No axillary dissection done
oestrogen)
• Aromatase inhibitor-
Anastrazole, Letrazole

• MC Cause of Lymphedema in Western Countries is due to RT to axilla** which can

lead to Stewart Treves Syndrome** ( Lymphangiosarcoma)

SURGERY SIXER APP BASED WORK-BOOK 2020 156


Chemotherapy:
✓ CAF - 6 cycles ( when given as adjuvant)

• Cyclophosphamide

• Adriamycin

• 5 FU

✓ Neoadjuvant _-3 cycles before surgery

✓ If her2 + - Herceptin (trastuzumab)- anti her2 antibody

Hormone therapy
✓ Tamoxifen – 10 mg BD x 5 years ( min) – 15 yrs ( max)

o Antagonist in breast

o Agonist on endometrium

o Mortality decreased by 30%

o Recurrence in opposite breast decresed by 40%

➢ Side effects:

o Endometrial cancer

o Thromboembolic manifestation

o Bone pain

o Increase incidence of cataract

➢ Added advantages

o Decreased osteoporosis ( Agonist in bone)

SERM: Selective Estrogen Receptor Modulators


o Raloxifene

o Tormefene

Other Cancer Breast related Lesions

Paget’s disease:
• Clinical Features: nipple discharge and nipple erosion

• Differential Diagnosis: Eczema

Eczema Paget’s disease


B/L U/L
Associated with skin lesions + -
Responds to steroids -

SURGERY SIXER APP BASED WORK-BOOK 2020 157


• Superficial manifestation of underlying breast cancer

• 40-50% patients we can palpate a lump

• If lump is negative- take a nipple scrap biopsy-

o Shows Paget’s cells – pale, vacuolated cells in epithelium

o It comes under Tis

• Treatment – do MRM

Inflammatory breast cancer ( T4d)


O/E :
• Mastitis like appearance

• Paeu de orange+

• Redness+

• Extensive subdermal lymphatic infiltration +

❖ Immediately start on neoadjuvent chemotherapy and do MRM

❖ Bad prognosis

Male breast cancer


Predisposed by
• BRCA 2 mutation

• Klinefelter syndrome

• Testicular feminization syndrome

NOTE: Gynaecomastia is not a predisposing factor

Figure: Male Breast Cancer

Management:
• TNM staging is same like females

• Indepth infiltration is more common

• No lobular cancer seen in males – only ductal cancer

• Investigations are same

SURGERY SIXER APP BASED WORK-BOOK 2020 158


• Biopsy by trucut biopsy

• Treatment is also same – MRM

• After MRM – study for ER

o If ER + ( mostly) – give them tamoxifen

NOTE:
• Gold standard investigation to look for mets – PET- CT ( for all LABC)

• Replaced by old investigations like bone scan and x ray flat bones.

Chapter 3C- Miscellaneous Diseases

Nipple discharges:
❖ Bloody-

o Young – duct papilloma

o Old age – CA breast

❖ Greenish yellow – duct ectasia

❖ Milky discharge

o Physiologic

o Pathologic – drugs

Surface discharge form breast – Eczema and Paget’s disease


Eczema Paget’s disease
B/L U/L
Associated with skin lesion + -
Responds to steroids -

Duct papilloma Duct ectasia( no pain)


Papilloma inside duct Duct is abnormally dilated
Young female Middle aged female
Bloody discharge Greenish yellow discharge
Periductal mastitis (Zuska disease)**
Pain +
Predisposed by smoking females
Ductography
Filling defect Dilated ducts
Treatment Hadfield’s operation
• Tennis racket shaped incision ( on skin) - conical excision of all ducts with apex at
and microdochectomy nipple
- Radical approach

SURGERY SIXER APP BASED WORK-BOOK 2020 159


- No lactation possible hereafter from this
breast
- Done only for incurable periductal mastitis

ANDI ( ABERRATION IN NORMAL DEVELOPMENT AND INVOLUTION OF BREAST)


Non proliferative lesions Proliferative lesions
Not premalignant No risk of malignancy Risk of malignancy
Duct ectasia Sclerosis Atypical ductal hyperplasia
Fibro adenosis Radial scar Atypical lobular hyperplasia
Cyst disease Fibroadenoma Atypical epithelial hyperplasia
Apocrine changes Duct papilloma

Fibroadenosis
• Young females

• Clinical features:

o Lumpiness in breast

o Painful cyclical mastalgia ( preovulation)

o Fullness of breast ( pre menstrual)

• Treatment : reassurance and analgesics

o oil of evening prime rose

o danazol

o tamoxifen

Fibroadenoma
• Most common benign lesion of female

SURGERY SIXER APP BASED WORK-BOOK 2020 160


• Most common <30 yrs

• 2nd MC tumor next to ca breast

• c/f: breast mouse ( Indian rubbery firm consistency mass)

• no pain

• Giant fibroadenoma = >5cm in size

• Types : pericanalicular and intracanalicular

Peri canalicular Intracanalicular


Hard fibroadenoma Soft
m/c type -
Painless
Young females (<30 yrs) 30- 50 yrs
Breast mouse Adherence to surrounding tissue
Slow growing Rapidly growing
Treatment:
Enucleation Excision
Or excision

• Management:

o Cosmetic incision

▪ Subareolar incision

▪ Gaillard Thomas submammary incision

o Non cosmetic – radial incision

• Mammography shows – Pop corn calcification

Figure: Popcorn Calcification

Mondor’s disease
• Superficial vein thrombophlebitis

SURGERY SIXER APP BASED WORK-BOOK 2020 161


• MC vein – lateral thoracic vein

o Thoracodorsal vein

o Superior epigastric vein

❖ C/F : painful, tender, cord like ( DDX : lymphatic permeation by tumor)

❖ Management: rest shoulder , arm restriction with analgesics

❖ If pain persists – excision

Figure: Mondor’s Disease

Phyllodes tumor:
• Arising from the intervening tissue of breast / supporting tissue

• Reason:

o Leaf like appearance on cutting

o USG – cystic spaces( hence also called as cystosarcoma phyllodes)- misnomer

• C/F : painless , massive size

o Benign, borderline, malignant

• No lymph node enlargement

• No lymphadenectomy required

• Treatment : wide local excision / simple mastectomy

SURGERY SIXER APP BASED WORK-BOOK 2020 162


Figure: Phyllodes Tumor

Gynecomastia – enlargement of male breast


SIMON’S GRADING
I. MILD

II. MODERATE

a. No skin redundancy

b. Skin redundancy

III. Marked with skin redundancy and ptosis of breast tissue

Causes:
I. Physiological – puberty

II. Pathological:

a. Estrogen excess

• Tumors in testis

• Endocrine disorders

• Cirrhotic liver

• Klinefelter’s syndrome

• Testicular feminization syndrome

b. Androgen deficient

• Hypogonadism

• ACTH deficiency

• B/L orchidectomy

c. Testis failure

• Orchitis

• Trauma

• Undescended testis

• Radiotherapy

d. Drugs: (DISCKO FM)

• Digitalis and Di ethyl stilbesterol

• Isoniazid

• Spironolactone and steroids

• Cimetidine

• Ketoconazole

• Oestrogen

• Flutamide and frusemide

SURGERY SIXER APP BASED WORK-BOOK 2020 163


• metronidazole

Figure: Gynecomastia

Management:
• Testosterone

• Danazol

• Treat the cause

Surgical :
• Liposuction

• Resection by sub mammary/ subareolar incision-

o Webster operation**

• Not precancerous except secondary to klinefelter syndrome.

Mastitis in lactating women:


• Most common – Staphylococcus aureus ( from baby’s saliva)

• Lead to breast abscess

o Latest treatment – USG guided aspiration of abscess**

o Can do repeated aspiration

• Avoid feeding from infected breast

• But express milk by pump

• Treatment : Cloxacillin with tight support for breast.

SURGERY SIXER APP BASED WORK-BOOK 2020 164


Figure: Mastitis going for Abscess

SURGERY SIXER APP BASED WORK-BOOK 2020 165


Chapter 4- Hernia

4a- Anatomy of Hernia

Inguinal canal/ House of Bassini


• Extending from deep ring to superficial ring

• 4cm

Deep ring Superficial ring Saphenous opening


• Present in fascia • Present in external • 4cm below and
transversalis oblique aponeurosis lateral to pubic
• Oval shaped • Triangular tubercle
• ½ inch above mid • Above and lateral to • Opening of
inguinal point pubic tubercle femoral canal

Femoral canal
• 1.75 cm in length

• Extends form femoral ring to saphenous opening

• Medial most compartment of femoral sheath

• Boundaries:

o Anteriorly- inguinal ligament

o posteriorly- iliopectineal ligament of cooper

o Medially – Lacunar ligament

o Laterally – femoral vein

• Content: Cloquet (or) Rosenmuller Lymphnodes

• Femoral hernia does not go inferiorly: due to HOLDEN’S LINE

o HOLDEN’S LINE: attachment of deep membranous layer of superficial fascia of

abdomen with fascialata of thigh

o Therefore femoral hernia is retort shaped.

Hesselbach’s triangle
• Boundaries:

o Laterally- Inferior epigastric artery

o Medially- Rectus abdominis

o Inferiorly- Inguinal canal

• Hernia here is direct hernia

• Indirect hernia goes from deep to superficial ring.

SURGERY SIXER APP BASED WORK-BOOK 2020 166


Indirect Hernia Direct Hernia
Young age Old age
Congenital / acquired Mostly acquired (except ogilvie)
Due to preformed sac. MC in Males
MC in Males Predisposed by
▪ Collagen vascular disorder
▪ Smoking
▪ Increased intraabdominal pressure
Eg. Constipation, difficulty in micturition,
BPH, chronic cough- COPD

Course:
Deep ring to superficial ring
Pyriform shaped Through hesselbach’s triangle
• Sac is lateral to inferior epigastric Hemispherical shaped
artery Sac is medial to inferior epigastric artery
• Sac is anteriolateral to cord structure Sac is Posteriomedial to cord structure

Treatment: • NO herniotomy (just push the sac


• Herniotomy + inside)
• Herniorrhaphy/hernioplasty • Only hernioplasty

Complications: Less chance of obstruction


High chances of obstruction (due to wide neck of sac)

SURGERY SIXER APP BASED WORK-BOOK 2020 167


In females:
• Indirect hernia is most common hernia in females.

• Direct hernia never occurs

• Femoral hernia is more common in females compared to males.

Males:
• Indirect hernia is most common hernia.

Contents of cord:

1. Vas deferns/ Round ligament of uterus


2. 3 Arteries:
• Testicular artery
• Artery to vas
• Artery to cremaster
3. Veins: corresponding veins forms pampiniform plexus
4. Nerves
• Genital branch of genitofemoral nerve
• Ilioinguinal nerve
• Iliohypogastric nerve

Anatomy of hernia sac:


• Has Mouth, Neck, Body, Fundus

• Neck less hernia- Direct hernia

o Therefore never goes for strangulation

Classification based on extent of sac:


1. Vaginal or complete hernia: sac extends till base of scrotum

2. Incomplete/ Funicular hernia: sac extends till root of scrotum

SURGERY SIXER APP BASED WORK-BOOK 2020 168


3. Bubonocele : Hernia is confined to inguinal canal.

Contents of sac:
Enterocoele Omentocoele
Contains bowel Contains omentum
Reducibility:
Initially: difficult Initially: easy
(due to narrow neck)
Later: easily reducible with gurgling sound Later : difficult
Due to adhesion of omentum to fundus
Elastic in consistency Doughy consistency

Clinical examination:
• Cough impulse:

▪ Sign of reducible hernia

▪ Absent in irreducible / obstructed hernia/ in dense adhesions

Tests done:
1. Three finger test( ZIEMANN’S TEST)

• Ask patient to cough. Impulse felt in

o Index finger- indirect hernia ( deep ring)

o Middle finger – direct hernia ( superficial ring)

o Ring finger – Femoral hernia ( saphenous opening)

2. Deep ring occlusion test ( DRO test)

▪ Occlude deep ring with thumb

▪ Ask patient to cough

▪ Swelling appears- direct hernia

▪ Swelling doesn’t appear – indirect hernia

False positive test:


o Deep ring is wide

o Pantaloon hernia

3. Finger invagination test:

• Done only in males

• Not possible in females due to absence of lax skin

• Invaginate little finger from scrotum towards superficial ring

• Ask patient to cough

SURGERY SIXER APP BASED WORK-BOOK 2020 169


• Impulse felt at tip – indirect hernia

• Impulse felt at pulp – direct hernia

o Other advantages of invagination test:

▪ Detect posterior wall weakness

▪ Superficial ring laxity is noted

Malgaigne’s Bulges
• B/L pyriform shaped swelling in inguinal region in old patient when asked to lift head

• Present due to muscle weakness

• Treatment of choice : Hernioplasty

Classification of hernia:
NYHUS:
TYPE 1 TYPE 2 TYPE 3 TYPE 4
INDIRECT HERNIA INDIRECT HERNIA POSTERIOR WALL RECURRENT HERNIA
DEFECT
Deep ring normal Deep ring is dilated Direct hernia
Pantaloon hernia
Femoral hernia

GILBERT CLASSIFICATION: 7 TYPES


1. Indirect + small defect

2. Indirect + medium defect

3. Indirect + large defect

4. Entire floor defect ( posterior wall ) – direct

5. Diverticular type- Direct

6. Pantaloon

7. Femoral

European Hernia Society Classification:


• Primary / Recurrent

• Lateral / Medial / Femoral

• Defect : 1 finger = 1.5 cm

o Eg. PL2 hernia = primary lateral 2 finger ( 3 cm defect)

SURGERY SIXER APP BASED WORK-BOOK 2020 170


MCQ points:

o Lumbar hernia – only natural weakness seen in the body


o Surgical scar : max 70% of initial strength ( irrespective of duration of healing)
o Weight lifting doesn’t cause hernia
o Obesity is inversely related to hernia ( Inguinal Hernias)
o Collagen vascular disorder, elderly, pregnancy increases incidence of hernia.

Management of Hernias:
TRUSS – hernia belt ( Not advised nowadays)
o Used only in reducible small hernia

o Disadvantage: dense adhesions will form – make surgery difficult

Figure: TRUSS

• Congenital hernia:

o Herniotomy

o Identify sac – preperitoneal pad of fat

o Dissect the sac and remove

o Ligate at neck

• Adults:

o Herniotomy + herniorrhaphy / hernioplasty

Herniorrhaphy
• No mesh- only suture is used

• Done only in emergency nowadays– due to increased chances of mesh infection

SURGERY SIXER APP BASED WORK-BOOK 2020 171


Types:
• Original Bassini – approximate conjoint tendon with inguinal ligament ( intermittent

sutures)

• Modified Bassini – Continuous sutures to approximate conjoint tendon with inguinal

ligament.

• Maloney’s Darning procedure: ( best – results equal to mesh repair)

o Dense continuous sutures between conjoint tendon and inguinal ligament.

Sutures themselves looks like a mesh.

• Shouldice repair- Double breasting of fascia transversalis

• Mc Vay’s repair- approximate Conjoint tendon with iliopecteal ligament of cooper. (

femoral hernia is also cured)

Hernioplasty
Gold standard- Lichtenstein tension free mesh repair
• Treatment of choice for direct hernia

(as in direct hernia – posterior wall is already weak- if we put sutures it becomes further
weak)

Also remember:
Lytle repair- narrow the deep ring with prolene sutures (in case of wide deep ring)
Tanner’s slide:
• It’s a incision made on anterior rectus sheath to relieve tension ( when bassini was

used – patient used to bend to that side due to tight sutures)

Femoral hernia surgeries:


• 20% patients have abnormal obturator artery medial to lacunar ligament

• Most common presentation – obstruction at lacunar ligament

Therefore 1st step- relieve obstruction


• Dissect lacunar ligament (don’t injure abnormal obturator artery)

• Then place a mesh between inguinal and cooper’s ligament

Skin incisions in femoral hernia:


1. Vertical incision – Mc Evedy’s operation

• for strangulated hernia

• Can also give access to peritoneum if required

2. Inguinal approach of Lotheissen

• Incision along inguinal ligament

SURGERY SIXER APP BASED WORK-BOOK 2020 172


3. Low incision of Lockwood ( along groin crease )

4. Midline incision- Henry’s procedure

• Midline extraperitoneal femoral hernioplasty

• Treatment of choice for femoral hernia

Other named Surgeries in Hernia:


GPRVS- Giant Prosthesis Reinforcement Visceral Surgery( STOPPAS repair)
• Huge mesh kept preperitoneally after reducing the hernia

• No sutures required

• Intraabdominal pressure will seal it.

Gilbert plug mesh


• Plug a mesh into the defect

Meshes in Hernia Repair

1. Mesh types

• NET (can be woven or knitted)

✓ Adherence is easy ( tissue enters mesh- adherence due to friction)

✓ Fixation may or may not be done

• Sheet

✓ Adherence is not there

✓ Fixation is must to prevent mesh migration

SURGERY SIXER APP BASED WORK-BOOK 2020 173


2. Materials used for mesh

I. Polypropylene mesh

✓ White colour

✓ Fix with 2.0 prolene / tackers

✓ Hydrophobic (Hence Prevents Bacterial ingrowth**)

✓ Prevents bacterial growth due to hydrophobic nature

II. Polyester mesh

✓ Hydrophilic

✓ Increases bacterial growth but also increased anti inflammatory cells

reaching there.- so it can be used and nothing to worry.

III. PTFE (Poly Tetra Fluoro Ethylene)

✓ Flat sheet

✓ No tissue ingrowth

✓ Used as non adhesive barrier between layers

3. Weight of mesh

Light weight= 40 g/m2 Heavy weight mesh = 80g/m2

o Better tissue integration o Goes for contracture


o Larger pores o Other features are opposite to light
o Less shrinkage weight
o Better comfort

4. Biological mesh

• They have a scaffold to encourage neovascular ingrowth and collagen deposition.

• Expensive

• Derived from

✓ Human / animal dermis

✓ Bovine pericardium

✓ Porcine submucosa

• Contraindicated in infection (mesh will be dissolved)

• Prevents attachment of bowel to mesh

• Used in burst abdomen

5. Absorbable mesh

• Used for temporary closure of abdomen in burst abdomen

SURGERY SIXER APP BASED WORK-BOOK 2020 174


• No role in hernia repair

• Made up of polyglycolic acid = also called as “VICRYL MESH”

6. Dual mesh/ Tissue separation mesh

• One side by prolene ( towards skin)

• Other side by vicryl/ collagen/ PTFE / cellulose ( visceral side)

• Prevents bowel adhesion

Mesh placement techniques:


1. Onlay mesh – between skin and anterior rectus sheath

2. Inlay mesh- between the muscle layers

✓ Sublay intraparietal – between rectus abdominis and posterior rectus sheath

✓ Sublay extraperitoneal – between oblique mucles and fascia transversalis

3. Intraperitoneal- along the posterior surface of peritoneum ( Inside the Peritoneal cavity)

✓ Use only dual mesh

• IPOM- Intraperitoneal Onlay mesh using DUAL mesh is done by Laparoscopic


surgeons commonly in practice

NOTE:
• In open surgeries – we can use inlay/ onlay/ intraperitoneal mesh

• In laparoscopy- use only intra peritoneal / sublay extraperitoneal mesh.

• Overlap of mesh- 2 – 5 cm on all sides (beyond the defect)

Complications of hernia
1. Irreducible hernia

SURGERY SIXER APP BASED WORK-BOOK 2020 175


• Hernia is not going back inside (adhesion)

• Not emergency – but plan for surgery

• Can become obstructed hernia

2. Obstructed hernia

• Bowel is viable

• Emergency

3. Strangulated hernia

• Bowel is not viable

Management of obstructed hernia:


▪ Take a inguinoscrotal incision

▪ Open the fundus and let the toxic fluid drain out

▪ Cut the constriction point

▪ Look for viability of bowel-

o Mesenteric artery pulsation

o Peristalsis

▪ If no pulsation and peristalsis + bowel looks bluish

o Keep a warm pad on bowel + 100% O2

o If pulsation appear- reduce the content

o And do hernioraphy

Complications of hernia surgery:

• Surgical site infection


• Nerve injuries

Most common nerve injured

• Laparoscopic hernia repair – lateral cutaneous nerve of thigh > Genito


femoral nerve
• Open repair- ilioinguinal nerve> Genital branch of gentiofemoral nerve
• Ischaemic orchitis due to testicular artery injury ( most serious complication)
• Injury to vas deferns
• Recurrence are most common in direct hernia repairs – most commonly near the floor.

SURGERY SIXER APP BASED WORK-BOOK 2020 176


4b Types of hernia

1. Sliding inguinal hernia

• Sac + intraperitoneal structures + some retroperitoneal structures ( eg..caecum,

descending colon)

• Most common on left side

• Most common content sigmoid colon

• If on right side- content is caecum (MC)

Management:
• Dissect sac and ligate

• Posterior dissection is not advised – to prevent injury to retroperitoneal organs.

2. Maydl’s/ W shaped hernia

• Most distal part is B- present inside the abdomen (more prone for ischaemia)

• If we see A and C being viable and push bowel inside

▪ Delayed bowel leak from B can occur

▪ Retrograde strangulation

Figure: Maydl’s hernia

3. Scrotal abdomen

▪ Huge indirect hernia

▪ May be associated with sliding hernia

4. Richter’s hernia

▪ Seen in femoral and obturator hernia

▪ One circumference of bowel is obstructed

▪ Fluid is secreted in response- hence presents as diarrhea

SURGERY SIXER APP BASED WORK-BOOK 2020 177


5. Internal Hernia
▪ Stammer’s Hernia : hernia through defect in transverse mesocolon

▪ Peterson hernia: hernia behind Roux limb

Gaps in ligaments causing hernia

▪ Ogilvie: Gap in conjoint tendon. Only congenital direct hernia


▪ Laguier’s hernia: Lacunar ligament
▪ Beclard hernia: Type of femoral hernia with defect in saphenous vein area
▪ Narath Hernia: Gap due to congenital dislocation of hip.
▪ Berger’s Hernia : Hernia in pouch of Douglas. Also called as sigmoidocoele.

Sportsman Hernia:
o No swelling

o No cough impulse

o Seen in rugby/ football players

o Young men

o C/F-

▪ Pain at inguinal region extending to scrotum and upper thigh

▪ ( DD- rupture of adductor tendon, Gilmores groin( inflammatory pain) )

o Investigation of choice- MRI

o Treatment- Laparoscopic repair.

Ventral Hernia

Hernia on the ventral surface of the body


• Umbilical and paraumbilical

• Epigastric

• Incisional

• Parastomal

• Spigelian hernia

• Traumatic

• Lumbar (Dorsolateral hernia)- Though coming under Ventral hernia it is seen on the

Dorsum side.

Though Inguinal and Femoral are seen ventrally – they are not put under this classification**
( So you can get in exam as EXCEPT)

SURGERY SIXER APP BASED WORK-BOOK 2020 178


1. Umbilical hernia

In children:
▪ Congenital ( umbilical ring fail to close)

▪ Defect is not so tight

▪ No strangulation

▪ Wait for 2 years

▪ Ideal age for surgery : simple anatomical repair

✓ 2 years ( NEET- FROM B&L)

✓ 5 YEARS ( AIIMS – From sabiston)

Mayo’s repair: “Pant on vest repair” – overlapping of rectus sheath . Not performed now.

In Adults:
• Abdominal pathology-

✓ Obesity

✓ Cirrhosis and ascites

✓ Pregnancy

• Strangulation is common in adults

• <2cm – wait and watch

• >2cm – surgery

✓ Anatomical repair

✓ Laparoscopic repair

✓ Mayo’s repair

2. Epigastric hernia of Linea alba/ Ghost hernia/ Sacless hernia/ Fatty hernia of linea alba

• Defect is transverse

• Preperitoneal pad of fat comes out first

• Undergoes strangulation – hence presents with pain

• Pain mimics peptic ulcer disease

• Young males (25-40 years)

• Cough impulse may or may not be present (based on strangulation)

• Ghost/ Sacless hernia – only preperitoneal fat pad comes ( no peritoneum)

• Management-

o Open entire linea alba while operating – look for minor defects

o Recurrent due multiple defects missed during previous surgery

o Adult- close with non absorbable surtures

o Child – absorbable suture

SURGERY SIXER APP BASED WORK-BOOK 2020 179


3. Incisional hernia

• 10-50% incidence

• Port site hernia (laparoscopy)- 1-5%

• Broad necked – less risk of strangulation

• We can operate electively

• Factors causing incisional hernia.

Surgery related Surgeon factors Patient factors


Wound infection Poor technique Old age
mc- midline> paramedian Used absorbable sutures ( Malnutrition
instead of prolene)
Mc – emergency Surgery> Excessive tension Diabetes
elective surgery
Jenkins rule – length of Obesity
suture material to be used
=4x length of incision
Ascites
Steroids
smoking
Cough/ vomiting – post op

Management
▪ Linea alba closure – poly prolylene/ poly-diaxonone (Longest absorbable suture)

o Continuous sutures used

o Intermittent sutures used ( Give less gap < 1cm between Sutures)

Burst abdomen happen on 6th day post op.

Treatment:
• Mesh repair

o Onlay technique ( by open method)

o Laparoscopic -IPOM ( intra peritoneal Onlay Mesh)- Dual mesh is used.

(Errata in Video : Sublay is kept above the peritoneum below the rectus
muscle*)
o e-TEP ( extended – Total Extraperitoneal Repair)

o Abdominoplasty ( remove excess fat)

4. Spigelian hernia

SURGERY SIXER APP BASED WORK-BOOK 2020 180


• Occur in spigelian fascia(connecting xiphi sternum to supra pubic)

• Inter parietal hernia- between muscles

• Occur at any age

• Most common site- infra umbilical

• Seen mostly above arcuate line** > than below Arcuate line

• C/F-

o Young age- pain + and Lump may or may not be palpable

o Old age – Pain seen with Cough impulse+ and defect palpated.

• IOC : CT abdomen/ USG

• High incidence of strangulation

• Repair:

o IPOM

o Open method- close the defect

Note: hernia with decreased incidence of strangulation:

• Umbilical hernia
• Incisional hernia

5. Lumbar hernia:

Superior lumbar triangle boundaries Inferior lumbar triangle( Petit’s triangle)


boundaries:
( Grynfelt Hernia- Rare) ( Petit’s Hernia – Most common)
• Superio lateral: 12th rib • Superiolateral : External oblique (
• Inferio lateral : internal oblique defect)
muscle • Inferiolateral: iliac crest
• Medially : sacrospinalis tendon of • Medially : Lattisimus dorsi
erector spinae

Treatment: DOWDPONKA OPERATION

SURGERY SIXER APP BASED WORK-BOOK 2020 181


Figure: Lumbar Hernias

6. Parastomal hernia

• Most commonly seen after end colostomy ( 50 % cases)

• To prevent this we keep a mesh

• SUGAR BAKER PROCEDURE (MESH- BOWEL – MESH)

7. OBTURATOR HERNIA

• Occurs through obturator foramen

• Mc in elderly females

• C/F-

o swelling in scarpa’s triangle of thigh

o Presents as strangulation

o Flexion ,abduction and external rotation of hip – makes swelling visible.

• Howship Romberg sign: Referred pain to knee ( due to obturator nerve

compression)

• Hannington kiff sign: absent obturator reflex

• Treatment: Posterior approach and keep a mesh

SURGERY SIXER APP BASED WORK-BOOK 2020 182


4c. LAPAROSCOPIC ANATOMY OF HERNIA

• Triangle of Doom :

o Apex- deep inguinal ring

o Medial- vas deferns

o Laterally – gonadal vessels

o Base – reflected peritoneum

▪ Content :

✓ External iliac artery and vein

▪ When we damage the vessels here person may die ( hence doom)

• Triangle of Pain:

o Laterally- iliopubic tract

o Medially- gonadal vessel

o Base- reflected peritoneum

o Contents:

✓ Lateral cutaneous nerve of thigh

✓ Anterior cutaneous nerve of thigh

✓ Femoral branch of genitofemoral nerve

✓ Femoral nerve

o Importance of this triangle:

✓ If trackers are put below iliopubic tract – it causes severe pain

✓ Most common – lateral cutaneous nerve of thigh – meralgia paraesthetica

Figure: Triangle of Pain and Doom


Laparoscopic hernia repair techniques:

SURGERY SIXER APP BASED WORK-BOOK 2020 183


TEP( TOTAL EXTRA PERITONEAL REPAIR) TAPP ( TRANSABDOMINAL PRE
PERITONEAL REPAIR)
ADVANTAGES: ADVANTAGES:
o Peritoneum not opened o Easy to do
o No paralytic ileus o Short learning curve
DISADVANTAGES: DISADVANTAGES:
o Long learning curve o Paralytic ileus
o Bowel injury

Extra Edge:

o Space of Retzius – Retropubic space ( SPACE BETWEEN THE BLADDER AND PUBIC
SYMPHYSIS)
o Space of Bogros – Retro inguinal space ( SPACE BEHIND THE INGUINAL CANAL
ANTERIOR TO PERITONEUM)
o We use these two spaces to do laparoscopic surgeries.

CORONA MORTIS/ CIRCLE OF DEATH:


• Formed by

o External iliac artery

o Internal iliac artery

o Obturator artery

o Abnormal obturator artery

o Inferior epigastric artery.

SURGERY SIXER APP BASED WORK-BOOK 2020 184


Abnormal obturator artery ( present in 20% patients) is required to complete corona mortis
Injury to it may lead to death .

Myopectineal orifice of Fruchad


o Medially – rectus abdnominis

o Laterally- iliopsoas

o Superiorly- conjoint tendon

o Inferiorly – pecten pubis

Application : all types of hernia can be prevented by keeping a mesh here.

NOTE:
o Treatment of choice for mesh infection in laparoscopic repair – laparoscopic removal of

mesh.

SURGERY SIXER APP BASED WORK-BOOK 2020 185


Space for Additional Points:

SURGERY SIXER APP BASED WORK-BOOK 2020 186


Section C - Gastrointestinal Surgery

Chapter 1: Esophagus
• 1a- Motility Disorders
• 1b- GERD, Hiatal hernia
• 1c- Miscellaneous
• 1d- Tumors in esophagus

Chapter 2: Stomach and Bariatric Surgery


• 2a- Peptic Ulcer disease
• 2b- Cancer stomach
• 2c- Miscellaneous
• 2d- Upper GI bleeding
• 2e- Bariatric Surgery

Chapter 3: Intestines
• 3a- Introduction
• 3b- Benign Topics intestines
• 3c- Small Intestine Tumors
• 3d- Inflammatory Bowel diseases
• 3e- Polyps
• 3f- Carcinoma Colon
• 3g- Appendix
• 3h- Intestinal obstruction

Chapter 4: Rectum and Anus


• 4a- Benign diseases
• 4b- Malignant diseases

Chapter 5: Miscellaneous- Mesentry and Peritoneum

SURGERY SIXER APP BASED WORK-BOOK 2020 187


Chapter 1: Esophagus

Introduction:
• Length- 25cm
• From Cricopharynx to OG junction
• Narrowest part of entire GIT- Cricopharynx ( 13 mm)- MC site of FB obstruction in GIT.
• Cricopharynx seen at 15 cm on endoscopy.

Narrowest parts in Esophagus- Natural constrictions:


• 15 cm- Crico pharynx
• 25 cm- Arch of Aorta
• 27 cm- Left bronchus
• 40 cm- OG junction
• Esophagus has no serosa ( GB has no submucosa)
• Esophagus – 10th Vertebra level enters diaphragm ( VOA- IVC, Oesophagus, Aorta)
• Normal IAP= 5-8 mmHg, Thoracic Pressure= -4 mmHg
• This increased Intra abdominal pressure is the most important mechanism to prevent
GERD**
• Minimum length of esophagus needed to prevent GERD- 1 cm **
• Angle of HIS- Esophago Gastric Angle.

Investigations in Esophagus:
• Upper GI Endoscopy – Best to study the esophagus. Length of tube =100 cm.
• Upper GI Scopy can pass upto D4.
• MC site of Esophageal injury- Cervical esophagus by Endoscopy

Endoscopy USG:
• IOC for T and N staging of Esophageal Cancer
Layers in Endoscopic Ultrasound: 5 layers are seen:
• There are 5 layers seen in endoscopy.

• Layer 1- Mucosa ( Hyper echoic)

• Layer 2- Muscularis Mucosa ( Hypoechoic)

• Layer 3- Submucosa ( Hyper echoic)

• Layer 4- Muscularis Propria ( Hypoechoic)

• Layer 5- Adventitial layer

Endoscopic appearance of malignant nodes:


• Hypo echoic node

• Well defined border

• Round shape

SURGERY SIXER APP BASED WORK-BOOK 2020 188


• Size > 1 cm

Figure- EUS showing Esophageal Cancer

• LEIOMYOMA arises from 4th Layer on EUS most commonly ( NEET SS Question)
• EUS is least sensitive for T2 ( NEET SS Question)

Physiology of Oesophagus and Motility disorders

Normal phenomenon Abnormal


Peristalsis: Tertiary peristalsis:
• Primary- With swallowing • Without food stimulus
• Secondary – clearing of food pipe, not • Seen in DES and NCE
associated with swallowing
Amplitude: DCI:
Contraction of Oesophagus • Decreased- Achalasia
• Value- 40- 80 mmHg • >450 mmHg.cm.S- DES
• Duration- 4-6 S • >8000mmHg.cm.S- Nutcracker
esophagus
Distal Contractile Integrity (DCI)
• Amplitude X Duration X Cm of
movement
• 450 mmHg.Cm.S

LES: LES pressure> 26 mmHg- seen in Achalasia


• Resting pressure: 10-26 mmHg
• Relax on swallowing

Integrated Relaxation Pressure( IRP) IRP- > 15 mmHg and LES fails to relax -
• Is the new value used for LES Achalasia
relaxation pressure relaxation:

SURGERY SIXER APP BASED WORK-BOOK 2020 189


• Normal value- < 15 mmHg

Achalasia means – failure to relax ( term coined by HURST)

Gold standard IOC for Motility disorder= High Resolution Manometry** (Uses 36 Sensors)
Classification of Motility disorders:
Chicago Classification:
Hypomotility disorders Hypermotility Disorders
Achalasia ( Most common overall) DES
NCE- Most common Hypermotility disorder
Type 1 – Most common
Type 2- Best prognosis
Type 3- Rare type

Achalasia and Esophago gastric junction disorders


Type 1 Achalasia ( Classic) • Mean IRP> 15 mmHg
• 100% failed Peristalsis
• Distal Contractile Integral(DCI) < 100
mmHg.s.Cm
Type 2 Achalasia with Esophageal • Mean IRP> 15 mmHg
Compression • 100% failed Peristalsis
( Old name Vigorous Achalasia) • Pan esophageal pressurization**
• DCI is difficult to measure.
• In Manometry- ISOBARIC Contraction seen at
30 mmHg
Type 3 (Spastic Achalasia) • Mean IRP> 15 mmHg
• No Normal Peristalsis
• Segmental increased DCI seen.
• Spastic Contractions with Distal Contractile
Integral(DCI) > 450 mmHg.s.Cm
• Rarely Pan esophageal pressurization seen
Achalasia in Evolution • IRP >15 mmHg
(Esophago Gastric Outlet • Peristalsis and DCI looks normal.
Obstruction)- NEET SS
Major disorders of peristalsis
Distal Esophageal Spasm • Mean IRP- Normal
• DCI>450 mmHg.S.Cm
• > 20 premature contractions
Jackhammer Esophagus • DCI >8000***for atleast 2 swallows**
( Hypercontractile Esophagus) • IRP is normal

SURGERY SIXER APP BASED WORK-BOOK 2020 190


Figure- HRM image

Achalasia Cardia
Clinical features:
• MC in young females
• MC Symptom – Dysphagia ( both Solids and Liquids – More for Liquids); Slow Progressive
type
• Triad of Achalasia- Dysphagia+ Weight loss+ Reflux
• Nocturnal Asthma ( Also seen in GERD cases)

Investigations:
• Barium Swallow- Bird Beak appearance, pencil tip , Abrupt Cut off
• Hurst Phenomenon – Flow of contrast beyond bird beak due to contrast load and weight

Figure: Bird Beak

Figure Hurst Phenomenon

SURGERY SIXER APP BASED WORK-BOOK 2020 191


Please note- Rat tail is a feature of cancer esophagus. ( Irregular Narrowing in Esophagus)

• X ray chest-Presence of mediastinal gas fluid level behind heart shadow


• X ray abdomen- Absent Fundus Gas shadow**
• IOC- High Resolution manometry ( Discussed above)

Ganglion cells in Achalasia Vs Hirschsprung disease


• Achalasia:
- Dilated part has absence of ganglion cells in Auerbach plexus
- Constricted part is normal

• Hirschsprung disease:
- Constricted part is having absence of ganglion cells in both Auerbach Plexus and
Meissner plexus
- Dilated part is normal

Complications of Achalasia Cardia:


• MC complication- Reflux Esophagitis
• Aspiration
• MC Tumor- SCC ( 8%)**
• Adenocarcinoma- Below Air fluid level ( NEET SS question)
• Sigmoid Mega oesophagus

Management:
Medical management Endoscopic Surgical
• CCB ( Nifedipine) • Botox injection • Modified Heller’s cardio
• Sildenafil citrate • Pneumatic balloon myotomy- Gold Standard
Dilatation Treatment
• Recent advance- POEM** • Myotomy done
( Per oral Endoscopic Anteriorly- by
Myotomy)- Best method for laparoscopically from OG
Type 3 Achalasia ( NEET SS) junction 1-2 cm below
and 6-8 cm above.
• Complication is GERD-
hence we also do
fundoplication .
• The best method for
fundoplication is DOR’S
type ( Anterior 180
degree)

SURGERY SIXER APP BASED WORK-BOOK 2020 192


Perforation of Esophagus during Surgery:
• Close the rent of mucosa alone and do a fundoplication anteriorly by DOR method which will
act as a Buttress to repaired mucosa

ECKARDT Score- Post treatment score of Achalasia to assess the response.

Diffuse esophageal spasm Nut cracker esophagus


• Middle aged female • MC hypermotility disorder
• C/F- Dysphagia+ Chest pain • M=F
• Aggravated by Cold fluids and stress • C/F- Mimics like MI and usually admitted
• Treatment is Medical- CCB in ICU
• Surgery can be done- Long Esophago • No surgery advised.
myotomy is advised upto Carina • Only medical treatment with CCB
• Barium Swallow- CORK Screw Esophagus • Barium swallow- PULSION or
EPIPHRENIC Diverticulum

Figure- Cork Screw Esophagus

Diverticulum in Esophagus

Kilian’s Dehiscence:
• Defect in Inferior Constrictor muscle between Thyropharyngeal and Cricopharyngeal muscle.
• Diverticulum via this defect is Zenker’s diverticulum

Zenker’s Diverticulum
Pathophysiology:
• UES- made of Cricopharynx
• Resting pressure is 40-60 mmHg
• Relaxes only for 2-4 seconds
• In old age, the UES is very tight ( CP Spasm or CP achalasia)- Results in Zenker.

SURGERY SIXER APP BASED WORK-BOOK 2020 193


Clinical features:
• MC in old men, MC in left side
• MC symptom- Dysphagia
• MC complication- Aspiration
• Other features- Halitosis, Lung abscess etc
• IOC- Lateral Oblique view of Barium Swallow

Treatment of choice- Surgical:


• 1st Step – CP myotomy
• < 2cm – CP myotomy alone is enough
CP myotomy+ Diverticulectomy is the Gold standard

Dohlman’s Endoscopic stapling Technique:


- Indication: > 3cm only
- Contraindication: Size < 3cm or patients having Trismus

Diverticulopexy- For old unfit patients the diverticulum is made upside down and fixed to
Posterior Pharyngeal wall or Prevertebral fascia*

Figure- DOHLMAN Procedure

Traction diverticulum
• MC in middle of esophagus due to Tuberculous nodes
• True diverticulum
• MC on right side
• MC cause in west- Histoplasmosis
• Treatment is Diverticulopexy
• Nowadays even in middle of esophagus MC type of diverticulum is Pulsion diverticulum

Epiphrenic diverticulum:

SURGERY SIXER APP BASED WORK-BOOK 2020 194


• Also known as Pulsion diverticulum
• Seen in last 10 cm in Motility disorders
• It is a False diverticulum

Section 1b- Gastro Esophageal Reflux Disease


• MC disease of GIT
Patho physiology::
• Normal pressure of LES= 10-26 mmHg
• Pressure less than 10 mmHg results in GERD.
• Decreased Intra abdominal length < 1cm is the most important mechanism for GERD.

Causes of GERD:
• Tea Drugs:
• Coffee • A-Anticholinergics
• Alcohol • B-Barbiturates
• Smoking • C-CCB
• Pregnancy • D-Diazepam
• Obesity • E- PG E1 Analogues
• Hight Fat diet

Clinical features:
• MC Symptom- Heart Burn
• MC complication- Reflux esophagitis- MC presentation is Retrosternal Discomfort.
• Noctural asthma / Paroxysmal Nocturnal Cough- Laryngo Pharyngeal Reflux (LPR)

Investigations:
• Gold Standard- 24 Hours Ambulatory pH monitoring
• Sensor kept 5 cm above OG junction
• Reflux of Acid at pH <4 is counted every time.
• Pre requisites to do this test:
o Stop PPI for 7 days before test
o Patient comes in Empty stomach and after catheter kept inside she is sent
back to do routine activities.
o Every time pH < 4 strikes the sensor will get a count.
• With all the values we calculate a score known as DEMEESTER SCORE> 14.7** is
diagnostic of GERD (NEET SS question)
• LPR is detected by 24 Hours Double probe pH monitoring** keeping 2 sensors one at
above OG junction and another at Cricopharynx level.

SURGERY SIXER APP BASED WORK-BOOK 2020 195


Complications of GERD:
• MC reflux esophagitis- IOC to find is by Endoscopy – Has 2 classification Los Angeles and
Savary Miller classifications** ( NEET SS Question)
• LPR causing – Nocturnal Asthma
• Peptic stricture- MC in lower end of esophagus, Thick stricture involving all layers and
hence needs surgical excision and keep Jejunal Conduit interposition,
• Schatzki’s ring- MC in Squamo-Columnar Junction ( Biospy above the ring is Squamous
epithelium and below is Columnar epithelium), Thin submucosal ring- and hence can be
easily treated by Dilatation+ Fundoplication.
• Barret’s Esophagus- Conversion of Squamous epithelium to Columnar epithelium . The
type of Columnar epithelium is Intestinal Columnar Epithelium with Biospy showing
GOBLET CELLS**. To confirm the dysplasia we must take Biospy at all 4 quadrant at
every 1cm of Barret’s esophagus – Known as SEATLE BIOPSY PROTOCOL
• Adenocarcinoma- 40 times risk in Barret’s esophagus. Incidence = 0.5 to 1% /year. Hence
keep Barret’s cases under Surveillance.

Management:
Initial:
• Conservative management with Life style modification
• 6-8 weeks of PPI given- Responders are given PPI Continuously.
• Non responders to PPI must be tested for Motility disorders by doing High Resolution
manometry.

Impedance manometry:
• IOC to find out GERD in patients who cant stop PPI
• IOC to find out the type of reflux ( Air reflux has high impedance, Solid reflux has low
impedance)
• IOC to find GERD in Bed ridden patients and ICU patients

Indications of Surgery:
• High volume reflux of acid ( good indication for surgery)

• Unable to modify life style (Good indication)

• Psychological distress ( poor indication – they wont be happy after surgery also)

• Poor compliance to drugs due to side effects of drugs.

• Patients with laryngo pharyngeal reflux.

• Complications of GERD

• Patient Having Life expectancy > 7 years

Please remember; Failure of PPI is not an indication for surgery** They must be evaluated for
motility disorders*

SURGERY SIXER APP BASED WORK-BOOK 2020 196


Surgical treatment:
Basics in Fundoplication operations:
• Pull the esophagus into peritoneal cavity atleast 3 cm

• Narrow the hiatus

• Wrap the lower end of esophagus using the fundus.

• Keep 52 Fr Bougie while wrapping

Techniques in surgery:
• 5 ports used.

• Pars flaccida technique – opening the lesser sac along the lesser curvature.

• Before wrap ensure the wrap is free by doing “ SHOE SHINE MANUEVER”**

Named surgeries**:
• Nissen Fundoplication- 360 degree posterior wrap- Can cause GAS BLOAT Syndrome**

• Toupet- 270 degree posterior wrap

• Dor- 180 degree Anterior wrap

• Watson- 90 degree Anterior wrap

• Belsey Mark 4 repair- Thoracoscopic approach . If there dense abdominal adhesions we do

this surgery.

MC complication of Fundoplication- Paralytic ileus> Pneumothorax**

Newer devices used for GERD by Endoscopic method:

• Radio frequency ablation device- STRETTA procedure**


• TIF Esophyx- Transoral Incisionless Fundoplication – A special device ESOPHYX is kept
at OG Junction by Endoscopy.

SURGERY SIXER APP BASED WORK-BOOK 2020 197


• Enteryx Procedure- Endoscopic injection of synthetic implants to enhance LES
strength.
• LINX is a new device made up magnets kept at OG junction to prevent reflux delivered
laparoscopically
• For patients who have shortened esophagus- COLLIS GASTROPLASTY is done.

Barret’s Esophagus- Surveillance: Follow up protocol ( Latest)

• Vagal Sparing Esophagectomy- Done via a stripper passed from below and the mucosa

and submucosa alone stripped.

Hiatal Hernias:
• 4 types:

• Type 1- Sliding

• Type 2- Para esophageal/ Rolling

• Type 3- Mixed

• Type 4- Massive( Along with Stomach- Small bowel, colon etc will go up)

SURGERY SIXER APP BASED WORK-BOOK 2020 198


• In which of these hernias- Angle of HIS is not disturbed?

▪ Type 2 Rolling/ PE hiatus hernia

Salient points:
• MC presentation of Sliding hiatus hernia- GERD

• Sliding hiatus hernia can be electively planned for surgery – Fundoplication

• Rolling Hiatus hernia is an emergency- they can bleed or perforate- immediate gastropexy

must be done.

Section 1c- Miscellaneous topics

Esophageal perforation

Causes:
• Iatrogenic- MC cause ( MC Site- Cervical esophagus)

• Barotrauma ( Boerhaave syndrome)

• Corrosive- Alkali ingestion

• Trauma

Highest mortality of all GIT perforations is with Esophageal perforation by BAROTRAUMA**


Barotrauma (Boerhaave syndrome) Mallory Weiss tear
• Vomiting after a binge alcohol or • Vomiting against a closed LES results

spontaneously against a closed in Mallory Weiss tear

GLOTTIS**

• MC site is lower left posterior • MC site in lesser curve near OG

esophagus junction at Cardia

SURGERY SIXER APP BASED WORK-BOOK 2020 199


• Full thickness rupture • Partial tear

• Mackler’s Triad: Vomiting, Chest pain • Characterised by Hematemesis- Self

and Left side pleural effusion limiting in 90%

• Mediastinitis

• Mortality

• Management- Urgent surgery by • Endoscopic Coagulation is advised

thoracotomy and repair

• Golden period = within 24 hours

primary repair done

• > 24 hours – Primary repair is mostly

impossible – If unhealthy we must do

Esophagostomy and ICD and FJ done

• Healthy bed even after 24 hours –

tissue is healthy we can do primary

repair.

Alkali poisoning Acid Poisoning


Consumed in large amounts Only few sips only they swallow due to severe
pain in mouth
Liquefactive necrosis Coagulative necrosis
Destroy all the layers and can cause Forms eschar and limited to superficial layers
esophageal perforation and no perforation
Esophagus more damaged than Stomach Stomach is more damaged than esophagus
Long term damage continues Long term damage is rare- Only Gastric
• Develops corrosive stricture for 3- 6 antral stricture can develop

months

Cameron Criteria: Non-Operative treatment of Esophageal Perforation:


• Endoscopy Perforation

• Small septic foci

• Draining collection in mediastinum

• Cervical esophagus perforation

Investigations for Esophageal perforation:


• 1st done investigation-X ray Chest – Shows Mediastinal Air known as Naclerio V sign*

• Early and suspected Esophageal perforation IOC- CT thorax with oral GI contrasts.

SURGERY SIXER APP BASED WORK-BOOK 2020 200


Figure: Naclerio V sign

Contrasts used in Esophageal Perforation:


• Non water soluble- Barium

• Water Soluble:

- High Osmolar- Gastrograffin ( Diatrazoate)

- Low Osmolar- Iohexol , Mypaque and Omnipaque

Barium Sulphate contrasts:


• Can cause chemical peritonitis in abdominal cavity and hence Contraindicated in Gastric,

Duodenal, Intestinal perforation.

• It is the contrast of Choice in Esophageal perforation.

• Gold standard Contrast.

• Highest sensitive contrast

• Mostly inert in mediastinum ( Rarely mediastinitis can happen)

Disadvantage:
• The contrast remains in mediastinum for long period and hence causes confusion in future

studies

SURGERY SIXER APP BASED WORK-BOOK 2020 201


• Rarely- Mediastinitis

Gastrograffin:
• 1st used Contrast

• MC used

Disadvantages:
• Less sensitive

• On aspiration patient may get pulmonary edema ( hence contraindicated in Unconscious

patients)

Iohexol:
• Safest of all contrast**

• Little Costly.

Score to predict the mortality in Esophageal perforation- PITTSBURGH SCORE** ( NEET SS)

Causes of Dysphagia:
1. Plummer Winson Syndrome/ Patterson Kelly Syndrome:
• Mc in 20-40 years females

• Presents with iron deficiency anemia+ Post cricoid web+ Dysphagia

• This phenomenon is known as Sideropenic Dysphagia*

• Predisposes to Esophageal Squamous cell cancer.

2. Schatzki’s Ring:
• MC seen at Squamo Columnar junction

• Submucosal ring – thin ring

• C/F- Intermittent Episodic dysphagia or Aphagia

• Managed by swallowing Proteolytic enzymes at times of dysphagia

• Endoscopic Dilatation

• Fundoplication can be done

3. Dysphagia Lusoria:
• MC caused by Aberrant right Subclavian Artery**

• Compresses the esophagus posteriorly.

• Incomplete ring around the esophagus

• Management:

o Symptomatic- Needs surgery

SURGERY SIXER APP BASED WORK-BOOK 2020 202


o Asymptomatic- no Surgery needed.

4. Congenital Tracheo Esophageal Fistula:


The five major varieties of EA and TEF :
• Most common type: EA with distal TEF (type C)- 85% **
• The next most frequent is pure EA (type A)-8-10%
• TEF without EA (type E)- 8% - Known as H type fistula**- presents in late childhood
• EA with fistula between both the proximal and distal ends of the esophagus and trachea
(type D) is seen in approximately 2% of cases- Also Known as Reverse K fistula*
• Type B, EA with TEF between the proximal esophagus and trachea, is seen in
approximately 1% of cases.

Associated Syndromes:
• Downs syndrome is associated with TEF*

• VACTERL anomalies- Vertebral anomalies, Anal atresia, CVS defects, Tracheo

oesophageal fistula, renal anomalies, Limb anomalies

Clinical features:
• Drooling of saliva

• Inability to pass Ryles tube and coiling seen

• H type fistula presents late.

• Type B fistula – will have cyanotic spells on feeding due to direct entry into lungs.

Investigations:
• IOC for A, B, C, D- Iohexol swallow

• IOC for E type fistula- Endoscopy + Bronchoscopy

Infective causes of Dysphagia:


• MC infection seen- Esophageal candidiasis**- Characterised by White Curdy patches

• Herpes simplex virus- Punched out ulcers

• CMV esophagitis- Serpentine ulcer / Geographical ulcer

SURGERY SIXER APP BASED WORK-BOOK 2020 203


Figure- Punched Out ulcer in HSV

Figure- Serpentine ulcer in CMV

Eosinophilic Esophagitis:
• Barium Swallow shows- FELINE Esophagus**

• Endoscopy Shows- Trachea like or Ringed Esophagus

• Treated by Steroids

Figure- Eosinophilic esophagitis showing Trachea like esophagus / Feline appearance

SURGERY SIXER APP BASED WORK-BOOK 2020 204


TUMORS IN ESOPHAGUS

Benign Tumors:
• Mc is Leiomyoma

• MC in lower third esophagus

• On CECT- Hypodense lesion seen

• EUS- this appears in 4th Layer and rarely 2nd Layer ( NEET SS)

• Treatment- Enucleation

Malignant Tumor:
• MC site- Middle third in World level and in India

• MC type is SCC in world / India

• MC type in Western countries- Adenocarcinoma

Predisposing factors:
Squamous cell cancer Adenocancer
• Hot tea • GERD

• Smoking • Barret’s

• Alcohol • Scleroderma

• Achalasia – 8% • Obesity

• Plummer vinson syndrome • High SE status

• Corrosive ingestion – 1000 times • Deficiency of Antioxidants

• Mb Deficiency

• Tylosis Palmaris et plantaris ** Alcohol is not a risk factor for Adenocancer


** Smoking is risk factor for both
• Low SE status

H.Pylori CAG-A Toxin:


• Causes Distal Gastric cancer

• Protects Esophageal Adenocancer

• No relation with Squamous cell cancer

Clinical features:
• Rapidly progressive dysphagia in 6 months

• More of Solids > than liquids

• To get dysphagia- 60% lumen or 2/3rd circumference must be involved.

• Tracheo Esophageal fistula- Aspiration+ Lung Infection

• 90% inoperable on presentation

SURGERY SIXER APP BASED WORK-BOOK 2020 205


TNM staging of Cancer esophagus:
T staging N staging M staging
• T1a- Muscularis mucosa N1- 1-2 nodes M1- Distant mets
• T1b- Submucosa N2- 3-6 nodes MC site - LUNGS
• T2- Upto muscularis propria N3- > 7 nodes
• T3- Invades adventita
• T4a- Resectable adjacent structures like Pleura,
Pericardium and Diaphragm
• T4b- Unresectable structures like trachea,
bronchus, aorta, vertebra

Investigations:
• IOC to diagnose- Endoscopic Biopsy

• IOC for T, N staging- EUS

• IOC for M staging- PET CT

• Overall staging of cancer esophagus- CECT thorax

Management:
Inoperable cases:
• Palliative SELF EXPANDING METALLIC STENT through the growth

• LASER Luminisation

• Feeding Jejunostomy

Operable Cancers:
• The tumor spreads submucosally for long distance- hence a clearance of 10 cm proximal

and 5 cm distal is needed for cancer esophagus.

• Three level nodes may be involved- Abdominal, Thoracic and Neck nodes

• Types of esophagectomy:

▪ Transthoracic

▪ Transhiatal

Ivor Lewis two stage procedure Mc Keown three stage procedure


• Subtotal esophagectomy • Total esophagectomy
• Two stage Lymphadenectomy ( • Three stage LN dissection ( all three)
abdominal and thoracic Nodes) • Three incisions- Abdominal, Thoracic
• Two Incisions- Abdominal and and Cervical ( Three regional nodes
thoracic ( Two regional nodes removed)
removed) • Neck Anastomosis
• Intrathoracic anastomosis** • Leak percentage is more
• Leak percentage is less
• Mortality if leaks is high

SURGERY SIXER APP BASED WORK-BOOK 2020 206


• Mortality is less as the contamination
into the mediastinum never happens
and secretions can be drained out.

Transhiatal Esophagectomy (THE) : ORRINGER’S OPERATION:


• Two incisions- Neck and abdominal

• Neck anastomosis

• Mortality is less as thorax is not opened

• Nodal clearance is less.

• Post Op radiotherapy is given after THE.

Conduits for Surgery:


• Best conduit for Oesophageal replacement- Stomach as it is highly vascular and less
chances of necrosis.
• Best Conduit in cases of Corrosive injury to Stomach and Esophagus and for LONG
SEGMENT replacement- Colon
• Best conduit for short segment replacement of esophagus- Jejunum, especially in peptic
stricture cases.
• Gastric Conduit is based on Right gastroepiploic artery and Right Gastric artery.

Routes of taking up the conduit:


• Posterior Mediastinal- Shortest and Most dependent route. 28 cm only

• Substernal route- 2nd option if posterior mediastinal route not available as in corrosive

ingestions. 30 cm length

• Subcutaneous route- Less commonly used. Cosmetically ugly, least dependent route

SURGERY SIXER APP BASED WORK-BOOK 2020 207


Parts of esophagus on basis of cancer:

• Cervical esophagus- 15 to 20 cm (Cricopharynx to Sternal notch)

• Upper thoracic- 20 to 25 cm ( Sternal Notch to Azygous vein)

• Mid thoracic- 25 to 30 cm ( Azygous vein and Inferior pulmonary vein)

• Lower thoracic- 30 to 40 cm ( IPV to OG junction

Site based management:

• Cervical esophageal cancer- Not operated and only RT is given

• Mid thoracic- Mc Keown operation

• Lower thoracic- Ivor Lewis operation

SURGERY SIXER APP BASED WORK-BOOK 2020 208


Chapter 2- Stomach and Bariatric Surgery

Vagus Nerve Branches:

• Left Vagus nerve is a thin Nerve


• Right Vagus nerve is a thick Nerve like a Guitar string

Anterior gastric nerve (Left)


• Hepatic branch
• Gastric branches
• Crow's foot (Nerve of laterjet) – Image Based Question**
Posterior gastric nerve (Right) **
• Criminal nerve of Grassi – Image Based Question**
• Crow's foot
• Coeliac branch

Truncal Vagotomy:
• Cutting the Vagus nerve at the Trunk at lower end of esophagus.
• Hepatic branch is lost- Bile stasis and Gall stones.
• Since pyloric Branches – Crow’s Foot function is lost and hence there will be Pylorospasm
( Function of Nerve laterjet is to do Pylorus relaxation)
• Coeliac branch is lost- Post vagotomy diarrhea
• Gastric branches lost- Loss of gastric tone

Complications of Truncal Vagotomy:


• Bile stasis+ Gall stones
• Pylorospasm

SURGERY SIXER APP BASED WORK-BOOK 2020 209


• Post vagotomy Diarrhea

Since there is pyloronspasm we must do- Drainage procedures:


• Posterior Gastro Jejunostomy
• Heinke Mikulicz Pyloroplasty
These drainage procedures have their own list of complications.

High Selective Vagotomy (Parietal Cell Vagotomy)


• Only the Gastric Branches are
selectively cut.
• No need of drainage procedure in
this operation.
• But the problem is we may miss
few branches during surgery and
hence risk of recurrence is more (
5-10%)

Figure: HSV- The gastric branches are cut 7 cm


proximal to pyloric orifice

Complications of TV with GJ:


1. Complications of losing the Vagus nerve- discussed already
2. Problems due to GJ:
• Malabsorption Syndromes
• Afferent and efferent Loop syndromes
• Dumping Syndromes

Malabsorption:
• Iron Deficiency Anemia ( MC deficiency in Gastric surgeries)
• Calcium Deficiency- Osteoporosis
• Vitamin B12 deficiency

Loop Syndromes:
Afferent Loop syndrome Efferent Loop syndrome
MC type LC type
Due to kinking or adhesion of afferent limb
There will be BILIOUS Vomit only There will be FOOD+ VOMIT **
NO FOOD in Vomit** Emergency case
Self limiting most of the time Urgent surgery needed

SURGERY SIXER APP BASED WORK-BOOK 2020 210


Radionuclide study will show stagnation of
contrast in Afferent limb is the IOC** ( NEET
SS)

Dumping Syndromes/ Post Cibal Syndrome:

Early Dumping Syndrome Late Dumping syndrome


Happens in Half hour Happens in 2 hours
Massive fluid shift towards Hypertonic food This happens due to Reactive Hypoglycemia**
particles
C/F- Dehydration, Thirsty and Giddiness C/F- Palpitation, Tremors and sweating
Diagnosis by PCV- there will be Diagnosis by PPBS value- Hypoglycemia
hemoconcentration

Prevention of dumping Syndromes ( NEET SS / AIIMS)


• Eat small split meals
• Dry meals
• High fat diet
• Low Carbohydrate diet

Treatment of Dumping Syndrome:


• Conservative treatment
• DOC- Octreotide**
• Revisional Surgeries- Reverse Roux limb interposition ( For NEET SS)

Helicobacter Pylori:
Microbiology:
• Incidence = 60-80% in India
• Gm-ve Cocco Bacilli
• Lopho trichous flagella
• Motile
• Can inhabitat only in Gastric type epithelium ( Stomach, Metaplastic Duodenal,
Meckels, Ectopic Gastric mucosa)
• Catalase+ve, Urease +ve ( Produces Ammonia from Urea)
• Due to ammonia causing local alkaline nature it produces more acid
• Two Toxins produced: Cag A toxin and Vac A toxin**
• Cag A toxin causes Distal Gastric Cancer and inhibits Esophageal Adenocancer and
Proximal Gastric Cancer. Relation with SCC is not known.**

SURGERY SIXER APP BASED WORK-BOOK 2020 211


Investigations:
• IOC ( If endoscopy done)- Invasive test- Rapid Urease Test**
• IOC non invasive test- ELISA for Serology
• IOC to look for eradication- C13/C14 Urea Breath Test
• New investigation for Eradication- Stool for H.Pylori Antigen
• GOLD STANDARD to diagnose H.Pylori- Staining and Culture
• Stains used for H.Pylori- Silver Starry Stain, Giemsa Stain, Warthin Starry stain**

Treatment:
• Triple drug regimen: 7 to 14 days
• LAC- Lansoprazole+ Amoxycillin+ Clarithromycin
• OAM- Omeprazole+ Amoxycillin+ Metrogyl

Diseases causes by H.Pylori:


• Peptic ulcer diseases- Duodenal Ulcer ( 90%) > Gastric Ulcer ( 80%)
• Type B gastritis
• Non ulcer dyspepsia
• MALTomas ( 100% association)
• Distal Gastric cancer
• Menetrier’s Disease

Not associated with


• Type A gastritis
• Proximal gastric cancer
• Lauren’s Diffuse Gastric Cancer.

SURGERY SIXER APP BASED WORK-BOOK 2020 212


Peptic Ulcer Disease

• Mc type of Peptic Ulcer- Duodenal Ulcer


Duodenal Ulcer Gastric Ulcer
MC site is 1st part of Duodenum Modified Johnson Classification:
• Anterior Located- Perforate • Type 1- MC – Incisura @ Lesser curve
• Posterior located- Bleed • Type 2- 2 Ulcers- One in Duodenum
• Bleeding is MC from Gastro duodenal and another in Stomach
Artery • Type 3- Prepyloric Ulcer
• Type 4- Proximal Gastric Ulcer in
Lesser curve side
• Type 5- NSAID induced ulcers
Unusual site of Peptic ulcers- D2, D3 is seen
in Zollinger Ellison Syndrome
Etiopathology: Etiopathology:
Increased acid Production • Due to loss of mucosal Barrier
• Except Type 2 and 3 Ulcers are due to
increased Acid Production.
Management : Management:
• Antiulcer drugs like H2 Blockers, PPI • Type 1,4 ulcers- Heals very well with
will be beneficial for Duodenal Ulcers. Ulcer healing agents or coating agents
• Give them 6-8 weeks of PPI like Sucralfate* and reduce the risk
factors
• Type 2 and 3 ulcers may benefit from
PPI.
Surgical treatment: Indications: Surgical Treatment: Indications:
• Intractable ulcer (when not healing • Intractable ulcer ( To r/o occult
with 8 weeks) malignancy)
• Bleeding- MC indication of surgery** • Bleeding
• Perforation • Perforation
• Gastric Outlet obstruction • Obstruction

Gastric Ulcer- Intractable Ulcers: Management:

Type 1 Ulcer Wedge resection


Type 4 ulcer • Pouchet Procedure
• Kelling Madlener Procedure
• Csendes Procedure
• Followed by Billroth 2 Reconstruction
Type 2/ 3 Ulcer Distal Gastrectomy including Gastric ulcer with Billroth-2
reconstruction

SURGERY SIXER APP BASED WORK-BOOK 2020 213


Along with above procedure we also do – Truncal vagotomy

Figure: Type 4 Ulcer surgeries.

Duodenal Ulcer Complications: Management:


o Intractable Duodenal Ulcer- HSV > TV+GJ
o Bleeding Duodenal ulcer- Endoscopic Management- Adrenaline injection/ Coagulation
Indications of Surgery in Bleeding Duodenal Ulcer:
- Failed Endoscopy > 2 times
- Rare blood group
- Hemodynamically unstable
- More than > 6 units of Blood transfused
- More than 3 units transfused per day
Duodenotomy and Three point U suture ligation of GDA and its Branch..
After duodenotomy we close the duodenum like a Heinke Mikulicz Pyloroplasty
Stable Patients- Truncal Vagotomy is also done.

o Perforated Duodenal Ulcer:


Clinical Features: Guarding, Rigidity and Obliteration of Liver dullness*
Hippocratic facies- Sick look face in septic shock
Investigations:
- X ray Chest/ Abdomen including Diaphragm in ERECT View: Air
under diaphragm: 1 ml air can be detected
- 70% of the cases will have air under diaphragm ( 30% will have no
air)
- Gold standard investigation= CECT abdomen ( 100 % sensitive)
- X ray Left Lateral view- 5-10 ml air detected

SURGERY SIXER APP BASED WORK-BOOK 2020 214


Figure: Air Under diaphragm

Other air under diaphragm signs:


o Cupola sign- Mushtaq like Air below Xiphisternal level
o Riglers Sign- Double wall sign ( Air inside and outside the bowel)
o Foot ball sign- Massive air in abdomen

Treatment:
o Emergency Laparotomy, Lavage, Identify the perforation
o Size: < 1 cm: Small: Direct Closure can be done with Polyglycolic acid
o Size > 1cm: Omental patch is used. ( Modified Graham’s Patch is a Live patch)
o Size > 3 cm ( Giant Ulcers) : Pyloric Exclusion procedure+ Jejunal Serosal Patch+ Gastro
jejunostomy to prevent Duodenal fistula

Gastric Outlet Obstruction:


o Correct electrolyte imbalance- Hypokalemic, Hyponatremic, Hypochloremic, Hypocalcemic
Metabolic alkalosis + Paradoxical renal aciduria
o Fluid of Choice- Normal saline + Potassium Supplements
o Surgery- TV + GJ

Consolidation:
Complications Duodenal ulcer Gastric Ulcer
Intractable HSV> TV/GJ • Type 1- Wedge resection
• Type 2/3- TV+Distal
gastrectomy +Billroth 2
• Type 4- Pauchet, Kelling
Madlener and Csendes
procedures

SURGERY SIXER APP BASED WORK-BOOK 2020 215


Bleeding Three point U sutures +/- TV Oversew the Bleeding vessel or a wedge
resection for unstable cases.
MC vessel to bleed is Left gastric
artery**
NEET SS- Distal Gastrectomy for Stable
Cases
Perforation < 1cm- closure Omental patch if patient unstable
>1cm- Graham patch Patient stable- Distal gastrectomy
>3 cm- Jejunal patch and
pyloric exclusion

Clinical points:
• Duodenal ulcer will have pain decreased on Eating and hence they will be obese
• Gastric ulcer will have pain increased on Eating and reduced on vomiting and hence they
will be slim and malnourished
• Periodicity is common in Duodenal ulcer.
• Hematemesis is MC than melena in Gastric ulcer
• Melena is MC than hematemesis in Duodenal Ulcer
• Gastric ulcer has 0.5% risk of malignancy**

Complication Questions:
• Most common cause of death in Peptic ulcer disease- bleeding**
• MC Complication of Gastric ulcer- Perforation
• MC Complication of Duodenal ulcer- Bleeding
• MC complication requiring Surgery- Bleeding***
• Complication with highest mortality rate – Perforation (15% cases)
• Deformities in Stomach- Tea Pot and Hour glass deformities in Chronic Gastric Ulcer

Extra Points:
Named Gastric Ulcers
• Cushing Ulcers- Stress ulcers in Head injury, MC in Proximal stomach near Cardia ,
more chances to perforate
• Curling Ulcers- Stress ulcers in BURNS, MC in stomach now than duodenum
• Cameron Ulcers- Vertical Riding Ulcers in Sliding Hiatus hernia, MC in fundus*

SURGERY SIXER APP BASED WORK-BOOK 2020 216


Cancer Stomach

MC site in India/ World- Incisura ( Antrum)


MC site in western- Proximal Stomach ( Bailey gives Western statistics only)
MC type is Adenocarcinoma.

Premalignant Risk Factors


6- S 4-A
S- Smoking A- Adenomatous Polyps
S- Spirit A- Atrophic Gastritis
S-Spicy foods A- Anemia ( Pernicious Anemia)
S- Salted Foods A- Blood Group A
S- Smoked Foods
S- Stored Foods

Others:
• Menetrier’s disease
• H.Pylori- Cag A toxin - Distal Gastric cancer
• H.Pylori is protective against OG junction Adenocancers
• H.Pylori is having no relation in Proximal Gastric cancer and SCC Esophagus

Genetic Syndromes:
• Lifraumeni Syndrome
• Lynch Syndrome
• FAP
• E- Cadherin mutations

Pernicious Anemia:
• Causes Type A Gastritis
• Antrum is Not affected
• MC affects proximal stomach
• Premalignant and causes cancer in proximal stomach.

Clinical Features:
STOMACH- features 5 MCQs
S- Silent Krukenberg Tumor:
T- Tumor - Bilateral Ovarian Deposit
O- GOO - Superficial Deposit
M- Melena - Seen in Premenopausal women only

SURGERY SIXER APP BASED WORK-BOOK 2020 217


C- Cachexia - On C/S- inside ovary is normal
H- Hematemesis - Transcoelomic Spread**

Sister Mary Joseph Nodule:


- Transcoelomic spread
- Umbilical deposit

Blumer’s Shelf:
- Per rectal deposit in POD**
- Transcoelomic spread

Virchow’s Node- Left Supraclavicular node ( Troisier sign)


Irish node- Left Axillary Node
The two nodes are due to Lymphatic spread

Clinical features to say the tumor is inoperable:


• Liver mets+
• Ascites+
• 5 MCQs mentioned above are inoperable features
In addition, if CECT shows Peritoneal mets, Pancreatic infiltration are also inoperable

T staging N staging M staging


T1-Tumor involves mucosa and Submucosa only N1- 1-2 nodes+ M1- Mets+
T2-Muscularis propria N2- 3-6 nodes+
T3- Penetrates subserosa N3a- 7- 15 nodes+
T4- Serosa and adjacent structures N3b- > 16 nodes+
(Adjacent structures doesn’t include Esophagus and
duodenum**)

Minimum Number of Nodes removed:


• Stomach- 16 nodes
• Breast- 10 nodes
• Oesophagus- 15 nodes
• Colon – 12 nodes

Early gastric cancer


• Tumor involving only Mucosa and Submucosa only
• With or without Lymphnodes
• Diagnosed by CON FOCAL Endoscopy and Narrow Band Imaging Scopy

SURGERY SIXER APP BASED WORK-BOOK 2020 218


• Japanese classification is used
• Treatment- EMR and ESD ( Endoscopic Mucosal resection and Submucosal Dissection)

Investigations:
• T and N Staging- EUS
• Diagnosis- Endoscopic Biopsy ( 6-8 biopsy taken from lesions)
• Tumor market- CA 72-4
• Mets- CECT abdomen
• To diagnose Peritoneal mets- Most sensitive test- Diagnostic Laparoscopy ( Diagnostic of
30% time in planned surgery cases)

Classification:
BORMANN'S CLASSIFICATION : Based on Gross examination
I. Polypoid
II. Ulcero proliferative ( Fungating)
III. Ulcerative
IV. Diffuse Linitis Plastica

Lauren’s Classification: Based on Microscopic appearances:


Histology INTESTINAL DIFFUSE
Epidemiology MC in Sporadic MC in Familial
Sex incidence Males more common Females more common
Age incidence Elderly Young
Risk factors Environmental Genetic
H.Pylori Positive Family H/O
Group A
Pernicious anemia
E- Cadherin Mutation –
prophylactic gastrectomy in
those patients.
Metastasis Hematogenous Mets Common** Lymphatic Mets MC
Transcoelomic Spread is More
common**
Peritoneal mets are mc
Hepatic metastasis Nodular Diffuse
Prognosis Good Bad*
Pathology Gland formation + Poor differentiated Signet
MC in distal Stomach Ring cells.
MC in Proximal Stomach
Genetics* APC gene mutation** Decreased E- Cadherin**

SURGERY SIXER APP BASED WORK-BOOK 2020 219


P53 and P16 inactivation P53 and P16 inactivation
Microsatellite instability

Siewart Classification of OG junction Cancers:


Types
• Type 1- Tumors 1-5 cm above OG junction in distal Esophagus.
• Type 2- Tumors at 1 cm above and upto 2 cm on the stomach side from OG junction.
• Type 3- Tumors from 2- 5 cm below the OG junction

Latest Guidelines:
• Types 1,2 are treated like esophageal cancer- by Transhiatal esophagectomy.
• Type 3 is treated like gastric cancer by Total gastrectomy*

Other classification systems:


• Ming- Chinese
• Early gastric cancer- Japanese classification

Lymph node stations during resection:


• 16 Lymph node stations are there as per Japanese Gastric Cancer association.
Level 1 nodes Level 2 nodes Level 3 nodes
Peri gastric nodes Beyond 3 cm from lesion,
( Nodes < 3 cm from tumor) along the major vessels
1-Right cardiac 7- Left gastric artery 12- hepatoduodenal ligament
2- Left cardiac 8- Common Hepatic artery 13- Retro pancreatic
3- Lesser curve 9- Coeliac artery 14- Superior Mesentric
4- Greater curve 10- Splenic Hilum vessels
5- Suprapyloric 11- Splenic artery 15- Middle Colic vessel
6- Infrapyloric 16- Para aortic
D1- Gastrectomy: D2- Gastrectomy: D3- Gastrectomy:
• Removal of all level
1,2,3 nodes.

SURGERY SIXER APP BASED WORK-BOOK 2020 220


• Removal of all level 1 • Removal of all level 1 • Done only in Japan
nodes+ Station 7 and level 2 nodes+
node. Station 12 nodes.
• Gold standard for
Cancer stomach
surgery*

Surgical resection:
• Proximal margin- 5cm clearance proximal ( NCCN guidelines says – 4cm margin
proximal)
• Distal upto Duodenal 1st part
• We either do Subtotal gastrectomy or Total gastrectomy,

Billroth -2 Reconstruction:
• Gastro jejunal anastomosis after subtotal gastrectomy.
• No more Gastro duodenal anastomosis ( Billroth 1 done now)
• Increased Bile reflux is seen in this reconstruction
• Duodenal stump Blow out:
- Happens on 4th /5th POD
- Bile and Pancreatic juice leaks
- No need of urgent surgery
- USG guided drainage is enough

Roux en Y reconstruction:
• Done after Total gastrectomy
• 2 anastomosis- Esophago-jejunal and Jejuno-jejunal anastomosis done
• Herniation behind Roux Limb is known as PETERSON Hernia**

SURGERY SIXER APP BASED WORK-BOOK 2020 221


Figure: Roux En Y anastomosis

Palliative procedures: for inoperable cancers.


• GOO cases- Palliative Gastro jejunostomy or Palliative gastrectomy
• For Bleeding and GOO cases Palliative resection is done and has increased survival
outcome than a Palliative GJ
• OG junction growth causing Dysphagia- SEMS can be done,

Non Hodgkin’s Lymphoma GIST


MC in stomach MC in Stomach
MC site- Antrum MC in Proximal stomach in gastric curve side
In Burkitt alone its mc in fundus near fundus
Types: Types:
• MC is Diffuse large cell Wild type- CD117 negative, CD 34 negative
• MALT ( 2nd MC) Normal GIST- Both are positive
• Mantle
• Burkitt
Origin- Lymphatic tissues on gut wall Origin- Cells of Cajal or Pacemaker ccells
PRIMARY GI LYMPHOMA
( DAWSON CRITERIA)
• No liver involvement
• No Spleen involvement
• No Bone marrow involvement
• No Generalised Lymphadenopathy
• Confined to one GI organ only
Diagnosis: for both
• Endoscopy is difficult to diagnose due to normal mucosa
• HENCE IOC- EUS guided Biopsy or CT guided Biopsy
Tumor markers: Tumor markers:
• Not mentioned in Books • CD 117 – tyrosine kinase receptor
• There is a named staging for them mutation positive in 90%
known as LUGANO STAGING** • CD 37- 40% positive

SURGERY SIXER APP BASED WORK-BOOK 2020 222


• PDGFRA- Platelet derived Growth
factor receptor Antibody
• Latest- DOG- Discovered on GIST
Management: Treatment:
• MALTOMA early- H.Pylori • Surgical resection with clear margin
eradication alone is enough • No lymphnode dissection needed.
• Advanced cases- R- CHOP regimen
o R- Rituximab Prognosis after surgery depends on Modified
o C- Cyclophosphamide Joenssu criteria:
o H- Hydroxy Adriamycin - Size ( >10 cm- Bad)
o O- Oncovin - Mitotic index ( > 5 mitotic index/
o P- prednisolone 50 HPF)
- Site ( Stomach – Good)
• DLC- R- CHOP
• For NHL of stomach- no need of After surgery:
surgery For High risk tumors- will give Imatinib
mesylate or Sunitinib
Indications of Surgery in a NHL Stomach:
• Bleeding
• Perforation
• Obstruction
• Intractable pain
• Not responding to chemotherapy

Other GI lymphomas like Small intestine and


Large intestine- Surgery advised first followed
by R- CHOP

Miscellaneous Topics

Congenital Hypertrophic Pyloric Stenosis


• Gross hypertrophy of Circular **musculature of pylorus
• Acquired Condition.
• Incidence- 1 in 3000 live births
• M/c surgical cause of vomiting in infants
• First born male infants are characteristically affected more**
• M/c age of presentation- 4 weeks **

Clinical feature:
• Non bilious vomiting increasingly projectile may also be non projectile*
• Inspection- shows VGP ( Must be done during feeding time**)

SURGERY SIXER APP BASED WORK-BOOK 2020 223


• Palpation- Right hypochondrial olive like mass+ (Done during non feeding time)
• Palpation of a olive live structure in right upper quadrant is pathognomonic.**
• Hypochloremia, Hypokalemia and Metabolic alkalosis happens due to severe vomiting

Investigations:
USG abdomen:IOC
• Thickened pylorus > 4mm
• Lengthened pylorus >16 mm
• Empty stomach
Barium meal- Double Track Sign** or String Sign**

Extra mile:
• Lump is not always palpable. But is palpated no other investigation is needed*
• The babies look healthy and not malnourished- as they consume enormous milk
during disappearance of mass time.

Treatment:
• Ramstedt’s pyloromyotomy*- pyloric mass is split horizontally without penetrating
the mucosa
• If the mucosa is inadvertently opened during surgery- close mucosa alone and keep
baby nil per oral for 48 hours.. ( In normal cases we start oral in 6 hours)
• Opened mucosa is repaired by 3’0 Vicryl sutures.

Intestinal atresias

Bilious vomiting in new born babies:


• Duodenal atresia- MC cause
• Jejuno ileal atresia
• Midgut volvulus

Duodenal atresia:
• MC site of atresia is distal to ampulla ( 80%)
• 20% atresia is proximal and they have non bilious vomiting
• X ray erect- Shows double bubble appearance** ( CHPS- Single Bubble appearance)
• Treatment is Duodeno duodenostomy ( Diamond shaped anastomosis)

Jejunal Atresia:
• Triple bubble appearance

SURGERY SIXER APP BASED WORK-BOOK 2020 224


Bezoars
• Trichobezoars – hair balls in stomach, Exclusively seen in Psychiatric young, female
Patients. Hair balls can lead to perforation, obstruction and GI bleeding. Diagnosis made
on endoscopy. Treated by Surgical removal of Bezoars.
• Rapunzel syndrome- Long hair of Trichobezoar extending from stomach upto Ileum.
• Phytobezoars – Vegetable matter in stomach in patients who have Gastroparesis like
Diabetes and in Post Gastric Surgery Patients, Also seen in Surgical patients.

Figure- Trichobezoar

Gastric Volvulus

• Stomach can rotate and go for volvulus either in Organo axial ( Longitudinal axis) or
mesentrico axial ( Vertical Axis)
• MC type of volvulus is Organo axial volvulus* and this associated with Diaphragmatic
hernia*

Figure A- Mesentrico axial, B- Organo axial volvulus.

• Organo axial volvulus associated with Diaphragmatic hernia


• Borchardt’s TRIAD:

SURGERY SIXER APP BASED WORK-BOOK 2020 225


▪ Sudden onset of constant retching epigastric pain
▪ Inability to vomit
▪ Inability to pass the Ryles tube** ( remember no vomiting)
• Urgent surgery must be done*- Gastropexy and Closure of diaphragm defects*

Gastric Antral Vascular Ectasia= Watermelon Stomach


• Antrum has vascular ectasia and looks like watermelon
• MC in old age in females
• Presents with melena and anemia
• Treatment is Argon plasma Laser coagulation

Figure: GAVE

Dieulafoy’s lesion
• The dilated submucosal bleeding vessel in lesser curve in proximal stomach – 6cm below
OG junction** is known as Dieulafoy’s lesion
• C/F- Spurting blood seen with severe hematemesis
• Treatment- Endoscopic coagulation
• If failed- Embolisation of LGA
• Last step – Surgical ligation

Menetrier’s disease
• Hypertrophied gastric mucosa looks like Brain cerebrum
• Pathology- Parietal cells are replaced by Mucus secreting cells- resulting in protein lose*
• Etiology: CMV in children; H.Pylori in Adults
• Growth factor elevated = TGF -Alpha**
• Premalignant
• Treatment- Protein supplement- if persists gastrectomy done.

Wilkie’s disease= Superior mesenteric artery syndrome


• SMA compresses the third part of duodenum and results in GOO*
• Predisposing factors:
 MC in tall thin young females on severe diet and weight loss
 Cast syndrome-due to spine immbolisation body cast
 Chronic immobilisation

SURGERY SIXER APP BASED WORK-BOOK 2020 226


 Tight dresses
• Aorto mesenteric angle ( Angle between SMA and Aorta= Normal: 35-65 degrees)
• If the Angle becomes < 22 degrees- Wilkie disease results
• Management:
 Conservative therapy
 Weight gain
 If not responding we do – Duodeno Jejunostomy ( NEET PG)
 Strong Procedure- Cutting Ligament of Trietz and releasing the 3rd part
duodenum away from compression ( NEET SS)

Figure: Wilkie’s disease

Image based questions:

Figure- Cameron ulcer in hiatus hernia

SURGERY SIXER APP BASED WORK-BOOK 2020 227


Figure- Peterson hernia in RYGB

Figure- Showing GIST in J manuever


Acute Upper GI Bleeding Protocol

Definition:
• Bleeding proximal to ligament of Treitz is known as Upper GI haemorrhage.
• Bleeding < 60 ml/day presents as melena
• Bleeding > 60 ml/day presents as Hematochezia

Management protocol:
• Start IV line
• 1st injection- Proton pump inhibitor
• Ryles tube decompression- To prevent aspiration and practically vomiting stops.
• Once patient is stable- refer to Medical Gastroenterologist to do Endoscopy
• IOC – Endoscopy
• Artery of Bleeding- Gastro duodenal artery

Causes of Upper Gi Bleeding on Endoscopy


Non variceal causes- 80% Variceal causes- 20%
Peptic ulcer disease – MC duodenal ulcer Esophageal varices- MC
Duodenitis Gastric varices
Gastritis Portal hypertensive gastropathy
Gastric ulcer
Esophagitis
Dieulafoy lesion
Mallory Weiss tear
AV malformation

SURGERY SIXER APP BASED WORK-BOOK 2020 228


Cancer- GIST is mc to bleed**

Bleeding duodenal ulcer- Classification:


FORREST Classification:
• Grade I: Active Bleeding
• Grade IIa: Non Bleeding visible Vessel
• Grade IIb: Adherent Clot
• Grade IIc: Black dot
( mnemonic for Grade 2- VCD- Vessel, Clot, Dot)
• Grade III: No signs of Bleeding
 Type 1 and IIa – high risk of rebleeding seen
 Hence we must clip or coagulate or injection Epinephrine done at bleeding point,

Extra Mile: Risk stratification scores for Upper GI hemorrhage


Blatchford score Rockall score
• BUN • Age
• Hb • Comorbid diseases
• BP • Magnitude of haemorrhage (Based on
• PR BP and PR)
• H/o of Melena, Syncope, Liver or • Transfusion requirement
cardiac dysfunction • Endoscopic findings**
• Stigmata of recent bleed
• Please remember Blatchford score doesn’t need endoscopy

Indications for surgery:


• > 6 units blood transfuse per day
• >3 units daily needed
• > 2 attempts failed
• Hemodynamically unstable
• Rare blood group cases
• Failed Endoscopic therapy

Surgical treatment of Bleeding ulcer:


• Duodenal ulcer- 3 Point U suture of GDA and its Branch- Superior Pancreatic duodenal
artery.
• Gastric Ulcer- Oversewing ulcer for unstable cases; Stable cases- Gastrectomy advised.

SURGERY SIXER APP BASED WORK-BOOK 2020 229


Bariatric Surgery

Bariatric surgery is a surgery done for Morbid Obesity. It is not done for Cosmetic purpose.

NICE 2014 Guidelines for Bariatric Surgery:


(National Institute of health and Care Excellence)
• BMI> 40.
• BMI>35 with Type 2 Diabetes past 10 years, other co morbid factors-
Hypertension, Sleep apnea, osteoarthritis, PCOD
• BMI 30-34.9 with Type 2 Diabetes in < 10 years.( Recent recommendation**)
• BMI value should be lowered for ASIAN origin.

Indication of Surgery:
• Age group between 16-65 years

Contraindications for Bariatric Surgery:


• Prader Willi Syndrome
• Psychiatrically unstable patient

Extra Points :
OS- MRS Scores: (Obesity Surgery-Mortality Risk Score)
To add one point for each of the below factors:
• Age > 45
• BMI > 50
• Male Gender
• Hypertension
• Increased risk of DVT or Pulmonary embolism.

The more the points, there is increased risk of mortality

Types of Bariatric Surgery


Restrictive
• Adjustable silicone gastric banding
• Sleeve gastrectomy
• Vertical banded Gastroplasty
Mostly restrictive (and partly malabsorptive)
• Short-limb (50–100 cm) Roux-en-Y gastric bypass
• Long-limb (150 cm) Roux-en-Y gastric bypass
Mostly malabsorptive
• Biliopancreatic diversion with or without duodenal switch

SURGERY SIXER APP BASED WORK-BOOK 2020 230


Discussion of Each surgery:
1. Sleeve Gastrectomy:
• Most commonly performed operation in India, but now there is decreasing trend for
Sleeve gastrectomy.
• The lesser curve based Gastric tube is constructed over a size 32-36 Fr Bougie.
• The Achilles heel of this operation is- Risk of Staple leak at Angle of His**
• Reduces the intake of food and produces weight loss.
• Least complicated surgery, Malabsorption is not seen

2. Lap adjustable Gastric Banding:


• Band wrapped at upper part of fundus with adjustable subcutaneous port.
• Lesser curve entered by PARS FLACCIDA Technique
• The access port is sutured to rectus sheath in upper abdomen for ease of access by a non
coring, HUBER Needle for Band Adjustment*
• Early satiety occurs and it is a type of pure restrictive surgery.

3. Roux En Y Gastric Bypass:


• Mostly restrictive and partly malabsorptive
• Stomach is made into a small pouch of 50 ml capacity.
• Roux limb made and brought to anastomose to the pouch. The length of Roux Limb varies
here from 100-150 cm . Y limb will be around 100 cm.
• Most commonly done surgery done now is this operation **

SURGERY SIXER APP BASED WORK-BOOK 2020 231


4. Biliopancreatic Diversion: ( SCOPINARO OPERATION)
• Mostly malabsorptive
• Stomach is made into 200 ml only. Distal gastrectomy done.
• Cut at level of ileum and Roux limb is made from Ileum and Gastro ileal anastomosis
done.
• Severe malabsorption happens. The food is not allowed to join with Bile.
• The common length is only 50-100cm
• Two anastomosis- Gastroileal and Jejuno ileal
• Highest weight loss
• Highest malabsorption
• Highest mortality
• One issue in this operation is Marginal Ulcer- as the feedback inhibition from duodenum is
not there and hence increased gastrin production causes- Peptic ulcer at margin. Hence a
modification with Duodenal switch done as shown.

5. Duodenal Switch Type BPD:


• Sleeve gastrectomy done and with stomach pouch D1 part is left as shown.
• The duodenum will send feedback inhibition to acid production.
• It can be done as two stage- Sleeve as 1st stage and 2nd stage completion can be done.
• There is no dumping syndrome also in these patients as pylorus is also preserved.

SURGERY SIXER APP BASED WORK-BOOK 2020 232


Figure: Biliopancreatic Diversion

Figure: Duodenal Switch Type BPD

Latest operation: Mini Gastric Bypass ( one anastomosis Gastric Bypass)


• This technique uses only one anastomosis- Antecolic Loop Gastrojejunostomy without a
Roux En Y configuration and a longer gastric pouch than a standard Gastric Bypass.
• Major complication is Biliary reflux esophagitis and long term risk of Barret’s Esophagus
and cancers.

SURGERY SIXER APP BASED WORK-BOOK 2020 233


Figure: Mini Gastric Bypass

Complications of Bariatric Surgery:


MC cause of death in Bariatric surgery:
• Immediate post op- Anastomotic leak and peritonitis
• within 30 days – DVT/Pulmonary embolism
• Overall- DVT/PE

Gastric Banding:
• Early complication- Port site infection
• Late complication- Slippage of Band and necrosis, Perforation, Weight regain
Sleeve Gastrectomy:
• Early complication- Leak from Angle of HIS, Bleeding, DVT and PE
• Late Complication- GERD, Weight gain

RYGB:
• Early complication- Anastomotic leakage, Bleeding, Obstruction, DVT/PE
• Late Complication- Peterson Hernia, Adhesive colicky, weight regain

BPD operations:
• Early complication- Anastomotic leakage, DVT/PE
• Late Complication- Severe Malabsorption

Post OP Supplements in bariatric Surgery:


After gastric banding:
• Multivitamin
• Minerals
• Thiamine if vomiting

SURGERY SIXER APP BASED WORK-BOOK 2020 234


• Vit. D
• Iron

All other surgeries ( RYGB, BPD, Sleeve gastrectomy)


• Same as above
• Selenium
• Copper
• Zinc
• Folic acid
• Vit B12
• Vitamin A,D, E,K

Space for Extra Points:

SURGERY SIXER APP BASED WORK-BOOK 2020 235


Chapter 3- Intestines

Introduction

Investigations in Intestine:
• Plain X ray Abdomen Erect showing Dilated
Bowel loop.
• The dilated bowel shown here is Jejunum.
• Based on the appearance of valves of Kerkring/
Valvulae Conniventes

• This is Colon
• Identified by presence of Haustrations

• Barium Meal
• Xray taken after half an hour
• Shows Stomach and Duodenum

SURGERY SIXER APP BASED WORK-BOOK 2020 236


• Barium meal follow through
• To study small bowel
• Xray taken at 2,4,6 and hours

Extra point:
• Jejunum has bird feather appearance
• Ileum is featureless

• Nasojejunal tube put inside and Gastrograffin


contrast is pushed inside.
• ENTEROCLYSIS or Small bowel Enema
• Its an invasive investigation
• Can be done by X ray, CT or even MRI
enteroclysis

Barium enema Double contrast Barium enema- Air or Co2 is


used

SURGERY SIXER APP BASED WORK-BOOK 2020 237


Small intestine Anatomy:
• Length= 6 metres ( Jejunum- 1.5 m . Ileum- 4.5 m)
• Jejunum has Long and few vascular arcades with wide diameter
• Ileum has short and many vascular arcades with less diameter.
• Ileum is featureless on X ray.
• Entire Small bowel supplied by SMA branches.

Ileocaecal valve- BAUHIN Valve**


Appendix valve- Gerlach valve**

Colon Anatomy:
• Appendices Epiploicae- is absent in CAR- Caecum, Appendix and Rectum
• Haustrations are seen in Colon.
• Mobile parts of colon – Transverse colon and Sigmoid colon
• Branches supplying Right Colon- Ileocolic, Right colic and Middle colic artery ( SMA
Branches)
• Branches supplying left colon- Left Colic artery and Sigmoid artery ( IMA branches)
• Marginal artery of Drummond** Supplies the colon all along the full length.
• Arc of Riolan**- Important collateral made between left branch of Middle Colic artery
and Ascending branch of Left Colic Artery.

Griffith Point- Ischemic Colitis MC in Splenic flexure


Sudeck Point- Ischemic Stricture MC in Sigmoid Colon

Barium enema appearances:


• Thumb print appearance**- Ischemic Colitis
• Lead pipe – Ulcerative colitis
• Hose pipe- Crohns
• Question mark – Hirschsprung disease
• Apple core- Cancer
• Claw sign/ Pincer sign- Intusussception
• Ace of Spade appearance/ Bird Beak appearance – Sigmoid volvulus
• Coiled spring appearance- Intussusception
• Pulled up cecum- Ileocaecal TB

Capsule Endoscopy
• Size= 2.6 cmX 1.1 cm size
• Contains camera, Light source, Battery and processor- Battery life= 8 hours
• Receiver collects all the images taken- 2 image/ second ( Total 50000 images)
• Swallow the capsule, Mild diet can be taken, No sedation needed.

SURGERY SIXER APP BASED WORK-BOOK 2020 238


Indications of Capsule endoscopy:
• SB bleeding ( Obscure GI Bleeding)
• SB tumors, Polyprs
• SB angiodysplasia
• Crohn’s Disease ( this is the IOC)**

Contraindications:
• SB obstruction
• SB stricture
• Gastroparesis
Always do Dummy capsule study if you suspect stricture- even if dummy capsule obstructed it
will dissolve in 48 hours.

Complication:
• Capsule retention ( 13%) – MC in CD patients.

Limitations of Capsule Scopy:


• Unable to locate the exact site of lesion
• Incomplete study may happen
• Biopsy cannot be taken

Conventional Colonoscopy Virtual Colonoscopy/ CT colonography


• 160 Cm length of Tube • Non Invasive colonoscopy
• MC site of perforation – sigmoid colon • Done with help of CT or MRI
( 0.5- 1%) reconstruction
• Biopsy can be done. • No risk of perforation
• Can be done in short time
• Small lesions can be easily picked up
• Extra luminal pathology is also picked.

Only one disadvantage- Biopsy not possible

SURGERY SIXER APP BASED WORK-BOOK 2020 239


Figure- Virtual colonoscopy

Ischemic Bowel Diseases

Acute Superior Mesentric Ischemia:


• MC due to embolus > Thrombus
• Embolus usually involves the distal vessel, Thrombus involves the Origin of SMA
• Clinical feature- Pain out of proportion to signs**, Once bowel goes for gangrene and
paralytic ileus- pain disappears.
• Embolus- Segmental bowel goes for Gangrene
• Thrombosis- Entire Small bowel goes for gangrene , colon escapes due to Marginal artery
supply from Retrograde IMA.
• Diagnosed on Laparotomy only
• Treatment- Resection of Gangrenous bowel- results in Short Bowel Syndrome

Figure: Acute SMA Embolus causing part of bowel gangrene

Chronic Superior Mesentric Ischemia:


• Due to slow forming thrombus at origin.
• Hence severe collateral formed
• Characterised by Intestinal angina Triad- Post prandial abdominal pain, Weight loss,
Chronic mesenteric ischemia
• Diagnosis- CT angiography
• Treatment- Endovascular stenting or Surgical Bypass with grafts.

SURGERY SIXER APP BASED WORK-BOOK 2020 240


Recent advance: SQUID
• Super Quantum Interference Device used to diagnose mesenteric ischemia and bowel
gangrene by non invasive method**

Mesentric vein Thrombosis:


• Seen in Hypercoagulable states like pregnancy, Protein C and S deficiency, Dehydration,
Diarrhea etc.
• The small tributaries of veins develop thrombosis and bowel becomes edematous- Venous
gangrene
• IOC is CT scan showing Bowel wall edema
• Treatment of Choice- Inj Heparin and rehydration ( No need of thrombolytics).
• No surgical intervention unless gangrene happens.

Lower GI Bleeding Protocol


• Bleeding distal to Ligament of Trietz

Colonic causes SB causes


Massive bleed Minor Bleed Angiodysplasia ( MC)
Diverticulosis ( MC) Hemorrhoids( MC) Polyps
Angiodysplasia Fissure Tumors
Ulcerative colitis Cancer Crohns disease
Mesentric Ischemia Solitary rectal ulcer
Radiation colitis Polyps

• 1st Done test- Proctoscopy


• 1st done Investigation- Upper GI Scopy because many times bleeding > 60 ml /day can
proceed Hematochezia.
• IOC – Colonoscopy
• If both Upper GI and Colonoscopy is Normal- We brand the case as Obscure GI bleeding*;
which is usually from Small Bowel.
• If Capsule endoscopy is normal- we must do Tc 99m RBC Scan- it detects 0.1 ml/
minute. Major disadvantage is – this cannot locate the lesion.
• If Tc 99m Scan shows- active bleeding immediately we must do Superior mesenteric
angiography- it detects bleeding of 0.5ml/minute.
• Please remember we will do Angiography only when active bleed is present**

SURGERY SIXER APP BASED WORK-BOOK 2020 241


Diverticular disease in GIT

Congenital- overall MC ( 2%) Acquired


Meckel’s Diverticulum: Small bowel- MC in 2nd part of duodenum
Large Bowel- MC in Sigmoid Colon
MC site of Acquired Diverticulum- Sigmoid
Colon
Classical points for Meckel’s Classical Points of Acquired:
• True diverticulum • False
• Antimesentric Border • Mesentric Border
• Incidentally detected Resect • No need to remove incidentally
detected Meckel’s

Meckel’s Diverticulum:
• Persistent of proximal part of Vitello intestinal Duct is Meckel’s ( Distal part of VID- is
Raspberry tumor/ Umbilical adenoma)
• Follows Rule of 2: for adults
- 2% individual
- 2 inch in length
- 2 feet proximal to IC junction
• Ectopic mucosa:
- Gastric Mucosa (MC)
- Pancreatic
- Colonic
- Brunner’s Glands
• Ectopic gastric mucosa disappears as the age advances.
• Children:
- Bleeding is the MC symptom
- Ectopic gastric mucosa seen in adjacent ileum and hence Bleeds due to peptic
ulcer
- IOC in Children – Tc 99m Pertechnate Scan*
- The sensitivity increases with Cimetidine and Glucagon*
- Hence while resecting we must do Resection and Anastomosis as adjacent
ileum contains ectopic mucosa.

• Adults:
- Obstruction is MC symptom
- Ectopic mucosa is not there and hence bleeding is less common.
- IOC – Enteroclysis
- Treatment- Wedge resection can be done

SURGERY SIXER APP BASED WORK-BOOK 2020 242


Complications of Meckel’s diverticulum: Incidence- 6.4%
• Bleeding
• Obstruction
• Hernia- Littre Hernia
• MC tumor – Carcinoid Tumor
• Diverticulitis

Due to such huge list complications Sabiston says- Resect the incidentally detected
Diverticulum.
Direct line from Bailey:
• Short, Wide mouthed and no nodularity- Leave it; Narrow mouth and nodularity
advised resection

Duodenal Diverticulum:
• Mc in 2nd part near ampulla ( periampullary diverticulum)
• MC complication- Bleeding
• Other complications- Pancreatitis, Cholangitis

Jejunal Diverticulum:
• Most commonly they are multiple and huge.
• MC complication- Diverticulitis
• Blind Loop syndrome- Bacterial overgrowth syndrome happens due to stasis of food and
multiplication of bacteria. The bacteria will consume the B12 vitamin and patient
develops B12 deficiency. Diagnosed by Schilling test. Treatment is by Antibiotics.
• Schilling test becomes negative only on antibiotics and not with B12 injection.

Sigmoid Colon Diverticulum:


• Mc site of acquired diverticulum is Sigmoid due to highest luminal pressure inside the
diverticulum.
• Multiple diverticulum is known as Diverticulosis*
• IOC for diverticulosis- Barium enema- Shows SAW TOOTH APPEARANCE**
• MC complication – Diverticulitis- Inflammation and perforation seen. Barium enema is
Contraindicated. IOC for this complication- CECT abdomen.
• HINCHEY Classification is used for Diverticulitis.
o Stage 1: Pericolic abscess
o Stage 2: Pelvic abscess
o Stage 3: Generalised purulent peritonitis
o Stage 4: Generalised fecal peritonitis

SURGERY SIXER APP BASED WORK-BOOK 2020 243


• Hinchey Type 1 and 2 are treated by Non surgical method by USG guided drainage of
abscess and antibiotics.
• Hinchey Type 3 and 4 needs emergency laparotomy and lavage and Hartmann’s
operation.
• Other complications:
- Bleeding: Right Colon diverticulum Most commonly bleed than left colon
- Vesico colic fistula: Clinical feature is UTI, Pneumaturia and fecaluria. Needs
staged procedure.
( MC cause of vesico colic fistula- Sigmoid Diverticulum)

Abdominal Tuberculosis
• Types: peritoneal and Intestinal TB
• Peritoneal TB ( MC)- characterised by ascites+; Omental Caking, Nodules+. Diagnosed by
Ascitic fluid analysis and AFB staining. CB- NAAT testing (Cartridge Based Nucleic Acid
Amplification Test, high Protein ascites +
• Definitive method is by Diagnostic lap and Peritoneal biopsy

Intestinal Type of TB:


• Mc involves terminal ileum and Ileocaecal junction*

Points Hyperplastic – Primary TB Ulcerative – Secondary TB


Etiology M.bovine primary ingestion from Secondary to swallowing infected
Raw milk. sputum
and when patients immunity is Patient having very low immunity*
good.
MC site Ileocaecal valve Longer parts of terminal ileum
Presentation As obstruction As transverse ulcers**and on healing
causes stricture.
(Typhoid presents with longitudinal
ulcers)
C/F Acute abdominal pain with Diarrhea and weight loss
intermittent diarrhea
Mass in right iliac fossa
Barium meal follow Pulled up caecum Absence of filling of lower ileum
through Ileocaecal angle becomes obtuse
Stierlin sign
Umbrella sign
Fleschner sign
Goose neck deformity

Management:

SURGERY SIXER APP BASED WORK-BOOK 2020 244


• They both cases presents with Small bowel obstruction
• IOC is ileo colonoscopic biopsy done.
• We start Category I- ATT and under cover of ATT we do limited resection of Ileo caecal
TB.

Short Bowel Syndrome


• Length of Small bowel= 600 cm- When we resect and it becomes < 200 cm there results
Short bowel syndrome.
• Adult causes- Acute SMA ischemia, Trauma, CD
• Paediatric – NEC, Midgut volvulus going for gangrene

Clinical features :
• Watery diarrhea- unabsorbed bile salts into colon
• Malabsorption of fat soluble vitamins
Failure of Enterohepatic circulation: Bile salt is reduced:
• Steatorrhea- reduction in bile salt pool
• Fat soluble vitamin deficiency ( A,D, E,K)
• Oxalate kidney stones- unabsorbed fatty acids bind with calcium
• Cholesterol gall stones- decreased bile salt in bile
Other problems:
• Megaloblastic anemia
• Increased gastrin secretion**- due to reduced hormonal inhibition from Small bowel is not
there- Acidity and Peptic ulcer disease.

Type of SBS and Pattern of Surgery done Length of jejunum needed to avoid
TPN
Type 1- End Jejunostomy- Good prognosis 100 cm
Type 2- Jejuno Colic Anastomosis 65 cm
Type 3- JejunoIleo Colic Anastomosis- Best prognosis 30-50 cm

SURGERY SIXER APP BASED WORK-BOOK 2020 245


Medical Management Surgical Management
• Fluid electrolyte correction • Intestinal lengthening operation of
• TPN initially Bianchi**
• Enteral Nutrition once ileus gets • Serial transverse Enteroplasty (STEP
settled procedure)
• High dose PPI/ H2 Blockers to reduce • Segmental Intestine reversal
gastric acid secretion • Interposition of colon
• Loperamide
• Octreotide reduces intestinal secretions Final resort in cases of patients with liver
• Recent drug- TEDUGLUTIDE – failure, Catheter related sepsis and Thrombosis
increases bowel growth- GLP 2 of major veins is Intestinal transplantation.
analogue**

Figure: Bianchi procedure

Enterocutaneous Fistula

• MC Cause- Iatrogenic
• Types – High output ( >500ml/day); Low output( <200 ml/day)
• MC complication of High output fistula- Skin excoriation**, Fluid electrolyte imbalance,
Sepsis.
• Highest electrolyte imbalance- Duodenal fistula

SURGERY SIXER APP BASED WORK-BOOK 2020 246


Principles in management of Enterocutaneous Fistula: ( SNAP)
• S- Sepsis Elimination
• N- Nutrition – a period of TPN
• A- Anatomical assessment
• P- Planned Surgery after 6 weeks

Factors preventing healing of fistula:


• F- Foreign Body in tract
• R- Radiation enteritis
• I- Inflammatory Bowel disease
• E- Epithelialization of tract
• N- Neoplasm
• D- Distal Obstruction
• S- Short tract ( < 2.5 cm tract)
• High Output fistulas also

Entero atmospheric fistula:


• Fistula is directly opening into the atmosphere and no intervening layers
• Poor healing

Congenital Megacolon= Hirschsprung disease


• Failure of migration of Neural crest.
• RET oncogene mutation is seen
• Absence of ganglion cells in Auerbach and Meissner’s plexus in constricted part of the
colon.
• Dilated part of the colon is absolutely normal

SURGERY SIXER APP BASED WORK-BOOK 2020 247


Figure explaining Hirschsprung disease

Locations:
• MC site- Rectum
• May involve entire left colon
• May involve entire colon ( Total Aganglionosis)
• May extend upto Small intestine also

Presentation:
• Newborn- Delayed passage of meconium
• Infants- Constipation without fecal soiling**
• Adults- Habitual constipation ( H/O self Enema)

Investigations:
• IOC- full thickness Biopsy of rectum above dentate line
• In newborn by Suction Biopsy
• Findings in Biopsy: Absence of ganglion cells in both myentric and submucous plexus* with
accompanying hypertrophy of nerve trunks and increased acetyl cholinesterase staining is
seen**
• Anal Manometry- Recto Anal Inhibitory reflex (RAIR) is absent** ( NEET SS question)
• Barium Enema-shows Question mark colon in Total aganglionosis**

Management:
• Resection of aganglionic segment and pull through operation.
• Duhamel, Swensen and Soave operations are types of pull through operations.

Angiodysplasia
• Abnormal dilated submucosal vessels known as Angiodysplasia
• MC in cecum and ascending colon> Rectum> Small bowel ( Jejunum)
• Produces Troublesome Bleeding by rupture of the vessels
• IOC- Colonoscopy and treated by Coagulation in same time.

SURGERY SIXER APP BASED WORK-BOOK 2020 248


• Heyde’s Syndrome- Type 2 VWD patients having- Angiodysplasia+ Valvular Aortic
Stenosis.

Small intestine Tumors


• Incidence = 2% inspite of such a major part in GUT.
• Reasons why tumors are rare in SB:
- Rapid transit of food particles and hence carcinogens don’t have exposure
- Peyer’s patches engulf the tumors
- Rapid epithelial turn over in Small bowel

MC’s Small Intestine Liver

Overall MC tumor Mets Mets


(MC from GIT Primaries) ( MC from Colorectal)
Extra GI Primary- Melanoma Extra GI Primary- Breast
MC benign tumor Adenoma ( Overall) Hemangioma
Leiomyoma ( Symptomatic)
MC primary Adeno carcinoma > Carcinoid HCC
Malignant tumor (MC site of Adenoca- Duodenum
MC site of Carcinoid- Appendix)
MC in Appendix Carcinoid > Adenocarcinoma

Adenocarcinoma in Small bowel


• Incidence-2%
• Associated with FAP, Crohn’s disease
• MC site- Duodenum 2nd part
• In Crohn’s Disease- Cancer is MC in terminal ileum
• MC in old age above 70 years among males
• Presents with mets mostly**
• MC clinical feature- Pain abdomen , Melena, Obstruction
• IOC- CECT Abdomen
• Elevated CEA is seen
• Resection with 10 cm margin is the treatment of choice including adjacent mesentry.
• Post op FOLFOX therapy is given.

Neuroendocrine tumors
• Also known as APUD omas ( Amine precursor Uptake and Deuptake tumors*
• All these tumors have Somatostatin Receptors and exhibit function.

Sites of NET:
• Lung- Small cell cancer
• Adrenal- Phaeochromocytoma
• Brain- Medulloblastoma

SURGERY SIXER APP BASED WORK-BOOK 2020 249


• CNS- Neuroblastoma
• Thyroid- Medullary cancer
• Pancreas- Insulinoma, Gastrinoma etc ( INSULINOMA has no Somatostatin Receptor)
• Intestine- Carcinoid**

Location Site Predominant Hormone

Foregut Bronchus and Thymus 5-Hydroxy Tryptophan and ACTH


( Low serotonin)*
Midgut Stomach, Proximal Duodenum, Serotonin, Histamine , Bradykinin
Ileum, jejunum, Right Colon
Hindgut Left Colon and rectum Somatostatin or Peptide YY**

• MC site – Bronchus > Appendix> Ileum( Terminal)


• MC in GIT- Appendix
• MC system affected is GIT system> Bronchus

Primary Carcinoids Secondary Carcinoids


Primary NET in GIT with no mets in liver NET in GIT + Liver mets also
The Carcinoid tumors produces substances Presents with Carcinoid Syndrome as the
which are detoxified into non toxic products active products are released into circulation
like 5- HIAA in liver.
Hence they won’t get any symptom
MC symptom- Pain abdomen> Diarrhea

Carcinoid syndrome:
• Vasomotor symptoms(80%)- cutaneous flushing is the MC symptom**.
Types of Flushing: Diffuse erythematous, Violaceous, Prolonged flushes and Bright red
patchy. MC type of Flushing seen in Gastric carcinoids- Bright red patchy
• GIT – explosive diarrhea. Due to serotonin.
• Cardiovascular –Right heart problems are more common. The three most common
cardiac lesions are pulmonary stenosis (90%), tricuspid insufficiency (47%), and tricuspid
stenosis (42%)*** MC valve affected is Tricuspid Valve. Pulmonary regurgitation is not
mentioned in books
• Asthmatic attacks- Broncho spasm
• Malabsorption and Pellagra (dementia, diarrhea, dermatitis) due to excessive diversion of
tryptophan.

Investigations :
• 24 hours urinary 5 hydroxy indole acetic acid(5HIAA)** are highly specific.
• Neuroendocrine tumour marker- chromogranin A and synaptophysin

SURGERY SIXER APP BASED WORK-BOOK 2020 250


• Localisation of tumor- Somatostatin receptor scintigraphy with indium-111 labelled
pentreotide was used previously.
• Somatostatin Receptor imaging with Gadolinium 68 DOTATATE CT is the most sensitive
to localise – 95% cases .

To look for metastasis:


• MRI is helpful only to detect Liver mets**
• FDG is taken up only in high-grade NETs (e.g., high Ki-67 expression), whereas most
NETs have low Ki-67 expression and are not apparent with this imaging modality- FDG
PET CT is only 38% helpful to detect mets
• DOTATATE is used with sensitivity of around 97% for mets now.( AIIMS 2019)

Extra edge points:


• Without liver mets- Carcinoid syndrome is seen in Retroperitoneal and Ovarian
carcinoids.
• Metastasis to liver depends on :
- Size ( < 1cm -10%; >1cm: 90%)
- Site ( Ileum has more chances , Appendix lowest)
- Ki 67 Index and Grade of tumor

• 2nd MC tumor in Carcinoid- Colorectal cancer


• Carcinoids may be multicentric nature

Management:
• Treatment- Surgical resection
o R0- No microscopic margin
o R1- Microscopic margin+
o R2- Macroscopic Margin+
• If there is liver mets- we must resect that also. Even with liver mets the NET have good
prognosis followed by Colorectal mets

• Carcinoid Small intestine- 10 cm margin resected with mesentry


• Carcinoid Appendix:
- Tip, < 1cm, No invasion into Mesoappendix: Appendectomy is enough
- More than > 2cm, Infiltration to base, Tumors in base- Right hemicolectomy
- 1-2 cm tumors in tip – Appendectomy is enough if no mesoappendix
involvement

Post surgical therapy:


• If residual tumor or micro or macro mets- LANREOTIDE is long acting Somatostatin
analogues. Pasireotide is latest.
SMART BOMBS:

SURGERY SIXER APP BASED WORK-BOOK 2020 251


• Radiolabelled Somatostatin Analogues that deliver radiation specifically to carcinoid
cells.

Inflammatory Bowel Disease

Crohn’s disease Ulcerative colitis


Incurable Curable
Bimodal Bimodal ( one at young and another at old
age)
IBD -1 ( CARD or NOD gene)- Chromosome IBD- 2 – Chromosome 12 q mutation
16q mutation
( Remember : BRCA -1 12 q) ( BRCA-2: 13 q)
Predisposing factors: Preventive factors:
- Smoking - Smoking
- Appendectomy
Involvement: Involvement:
• Mouth to anus • MC involves rectum.
• Unaffected- rectum** • Spreads retrograde upto caecum.
• MC involved: terminal ileum**- known • Unaffected – Anus**
as Terminal ileitis or regional enteritis. • Backwash Ileitis- Terminal ileum
involvement in 20% cases.
Vianna Classification: Types:
24 types based on (A,B, C) • Proctitis
• Age- <40 or >40 • Left Colitis
• Behaviour- Stricture/ Fistula etc • Pancolitis
• Site- Location
Clinical features: Clinical Features:
• MC symptom- Pain abdomen • MC symptom- Diarrhea
• Others- Bleeding and Diarrhea • Bleeding is MC here
• Earliest feature- Apthous ulcer** • Rare feature- Toxic Megacolon
• Strictures+ • Perforation
• Fistulas+
• Intestinal obstruction+
Extraintestinal Manifests: Extraintestinal Manifests:
• MC- Erythema Nodosum** • MC- Hepatic manifest ( Fatty liver)
• Ankylosing Spondylitis • MC skin manifest- Pyoderma
• Peripheral arthritis gangrenosum
• Kidney stones • Primary Sclerosing Cholangitis (
• Cholesterol Gall stones PSC)**
• Uveitis

SURGERY SIXER APP BASED WORK-BOOK 2020 252


• Scleritis
All the extraintestinal manifests disappears on
treatment- except PSC> Ankylosing
Spondylitis
Pathological features: Pathological features:
• Serpentine Ulcer**/ Bear claw ulcer • Collar button Ulcer
• Apthous ulcer • Partial thickness only involved
• Transmural involvement • Continuous lesion
• Skip lesions • No fistula, Stricture, Granuloma or
• Fistula fat wrap
• Non Caseating granuloma • Pseudopolyps appearance of
• Mesentric fat wrap uninvolved areas ( they are not
• Cobble stone appearance premalignant)
• Death- MC due to malignancy • Crypt Abscess, Cryptitis
• Toxic megacolon- Full thickness
involvement

Figure- Serpentine Ulcer

Figure: Pseudopolyps

Figure: Cobble stone

Investigations: Investigations:
• IOC/ Gold standard – Capsule • IOC – Colonoscopic Biopsy
endoscopy will show > 3ulcer in • Every 5 cm- 4 quadrant Biopsy is
absence of NSAID taken.
Biopsy: • Around 36 biopsies will be taken at
• Push endoscopy ( 90-150 cm), surveillance to look for dysplasia
• Double Balloon Endoscopy ( 240-360 • Antibody test: P- ANCA is positive
cm) ( Anti neutrophilic Cytoplasmic ab)

SURGERY SIXER APP BASED WORK-BOOK 2020 253


• Ileo Colonoscopy ( Terminal 50-80 cm
can be entered)
Stool for Lactiferrin and Calprotectin tests Instead of Barium enema- Instant Enema** is
used- With help of Gastrograffin and in an
• ASCA ( Anti sachharomyces Cervicia unprepared bowel.
antibody)+ in 90%

Figure- Double Balloon Endoscopy


Barium Investigations:
• String sign
• Stierlin sign
• Garden hose pipe appearance ( Bailey
says Lead pipe for Crohn’s by mistake)

Barium enema showing Loss of haustration


and known as Pipe stem/ Lead pipe
appearance.
CECT Abdomen:
Comb Sign- Increased vascularity to bowel
shown as comb sign*

Figure: Comb Sign


Recto anal features: Recto anal features:
• MC affected area in Colorectal and • Not involves anus
anal area- ANUS • MC causes Proctitis**

SURGERY SIXER APP BASED WORK-BOOK 2020 254


• Fissures- Multiple,Lateral position are
MC
• Fistulas-Multiple and doesn’t obey
GOODSALL rule
• Treatment of Crohn’s fistula- SETONS
and Infliximab are used instead of
surgery.
Complications: Complications:
• Fistula • Intractability
• Perforation • Toxic Megacolon
• Abscess • Perforation
• Stricture • Bleeding PR
• Intestinal obstruction ( MC due to • Cancer- MC site – Equally
stricture) distributed**
• Cancer ( MC in Terminal Ileum*) • In PSC+ UC- Cancer is mc in right
colon
Same list stands for Indications for Surgery** • Cancer risk increases after 8 years in
along with Intractable pain** Pancolitis and 12 years in Left colitis.
• MC indication for surgery is Stricture
causing obstruction
Cancer risk in colon with CD and Ulcerative
colitis is same risk ( Bailey mentions Cancer
risk in Table +++ for Crohns and ++ for
Ulcerative colitis )

MC cause of death in Both CD and UC-


Cancer

Management of Stricture: Gold standard Surgery:


• Single stricture-Resection • Total proctocolectomy + Ileal Pouch
• Multiple Short Strictures- Anal Anastomosis is advised after 8
Stricturoplasty by Heinke Mikulicz years in pancolitis or 12 years in Left
type Colitis. If they are not willing they are
• Multiple Long Strictures (>10cm)- advised to be in screening surveillance.
Stricturoplasty by Finney’s Type

Medical treatment:
1st Line drugs:
• Amino salicylic acid compounds- Sulphasalazine, Balasalazine, Olsalazine
• Mainly to maintain the remission
• Newer Drug- Mesalamine formulations releasing 5-ASA

SURGERY SIXER APP BASED WORK-BOOK 2020 255


o Timed release microgranules that release 5ASA throughout the Small intestine-
PENTASA
o Ph Sensitive resin that dissolves at pH 6-7 are Asacol , Lialda.
o Rowasa- Enema formulation
o Canasa- Suppository

Immunosuppressants:
• Azathioprine and mercaptopurine are 2nd line drugs

Initiation of remission:
• DOC- Steroids

Monoclonal Antibodies:
• Infliximab- TNF alpha inhibitor- Heals fistulas
• Natalizumab- Alpha 4 integrin ab (Causes Progressive multifocal leukoencephalopathy and
hence banned now- NEET SS**)
• Vedolizumab- Monoclonal antibody against Mad CAM-1 ( Mucosal addressin Cell adhesion
molecule)
Ileal pouch Anastomosis:
• MC used pouch is J pouch made with last 30 cm ileum ( two 15 cm limb folded) and
anastomosed to ileum
• MC complication after this procedure- Small bowel obstruction
• 2nd MC complication- Pouchitis**
• MC cause of pouchitis- Altered Colonic bacterial flora
• MC symptom of pouchitis- Increased bowel movements
• 1st line treatment for pouchitis- Probiotics, 2nd Line- Antibiotics, 3rd line- ASA
compounds. If they are not recovering with all the above- we do pouchectomy.
• Non responsiveness may be due to missed Crohn’s disease in which you made a pouch.

Figure- J pouch
Emergency cases:
• Toxic megacolon
• Perforation
• Bleeding massive

SURGERY SIXER APP BASED WORK-BOOK 2020 256


• Fulminant colitis

In these cases we don’t do TPC with IPAA- we go with Total colectomy with ileostomy and rectal
closure and we reopen and remove the rectum after some time and do Ileal pouch anal
anastomosis.

Types of Pouch:
• J , S and W pouch
• Best is J pouch- Length is 10-15 cm

Indications of Surgery in Ulcerative colitis


• 20% of the patients will undergo colectomy.
• Most of them will undergo surgery in 1 year itself
Elective indications Emergency Indications

1. Steroid related 1. Toxic megacolon


- Resistant 2. Fulminant colitis
- Dependency 3. Perforation
- Complications 4. Bleeding
2. Intractability
3. Dysplasia in colon
4. After 8 years in Pancolitis and 12 years in Left
colitis

Cancer and Ulcerative colitis: High Yield Points:


• Cancer is related to duration and extent directly proportional
• UC related cancer arises from Flat Dysplasia or Dysplasia associated mass lesions- But
sporadic always follows Adenoma carcinoma sequence*
• Evenly distributed in colon through out ( Sporadic Cancer mc in left side)
• Multiple Synchronous lesions are seen in UC
• Mucinous cancers are MC in UC, Mucinous cancers very rare in Sporadic type.
• Diagnosis is delayed because of Symptoms overlap
• Prognosis of cancer in UC related and Sporadic is almost same**

SURGERY SIXER APP BASED WORK-BOOK 2020 257


• Think of malignancy in Ulcerative colitis if they present with stricture with below
features:- MORE PROXIMAL TO SPLENIC FLEXURE, INCREASED DURATION AND
PRESENTS WITH OBSTRUCTION ( NEET SS question)

Polyps and Cancer

Hamartomatous polyps Hyperplastic Polyps Adenomatous polyps


Less risk of malignancy Around 2% risk • Tubular- 5-10% risk
<1% • Tubulo Villous- 10-20% risk
• Villous – 40%
• Pseudo polyps and inflammatory polyps have no risk of malignancy**

Hamartomatous Polyps syndromes:


Peutz Jeugher Syndrome:
• STK 11 Mutation- Chromosome 19 p
• MC site of Polyps- Jejunum
• Cutaneous marker- Oral and lips pigmentation
• Other cancers- Breast, Ovary and Thyroid
• 100 times risk of pancreatic cancer**
• Bile duct and GB cancer
• Colon cancer also seen rarely
• MC symptom – Pain abdomen and Bleeding
• Pain due to intussusception.

Juvenile Polyposis Syndrome ( JPS)


• SMAD -4 mutation ( Chromosome 10 q)
• DEFINED by > 10 polyps in colon.
• Mc site of polyps- Colon
• Single polyp- Polypectomy
• Multiple polyps- Colectomy needed.
• Associated with:
- Hydrocephalus
- Malrotation
- Meckel’s diverticulum
• Increased risk of Colon and Pancreatic cancer

Cowden Disease:
• Predisposing factor of Cancer breast ( 50%) and Thyroid ( 10%)
• Chromosome 10 q mutation
• Polyps seen in Stomach and colon

Banayan Ruvalcaba syndrome:

SURGERY SIXER APP BASED WORK-BOOK 2020 258


• Chromosome 10 q mutation
• All syndromes are AD
• Hamartomatous polyps in colon
• Other features:
- Glans penis pigments
- Dysmorphic features
- Macrocephaly

Cronkite Canada syndrome:


• Juvenile polyps in colon is also hamartomatous nature

Haggit’s Criteria of Polyps invaded by Malignancy:


Adenomatous polyp:
• Level 1- Involvement of Head only upto Submucosa
• Level 2- Involvement of Neck upto submucosa
• Level 3- Involvement of Stalk upto submucosa
• Level 4- Involvement of Base upto submucosa
Sessile Polyp:
• Only one level – Level 4

Figure: Haggit’s criteria

Tubular adenomatous polyps syndromes:


Familial Adenomatous polyps HNPCC
5q mutation in APC gene Microsatellite Instability
Autosomal dominant syndrome Mutation of MMR gene
MLH-1( MC), MSH -2, MSH-6

SURGERY SIXER APP BASED WORK-BOOK 2020 259


More than 100 polyps seen in Colon Name hereditary non polyposis is a misnomer,
They also have polyps- Polyps < 100 number
Incidence-1% Incidence- 3-5%
History of FAP: History of HNPCC:
• 10 years polyps develop • 20 years polyps develops
• Before high school( <20 years)- • Around 40-50 years cancer develops
Develops cancer • 2nd mc cancer- Endometrial cancer.
• By 40 years they all die

Most common mode of screening is by To do screening we must select the high risk
checking for APC Truncation mutation** cases by using the following Criteria:
Confirmation by DNA Sequencing** 1; Amsterdam Criteria:
- 3 members affected
• Classic FAP have mutation in 1250 to - 2 are 1st degree relative
1450 codon in 5’---3’ gene of APC. - 1 fellow < 50 years
• Mutation proximal to 1250 or distal - FAP excluded
to 1450 are attenuated FAP and have
less polyps. 2. Modified Amsterdam criteria:
• Rectal polyps are absent in Mutation - Along with above other cancers are
proximal to 1250 codon ( NEET SS) included- CESUR
in 5’ end - C- colorectal, E- Endometrial, S-
Stomach and SI, U- Urinary tract and
R- Renal

3. Bethesda Criteria:
- Includes polyps also in criteria
Surveillance: Surveillance:
• Colonoscopy screening- from 10 years • Colonoscopy surveillance- from 20
• Before 20 years we must do years
Prophylactic Total Proctocolectomy** • No prophylactic surgery- Total
• 2nd MC cancer- Duodenal and colectomy done between 40-50 years
periampullary cancer once the cancer develops.
• Hence Endoscopy screening started • Endometrial cancer screening done
from 20-25 years onwars from 25 years onwards by TVS and
D&C.
Other features: Other features:
MC site of Cancer- Left Colon MC in right colon
Synchronous and metachronous tumors seen
Associated syndromes: Associated syndromes:
Gardner syndrome: AD Lynch -1 :
- FAP + • Only CRC
Lynch -2: ( EGO- SP-TUB)

SURGERY SIXER APP BASED WORK-BOOK 2020 260


- CHRPE (MC extraintestinal manifest)- • CRC Associated with other cancers
Congenital Hypertrophy of Retinal • Endometrial cancer ( 50%)
Pigment Epithelium • Gastric cancer
- Desmoid Tumors • Ovarian cancer
- Exostosis of Bone • SI cancer, Pancreas cancer, Thyroid
- Epidermoid Cyst( Multiple Sebaceous Cancer, Urinary tract cancer and
cysts) Brain tumor** ( SP- TUB)
Turcot Syndrome: AR • MC brain tumor is Glioblastoma
- FAP + Brain tumor ( Medulloblastoma) Muir Torre syndrome:
• HNPCC with multiple Sebaceous cyst
Management: Management:
Prophylactic TPC+ IPAA • Once they develop cancer at around
40-50 year we do Total colectomy
and ileorectal anastomosis

Extra edge points:


• Spigelman classification: Duodenal polyps in FAP patients going for Cancer. Based on
this classification we will plan further management of Duodenal polyps.
• Special Management of FAP in young males:
- Young males with mutation at 5’ end before 1250 codon with rectal polyps we can
preserve the rectum and do Total colectomy+ Ileorectal anastomosis
Cancer Colorectal region

Right colon Left colon


[cauliflower/ fungating type} [ stenosing type or stricture type]
- Chronic blood loss - Alteration in bowel habits
- Anemia - Hematochezia
- Melena - Obstruction

SURGERY SIXER APP BASED WORK-BOOK 2020 261


- Mass in RIF

Fearon Vogelstein sequence of Adenoma carcinoma:


• Initiation- APC gene ( 59 gene)- 1st hit
• Promotion- HNPCC gene(MSI)
• In between- K ras ( 12p); DCC( 18q);
• Last hit- P53 gene (17 p)- MOST COMMON MUTATION IN CRC**

Predisposing factors of Carcinoma Colon:


Dietary factors:
- High Animal fat diet
- Low fibre diet
- Low Carbohydrate diet
- Lack of anti oxidants

Inflammatory Bowel Disease:


- Ulcerative colitis
- Crohn’s disease

Iatrogenic:
- Radiotherapy to colon
- Uretero-sigmoidostomy after bladder resection
Investigations for CRC:
• Diagnosis- Colonoscopic Biopsy
• T,N and M staging- CECT abdomen ( MC site of Distant mets- Liver)
• Tumor marker- CEA** ( Normal value < 5 ng/ml)
• Apple core in Barium enema

Figure shows Apple core

TNM Staging:
• T1- Mucosa, Submucosa
• T2- Muscularis propria
• T3- Pericolic tissues and upto serosa only
• T4- Adjacent structures beyond serosa

SURGERY SIXER APP BASED WORK-BOOK 2020 262


• N1- 1-3 nodes+
• N2- 4 or more nodes+
Stage 1- T1 and T2
Stage 2- T3 and T4
Stage 3- Nodes+
Stage 4- Mets+

Modified Astler Coller Staging System:


A- Tumor Limited upto Mucosa
B1- Tumor up to SM and Muscularis propria C1- B1 + nodes
B2- Tumor beyond Muscularis propria C2- B2+ nodes
B3- Tumor beyond Muscularis propria with C3- B3+ Nodes
adjacent structure involvement
D- Distant mets

• Prognosis of the Cancer depends upon DEPTH OF INFILTRATION** into colonic lumen(
BEST prognostic )Lymph node mets, CEA level and presence of mets.

Surgical treatment of Colon Cancers:


Right hemicolectomy Extended Right Left Hemicolectomy
Hemicolectomy
Last 30 cm ileum Extended resection upto Removal of
Cecum splenic flexure is done Left 1/3rd transverse
Appendix Descending
Ascending colon Sigmoid
Upto right 2/3 rd colon Upper part of rectum
- Ligation of ICA, RCA
and MCA happens - Ligation of LCA and
All 4 level lymph nodes Sigmoid arteries
removed - Previously high
ligation of IMA done,
now ligated leaving
some stump at IMA
level.
Indications: Indications: Indications:
- Caecal cancer - Transverse colon - Splenic flexure
- Ascending colon growth - Descending colon
cancer - Hepatic Flexure - Sigmoid Colon
growth
- Splenic flexure growth

SURGERY SIXER APP BASED WORK-BOOK 2020 263


We must also remove 4 level of nodes while operating
• Epicolic nodes- On colonic wall
• Paracolic nodes- On marginal artery
• Intermediate nodes- Along branch vessels
• Principal nodes-At origin of SMA and IMA

Minimum number of nodes removed= 12 nodes**

Adjuvant therapy is only Chemotherapy:


Stage of cancer in HPE Post op therapy
Stage 1- T1 /T2 No further treatment
Stage 2- T3/T4 No further treatment except high risk cases in Stage 2
High risk cases in Stage 2:
- < 12 nodes removed
- Poorly differentiated cancers
- T4 cancers
- Perforated and obstructed cancers

FOLFOX therapy is given now adays for 6 cylces


• Folinic acid ( Leucovorin)
• 5-FU ( Continuous infusion advised ) or Oral form
Capecitabine can be used
• Oxaliplatin
Stage 3- Nodes + FOLFOX for 6 cycles
Stage 4 – Mets+ Lung met or Liver mets if resectable and < 3 numbers we can
resect them .
Provide 6 cylces of FOLFIRI instead of FOLFOX
Irinotecan is used**
All stage 4 must undergo K- ras negative= Wild K-ras cases:
testing for K-ras mutation • Additional Monoclonal ab has benefit.
• Cetuximab and Panitumab – EGFR antibodies are
given

SURGERY SIXER APP BASED WORK-BOOK 2020 264


• Bevacizumab- VEGR antibodies

Follow up of colorectal cancer:


• CEA- 3 monthly for 2 years , 6 monthly for 2-5 years
• Colonoscopy- Yearly once for 5 years. ( No need after 5 years)
• CECT abdomen – 1-3 years annually ( No need after 3 years)

Appendix

• Appendix is a vestigial organ**


• But some procedures use appendix:
• Mitranoff procedure: Appendico vesicostomy for Chronic Bladder
catheterization*
• Malone Procedure: Conduit for repeated Antegrade enema or
medication via Appendix to the rest of colon.

Anatomy:
• MC position of appendix- Retrocecal ( 60%), Pelvic position ( 20-25%), others are
preileal, post ileal and subcecal positions.
• Valve in appendix- Gerlach valve**

Named valves:
• Heister- Cystic duct and GB
• Houston- Rectum
• Gerlach- Appendix
• Bauhin- Ileocecal valve
• Kerkring/ Conniventes- Jejunum

Acute Appendicitis:
• MC emergency done in worldwide- Acute appendicitis
• MURPHY’S TRIAD- Migratory pain, vomiting, fever

Common Signs in Appendicitis


• Mc Burney’s Point tenderness- tenderness at the point ( Medial 2/3 and lateral 1/3rd
of spino umbilical line- Mc Burney point)
• Blumberg sign- Rebound tenderness
• Rovsing’s sign- palpation of left iliac fossa produces pain in right iliac fossa by shift of
bowels.
• Sherrens triangle hyperaesthesia- (triangle formed by ASIS, umbilicus and pubic
symphysis) due to irritation of lower abdominal nerves

SURGERY SIXER APP BASED WORK-BOOK 2020 265


• Copes psoas sign- Retrocaecal appendicitis* on extension of hip produces pain due to
irritation over psoas major[ muscle that flexes the hip]
• Copes obturator sign- Pelvic appendicitis* , Flexion and Medial rotation produces pain

Not done signs:


• Dunphy’s Sign- Pain on Coughing at RIF.
• Ten Horn Sign- Pain in RIF on gentle traction of Right testicle.
• Aaron Sign- Pain on epigastrium on persistent pressure at Mc Burney’s point
• Bastedes Sign- Pain and tenderness in RIF on rectal air insufflation

ALVARADO SCORING in Acute Appendicitis (Mnemonic- MANTRELS)


o M-migratory pain
o A- anorexia
o N- nausea and vomiting
o T-tenderness* ( 2 points)
o R- rebound tenderness-
o E- elevated temperature
o L- Leukocytosis* ( 2 points)
o S- shift to left, segmented neutrophils

• All are given 1 point except tenderness and Leukocytosis given 2 points**
• SCORE OF > 7 IS STRONGLY SUGGESTIVE OF APPENDICITIS
Investigations :
1st done- USG abdomen: Findings in Acute appendicitis- 80% sensitive
- Blind ending
- Non Peristaltic
- Non compressible
- > 7 mm Thickness

Figure – Acute appendicitis

IOC for acute appendicitis: CECT abdomen:


- Extremes of Age
- Doubtful cases
- 2nd trimester of pregnancy ( If MRI is not available as MRI is the IOC**)

SURGERY SIXER APP BASED WORK-BOOK 2020 266


Etiology for Acute appendicitis:
• MC – fecolith**
• Ascaris
• Tumors

MC organism isolated – Bacteroides fragilis**


MC aerobe- E.Coli

Complications of Acute appendicitis:


1. Rupture/perforation
2. Abscess
3. Appendicular mass

Appendicular rupture:
• MC in Extremes of age, Fecolith, Diabetic and Immunocompromised patients
• MC in Antimesentric border at the tip.
• MC in pelvic position of appendix**
• Rupture in < 24 hours- Diffuse peritonitis
• Rupture in > 24 hours- Due to omental covering localised peritonitis and its better.
• Early antibiotics doesn’t prevent rupture.
• C/E- Guarding and rigidity+
• Emergency appendectomy done

Appendicular abscess:
• Retroperitoneal or pelvic abscess happens.
• C/E- fever+ Chills and rigors.
• Treatment- USG guided drainage of abscess.
• Retrocecal abscess- retroperitoneal drainage
• Pelvic abscess- via PR/PV
• Index appendectomy during same admission advised.

Appendicular mass:
• The regimen followed is OCHSNER SHERREN REGIMEN**
• After a gap of 6 weeks we must give Interval appendectomy**

Ochsner Sherren regimen:


• Conservative treatment of mass
• The mass is formed by surrounding omentum, small intestine and dense inflammation;
if we dissect at this time EC fistula can happen.
• In case you open the abdomen and found a mass- don’t dissect and follow this regimen.
• Nil oral until paralytic ileus recovers

SURGERY SIXER APP BASED WORK-BOOK 2020 267


• IVF, Antibiotics and Input/ Output monitoring, Electrolyte correction
• Charts for Pulse temperature and mass size.
• Once symptoms resolve- discharge and ask them to come after 6 weeks for surgery
Interval appendectomy.

When will you abandon this regimen and operate?


• Increased PR
• Increasing Temperature
• Vomiting+ or Increased Ryles tube aspiration
• Mass increasing or not resolving
• Rule out underlying cancer cecum in old age cases.

Surgical treatment:
Conventional Open method:
• Grid iron Incision at mc burney point known as GRID IRON Incision- Injury to Ilio
hypogastric nerve can happen and patient may develop DIRECT HERNIA.
• Females- LANZ horizontal cosmetic incision.
• Usually we reach the appendix by muscle splitting
• Muscle cutting incision- Rutherford morrison Incision*

Laparoscopic method:
• 3 ports are used
• Endoloop is used to tie the base.
• 1st Done by KURT SEMM**

Recent updates in Sabiston 20th edition:


Special situations during Surgery:
Situation 1: Normal appendix found during surgery ( Negative appendix)
• Examine for other pathologies like Meckel’s, Diverticulitis, Uterine pathologies etc.
• Appendicectomy is done to avoid future confusions.

Situation 2: CROHN’S and APPENDIX


• Active Crohn’s seen at base of a normal appendix and Caecum:To avoid development of
Fistula leave the appendix there itself- Don’t remove
• No Crohn’s disease involvement at the base but actively involves only terminal ileum - we
can remove the appendix** to avoid future confusions.
• Active Crohn’s at base and appendix also turgid and inflamed- Take risk and remove the
appendix; don’t crush the base in such cases.

Situation 3: Can we perform Incidental appendectomy during some other surgery?


• No, don’t do. Its not cost effective.

SURGERY SIXER APP BASED WORK-BOOK 2020 268


Situation 4: Uncomplicated appendicitis- Can we manage by Non Operative method?
• Yes, in selected patients with high risk of surgery we can manage conservatively with
antibiotics and IV fluids

Situation 5: Pregnancy with appendicitis:


• MC surgical Non-Obstetric emergency in pregnant
• MC in 2nd Trimester
• Laparoscopic Appendectomy is advised by Open Port technique*
• To confirm the diagnosis USG is used , but in cases of Doubt CT scan is also advised
which avoids the risk of negative appendectomies.
• IOC is MRI in pregnancy.
• NO wait and watch advised in pregnancy

Tumors of Appendix

NET tumors- Carcinoid Epithelial tumors in Appendix (DELPHI Classification)


MC tumor in Appendix- 0.9% Adenomas:
- Tubular
- Tubulo villous
- Villous

Adenocarcinoma ( <0.5%)
- Mucinous
- Signet ring cell type

Types of Mucinous tumors:


• Low grade appendiceal mucinous neoplasms
(LAMN)
• High grade appendiceal mucinous
neoplasms (HAMN)
• Mucinous adenocarcinoma
These 3 cases can rupture and produce
PSEUDOMYXOMA PERITONEI**

Pseudomyxoma Peritonei (PMP)


MC follows appendiceal neoplasm rupture> Ovarian neoplasm
• LAMN- Just keep them on follow up
• HAMN- Chances of rupture more and hence advise Right Hemicolectomy+ Limited
Peritonectomy+ Bilateral Salphingo oophorectomy
• Established PMP- Cytoreductive surgery+ HIPEC ( SUGARBAKER PROCEDURE)

SURGERY SIXER APP BASED WORK-BOOK 2020 269


• Cytoreductive surgery ( CRS)- Right hemicolectomy+ BSO+ Cholecystectomy+
Splenectomy+ Omentectomy+ peritonectomy+ sometimes gastrectomy+ Colectomy+ Rectal
resection and this procedure takes 10 hours
• HIPEC- Hyperthermic Intraperitoneal Chemotherapy**
• Inspite of all therapies they may recur and have poor prognosis

Intestinal Obstruction

Classification of Dynamic Obstruction


Paediatric causes Adult Causes
Newborn: Small bowel causes:
• MC cause is Duodenal atresia • Adhesions ( MC)
• Jejunal atresia • Hernia
• Ileal atresia • Stricture
• Annular pancreas • Tumors
• Midgut Volvulus • Congenital bands

Other children: Large bowel causes


• MC cause is Intussusception • Cancer
• Hernia • Volvulus
• Tumors
• strictures

Four Cardinal features of Dynamic Obstruction:


• Colicky pain-If the bowel goes for gangrene it becomes Dull aching continuous pain
• Abdominal distension- MC by Swallowed gas> GI secretions
• Vomiting- Projectile Bilious Vomiting
• Obstipation- Not pass motion + Flatus
Causes of Small bowel obstruction which produces No obstipation:
• Richter’s hernia (Diarrhea seen)
• Acute SMA ischemia
• Gall Stone ileus

On opening the abdomen- bowel is distended massively: How to differentiate it’s small bowel or
Large bowel by seeing one organ?
• Cecum
• Cecum Distended is Large Bowel obstruction
• Cecum is collapsed is Small Bowel Obstruction
• If Ileocecal valve is not patent- Cecum will not be dilated even in Large Bowel obstruction
and entire small bowel also dilated**

SURGERY SIXER APP BASED WORK-BOOK 2020 270


Paediatric causes of Intestinal obstruction

Intussusception:
• MC in age group 6 months – 6 years
• MC cause due to Rhinovirus infection causing Hypertrophic peyers patches and that pulls
the bowel and causes intussusception.
• Anatomy:
- Apex: Lead point
- Innertube: Intussusceptum
- Outer tube: Intussuscipien

Figure: Anatomy of Intussusception


• MC type is Ileo Colic
• MC type in Old Age – Colo Colic
• Apex: Lead points:
- MC lead point: Hypertrophied payers patches
- Benign tumors
- Meckels
- Cancer ( MC in old age)

Clinical features in 6 months Baby:


• Red currant jelly stool
• Vomiting
• Le dance sign- Empty Right iliac fossa
• Sausage shaped mass towards umbilicus

USG features/ CT Features:


• Target sign
• Pseudo kidney sign
• Bull’s eye sign

SURGERY SIXER APP BASED WORK-BOOK 2020 271


Figure: Target sign

Barium Enema:
• Claw Sign/ Pincer sign
• Coiled Spring Sign

Figure: Coiled Spring Sign

Figure : Claw sign


Management:
• Infants: USG guided saline or air enema is done
• Others: Surgical reduction is done- Don’t pull the inner tube from back , Always push it
from above downwards by Milking.

Meconium Ileus:
• New born Babies pass motion in 48 hours usually.
• If not passed after 48 hours think of meconium ileus
• MC cause- Cystic fibrosis ( Mucoviscidosis)

SURGERY SIXER APP BASED WORK-BOOK 2020 272


• X ray shows: No air fluid level due to thick meconium**
• Egg shell Calcification seen in cases of meconium peritonitis**
• IOC- Gastrograffin Contrast study
• 1st Line treatment- MYPAQUE ENEMA**
• 2nd Line Treatment- Bishop koop operation** ( Chimney shaped Ileostomy)

Figure: Chimney Shaped Ileostomy of Bishop koop

Malrotation of Gut:
• Midgut is supplied by SMA*, by 6th week of IUL umbilicus is opened and the entire midgut
comes out and goes back by 10th week and goes to normal anatomy.
• Ladd’s Band if present extends from Duodenum to Ascending colon**
• This Ladd’s Band prevents rotation and normal fixation of bowel.

Problems encountered:
• Non rotation of GUT with straight duodenum with unrotated bowel . The Ladd’s band
may compress the duodenum and causes vomiting. May present late in life also.
• Partial Malrotations
• Dangerous: Midgut Volvulus: The bowel goes for rotation in a clockwise rotation** and
results in bowel ischemia. Serious emergency in new born.

SURGERY SIXER APP BASED WORK-BOOK 2020 273


Midgut Volvulus:
• Bilious vomiting+ Pain abdomen
• IOC – GI contrast study is done.
• CECT abdomen- Whirl Pool Sign**
• Treatment for Both Non rotation or Volvulus is LADD’S PROCEDURE**

LADD’S Procedure:
• Untwist the volvulus
• Remove the Ladd Band
• Keep all small bowel in right side and large bowel in left side
• Appendix also removed.

Adult causes of Small bowel Obstruction

Erect X ray findings:


• Normal air fluid level: 3-5
• > 5 air fluid level is pathological
• Air fluid level with vertical diameter: > 2.5 cm it is pathological**
• Step Ladder pattern is seen**
• String of pearls appearance is also seen if more of fluids present and less gas seen.
• Xray shows- No gas on Colon and rectum

Figure: Step ladder pattern

Dynamic Obstruction Non Dynamic Obstruction ( Paralytic ileus)


MC cause- Adhesions* MC cause -Post operative patient**
Electrolyte imbalance:
- MC hypokalemia**
- Na, Mg, Cl disturbances
Hypothyroidism
Neurological causes
Four cardinal features are seen Pain is absent**

SURGERY SIXER APP BASED WORK-BOOK 2020 274


Bowel sounds +++ ( Borborgami) Absent Bowel Sounds
Xray shows: Xray shows:
• Multiple Air fluid levels • Gas + in rectum and colon
• No gas in rectum
Management: Management:
Adhesive obstruction: • NPO
• NPO, IVF • IVF
• For 24-48 hours • Electrolyte correction
• > 48 hours : Lap adhesiolysis** • Wait for 48-72 hours
Other causes: • If not resolved: CATCHPOLE
• Surgery Immediately done** REGIMEN** Injection Neostigmine**
( Sun should never set or raise)

Order of recovery of bowel as per Bailey in Ileus:


• Small bowel- 24 hours
• Colon
• Stomach (Last to recover)

Adult causes of Large Bowel Obstruction


• MC causes- Cancer
• 2nd cause- Volvulus

Caecal Volvulus Sigmoid Volvulus


• Misnomer- its actually a Caeco colic • MC type
volvulus • MC in old men
• LC • Predisposed by High fibre diet**
• MC in young Females
• Predisposed by repeated pregnancy
• MC in clockwise direction • MC in Anticlockwise > Clockwise
• Please remember antero posterior
rotation of only caecum alone is
known as Caecal Bascule**
• X ray shows- Comma shaped colon • Xray shows: Coffee bean shaped
• Barium enema- Ace of spade or Bird
Beak appearance
• Treatment: Always Right • 1st line: Sigmoidoscopic reduction
Hemicolectomy • Definitive treatment: Sigmoid
• Cecopexy is contraindicated colectomy

SURGERY SIXER APP BASED WORK-BOOK 2020 275


Figure: Coffee Bean appearance

Pseudo obstruction of Colon:


• Acute- Ogilvie syndrome
• Chronic-Neurological causes, Laxative abuse, Self enema .

C/F- Abdominal distension, Painless, severe constipation


Cecum if becomes distended > 14 cm and not passing motion – colonoscopic decompression is
done
2nd Line treatment:
• Injection Neostigmine by making the patient sit in commode.

Final treatment:
• Subtotal colectomy**

Hartman’s operation: Paul Mikulicz Operation:


Composed of three components: • Patient very sick
• Resect the diseased colon • Bring the proximal colon as colostomy
• Proximal colostomy and leave the pathology thereitself and
• Distal closure and leave inside bring the distal bowel as mucus fistula
Redo Anastomosis done after 6 weeks

SURGERY SIXER APP BASED WORK-BOOK 2020 276


Figures: Hartman’s and Paul Mikulicz operations

Stomas

• Content from Ileostomy is liquid and more damaging to skin


• Content from Colostomy is solid stool and no skin damage
• Types of stoma: End or Loop type

Indications for Stoma:


Elective Emergency
• Diversion purpose after low anterior • After Hartman’s
resection cases. • After Paul Mikulicz operation
• For divert the stools in distal
obstruction

Indications of permanent stoma:


• Ulcerative colitis involving anus also- TPC+ Anus also removed and do Permanent
ileostomy

SURGERY SIXER APP BASED WORK-BOOK 2020 277


• In Cancer rectum- After APR- Permanent Colostomy*
Complications of Stoma:
Ileostomy:
• MC complication- Skin Excoriation
• MC early complication- Ileostomy Necrosis
• Fluid Electrolyte imbalance
• Parastomal hernia
• Stomal prolapse

Colostomy:
• Stomal prolapse
• Parastomal hernia

Figure: Stomal prolapse

• MC complication of Colostomy- Parastomal hernia**


• MC indication of redo surgery in Colostomy- Parastomal hernia
• SUGARBAKER- Double mesh technique is done for parastomal hernia

NEET SS question:
Parastomal hernia is MC after End Colostomy
Stomal Prolapse is MC after Loop Colostomy ( Distal loop prolapses more than proximal)

Chapter 4: Rectum and Anus

Basic anatomy:
• Rectum is 12-15 cm length
• 3 houston valves:
o Left side: upper and lower are present
o Right side- Middle is seen
• Puborectalis muscle is seen at anorectal junction
• Anus is 4 cm

SURGERY SIXER APP BASED WORK-BOOK 2020 278


• Upper 1/3rd rectum is fully intraperitoneal
• Middle 1/3rd anterior part is intraperitoneal
• Lower 1/3rs is fully extraperitoneal

Anal anatomy:
• Divided into parts based on Dentate line ( Pectinate line)
• Above the line is columnar epithelium
• Below the line is Squamous epithelium

Above pectinate line Below pectinate line

Embryology- derived from primitive Derived from proctodeum**


anorectal canal
Lymphatic drainage- Internal iliac nodes Superficial inguinal nodes

Autonomic nerve supply Somatic – Inferior rectal nerves

Artery – Superior rectal artery Inferior rectal artery

Pathology in this zone is painless Pathology in this zone is painful

Muscles around the Anus:


• Circular muscle of rectum continues as Internal sphincter of anus.
• Internal sphincter is involuntary control and maintains the resting pressure of anus**
• Three parts of External sphincter- Deep, Superficial and Subcutaneous parts.
• Ischiorectal fossa is seen outside the External sphincter

SURGERY SIXER APP BASED WORK-BOOK 2020 279


Haemorrhoids:
• Prolapsing Anal Cushion is known as haemorrhoids
• Painless bleeding PR seen in Grade 1 and 2
• Painful Bleeding PR seen in Grade 3 and 4

Grade 1- No prolapsing mass.


Grade 2- Prolapse via anus but gets reduced spontaneously
Grade 3- Prolapse via anus and gets reduced manually by himself
Grade 4- Prolapsed and not reducible

• Grade 1 and 2 hemorrhoids are done as office( OP procedure)


• Grade 1- using GABRIEL Syringe we can inject 5% phenol in almond oil as sclerosant
• Grade 2- We can do Banding.
• Grade 3 and 4 are operated under Spinal Anesthesia

Surgical techniques:
• Miligan Morgan operation- Open Hemorrhoidectomy – removal of haemorrhoids at 3,7
and 11’o Clock position. After the surgery “anus looks like a clover the problem is over “-
it means there wont be future stricture.
• Ferguson operation-Closed hemorrhoidectomy*
• Newer operations: LONGO procedure ( Using Circular stapler )
• HALO- Hemorrhoidal artery Ligation operation

SURGERY SIXER APP BASED WORK-BOOK 2020 280


Figure: Miligan morgan operation – Clover shape
Please note; 5 day self subsiding haemorrhoid is thrombosed external haemorrhoids.
Fissure in Ano:
• MC in 6’ o clock
• In pregnancy- 12’0 clock
• In Crohn’s- Lateral positions
• C/F- Severe painful defecation with Spasm and Bleeding P/R
• Sentinel tag- Protective tag present in Chronic Fissure.
• Management:
- Conservative: Laxatives, Glyceryl trinitrate ointment local application, Diltiazem
Application, SITZ bath
- Surgical treatment of Choice- Lateral Sphincterotomy** - Internal sphincter cut at
lateral position ( 3/9’0 Clock position)- NOTARO’S Operation

Fistula in Ano:
• Communication of Anal epithelium with Perianal skin
• MC cause is Cryptoglandular Abscess*
• Park’s Classification:
o Type 1- Inter sphincteric Fistula ( MC type)
o Type 2- Trans sphincteric Fistula
o Type 3- Supra sphincteric fistula
o Type 4- Extra Levator Fistula
o Type 1 and 2- Low Fistula
o Type 3 and 4 – High Fistula

SURGERY SIXER APP BASED WORK-BOOK 2020 281


Goodsall’s Rule:
• Anterior Fistulas drain straight
• Posterior Fistulas drain by forming a Horse shoe tract
• If tract is > 3.5 cm anteriorly– it is an exception – It forms a long tract and drain
posteriorly.

Management of fistula:
• Type 1 and 2- Fistulectomy or Fistulotomy
• Type 3 and 4- Seton application done
Newer treatments:
• Biological treatment- Small intestine Submucosa of Porcine ( SIS Plug)
• VAFT- Video assisted Fistula tract excision
• LIFT- Ligation of intersphinteric Fistula Tract

Pilonidal sinus:
• Hair in the gluteal region enters gluteal cleft in jeep drivers and hence known as
JEEPER’S Bottom or Pilonidal sinu
• The tract will have hair
• They get infected and cause pain and discharge*
• Lay open the tract and core the tract completely.

SURGERY SIXER APP BASED WORK-BOOK 2020 282


• Treatment for recurrence: Bascom’s technique or Karydakis flap or Rhomboidal flap(
Limberg flap) operations are done.
• Operation is done in PRONE JACK Knife position**

Figure: Limberg Flap

Rectal prolapse:
• Partial and complete types
• Pathophysiology- repeated pregnancy, Psychiatric patients, Old age
• C/F- mass descending, Bleeding, discharge and decubitus ulcers.
• Surgery is advised for Complete rectal prolapses.

Perineal approach Abdominal approach


Useful in very old age who cannot withstand Procedure of choice \ in young age
abdominal operation
Recurrence is more ( >20%) Least recurrence
• Delormes mucosectomy operation of • Ripstein’s anterior rectopexy
removal of rectal mucosa and • Well’s posterior rectopexy
placating the muscle layer* • Resection rectopexy (Frykman and
• Thiersch anal encirclement operation Goldberg) for redundant sigmoid colon
• Altemier’s perineal recto • Anterior resection
sigmoidectomy procedure with Colo
anal anastomosis*- Resection
procedure**

Solitary rectal ulcer syndrome: (SRUS)


• Lesion is seen on anterior wall of rectum
• It’s a misnomer- It can be an ulcer or polyp
• It’s another misnomer- It can be multiple in 20%
• MC in young females* ( MC in female nurses and doctors)
• C/F- Constipation, After defection they feel something remaining in rectum- they will do
digital evacuation*; sense of incomplete evacuation
• Treatment- Laxatives, Biofeedback Exercises**
• The one etiology that may cause this known as internal rectal prolapse, and it can be
operated.

SURGERY SIXER APP BASED WORK-BOOK 2020 283


Investigations for rectum and Anus:
• Proctoscopy- examine 8-10 cm
• Rigid Sigmoidoscopy- 16-18 cm
• Flexible sigmoidoscopy- 60 cm
• Colonoscopy- 160 cm

Figures showing proctoscope and Rigid sigmoidoscope

• IOC / Gold standard for Fistula- MRI Fistulogram


• IOC for internal rectal prolapse- MRI defecogram

Imperforate Anus:
• Cloaca fails to open and there is no anal opening seen at birth
• Now as soon as the baby is born our aim is to find out where the GIT ends( rectum ends
inside abdominal cavity)
• Types of Imperforate Anus:
o High – The ending of bowel is above pelvic floor
o Low- The ending of bowel is below pelvic floor
• IOC to detect the type – INVERTOGRAM
• Invertogram taken after 24 hours ** ( Latest answer)- the gas swallowed by the baby
acts as a contrast.
• Treatment for Low Imperforate anus- PSARP ( PENA operation)- Posterior Sagital
Anorecto plasty operation.
• High imperforate anus- Staged operation- 1st stage I will do colostomy and repair in 2nd
sitting.

SURGERY SIXER APP BASED WORK-BOOK 2020 284


Image based questions

Gabriel Syringe
Longo operation

Jack knife position

Invertogram

Malignant diseases of Rectum and Anus

Classification

Rectal cancer Anus cancers

• Adenocarcinomas From Columnar epithelium From Squamous epithelium


• MC site of Colorectal Anal Canal tumors Anal Margin Tumors
cancers • Adeno cancers • SCC ( MC type
• MC symptom- • Epidermoid overall)
Bleeding P/R cancers • BCC
• Melanomas • Verrucous
• Bowen’s
Melanoma is mc in skin>
Eyes> anus (3rd)

Clinical features:
• MC symptom- Bleeding P/P
• Early morning Spurious Diarrhea

SURGERY SIXER APP BASED WORK-BOOK 2020 285


IOC:
• Diagnose- Biopsy by Sigmoidoscopy
• T/N staging- MRI pelvis ( Surface coil); Old answer was Endorectal MRI, ERUS**
• M staging – CECT abdomen and pelvis ( MC site Liver> lung)
• Tumor marker- CEA**

Anatomy related to Surgery:


• Margin given distally- 2cm**
• Margin given in post neoadjuvant Chemo RT cases- 1cm**
• If there is a cancer in Rectum proximally we cut upto Sigmoid colon.
• If growth is very proximal we do resection and do anastomosis- Colorectal anastomosis-
This surgery is known as Anterior Resection**
• If growth is very distal involving the sphincter- we have to resect the anus along with
tumor and proximal permanent colostomy done- Abdominal Perineal resection**

Figure : Anterior Resection

If anterior Resection done:


• If anastomosis done above peritoneal reflection - High AR
• If anastomosis done below peritoneal reflection- Low AR
• If anastomosis done at level of Pelvic floor- Ultra Low AR
• In LAR and Ultra Low AR we will do Covering Ileostomy for the patients to avoid pelvic
sepsis in selected patients.

Abdominal perineal Resection ( APR- Miles operation)


• If you are unable to give 2cm clearance and the tumor is very close to the sphincter we
do this morbid operation
• Patient will land up in life long – Colostomy

SURGERY SIXER APP BASED WORK-BOOK 2020 286


Nutshell of Selection:
• Growth > 6 cm from Anal verge- AR
• Growth <6cm from Anal verge- APR
• Growth >4cm from Dentate line- AR
• Growth < 4cm from Dentate Line- APR

Mesorectal excision:
• To prevent recurrence – we must also resect the mesorectum
• Mesorectum predominantly posterior
• Mesorectal clearance- 5cm given
• Lower rectal cancers- Total mesorectal excisision ( TME**)

SURGERY SIXER APP BASED WORK-BOOK 2020 287


Margins in nutshell:
• Lumen margin Distal- 2cm
• Lumen margin Distal in post chemoRT cases- 1cm
• Mesorectal clearance- 5cm
• Circumferential resection margin (CRM)- minimum 1 mm

Management differences

Cancer colon Cancer rectum

NO neoadjuvant advised for Non metastatic Neoadjuvant Chemo and RT available for T3
Colon cancers** and T4 cases
• Long Course RT- 6 weeks RT given
and operate after 6 weeks
• Short Course RT- 5 days RT and
operate in next week itself
• Papillion Technique of RT- Intra
cavitary RT for rectal cancers.

Surgeries done: • APR


Right Hemicolectomy • AR
Extended hemicolectomy • Brunschwig operation- Pelvic
Left Hemicolectomy eventeration operation involving
pelvic organ removal**

Transanal Excision of Cancer rectum:


• Local resection by minimally invasive methods – TEMS, TAMIS
• TEMS- Transanal Endoscopic Microsurgery
• TAMIS-Transanal Minimally Invasive surgery

SURGERY SIXER APP BASED WORK-BOOK 2020 288


Indications:
• T1 (mucosa and Submucosa )
• Mobile Tumors
• <3cm
• < 30% circumference
• Tumor within 4-8 cm from Anal verge**
• Tumor should be moderately differentiated.
• No nodal mets

Management of Cancer Anal Canal:


• Squamous Cell Cancer: Nigro regimen- 5FU based Chemotherapy followed by
Radiotherapy*
• Adenocarcinoma Anal canal- APR
• Malignant Melanoma- Wide Local excision > APR

Chapter 5: Miscellaneous

Mesentry

Acute Mesentric Adenitis:


• MC organism- Yersinia
• Predisposed by Respiratory infection.
• C/F- Mimics appendicitis; Central abdominal pain for 10-30 minutes associated with
vomiting*
• Klein’s Sign** - Shifting tenderness
• Self-limiting
• In doubtful cases- D.Lap advised

Misty Mesentry:
• Pathological increase of fat attenuation in mesentry on CT scan.
• Conditions include- Edema, Inflammation, Neoplastic infiltration and Hemorrhage*

Mesentric Panniculitis:
• A term to be used for mesentry involvement by
• Weber Christian disease,
• Lipodystrophy and
• Lipogranuloma

Mesentric Cysts:
• Cysts may occur in small intestine or Colon – MC in Small intestine 60%
• Four types are there:
• Chylolymphatic ( MC)

SURGERY SIXER APP BASED WORK-BOOK 2020 289


• Enterogenous
• Urogenital remnant
• Dermoid

CHYLOLYMPHATIC CYST ENTEROGENOUS CYST

Usually congenital as a result of sequestration of Derived from diverticulum of mesentric border


lymphatics that has become sequestrated

Cyst wall is thin, lacks muscular wall, not lined by Thick wall lined by mucosa
mucosa
Usually solitary, often unilocular with clear lymph Content is mucinous
or chyle

Most frequently seen in mesentry of ileum

Independent blood supply Common blood supply as adjacent bowel

Enucleation is enough** Resection anastomosis needed**

SURGERY SIXER APP BASED WORK-BOOK 2020 290


Tillaux Triad:
• Central area of dullness with zone of resonance all around
• Swelling moves perpendicular to mesentry freely
• Soft fluctuant swelling in the umbilical region

Complications of Mesentric Cyst:


• Acute abdominal pain due to
• Torsion of mesentry,
• Rupture of cyst,
• Haemorrhage into cyst,
• Infection of cyst

Retroperitoneal fibrosis
• The process of fibrosis begins at aortic bifurcation and spreads Cephalad** upto renal
artery generally.
Types:
• Primary- Idiopathic [Ormond’s disease]**
• Secondary

Secondary Causes:
• Inflammatory- Chronic pancreatitis, histoplasmosis, tuberculosis, actinomycosis
• Drugs – Methysergide, Alpha methyldopa, Beta blockers, Hydralazine.
• Malignancies- Prostate, NHL, sarcoma, carcinoid, gastric cancer

SURGERY SIXER APP BASED WORK-BOOK 2020 291


• Autoimmune- Ankylosing spondylitis, SLE, PAN
• Radiotherapy

Salient Points:
• M/c in men
• Dull aching pain, nausea, anorexia
• Compression of ureter, aorta, IVC ( MC affected structure – URETER**)
• DIAGNOSIS- Intravenous pyelogram – shows laterally deviated ureter
• Currently – CT SCAN IS THE INVESTIGATION OF CHOICE
• If renal function impaired- MRI is the choice

Treatment:
• Ormond Disease:
• Ureteral stenting and Immunosuppression ( Tamoxifen, Azathioprine,
Penicillamine and Steroids)
• Secondary RP fibrosis:
• Ureterolysis and wrapping the ureter with omental flap is done.

Peritonitis
• Spontaneous bacterial peritonitis – E.coli(most common) Klebsiella(2nd mc)
• Secondary bacterial peritonitis – E.coli and Bacteroides
• MC organism in Children- Streptococci*

Spontaneous Bacterial Peritonitis:


• Bacterial translocation from gut traversing the intestine into mesenteric lymph nodes
leading to bacteremia and seeding of ascitic fluid*
• Predisposed by Dehydration, Bowel preparation and hypoproteinemia*
• MC in patients with GI hemorrhage
• Diagnosed by presence of> 250 PMN cells + Culture shows growth of one organism**( if
more than 2 organisms are identified think of perforated viscus)**
• Treatment: Cefotaxime+ Albumin infusion
• Mortality – 20%

Cirrhotic patients with SBP:


• SBP in cirrhotic patients carry bad prognosis with less than 20% survival at 2 years.
• T. Norfloxacin as prophylaxis during GI variceal bleeding is protective in cases of DCLD
with Ascites patients.

Primary Pneumococcal peritonitis:


• Primary Pneumococcal peritonitis may complicate nephrotic syndrome* or Cirrhosis in
Children.

SURGERY SIXER APP BASED WORK-BOOK 2020 292


• Girls 3-9 years: Infection happens via Vagina and Fallopian tube.
• Male child-Blood borne (secondary to Respiratory infection or middle ear infection)
• C/F- Fever, Vomiting, Increased micturition and guarding++
Diagnosis :
• WBC count> 30000 /micro Litre( 90% polymorphs)
• Early surgery – Laparotomy done
• Exudate is odorless and sticky( Diagnostic of Pneumococcal)

Periodic Peritonitis:
• Caused by Familial Mediterranean fever.
• Disease is familial (seen in Arab and Jews)
• MC in children* (but onset may also occur in Adult)
• Duration of attack in 24- 72 hours with complete remission
• H/o Appendicectomy seen in most cases.
• On Laparotomy - the peritoneum is inflamed, particularly in the vicinity of the spleen
and the gall Bladder
• Colchicine therapy is used during attacks and to prevent recurrent attacks

Types of Ascites
High SAAG ( >1.1 gm/dl) Low SAAG ( < 1.1 gm/dl)
Transudate Exudate
Mostly liver causes*
• Cirrhosis ( M/C) • Peritoneal carcinomatosis (MC)
• Hepatitis • TB
• Liver failure • Pancreatic ascites
• Massive liver mets • Bowel Obstruction
• Budd Chiari syndrome • Biliary ascites
• PV thrombosis • Nephrotic syndrome
• Sinusoidal obstruction • Post OP Lymph leak
• Fatty liver in pregnancy • Serositis in Connective tissue disease
• Myxedema*
• Cardiac Ascites*

Pseudomyxoma Peritonei
• MC arises from
• Ruptured appendiceal neoplasm ( MC)
• Ovarian Adeno carcinoma
• Colorectal mucinous adeno carcinoma
C/F:
• Peritoneum gets filled with mucus secreting tumor with tenacious semi solid mucus
material and loculated cystic masses.

SURGERY SIXER APP BASED WORK-BOOK 2020 293


• MC in 40-50 years (M=F)
• Mostly asymptomatic, patient becomes ill health as a whole.
• Non-specific abdominal pain and distension is seen.
• Non Shifting dullness ASCITES***and a palpable abdominal mass**

Management:
• IOC- CECT will show the diagnosis and resectability.
• Preoperative colonoscopy is advised to rule out colonic mucinous adenocarcinoma**
• Treatment: Cytoreductive surgery+ Intra peritoneal Hyperthermic chemotherapy.
(Cyto reductive- Omentectomy+ Stripping peritoneum+ Resect involved organs+
appendectomy+ right Hemicolectomy)
• Best prognosis is seen with appendiceal adeno carcinoma compared with other causes*

Intra abdominal abscess


• Mc site of Abscess/Fluid Collection in Post op patient (Supine Lying )- Morrison’s Pouch (
Subhepatic pouch)
• MC site of Abscess/Fluid Collection in Sitting or Mobile patient- Pelvis**

Subphrenic spaces:
• Intraperitoneal
• Left anterior( left subphrenic space)- splenic abscess

SURGERY SIXER APP BASED WORK-BOOK 2020 294


• Left posterior ( Lesser sac)- Pancreatitis, Perforated posterior gastric ulcer
• Right anterior ( Right sub phrenic space)- appendix, gall bladder and intestinal
infections
• Right posterior ( Right sub hepatic- Morrison’s)- m/c site of sub phrenic abscess**
• Extra peritoneal spaces:
• Right – Bare area of liver
• Left – Lies around left supra renal and kidney- site for perinephric abscess.

Space for Additional Points:

SURGERY SIXER APP BASED WORK-BOOK 2020 295


Section D- Hepato Biliary and Pancreatic System

Chapter 1: Gall bladder and Biliary System


• 1a- Introduction
• 1b- Gallstone disease
• 1c- Bile duct injury
• 1d- CBD stones
• 1e- Miscellaneous
• 1f- Cancer Gall bladder and Bile duct

Chapter 2: Pancreas
• 2a- Benign Diseases
• 2b- Tumors in Pancreas

Chapter 3: Liver
• 3a- Introduction
• 3b- Portal Hypertension
• 3c- Liver surgical infections
• 3d- Tumors in Liver

Chapter 4: Spleen

SURGERY SIXER APP BASED WORK-BOOK 2020 296


Chapter – 1: Gall Bladder and Biliary System

1a- Introduction
• Crypts of Luschka*: mucous folds in GB.
• Common hepatic duct- 2.5 cm*
• Common bile duct- 7.5 Cm*
• CBD enters the duodenum at ampulla located posteriorly 10 cm from pylorus*
• Cystic artery*- arises from Right hepatic artery behind common hepatic duct
• Accessory cystic artery- From Gastro duodenal artery*
• Valves of Heister*- Valves present in the Neck of Gall Bladder and Cystic duct

Calot’s triangle Hepato cystic Triangle:

Formed between Formed between


• Inferior- Cystic duct and GB. • Inferior- Cystic duct and GB.
• Medial- Common hepatic duct • Medial- Common hepatic duct
• Above- Cystic artery • Above- Inferior surface of Liver

• Node seen in Calot’s Triangle- Lund Node**

SURGERY SIXER APP BASED WORK-BOOK 2020 297


• Right Hepatic artery taking a Tortuous
course and running in Calot’s
Triangle- is known as Caterpillar Turn
or Moynihan’s Hump**

Figure: Moynihan’s hump

Sphincter of ODDI:
• Made of 4 sphincters as a complex
• Superior and Inferior Choledochal
sphincter, Sphincter pancreaticus and
Sphincter Ampulla.
• Always closed at Resting pressure
from 12 to 120 mmHg
• The pressure in SOD is always greater
than Duodenal Pressure and this
prevents contents entering the SOD
from Duodenum.
• Only one Hormone acts here and
opens it- CCK*
Figure: Sphincter of ODDI
• CCK- Contracts Gb and Opens the
SOD.
Applied Anatomy:
• CCK is stimulated by Fatty chyme*
• If GB contracts and Ampulla not
opens is known as Sphincter of ODDI
dyskinesia.
• Characterised by Biliary colicky pain.
• Can also cause Pancreatic pain.
• Classification is known as Milkwauke
Classification ( NEET SS)
• Treatment: ERCP and Sphincterotomy
procedure

SURGERY SIXER APP BASED WORK-BOOK 2020 298


Endoscopic Retrograde Cholangiopancreatography ( ERCP)
• Done with help of Side viewing Scopy.
• We cut the sphincter at 11’0 or 1’0 clock position- known as Sphincterotomy
• The SOD pressure falls down and SOD will be always relaxed and opened.
• The contrast is passed inside the Biliary and Pancreatic system studied.
• We will keep a stent inside the Bile duct known as Double Pig tail or DJ stent.

Figure: Normal ERCP

Indications for ERCP Complications


• Sphincterotomy- SOD dyskinesia • Most Common – ERCP induced
• Stone removal Pancreatitis**
• Stenting ( Obstructive jaundice)- • Cholangitis
SEMS self expanding metallic stent for • Bleeding
growth in Bile duct • Perforation of Duodenum
• Biopsy • Mortality**- 0.1%
• Pancreas Divisum – Minor duct
papillotomy
• Gall stone pancreatitis

Magnetic Resonance Cholangio Pancreatography: (MRCP)


• Non invasive
• Has replaced ERCP in diagnostic purposes.
• IOC for BD stones, BD malignancy and Pancreatic duct pathologies.
• No contrast is used- Bile and pancreatic juice acts as contrast

SURGERY SIXER APP BASED WORK-BOOK 2020 299


Figure: MRCP

Percutaneous Transhepatic Cholangiography:


• Invasive procedure
• Cannulate the intrahepatic Bile duct- and push contrast and study the Bile duct
• It’s diagnostic as well as therapeutic.
• Therapeutic to drain Bile in Obstructive Jaundice- known as PTBD ( percutaneous
Transhepatic Biliary Drainage.
• This is the IOC for studying the proximal Bile duct**

Hydroxy Imino Diacetic Acid (HIDA Scan)


• It’s a radio tracer substance injected into Blood stream- taken up by liver and excreted
into Bile duct without metabolization. On the way it gets concentrated in Gall bladder.
Applications:
• Detects physiology of Bile flow
• Detects Bile leak
• Gold Standard for Acute Cholecystitis**- Non Visualisation of Gall bladder.

Figure: HIDA scan

SURGERY SIXER APP BASED WORK-BOOK 2020 300


Protocol for Biliary Pathology:
• 1st Investigation done- USG abdomen
• Based on pathology in USG abdomen we have so many other investigations as choices.

Findings in USG Salient Points


1. Gall stones are diagnosed • Posterior Acoustic Shadow
• 95% sensitive
• Mercedes Benz and Seagull sign are seen

Figure: Gall stone


2. Acute Cholecystitis • Shows thickened Gall bladder wall.
• IOC is USG abdomen
• Gold Standard is HIDA scan*
3. CBD stone • IOC is MRCP**
• Gold Standard is ERCP as we can remove the
stone via this procedure.

Other investigations:
1. Oral Cholecysto gram: (Graham Cole Test)
• Using Iopanoic Acid
• To diagnose the 90% radiolucent stones.

2. Xray Abdomen in Bile system:


• 10% Gall stones
• Porcelain GB- Calcified GB- Premalignant- Surgery Advised.
• Limey Bile- Tooth paste material in GB in Chronic Pancreatitis- Non premalignant
• Air in Biliary system known as Pneumobilia**

Pneumobilia Causes:
• Fistula between GB and Biliary system
• Following ERCP
• Following Emphysematous Cholecystitis

SURGERY SIXER APP BASED WORK-BOOK 2020 301


One liners from Introduction:
• Moynihan’s Hump- Caterpillar turn of RHA
• Valves of Heister- Cystic duct and neck valve
• Phrygian cap- MC anomaly of GB, Fundus of GB folded upon itself.
• Von Meyenburg Complexes
Also Known as Biliary Hamartomas
They are embryological remnants of Bile duct. Associated with PCKD**.
They are premalignant and can produce Cholangio carcinoma**
- IOC to diagnose this is MRCP( MRI)
• Mercedes Benz Sign or Seagull Sign is air in Gallstones
• Meniscus sign- Feature of CBD

Figure: Mercedes benz Sign

Figure: Meniscus sign in CBD stone

Benign Gall bladder diseases:


1. Cholesterosis: @ Strawberry Gall bladder:
• Contains Multiple Cholesterol Stones seen
• Inflammatory Pathology.

2. Adenomyomatosis
3. Diverticulosis
4. Polyps:

SURGERY SIXER APP BASED WORK-BOOK 2020 302


• MC type is Cholesterol Polyp**
• Other one is adenomatous polyp- premalignant
• Risk of malignancy in Polyp in GB:
- Age > 60 year
- Polyp associated with stone
- Size > 10 mm
- Increasing size of polyp
- Family H/o Gall bladder cancer
• Above cases must be operated as there is risk of malignancy inspite of them being
asymptomatic also.
• Surgery by Lap or Open Cholecystectomy. Informed consent obtained regarding re
opening if there is malignancy* in Biopsy

1b- Gall stones

Types of Gall stones:


Cholesterol • MC type of stone in western Composed of Cholesterol
countries Monohydrate crystals**
• Golden Yellow Color 15% of these stones are radio
• Hard stones opaque
Mixed • MC type of Stone in world Composed of
• Cholesterol Component is more • Insoluble Bilirubin
than 30% • Calcium Phosphate
• Calcium carbonate

Pigment • Black and Brown pigment Composed of


stones • Ca. Palmitate
• Brown pigment is mc type in • Ca. Stearate
Asians and Indians • Ca. Bilirubinate
• Black stone is mc type in
Children 50% of Black stones are radio
• In pigment stone Cholesterol opaque
Component is less than 30% All brown stones are radio lucent.
• Black stone is also hard
• Brown stone is soft and earthy
stones**

Cholesterol Stones:
• Formed by supersaturation of Cholesterols
• Ratio of Cholesterol: Phospholipids (Bile salts)= 1:20 is ideal ratio; if the ratio becomes
1:13 i.e the cholesterol content increases and Bile salts decreases- Stone forms.

SURGERY SIXER APP BASED WORK-BOOK 2020 303


Increased Cholesterol Decreased Bile salt
Obesity Decreased Entero Hepatic Circulation:
Cholesterol rich diet • Short Bowel Syndrome
Clofibrate therapy • Ileal resection
Stasis of Bile: • Crohn’s disease
- Fasting
- TPN Decreased Bile salt production:
- Pregnancy • Cirrhosis of Liver
- OCP • 7 alpha hydroxylase deficiency
- Octreotide

Obese lady with Gall stone:


o Saint’s Triad- Gall stones+ Diverticulosis colon+ Hiatus hernia

BROWN PIGMENT BLACK PIGMENT


(Calcium Bilirubinate+ Calcium Palmitate+ Calcium ( Insoluble Bilirubin Pigment polymer+ Calcium
Stearate+ Cholesterol) Phosphate + Calcium Carbonate)
• Common in Asia* M/c in hemolytic states
• Always radiolucent stone • Hereditary spherocytosis, sickle cell
• Due to bile stasis and infection** disease
• M/C stone in infection– E.COLI*, Ascaris • Heart valves [mechanical]
and Clonorchis sinensis • Liver cirrhosis
• M/C in presence of FB, clips, stents, • Gilbert’s syndrome
parasites** • Cystic fibrosis
• MC type of stone seen in Hepatolithiasis ( • Ileal resection
Oriental Cholangiohepatitis)- intra hepatic
stone.
• Primary bile duct stone formation** (
Formed in Bile duct itself)- MC primary
Bile duct stone is Brown Stone
• Patient coming with CBD stone after
cholecystectomy:
- <2 years: Retained stone
- >2 years : Recurrent stone.
- MC recurrent stone is Brown Stone

SURGERY SIXER APP BASED WORK-BOOK 2020 304


Clinical features of Gall stones:
• Mostly asymptomatic
• MC symptom- Biliary Colicky** (Pain after eating fatty foods)- Misnomer- it’s not a
typical colicky pain. The pain increased gradually after food intake and gets relieved once
the stone in cystic duct passed out or falls inside the GB. Usually pain starts after 45
minutes. Surgery advised for such cases.

Complications of Gall stones:


1. Acute Cholecystitis- MC complication
2. Chronic Cholecystitis
3. Stone impaction at Neck of GB/ Cystic duct- Mucocele, Empyema and Perforation.
4. Stone near Hartman’s pouch or Neck and compressing CBD- Mirizzi Syndrome
5. GB can adhere to adjacent bowel and result in Cholecysto duodenal fistula- Gall stone ileus**
6. Cancer GB- 0.5%
7. CBD entry of Stone- Cholangitis, Gall stone pancreatitis

Discussion of Each complication:


1. Acute Cholecystitis:
• C/F- Pain in right hypochondrium
• Murphy’s Sign- Tenderness in 9th Costal cartilage on Deep Breath
• Moynihan’s Method- Patient lies down and the tenderness elicited by Palpation using
Thumb.
• Boas sign- Sign of Hyperaesthesia**
• IOC- USG abdomen
• Gold standard- HIDA scan shows non visualisation of GB

Tokyo Guideline of Acute Cholecystitis


Mild Cholecystitis Moderate Cholecystitis Severe Cholecystitis
Grade 1 Grade 2 Grade 3
• Only Clinical • Clinical features+ • Signs of organ
features seen • Elevated WBC failure++
count+ ( >18000) • Renal
• Palpable Mass+ • Respiratory
• Gangrene+ • Hypotension
• Localised features • Altered sensorium
only
• Advised Lap • Immediate surgery • Treatment is
cholecystectomy by Lap or open Urgent
Cholecystectomy Percutaneous
Cholecystotomy

SURGERY SIXER APP BASED WORK-BOOK 2020 305


Management:
o Nil oral
o IVF
o Ryles tube aspiration
o Higher Antibiotics
o Early or Interval Cholecystectomy
o Early Cholecystectomy ( Bailey says in 72 hours- Other Books say up to 7 days)-
Mostly followed. The only problem is more chances of conversion from lap to open
method.
o Interval cholecystectomy or Delayed one is done after 6 weeks

2. Mirizzi Syndrome:
o Stone in Hartman’s of Neck causing compression of CBD
o Causes Obstructive Jaundice
o Csendes Classification:
▪ Type 1- Compression only
▪ Type 2- 1/3rd circumference is eroded
▪ Type 3- 2/3rd Circumference eroded
▪ Type 4- Fistula between GB and Bile duct or adjacent fistula
( Choledocho cholecysto Duodenal fistula)
o Treatment: open cholecystectomy is best option as calot’s triangle is disturbed.
o If fistula is seen- GB wall is used as a Buttress to close the fistulous part of Bile
duct.

Figure: Mirizzi syndrome


3. Gall stone ileus:
o Inflammed Gb adhered with duodenum and huge stone passes from Gb to
Duodenum in the fistula . ( usual size >3cm stone)
o Can cause obstruction in Duodenum- Bouveret Syndrome
o Can cause obstruction in terminal ileum- Intestinal obstruction- Gall stone
Ileus
o Triad : Rigler’s Triad:
▪ Tumbling Obstruction of Bowel ( motion passed due to
movement of stone and there is Tumbling Obstruction)
▪ Fistula between GB and Bowel
▪ Pneumobilia
o Treatment:

SURGERY SIXER APP BASED WORK-BOOK 2020 306


o Unstable patients- Enterotomy and stone removal
o Stable patients- Along with enterotomy and stone removal, we will also remove
the GB and close the fistula also.

4. Gall Bladder Cancer:


o Incidence= < 0.5%
o Risk increased - stone > 2.5 cm size, Stone + polyp, Family
history
o The above cases to be operated even without symptoms.

Management of Gall stones

Medical Treatment:
• Useful only for cholesterol gall stones not for pigment stones*
• Mechanism is by inhibiting HMG CO –A reductase in cholesterol synthesis, thus decrease
cholesterol Super saturation or by giving Urso deoxycholic acid ( UDCA) and increasing
Bile salts.

Useful only in**


o Radiolucent
o Size <15 mm
o Functioning gall bladder
o Non acute symptoms

Surgical Treatment:
• Lap or Open Cholecystectomy ( MC done is Laparoscopic method)
• 1st Done Lap Cholecystectomy- ERIC MUHE
• 4 ports needed- Primary port at umbilicus and 3 secondary ports along Right
Hypochondrium ( Epigastrium, MCL and Anterior Axillary Line)
• Two Methods:
- Retrograde method- From calot’s triangle to fundus ( MC done method)
- Antegrade method – Fundus First Approach* when calot’s triangle is not clear.

SURGERY SIXER APP BASED WORK-BOOK 2020 307


Critical view of Strassberg:
• Prevents Being Clipped.
• On Laparoscopy we must see only 2 tubular structures entering GB- Cystic artery and
Cystic Duct

Indications of Surgery:
• All symptomatic stones operated.
• Asymptomatic cases:
- Size > 2.5 cm
- Gall stone associated with Polyps
- Anomaly in GB
- Porcelain GB
- Family H/o of GB cancer
• Other indications:
- Heart or lung Transplantation – Cyclosporine therapy can cause Gallstones
- Bilio pancreatic Bypass surgery- Prophylactic surgery done as there is more chances of
Stones.

• Rare indications:
- Typhoid carriers
- TPN on long term can cause gall stasis
- Immuno-suppressed patients- Chemotherapy cases, HIV cases
- Diabetes Mellitus with Asymptomatic Gall stones is an indication according to Bailey and
Love.

The following is not indication for asymptomatic stone-Multiple Gall stones**

Indications for open Cholecystectomy ( Contraindications for Laparoscopy)


- Cirrhosis with Portal Hypertension
- Bleeding Disorders
- Poor cardio pulmonary status ( COPD, CAD)
- Suspicious of GB cancer** ( To avoid Dissemination of Malignancy)

SURGERY SIXER APP BASED WORK-BOOK 2020 308


- Abdominal pathologies like Adhesions, Morbid Obesity, Pregnancy, Perforated GB peritonitis
are relative Contraindications.

Acalculus Cholecystitis:
• Highest mortality among all cases of Cholecystitis
• Because this is common in patients having other pathologies- sepsis, Major surgery,
Burns or TPN, Typhoid or Streptococcal infections.
• Incidence- 5%
• IOC- USG
• Gold standard- HIDA scan ( Absent filling of GB)
• Immediate cholecystectomy advised if patient is fit by open method.
• If patient is unfit percutaneous Cholecystostomy advised.

1c. Bile Duct Injury - Protocol

• Post cholecystectomy one patient is in your ward- C/0 Abdominal pain + Fever
• O/E- Severe tenderness in Right Hypochondrium and increased Pulse rate.
Suspect Bile duct injury:
• Incidence- 0.3- 0.8% in lap and 0.1 -0.2% in open Surgery
1. 1st Done investigation is USG abdomen. USG abdomen-Shows Collection in RIF known as
Bilioma
2. Immediately Percutaneous drainage of Bilioma done.
3. PCD draining Bile and gradually gets reduced in 2 more weeks
4. After 2 weeks we must do the IOC for Bile duct injury – ERCP**

ERCP Shows: Strassberg Classification for Bile duct injury and leak:
o Type A- Cystic duct leak
o Type B- Aberrant duct clipped
o Type C- Aberrant duct cut
o Type D- Lateral injury
o Type E- Transection of CBD

SURGERY SIXER APP BASED WORK-BOOK 2020 309


• MC type of injury- Type A, Cystic Blow out
• In whom there is no leak- Type B injury, ERCP looks normal ( NEET SS Question)
• Type A injury no need to worry , as the leak stops by itself. If the leak not stops > 2week
we will do ERCP sphincterotomy and Stenting is done.

5. After 8-12 weeks ; the injuries will heal with Strictures:


IOC for Bile duct Strictures- MRCP*

Classification – Bismuth Classification


• TYPE 1- Low common hepatic duct stricture. stump>2cm
• TYPE 2- Proximal stricture. Stump< 2 cm
• TYPE 3- Hilar stricture. Confluence intact

• TYPE 4- Destructed confluence. Right and left ducts separated


• TYPE 5- Involvement of right aberrant sectoral duct alone or along with stricture at
CHD.

• Type 1,2,3 has confluence preserved and hence one anastomosis only needed
• Type 4- 2 anastomosis will be needed.

Gold Standard treatment of Bile duct stricture- Hepatico Jejunostomy

SURGERY SIXER APP BASED WORK-BOOK 2020 310


Bile duct injury Diagnosed during Surgery itself:
• Injured duct < 3mm- Ligate it
• Injured duct > 3mm- Important duct and hence must be repaired.
• Circumferential injury : < 50%- Keep a T tube and repair
• Circumferential injury: >50% - Hepatico jejunostomy done
• Repair done by using 4’0 Vicryl ( Polyglactic acid)

If you are inexperienced Surgeon and encountering an injury like this – What will you do?
• Keep a drain and refer to a Experienced Hepato Biliary Surgeon for immediate repair.

1d. CBD Stones


Types:
• Primary CBD stone- Formed in Bile duct itself
• Secondary CBD stone- Formed in GB and moves to CBD
Clinical Features:
Charcot’s Triad- Cholangitis (Inflammation of Bile duct)
• Obstructive jaundice
• Pain
• Fever with Chills
Mortality- 10%
• MC organism- E.Coli and Klebsiella
• Reynauld’s Pentad- Above 3 features+ Septic shock + Mental status changes.
• Mortality is 50% in cases of Reynauld’s pentad.
• IOC is MRCP**

SURGERY SIXER APP BASED WORK-BOOK 2020 311


CBD stone with no Cholangitis CBD stone with Cholangitis
• Treatment of Choice is Immediate next treatment:
1. ERCP + Sphincterotomy +removal of stones • Start Antibiotics+ IVF :
and stenting followed by Lap cholecystectomy PIPERACILLIN+TAZOBACTUM or
(Two stage procedure) Meropenam
2. Laparoscopic Removal of CBD + • See for resolving cholangitis.
Cholecystectomy (Single stage procedure)
3. Open Cholecystectomy+ CBD exploration If fever Subsides and patient recovers from
and removal of stone Cholangitis:
- Plan elective ERCP and stone removal
• Most commonly followed Procedure is - Followed by laparoscopic
ERCP followed by Lap Cholecystectomy
cholecystectomy.
If fever doesn’t subside and persistent
CBD Exploration: cholangitis:
• Opening the Bile duct Vertically and - Emergency ERCP, sphincterotomy and
removal of stone. stone removed with Stenting by Medical
• If CBD <12 mm- T tube and closure Gastroenterologist.
• If CBD >12 mm- Choledocho-
duodenostomy. If ERCP failed- call the interventional
radiologist to remove the infected bile by
ERCP Contraindications: putting a percutaneous Biliary drainage tube
• CBD stone> 1.5 cm transhepatically. ( PTBD)
• Multiple Impacted stones
• Ampullary Stenosis If PTBD also failed- Call the Surgical
• More proximal stone Gastroenterologist for CBD stone removal .
This surgical procedure has high mortality

CBD Exploration ( Choledochotomy) and Stone


removal:
- Vertical incision
- Desjardin forceps to remove the stone
- T Tube kept inside.
- CBD <12 mm- T tube drainage
- CBD> 12 mm-
Choledochoduodenostomy

SURGERY SIXER APP BASED WORK-BOOK 2020 312


T Tube Protocol:

Why T Tube is kept?


• For temporary diversion of Bile we keep a T tube as there will be ampullary spasm.
• To prevent leak
• To drain the bile in presence of any obstruction

As the patient is passing yellow stools means T tube is draining Bile into the Duodenum

Removal of T Tube:
• After doing T tube Cholangiogram – usually done on 7th/8th POD
• If T tube cholangiogram is normal- Remove the T tube on 10-14 days and discharge

If there is a stone in CBD on T tube cholangiogram:


• The name given is Missed or Retained stone ( Name given for stones in < 2 years)
• Step 1- Wash with Saline or Methyl Terbutyl Ether
• Step 2- Come back after 6 weeks- The tract will be well formed now and via the T Tube
tract; We go inside using Cholangioscope and remove the Missed stone ( THIS TECHNIQUE
IS KNOWN AS BURHENNE TECHNIQUE**)

Retained or missed stone Recurrent Stones


( <2 years ) ( >2 years)- Brown Pigment stone
ERCP and Sphincterotomy and Balloon The recurrent stones are usually bigger ( >
clearance 2cm size) and usually not feasible for ERCP.

Choledochotomy and stone removal followed


by CHOLEDOCHO DUODENOSTOMY**

SURGERY SIXER APP BASED WORK-BOOK 2020 313


1e. Miscellaneous Lesions

Choledochal Cyst Extra Hepatic Biliary Atresia Primary Sclerosing cholangitis


Abnormal dilatation of Bile Inside the liver Bile duct is Abnormal dilatation and
duct is known as Choledochal normal stricture of Bile duct.
cyst. Once outside the liver the bile
• MC in Infants, Female ducts are atretic* • MC in Males
• MC in Newborn • MC in 40 years
• Equal in male and
female
Clinical features: C/F: C/F:
Triad of: • Obstructive jaundice • Obstructive jaundice
• Pain • Itching • Cholangitis
• Jaundice • Cholangitis
• Mass
Complications: Complication: Complication:
• Stones • Cholangitis • Cholangiocarcinoma
• Cirrhosis • Liver cirrhosis- • 10% risk every year
• PV thrombosis Secondary Biliary
• Cholangitis type
• Malignancy risk – in
Bile duct, Pancreas,
Liver or GB. MC on
posterior wall of Cyst
Todani or Alonso Classification: Types of EHBA: Types of PSC:
• Type 1- Fusiform • Type 1- Common Bile duct
Dilatation level atresia- GB Enlarged • Primary PSC- Unknown
• Type 2- Diverticulum type • Type 2- Atresia at CHD, no etiology, HLA association,
• Type 3- Ampullary level GB enlarged Reidel’s Thyroiditis or
Diverticulum, • Type 3- At porta Hepatis Dequervain’s
Choledochocele level- 90% . MC type
• Type 4: • Secondary PSC: UC > CD.
✓ 4a- All ducts Intra and This is most common
extra hepatic dilated type
✓ 4b- Extra hepatic
Dilatation
• Type 5: ( Caroli’s
Disease)
✓ Intrahepatic dilatation
Investigations:
IOC- MRCP IOC – LIVER BIOPSY is Gold IOC- ERCP
• Caroli’s Disease- standard now ( Old answer Appearance is Prune tree or
Shows a sign on CECT was Percutaneous Beaded Appearance**
abdomen: Central Dot Cholangiography)
sign**- Portal vein
radicle appears like USG abdomen- Triangular
that cord sign seen.

SURGERY SIXER APP BASED WORK-BOOK 2020 314


Management: Management: Only treatment is Liver
It’s premalignant cyst formed 1st Line- Kasai Procedure transplant
by Anomalous Pancreatic bile “Porto enterostomy”
duct Junction ( APBDJ) with Bridging procedure for Liver
pancreatic reflux. transplant which is Gold
standard treatment.
It needs to be resected as
there is malignancy risk Good Prognosis of kasai
depends on:
• Type 1 and 2- Resect • Age < 6 weeks
the cyst and do • Size of ductules > 150
Hepatico jejunostomy microns
• Type 3- Trans • No Cirrhosis
duodenal • No Cholangitis
sphincteroplasty
• Type 4 and 5- Liver
transplantation

Lilly Procedure: if posterior


wall is very adherent to
Portal vein – we remove the
anterior wall and remove the
mucosa of posterior wall
leaving the muscular part
behind itself. ( Not Done now)

• Liver transplantation is the final resort in all patients


• MC indication in New born – EHBA**

Figure: Todani Classification

SURGERY SIXER APP BASED WORK-BOOK 2020 315


Figure: Central Dot sign in caroli disease

Figure : Kasai Procedure ( porto enterostomy)

1f. Cancer Gb and Bileduct


• MC site of malignancy in Biliary system- Gall bladder
• In Bile duct cancer common at bifurcation- Klatskin Tumor**

Clinical features:
• GB malignancy is associated with stones in 90% cases and mimics symptoms of Gall stones.
• Pain, Jaundice and weight loss.

Obstructive Jaundice: is MC due to Stones and Cancer.


Clinical examination method- Courvoisier’s law to find the etiology by Examination in Obstructive
jaundice patients:

SURGERY SIXER APP BASED WORK-BOOK 2020 316


Courvoisier’s law states:
• When Gallbladder is enlarged- its due to cancer
• When Gall bladder is not enlarged- its due to stones in CBD, because the stone would
have already caused fibrosis and won’t enlarge anymore.

Exceptions to Courvoisier’s Law:


• Cancer case but no Gb enlarged – Hilar cholangiocarcinoma ( Klatskin tumor)
• Stone case but GB enlarged- Double Impacted stone ( One at CBD and one at Cystic
Duct)

Gallbladder Cancers:
Predisposing factors:
• MC – Gall stones
▪ 0.5% of Gall stones leads to cancer
▪ 95% of Cancer GB will have stones**
▪ Incidence is 10% if stone size 3cm or more.
• Polyps > 10 mm
• Typhoid carriers
• Oestrogen
• Clonorchis sinensis
• Ulcerative colitis
• Choledochal cyst, APBDJ
• Primary sclerosing cholangitis
• Carcinogens- Nitrosamines, Rubber and Dyes and chemicals

SURGERY SIXER APP BASED WORK-BOOK 2020 317


Investigation:
• USG usually shows Gallstones or Gallstones with Polyps or a mass lesion is usually the 1st
done investigation
• IOC – CECT abdomen with Triple Phase contrast
• Tumor Marker- CEA

For Bile duct Cancer:


IOC for staging- MRCP+ MRA
Tumor marker- CA 19-9 and CEA

Management and Staging:


TNM Staging Procedure
T1a- Tumor into Lamina Propria Laparoscopic /simple cholecystectomy is enough

T1b- Tumor into Muscle layer If Perineural invasion, Vascular invasion, LN or margins
are not involved- laparoscopic cholecystectomy is enough.

If above are positive- We do Extended Cholecystectomy


• Liver resected with a margin of 2cm
• Bile duct is removed if Cystic Duct margin +ve
• Nodes removed- Peri choledochal nodes,
Periportal nodes, Coeliac nodes, Hilar and retro
duodenal nodes.
T2- Tumor into peri muscular
connective tissue
▪ T2a- Invade the
perimuscular connective
tissue on peritoneal side
with no extension to serosa
▪ T2b- Invade the Radical Cholecystectomy:
perimuscular connective • Remove segments – 4b +5
tissue on the hepatic side • Remove all nodes mentioned already.
with no extension into liver.

SURGERY SIXER APP BASED WORK-BOOK 2020 318


T3- Serosal Perforation and/or
Direct invasion of the Liver or any
other single extra hepatic organ

Options:
• Radical Cholecystectomy- 4b+5
• Extended Right hepatectomy- 4a+4b+
5,6,7,8
• 3. Central (Taj mahal) hepatectomy-
4b+5+8
T4- Invades main PV or HA or two Inoperable cancer
or more extrahepatic organs.
• N1- Metastasis in 1-3 regional nodes – Pericholedochal nodes ( CD, CBD, HA
and PV)
• N2- Metastasis in 4 or more regional nodes- Distant nodes ( Periaortic,
pericaval, SMA and Coeliac nodes)
• M1- Distant mets

Recent advances:
• No need to remove the port sites in present scenario on resection cases after
biopsy report.
• If already diagnosed as GB cancer and planning a radical surgery- one
investigation that must be done now is D.Lap. In around 30% cases there are
small peritoneal deposits or liver mets. D.Lap is a staging investigation

Scenario 1: Incidental GB cancer in specimen:


• Lap cholecystectomy done for Multiple GB stones.
• Pathologist is telling there is a cancer in the specimen.
What next?
• The further management depends on staging:
T1a- Already done Lap cholecystectomy is enough
T1b- if everything is negative ( PNI, LN,Margins)- Lap cholecystectomy is enough
T1b- if any of the above positive- Reopen and do Extended Cholecystectomy
T2- Reopen and do Radical Cholecystectomy

SURGERY SIXER APP BASED WORK-BOOK 2020 319


T3 and T4 will not seen in Incidental as we can see the growth on putting the laparoscopy itself.

Scenario 2: On Lap cholecystectomy 1st port you are seeing a cancer in GB:
• Unexperienced surgeon- Remove the port and refer to Hepato Biliary surgery
• Hepatobiliary surgeon- Convert to open and proceed with radical surgery

Adjuvant therapy:
• Gemcitabine therapy is given

Bile duct cancers:


Predisposing factors:
• Inflammatory causes- PSC, UC and Hepatitis C and Oriental Cholangiohepatitis
• Parasites- Clonorchis sinensis, Opisthorcis Vivernii**
• Chemicals- Thorium, Asbestos, Vinyl Chloride
• Congenital- Caroli disease and Choledochal cysts
• Post op causes- Bilio enteric anastomosis

MC type is Adenocarcinoma
Types:
• Sclerosing type has worst prognosis
• Papillary type has best prognosis
• Nodular types

Bismuth Corelette Classification:


• TYPE 1- At common hepatic duct only.
• TYPE 2- Involving confluence without involvement of secondary ducts
• Type 3-a- Involving right secondary intra hepatic ducts
• Type 3 b- involving left secondary intra hepatic ducts
• TYPE 4 – Involves secondary ducts on both sides

Fig. Bismuth Corelette classification of Hilar cancers

SURGERY SIXER APP BASED WORK-BOOK 2020 320


Treatment:
o For 1 and 2- Bile duct resection done radically and Hepatico jejunostomy done
o For 3a- we do extended Right Hepatectomy( Right trisectionectomy)- 4a+4b+
5,6,7,8
o For 3b- we do extended Left Hepatectomy ( Left trisectionectomy)- 2,3,4a,4b+ 5
and 8
o Type 4- inoperable

Criteria for Unresectability of Hilar Cholangiocarcinomas:


o Bilateral hepatic ducts upto secondary radicles involved (Type 4)
o Main PV Involvement
o Atrophy of one lobe (One Hepatic artery involved) with Contralateral PV
o Atrophy of One lobe (One Hepatic artery involved) with Contralateral
Secondary Radicles involvement
o Distant Mets, Ascites

SURGERY SIXER APP BASED WORK-BOOK 2020 321


Chapter 2: Pancreas
Embryology:
• Pancreas derived from Dorsal and ventral bud
• Ventral bud rotates and fuses with dorsal bud by 6 weeks.
• If the ventral bud not rotates- Annular pancreas
• If the ventral and dorsal bud not fuses with each other- Pancreas Divisum

Annular Pancreas Pancreas divisum


• Ventral and dorsal bud encircles the • Minor duct is draining the major part
pancreas like a ring** of pancreas.
• C/F- Gastric outlet obstruction • Hence there will be recurrent
• Associated with Down’s Syndrome, pancreatitis.
Duodenal atresia • IOC- ERCP> MRCP
• IOC – CECT abdomen • Treatment- Minor duct
• ERCP will show absence of ventral sphinterotomy**
bud.
• Treatment:
Duodeno Duodenostomy ( Bailey) or Duodeno
jejunostomy**
Resection not needed.

Anatomy of Pancreas:

• Tunnel of Love- in neck of pancreas. SMV enters and comes out as portal vein.
• Major duct of Wirsung drains- Tail, Body, part of head And uncinate process.
• Minor duct of Santorini drains small part of Head.
• Wirsung Duct is formed by Distal part of Dorsal bud and full ventral bud**
• Santorini Duct is formed by Proximal part of Dorsal Bud.

SURGERY SIXER APP BASED WORK-BOOK 2020 322


Ectopic Pancreas:
• MC in Duodenum > Stomach > Meckel’s Diverticulum
Acute Pancreatitis

ETIOLOGY OF PANCREATITIS; ( GET- SMASHED- I)


• G-gallstones ( MC cause in adults)
• E-ethanol
• T-trauma ( MC cause in Children)
• S-steroids
• M-mumps
• A-autoimmune(PAN,SLE etc.,)
• S-scorpion venom
• H-Hyper lipidaemia, Hyper calcaemia, Hyperparathyroidism
• E-ERCP ,Emboli
• D-Drugs (Azathioprine, Thiazides, Furosemide, Tetracyclines, 6- Mercaptopurine etc)
• I- Infections (Cox-Sackie, CMV, Echovirus, Ascaris Lumbricoides and clonorchis sinensis
etc)

Drugs that cause Acute pancreatitis are:


• Corticosteroids
• Azathioprine
• Asparaginase
• Valproic Acid
• Thiazide diuretics
• Estrogens

Clinical features:
• Mohammed Prayer Sign: Pain radiating to back from Epigastric region making patient sit
like a Muslim Praying appearance.
• Hemorrhagic pancreatitis:
- Cullen’s Sign: Periumbilical pigmentation
- Grey Turner’s Sign: Loin Pigmentation
- Fox Sign: Inguinal ecchymosis

SURGERY SIXER APP BASED WORK-BOOK 2020 323


• Differential Diagnosis: Perforation

X ray Abdomen erect findings:


▪ Gasless abdomen**
▪ Colon Cut off sign**
▪ Ground Glass appearance
▪ Sentinel Loop Sign
▪ Renal Halo Sign
Investigations:
• Serum Amylase: 4 times increased than normal , Not specific. It can be elevated in other
GI pathologies – Perforation, Obstruction, Diverticulitis, mesenteric ischemia
• Serum amylase usually becomes normal after 3 days after attack
• Most Sensitive test- Serum Lipase**
• Most specific test- Serum trypsinogen > Serum Lipase**
• CRP: >150 implies severe AP
• Radiological IOC- CECT abdomen with oral and IV contrast (usually advised after 72
hours)- Balthazar Score or CTSI ( CT severity Index)

Scoring Systems in Acute pancreatitis


• Ranson’s scoring – Most widely used- 3 is severe
• Apache 2- Uses 12 factors: >8 is severe
• Modified Glassgow score > 3
• CT severity index/ Balthazar et al scoring- Best Scoring **
• C –reactive protein- more than 150 mg/dl is severe
• BISAP score – Bed side Index of Severity of Acute Pancreatitis**
• Revised Atlanta Scoring System(RAS)*
• Modified Marshall Score
• q SOFA score used in Sepsis

BISAP Score:
• B- Blood urea Nitrogen
• I- Impaired Mental status
• S- SIRS
• A- Age > 60 years
• P- Pleural effusion
Ranson Score: ( Mnemonic – LAGAW BUCHOW)
On admission Within 48 hrs
o L- LDH >350 u/l o B-Base deficit > 4 mmol/l
o A- AST >250 sigma frankel units % o U- Blood Urea N2 >5 mg/dl
o G- Glucose >200 mg/dl or > 1.1 o C- Calcium <2.0 mmol/l
mmol/l o H- Haematocrit fall >10%

SURGERY SIXER APP BASED WORK-BOOK 2020 324


o A- Age>55 yrs o O- Arterial 02 saturationPa02 <60
o W- WBC Count >16000/mm3 mmHg
o W- Water sequestration >6l
• Ranson’s score > 3 is said to be severe AP.

CT Severity Index ( old Name Balthazar )


Pancreas inflammation Necrosis
• Normal- 0 • <30% necrosis: 2
• Focal/ Diffuse Enlargement- 1 • 30-50% necrosis: 4
• Intrinsic changes – 2 • > 50% necrosis: 6
• Single fluid collection- 3
• Multiple fluid collection- 4
• Total score= 10
• Severe AP= 7-10 ( mortality – 17%)

Revised Atlanta Classification:


Organ failure Local Complications Systemic Complications
• Shock- SBP<90 mmHg • Necrosis • DIC
• ARDS- PaO2< 60 mmHg • Pseudocyst • Metabolic disturbances
• ARF- Se. creatinine >2mg • Abscess • Fibrinogen level
• GI bleed> 500 ml/day decreasing

RAC Classification takes into account all the above:


• Mild: No Local Complication or organ failure ( only amylase Increased)
• Moderate: Organ Failure +, but it’s transient and it resolves in 48 hours**+ Local/
Systemic Complications but no persistent organ failure ( in 48 hours patient recovers.
• Severe: > 48 hours: there is persistent organ failure+ Local or systemic complications

Complications of Acute Pancreatitis:


• Release of Trypsinogen which will activate all the enzymes from pancreas and does
Autodigestion and results in list of complications:

Local Complications Systemic Complications


caused by IL-1,6 and TNF
alpha
• < 4 weeks: Acute Pancreatic fluid collection ( APFC) • Shock
• > 4 weeks: Pseudocyst • ARDS
• < 4 weeks: Acute Necrotic fluid collection ( Necrotic • ARF
material also seen) • GI Bleed
• > 4 weeks- Walled Off Pancreatic necrosis ( WOPN) • MODS ( MC cause of
death in AP overall)

SURGERY SIXER APP BASED WORK-BOOK 2020 325


• MC complication – Infection ( Pancreatic abscess)- MC
cause of late death and MC local complication causing
death.
• Pancreatic ascites
• Pancreatico pleural fistula
• Pleural effusion
• Splenic vein thrombosis
• Splenic artery Aneurysm ( False aneurysm caused by
Elastase enzyme)
• Colonic gangrene or perforation

Pseudo Cysts:
• Follows Acute or chronic pancreatitis
• MC site is lesser sac. ( But can happen anywhere inside Abdomen)
• MC near Body and tail of Pancreas**
• D’egidio Classification:
o Type 1- Follows Acute Pancreatitis; Duct is normal
o Type 2- Follows Acute on Chronic Pancreatitis. Duct is abnormal with no
stricture*
o Type 3- Follows Chronic Pancreatitis; Communication seen with Duct and duct
has strictures**

Figure: Pseudocyst in Tail of pancreas.

• C/F- Presents after 4 weeks with persistent pain and epigastric fullness and vomiting
and early satiety**
• Persistent elevation of Serum Amylase
• IOC – CECT abdomen;
• EUS guided Fluid analysis shows – Increased amylase and Normal CEA ( To
differentiate from Cystic Neoplasm)

Management:
• Surgery Advised for Symptomatic pseudocysts only and for complications.

SURGERY SIXER APP BASED WORK-BOOK 2020 326


• Old Rule : Rule of 6 : Surgery done if cyst > 6 weeks, > 6 cm diameter or thickness 6
mm wall.

Complications in Pseudocyst:
• Mc complication- Infection ( Pancreatic abscess)
• Rupture into abdominal cavity causes pancreatic ascites
• Rupture into Thoracic cavity – Pancreatico pleural fistula
• Bleeding ( Known as Haemosuccus Pancreaticus**)

Surgical treatment:
• Internal Drainage : Cysto jejunostomy > Cystogastrostomy or Cysto duodenostomy.
• Cysto gastrostomy done by open method , Endoscopic method or laparoscopic method
• Infected Pseudocyst – External drainage done

Pancreatic ascites:
• Communication seen with pancreatic duct
• Increased amylase seen
• High Protein Ascites ( Low SAAG < 1.1)
• Treatment : Paracentesis + Injection Octreotide
• 2nd Line : Endoscopic Therapy – ERCP and Pancreatic stenting done through the duct
disruption.
• Last option – Surgeon called only if there is no resolution on ERCP. Procedure done is
Lateral Pancreatico jejunostomy of Puestow**

Pancreatic necrosis:
• CTSI: < 30 % necrosis= 2 points, 30-50% =4 points; > 50% necrosis= 6 points
• Mostly the necrosis is sterile.
• If it gets infected – there will be air, Increased WBC count and temperature.
• Infected necrosis cases we start antibiotics.
• Under antibiotic coverage – we must remove the necrosis.

Figure: Pancreatic necrosis

SURGERY SIXER APP BASED WORK-BOOK 2020 327


• Step up approach- From minimally invasive to maximally invasive procedure. USG
guided PCD done 1st, Later as patient improves , Minimally invasive method by
Laparoscopic Necrosectomy or Video assisted Retroperitoneal debridement ( VARD)-
via a nephroscope introduced into the PCD tract.

• Step Down approach: Open laparotomy and keep wide drains for regular lavage.

Management of Acute pancreatitis:


• Key step in management- Fluid replacement enormously expecting good urine
output**
• Octreotide has a doubtful role and hence advised and given for pancreatitis cases.

Mild Acute pancreatitis Severe Acute Pancreatitis


In General ward In SICU
Single Surgeon can treat Multidisciplinary management needed
No antibiotics needed Only in infective necrosis- antibiotics needed.
Analgesics advised- opioids are used DOC- Metamizole and Tramadol are given
( don’t give NSAIDS)
Nutrition: Nutrition:
• Normal enteral feeding advised. • Patient is in shock we have to go
• NG feeding is method of choice if for TPN.
patient cant take oral diet**
• NJ is second option if patient cant
tolerate NG tube feeding and
there is vomiting
Intervention: Intervention:
• Gall stone Pancreatitis + • Necrosectomy Management
Cholangitis - ERCP and • Pseudo cyst management
sphincterotomy and stenting can • Pancreatic ascites management
be done in < 72 hours
• Mostly no intervention needed

Chronic pancreatitis

It is a triad made of
• Endocrine Loss- > 90% lost results in Diabetes
• Exocrine loss- > 90% lost Steatorrhea
• Strictures and stones- Severe Pain Epigastric region radiating to Back: pain is not due
to Enzymes; it is due to Ductal Hypertension.

TIGAR- O Classification of Aetiology:


• T- Toxic ( Alcohol )- MC causes

SURGERY SIXER APP BASED WORK-BOOK 2020 328


• I- Idiopathic/ Tropical– Unknown cause; Eating Casava Root fibres in kerala
• G- Genetic-PRSS gene mutation, SPINK -1 gene mutation, CFTR gene mutation
• A- Autoimmune
• R- Recurrent Acute Pancreatitis ( like in Pancreas Divisum)
• O- Obstructive

Clinical features:
• Pain abdomen; severe recurrent pain; addicted to Opioids
• Diabetes- responds only to Insulin
• Steatorrhea

Investigations:
• Old Tests not done now- Lundh meal test, Pancreaolauryl test
• Latest for steatorrhea:
o Fecal measurement of Elastase: < 100 : severe deficiency; >200 is normal
o Fecal fat excretion ( on 100 gm fat given): > 7 gm/ day in stools is steatorrhea
• IOC to diagnose Established Chronic Pancreatitis: CECT abdomen- Stricture, Stones and
atrophy
• Gold standard to diagnose Early Cases of Chronic Pancreatitis: ERCP- Chain of lakes
appearance seen**
• Recent Gold standard to diagnose Early cases of Chronic Pancreatitis: EUS
• ERCP Criteria- Known as CAMBRIDGE Criteria**
• EUS criteria- Known as Rosemont Criteria**

Management:
Pain management:
• Analgesics is given initially
• Coeliac plexus Block using Absolute Alcohol**- Laparoscopic or CECT guided method
• Thoracoscopic Sphlanchinectomy also available

Steatorrhea and weight loss:


• Pancreatic enzyme supplements- Creon, Pancrealipase
• The enzyme supplements must be given along with food**
• Acid reduction with PPI is needed while giving Pancreatic enzyme tablets.

Diabetes Mellitus:
• Best is by giving Inj. Insulin**

SURGERY SIXER APP BASED WORK-BOOK 2020 329


Surgical Management:
Drainage Procedures Resection+ Drainage Procedures Resection procedure
(Hybrid Proecdures)
• Dilated duct: > 7mm • Frey’s Procedure- Head Distal Pancreatectomy and
diameter:- Modified coring along with Lateral splenectomy for problems
Puestow procedure- Pancreatico Jejunostomy in Body and Tail- Child
Lateral Pancreatico Duval Procedure
jejunostomy procedure.
Presence of head mass
Duodenum Preserving
• For Undilated duct:- Pancreatic head resection
Izbicki Procedure in which (DPPHR)
V’ Shaped excision of the - Beger’s Procedure
duct done. - Bern’s Modification

If suspicious of malignancy:
Whipple’s procedure

2b- Tumors in pancreas

Types:
• Mc are adenocancers
• Endocrine tumors of Pancreas
• Cystic neoplasms of pancreas

Adenocarcinoma of Pancreas:
• Tumor within 2cm of ampulla is known as Periampullary cancers
• Head of pancreas- Most common site
• Body and tail.
Periampullary Cancers Pancreatic head cancers Body and tail
• C/F- Painless • C/F- Pain abdomen+ • Worst prognosis
intermittent jaundice Jaundice • Presents with Mets in
due to sloughing out • Late diagnosis 100% cases
of tumor. • Only 10% operable
• Early presentation • 90% are inoperable
and hence has good
prognosis
• 60% are operable

Genetic factors associated with Cancer pancreas:


• MC mutation- k- ras**
• Others- P16, P53,

SURGERY SIXER APP BASED WORK-BOOK 2020 330


• PRSS gene- Trypsinogen gene mutation
• SPINK- 1 mutation
• CFTR mutation
• Peutz jeugher syndrome ( STK-11)- 100 fold risk** ( NEET SS)
• Familial adenomatous polyposis
• Familial atypical melanoma syndrome
• BRCA-2
• Lynch 2- HNPCC**

Clinical features:
• Pain abdomen – Head tumors
• Painless intermittent Jaundice- Periampullary cancers
• Vomiting- MC cause is due to external compression of Duodenum by Tumor**
• Courvoisier’s Law- Gall bladder is enlarged in Pancreatic cancers.

TNM classification:
T1- <2cm
T2- 2-4 cm
T3- > 4cm
T4- Involvement of SMA, Common Hepatic artery and Coeliac artery
• Infiltration into SMV or Portal vein doesn’t change the staging

Resectable tumors Borderline resectable Unresectable


• Not infiltrating into • SMV / PV infiltrated • Involvement of SMA,
SMV/ Portal vein or even if more than Coeliac, Coeliac artery
infiltration < 180 180 degree > 180 degree
degree • SMA/ CHA or coeliac • Mets seen
• No mets artery involved < 180
degree**

Investigations:
• CECT abdomen is the IOC for staging- Pancreatic protocol CT is taken in which the cuts
are made at every 2-3 mm to look for small lesions and infiltration.
• Tumor Marker- CA- 19-9 **
• ERCP/ MRCP - Double duct sign seen*
• Barium meal shows- Frost berg sign or Reverse 3 sign** ( Epsilon sign)
• Biopsy is not mandatory for operable pancreatic cancers**. We advise Biopsy only for
inoperable cases.

SURGERY SIXER APP BASED WORK-BOOK 2020 331


Figure: Shows Double Duct sign

Management of Pancreatic cancer:


• Resectable cancer- in Head of pancreas / Periampullary cancer is Pylorus preserving
Pancreatico duodenectomy ( best answer) or Whipple’s operation is done.
• Distal pancreatic cancers- Distal pancreatectomy+ Splenectomy
• Inoperable patients: Palliative procedures
o Pain- Ceoliac plexus Blockade using Absolute alcohol
o GOO- Duodenal stenting by Endoscopy or Bypass procedures
o Jaundice- SEMS into biliary system or Bypass procedures
o Triple Bypass procedure is the one done for inoperable pancreatic cancer with
jaundice and GOO- Gastro jejunostomy+ Cholecysto jejunostomy+ Jejuno
jejunostomy

Operable cancers: Whipple’s Procedure

• The most important step in this operation- Pancreatico jejunostomy**

Complications of Whipple’s operation:


• Delayed Gastric emptying- increased Ryles tube aspirate or Vomiting- Incidence 18%

SURGERY SIXER APP BASED WORK-BOOK 2020 332


• MC cause of death- PJ anastomosis leak**; the active pancreatic enzymes leaking will
damage all the anastomosis and patient dies.
(Suture material for PJ anastomosis- Prolene or PDS- Don’t use Vicryl)
• Pancreatic Fistula: Drain kept after Whipple’s operation shows fluids which is having
increased amylase content:
- Rule of 3: 3rd POD, DT fluid amylase is 3 times higher than serum amylase**
• Bleeding

Pylorus preserving Pancreatico Duodenectomy ( Longmire and Traverso operation)


• This modification is done only to prevent Delayed Gastric
emptying** - < 1% as nerve of Laterjet is preserved.
• This is the procedure of choice for pancreatic cancer now.

Endocrine tumors of pancreas

• MC NET of pancreas- Pancreatic Polypeptidomas ( PPomas)- Non functional


• MC Functional NET – Insulinoma in sporadic**; Gastrinoma in MEN-1 Syndrome**

Insulinoma Gastrinoma
MC NET MC NET in MEN-1 Syndrome
MC site= Equal in H=B=T MC in Passaro triangle
• Junction of CD/ CBD
• Neck of Pancreas
• 2/3rd Part Duodenum

MC site – Duodenum> Pancreas

Mostly benign Mostly Malignant


Clinical features: Whipple’s Triad Clinical features: Zollinger Ellison Syndrome

SURGERY SIXER APP BASED WORK-BOOK 2020 333


• Signs and symptoms of hypoglycemia • Unusual peptic ulcer disease
• Blood glucose < 40-50 mg/dl • Diarrhoea
• Relief of symptoms by administration
glucose

Hence there is weight gain.

IOC to diagnose: IOC to diagnose:


• 72 hours FBS and Insulin level • Serum Gastrin >1000 pg/ml
• C peptide value increased ( To • In border line elevated cases we must
differentiate from Exogenous insulin do- Secretin Stimulation test**
injection cases- Normal in Exogenous • On SSI- the value increases from
administration) baseline to 200 pg /ml above in 15
minutes.
Localisation : ( No Somatostatin receptor) Localisation:
• Intra op USG ( Best ) • Somatostatin Receptor Scinitigraphy**
• Endo USG
• Selective Arteriography venous
sampling of blood
Management: Management:
• Enucleation is enough as this is a Malignancy and hence radical resection
Benign tumor** needed.
• Whipple’s is TOC ( PPPD is
contraindicated ) for Head tumors
• Distal Pancreatectomy for Tail and
body cases
Adjuvant therapy/ Inoperable/ metastatic: Inoperable cases/ Metastatic cases:
• Streptozocin • Octreotide
• Diazoxide • Triple dose PPI
• Total Gastrectomy may also be done

Other NET:
1. Glucagonoma:
• Presents with Diabetes+ Dermatitis
• Dermatitis- Necrolytic migratory erythema**
• MC in Head*
• Mostly Malignant

2. VIPoma:
• Also known as Verner Morrison Syndrome/ WDHA syndrome/ Pancreatic cholera
• WDHA syndrome- Watery Diarrhea+ Hypokalemia+ Achlorrhydria
• MC site is Tail**

SURGERY SIXER APP BASED WORK-BOOK 2020 334


Cystic neoplasms of Pancreas

Salient Points:
• IOC for Cystic Neoplasms- EUS guided Biopsy and Fluid analysis
• Fluid analysis shows elevated CEA level** in IPMN and Mucinous neoplasm
• CECT- SCN- Sun burst calcification
• CECT- MCN- Egg Shell Calcification
• ERCP for IPMN shows- Fish Mouth Appearance in Ampulla ( NEET SS)
• Solid Papillary Mucinous Neoplasm (SPMN)- Frantz tumor

SURGERY SIXER APP BASED WORK-BOOK 2020 335


SURGERY SIXER APP BASED WORK-BOOK 2020 336
Chapter 3: Liver and Portal Hypertension

3a- Introduction

Anatomy of liver:
Portal triad structures:
• Posterior – Portal Vein
• Anterior to right- CBD
• Anterior to left- Hepatic artery

Hepatic artery:
• Aberrant/ Abnormal/ Accessory Right Hepatic artery arises from Superior mesenteric
artery and runs posterior to portal vein.
• Aberrant/ Abnormal/Accessory Left Hepatic artery arises from Left gastric artery
• Abnormal cystic artery from GDA*

• Bile Duct : Blood supply arises from Right Hepatic artery at 3’0 and 9’0 Clock position**
from above downwards and from GDA from below upwards.
• Major Blood supply to liver- 70-80% by Portal vein, 20% by Hepatic artery.
• Major oxygenated blood to liver is by Hepatic artery
• 100% blood supply to a tumor is from Hepatic artery only
• The venous drainage is via- right, left and middle hepatic vein into IVC and systemic
circulation.

SURGERY SIXER APP BASED WORK-BOOK 2020 337


Coiunaud’s Surgical Segments of Liver:
• Total segments in liver= 8 segments
• The Middle Hepatic vein corresponds to an imaginary line CANTLE’S Line- runs from Gall
bladder fossa to right of IVC. ( Cantle’s line- Cholecysto caval line)
• Left and Right Hepatic veins further divides the liver and runs along Left and Right main
fissures*
• Portal veins also divides the liver by running on the Right and left scissura**

Figure: Liver segments

• Segment 1 is located on Posterior surface of liver


• Segment 2 and 3 are seen medial to falciparum ligament- Left lateral segments
• Segment 4a and 4b are seen lateral to Falciparum ligament- Left medial segments
• Segment 6 and 7 are known as Right posterior segments
• Segment 5 and 8 are known as Right anterior segments
• Each segment is a separate functional unit and contains HA, BD, PV and HV branches.
• Because of this concept- We can now do segmental resection of Liver and Live Donor
transplant**
• Segment 1- Caudate lobe; Directly drains into IVC**
• Caudate lobe is further divided into- Segment 9 and Spiegel lobe.
• Caudate lobe is enlarged in Budd Chiari Syndrome*

Brisbane 2000 is the latest naming for hepatectomy:


• Left hemihepatectomy= 2,3, 4a,4b
• Right hemihepatectomy= 5-8
• Left lateral hepatectomy= 2,3 removal ( lateral to Falciparum ligament)
• Right Posterior hepatectomy= segment 6 and 7 removed
• Radical Cholecystectomy= 4b +5
• Central hepatectomy= 4b+ 5+ 8 (Tajmahal Hepatectomy)
• Right Trisectionectomy= Extended Right hepatectomy= 5-8+ 4a and 4b

SURGERY SIXER APP BASED WORK-BOOK 2020 338


- Done for Type 3a Hilar Cholangiocarcinoma
• Left Trisectionectomy= Extended Left hepatectomy= 2,3,4,5 and 8
- Done for Type 3b Hilar Cholangiocarcinoma
• Paediatric liver transplant donated Segments 2 and 3 are used
• Adult Liver Transplant Donated segments-5,6,7,8

Scans in Liver:
Liver parenchyma is made up of two types of cells
• Hepatocytes – Perform excretory and synthetic functions.
• Kupffer cells- They have Reticulo endothelial function
• Both of these cells can be investigated with Radionuclide scanning.
• ITO cells ( Stellate cells)- used for Vitamin A storage

Two types of radio nuclide imaging procedure are used in liver


HIDA Scan Sulphur Colloid Scan
• This Scan is based on • This is based on the functioning RES
Functioning Hepatocytes. cells (Kupffer cells) of liver

IDA compound are taken up by the Hot spots- mass lesions which contain
functioning hepatocytes, excreted functioning RES cells ( kupffer cells)
unchanged in bile ducts and gallbladder.
Non-functioning areas- mass lesions which
Clinical Uses: do not contain kupffer cells.
• Assess regional liver function
• Bile leaks in liver trauma Clinical Uses:
• Investigation of biliary • FNH- only tumour which consistently
obstruction contains functioning RES cells and
• Choledochal cyst therefore shows uptake of colloids.
• Acute cholecystitis (Gold • Hepatic adenoma- nonfunctioning
standard) kupffer cells, hence do not uptake
• Differentiate hepatocellular
tumors

HCC will not take up colloid or HIDA scan**

Liver Function tests:


• AST/ ALT- Liver specific is ALT** ( AST is not specific)
• Alkaline Phosphatase- Obstruction of Bile duct
• GGT- Alcohol induced liver damage**
• Chronic Liver disease:
o Albumin is a marker of Chronic liver disease.
o PT/INR will be elevated

SURGERY SIXER APP BASED WORK-BOOK 2020 339


Scoring Systems in liver

Modified Child Pugh Score ( Mnemonic- BAAPE)


CLINICAL** 1 2 3
Encephalopathy None 1 or 2 3 or 4
Ascites None Mild moderate
Bilirubin (mg/dL) 1-2 mg/dl 2.1- 3 mg/dl ≥ 3.1 mg/dl
Albumin (gm/dL) ≥3.5 gm/dl 2.8- 3.4 gm/dl ≤ 2.7 gm/dl
Prothrombin time 1-4 4.1 - 6 ≥ 6
(increase in seconds)
 Grade A: 5-6
 Grade B: 7-9
 Grade C: 10-15 points

MELD: Modified End Liver Disease Scoring: Based on: “CBI”


• Creatinine
• Bilirubin
• INR
o Recently Waiting list for transplant is made on the basis of MELD score*
o Initially MELD score was used to predict mortality after TIPSS
o Values vary from 6-40*. Liver transplant is allowed only for MELD > 15** ( ref. Sabiston
Page 658)

Paediatric End Stage Liver Score: (Mnemonic “BANIA”)


• B- Bilirubin
• A- Albumin
• N- Nutritional status
• I- INR
• A-Age

King’s College Selection Criteria for Liver Transplant in Acute Liver Failure:
Paracetamol Induced Non Paracetamol Induced
• pH <7.30 (Irrespective of grade of • PT > 100 Seconds (Irrespective of
encephalopathy) grade of Encephalopathy)
(or) (Or)
• Prothrombin Time( PT ) > 100 Any three of the Following :** NEET PG 2020
Seconds + Serum Creatinine > 300 • Age <10 years or >40 years
Micro mol/litre + Encephalopathy of • Etiology being Non A , Non B,
Grade 3 or 4 ( all three together) Halothane or Drug reaction

SURGERY SIXER APP BASED WORK-BOOK 2020 340


• More than 7 days Jaundice before
Encephalopathy
• PT> 50 seconds
• Bilirubin > 300 Micromol/ Litre

Chapter 3b- Cirrhosis with portal hypertension

Explanation for image given above:


• Portal vein formed by SMV+ SV joining
• Portal vein = 5-8 cm length
• PV divides into Right and left and divides into multiple sinusoids in liver.
• Three hepatic veins drains from sinusoids into IVC
• Normal PV pressure= 5-8 mmHg

Causes of Portal Hypertension:


Presinusoidal Sinusoidal ( MC) Post sinusoidal
• Intrahepatic- • Cirrhosis causing • Intrahepatic- Veno
Schistosomiasis (MC ), fibrosis ( MC) occlusive disease- Sub
NCPF lobular venules
• Extra hepatic- obstructed
EHPVO • Extra hepatic- Budd
• Left sided Portal Chiari Syndrome- HV
Hypertension ( or IVC occlusion

SURGERY SIXER APP BASED WORK-BOOK 2020 341


Sinistral portal
hypertension)

Pathophysiology:
• Due to high portal pressure- the collaterals will open..
• Collaterals between Systemic and Portal venous system:
o Left gastric vein with Hemi azygous vein- Esophageal varices
o Inferior mesenteric vein ( Superior rectal vein) with Middle and inferior rectal
vein- Hemorrhoids develop
o Left Umbilical ( Paraumbilical vein) opens up- Caput medusae
o Bare area
o Retroperitoneum
• Ascites due to increased SMV pressure
• Splenomegaly

Complications:
• MC cause of death- liver failure ( only option is Liver transplant)
• 2nd MC cause of death- Bleeding**
▪ Esophageal variceal bleed ( MC)
▪ Portal Hypertensive gastropathy

Hepato-pulmonary syndrome Porto Pulmonary Hypertension


There is pulmonary vasodilatation There is Vasoconstriction of the vessels
resulting in Hypoxia, Platypnea resulting in Pulmonary artery pressure
increases > 25 mmHg

IOC- Echo IOC- Wedge Pressure of Pulmonary Artery


( PCWP)
Treatment- Liver transplant Treatment- Liver transplant
Bad prognosis and more serious
• If Pulmonary pressure > 50
mmHg- Liver transplant will be
contraindicated**

Protocol in Oesophageal variceal Bleeding:


• Among upper GI bleed- 20% is due to Portal hypertension.
• Causes of Bleeding – Variceal bleed in Esophagus or Stomach and PHTG
• Varices formed at PP> 10 mmHg
• Varices Bleeding occurs at PP >12 mmHg

SURGERY SIXER APP BASED WORK-BOOK 2020 342


1st Step: Massive hematemesis:
• IVF, Ryles Tube aspiration, Foley’s catheter
• DOC for variceal Bleeding- Injection Octreotide** infusion ( Please don’t forget the first
drug given in Upper GI Bleed= PPI)
• Other drugs- Vasopressin or Terlipressin along with GTN to avoid Coronary ischemia*

2nd Step: Ryles tube shows continuous Bleed:


• Modified Sengstaken Blakemore Tube:
o Kept in freezer
o Two Balloons
o 4 lumens
o Made up of rubber
o 1st Inflate Gastric Balloon using 300 ml of Air ; followed by Esophageal
Balloon upto 40 mmHg
o Inserted via Mouth or nose.
o Lumens are used to Inflate and aspirate the contents.
o Tamponade effect is produced on Varices
o Tamponade effect can be used only for Varices and not for Dieulafoy’s lesion,
Mallory Weiss tear as they are arterial bleeding** ( NEET SS)
o Maximum Time to retain the Balloon- 12 hours
o Extended pressure results in Esophageal necrosis and perforation.

Figure: Modified Sengstaken Blakemore Tube

3rd Step: Patient lands up in MGE unit:


• Treatment of Choice for EV is Endoscopic Banding > Endoscopic Sclerotherapy
• Sclerosants used are- Sodium Tetra decayl.
• Gastric varices are treated by Endoscopic Glue injection- Cyano acrylate Glue**

4th Step : Failed Endoscopy by MGE – Call the Interventional radiologist:


Transjugular Intrahepatic Shunt ( TIPS)
• Intra hepatically a stent kept between HV and PV by trans jugular Route**
• The stent will now reduce the Portal pressure immediately and Varices bleed stops.
• Stent is made up of Polytetra floroethylene ( PTFE)

SURGERY SIXER APP BASED WORK-BOOK 2020 343


Complications of TIPS:
• Early Complication**- Capsule Rupture ( NEET SS/PG) with intra peritoneal
haemorrhage.
• Within 1 month- Encephalopathy can happen**
• Shunt thrombosis also develops in 1 month
• Shunt stenosis occurs in 1 year ( Most common)
Contraindications for TIPS:
Absolute Contraindications Relative Contraindications
• Right Heart Failure • Portal vein thrombosis
• Poly cystic Liver disease • Encephalopathy
• Hepato Pulmonary syndrome • Liver tumors
• Porto Pulmonary
hypertension

Added Advantage of TIPS – Ascites also resolves**

5th Step : if TIPS not possible or feasible:


Surgical Shunt operations:
Non Selective shunts Selective Shunts
Entire Portal Blood is diverted into systemic Only Splenic blood diverted ; Portal Blood is
circulation and hence Encephalopathy flowing into liver and metabolised and hence
develops* no Encephalopathy; But there is ASCITES
formed
Types : Types :
• End to Side Porto caval • Warren’s= Distal Spleno renal shunt:
• Side to Side Porto Caval the distal splenic vein is anastomosed
• 16 mm Interposition Portocaval to left renal vein. Increased pressure in
• TIPS also Non selective SMV causes Ascites**

• LINTON= Conventional= Proximal • Inokuchi shunt: Left gastric vein (


Splenorenal shunt: Splenectomy is Coronary vein) anastomosed to IVC*
done and Proximal splenic vein
anastomosed to Left Renal vein

Contraindications of Shunt Operations:


• Splenic vein thrombosis
• SMV thrombosis
• Splanchnic vein thrombosis

SURGERY SIXER APP BASED WORK-BOOK 2020 344


6th Step : if Shunt operations not possible: Devascularisation operations:
Sugiura operation:
• Ligation of veins near
Esophagus.
• Vagotomy
• Pyloroplasty
• Esophageal transection and
anastomosis
• Splenectomy

Final resort in all cases of PHT= Liver transplant

SURGERY SIXER APP BASED WORK-BOOK 2020 345


Figure: Variceal Bleed in Nutshell

Various causes of portal hypertension

Sinistral / Left PORTAL hypertension


• There is a splenic vein thrombosis due to Chronic pancreatitis
• Hence Short Gastric vessels dilate and develop only left side PHT with Isolated Gastric
Varices+ Splenomegaly**
• Treatment of Choice- Splenectomy.

Non Cirrhotic Portal fibrosis ( NCPF) EHPVO ( Extra Hepatic PV obstruction)


2nd and 3rd order of PV branches are fibrosed Main or 1st order Portal vein obstructed
MC in age 20-40 years MC in < 10 years
Hence stunted growth seen
Common Features:
• Esophago gastric varices rypture and hematemesis ( MC presentation)
• Splenomegaly ( More bigger in NCPF)
• Liver histology is normal
• LFT is normal
• No ascites**
Treatment:
• Splenectomy + Proximal Splenorenal shunt ( LINTON)
• They don’t develop encephalopathy as their liver is normal
• REX shunt- most physiological shunt done for EHPVO between SMV to Left portal vein
using IJV of the baby.

SURGERY SIXER APP BASED WORK-BOOK 2020 346


Figure : REX shunt for EHPVO

Post sinusoidal causes of Portal Hypertension- VOD Vs BCS


Veno occlusive disease:
• Sublobular venules gets thrombosis
• MC etiology: Follows Bonemarrow transplant
• Others- Pyrazolidone alkaloids, Azathioprine
• DOC – Defibrotide
• Venography shows HV and IVC normal ; Wedge HV pressure is increased.

Budd Chiari Syndrome:


• 2 causes- HV thrombosis and IVC occlusion by Congenital membranous webs
• BCS- MC cause in western is HV thrombosis**- The thrombosis mc due to Polycythemia
Rubra vera**
• BCS – MC cause in Asians/ Indians is IVC webs
• IOC to diagnose- Hepatic Venography is gold standard- SPIDER WEB appearance**
• CECT abdomen- shows Enlarged Caudate Lobe – HOT SPOT of caudate lobe seen
Management:
• No role of Thrombolytic agents as it is a well formed thrombus.
• Liver is normal – Shunt operations are planned as shown below:
• Liver failure cases- Liver transplant is TOC

SURGERY SIXER APP BASED WORK-BOOK 2020 347


Figure: Shunt operations in BCS
Shunt operations:
• HV thrombosis- Side to side Porto caval shunt ( SSPCS)done
• IVC webs- Double shunt operation done ( SSPCS+ cavoatrial shunt ) or SSPCS with IVC
stenting

Clinical method to find – HV thrombosis alone or IVC web?


• Yes we can find out seeing the Multiple collaterals running from Below upwards in IVC
obstruction cases.

Figure shows- IVC web causing BCS and Liver failure**

Chapter 3c. Surgical infections in Liver


• Pyogenic liver Abscess
• Amoebic Liver Abscess
• Hydatid Cyst

Routes of Infection to liver:


Biliary Route Hematogenous Portal Vein route Direct spread from
(Hepatic Artery ) adjacent organs or
trauma
• MC route of • Septicemia • Infection from
infection carries Blood Colon like
• Follows infected to Liver Appendicitis,
Ascending via Hepatic Diverticulitis,
Cholangitis. artery. Amoebic
• MC organism • MC organism- infections are
associated – Staphylococcus** carried to liver.
E.Coli ( World)
Klebsiella ( Asians)

SURGERY SIXER APP BASED WORK-BOOK 2020 348


• Amoeba
transferred only
by this route
• Abscess will be • Abscess will be • Abscess will be
multiple Macro multiple Micro solitary and
abscess** abscess Large in
• Multi organisms • Single organism Amoebic abscess
are isolated isolated

Figure: Route of Infection

Pyogenic abscess Amoebic liver abscess


Sick patient with Fever, Chills and Rigors with Hepatomegaly + Pain
Hepatomegaly. (MC symptom- Pain)
Severe Right Hypochondrial tenderness++
M:F= 1.5:1 M>>>>>>> F
In Menstruating females it is never seen
Iron Deficiency anemia is protective
Predisposing Factors: Predisposing factor:
• MC in immunocompromised cases • Alcohol**
• Diabetes mellitus
Lab Values: Lab Values:
• Increased AST,ALT, Alk.po4, Bilirubin • All values Normal
• Only PT may be elevated **(NEET SS)
Serology: Negative for Amoeba Serology: Amoebic antigen+
Blood Culture is positive Blood Culture is negative*

SURGERY SIXER APP BASED WORK-BOOK 2020 349


CECT abdomen: CECT Abdomen:
• Clustered Abscess seen with Peripheral • Well defined abscess with no
rim Enhancement** peripheral enhancement but there
• They may be 50: 50 Single or may be peripheral wall edema++
multiple • Trophozoites are present in the wall.
• Mostly they are solitary**
• MC in segment 7**

Complications : Complications:
• Sepsis Color of pus- Anchovy sauce color
• Endogenous Endopthalmitis ( MC in Odourless.
Diabetes- 3% of patients will develop • MC complication is reactive pleural
due to Klebsiella) effusion in Right side.** ( Not rupture)
• Osteomyelitis • MC site of Rupture into Peritoneal
• Infective Endocarditis** cavity**
• Rupture into Pleura- Put an ICD**
• Rupture in Bronchus ( Broncho Pleural
fistula)- Cough shows Anchovy sauce
sputum. Treated by Lying in Prone
position and Bronchodilators+ Chest
Physiotherapy. Pus drained via
sputum.
• Rupture into Mediastinum

Treatment: Treatment:
• Immediate Treatment of choice is 1/3rd of patients has H/o Amoebic dysentery
aspiration of abscess 2/3rd develops from Subclinical infection
• Based on Pus C/S and Blood C/S- • TOC – Injection Metronidazole 750
Higher antibiotics are given. mg TDS for 2 weeks and convert to
oral
• Luminal Amoebicides- Diloxanide
furoate, Emetine etc X 10 days
• Aspiration- Indications:
- Doubtful diagnosis
- > 5 cm sized abscess

SURGERY SIXER APP BASED WORK-BOOK 2020 350


- Located on Left lobe
- Superficial located abscess
- Not responding for >3 days.

Surgical treatment:
• DP stent under USG guidance . Stent
left for 2 weeks to 1 month.
• Even after complete drainage of
abscess- the residual cavity remains
upto 6 months – 9 months

Hydatid Cyst Liver


• Caused by Echinococcus granulosus, Echinococcus Multilocularis
• E. Multilocularis is very rare and dangerous ( Malignant Hydatid Cyst) . Always need liver
resection.
• E. Granulosus- Dog Tape worm.
• Definitive host- Dogs
• Intermediate hosts- Sheep and Goats
• Accidental Host- Human (End host – Will not spread from one man to other)

Pathophysiology:
• The MC site of hydatid cyst- Segment 6 and 7**
• The cyst wall is formed by: 3 layers
o Outer Ectocyst/ Pericyst/ Adventitial Layer- Derived from Host Tissue
o Middle Laminated membrane- Thin membrane allows some essential
substances
o Inner Germinal Layer- Living layer producing Daughter cysts**
• MC site- Liver
• Other sites- Lungs, Brain, Kidney
• Ectocyst is derived from Host tissues and is absent in Lung and Brain Hydatid cyst**
• Ectocyst is seen in Liver and kidney only

Clinical features:
• MC feature- Hepatomegaly ( No pain)
• Jaundice- Hydatid rupture into Biliary tree causes jaundice
• Passing of grape like membranes in motion

Investigations:
• IOC to Diagnose- Immunoblast assay**
• Others- ELISA , Immune Electrophoresis, Casoni’s Intradermal test
• Radiological IOC- CECT Abdomen

SURGERY SIXER APP BASED WORK-BOOK 2020 351


Gharbi et al Classification of Hydatid Cysts is old classification ,
Latest classification is WHO classification:
o CE 1- Unilocular cyst
o CE 2- Multiseptated ( Honey comb/Rosette like)
o CE 3- Floating membrane ( water lilly sign)
o CE 4-Heterogenous cysts with partial calcification
o CE 5- Calcified wall

Type 1,2,3 are active cysts


Type 4,5 are Inactive and dead cysts

Figure; CE1,2,3 Hydatid cysts in USG

Figure: CECT shows Hydatid cyst

Complications:
• MC is biliary rupture- 30%**
• Secondary infections

Extra Mile
Risk of biliary-cyst communication. ( For NEET SS from Blumgart repeat)
• Male patients
• Abnormal preoperative serum alkaline phosphatase and γ-glutamyltransferase (GGT);
• Multiple cysts, multilocular and degenerated cysts
• Cysts near the biliary bifurcation,
• Presence of bile-stained or purulent cyst

SURGERY SIXER APP BASED WORK-BOOK 2020 352


• Cyst diameter greater than 10 cm was an independent clinical predictor for the
presence of intrabiliary rupture.

Management:
• T. Albendazole started.
• Under cover of T. Albendazole after 2 weeks we plan for Surgery / PAIR

Surgical treatment options:


• Partial pericystectomy ( Leaving part of pericyst thereitself)- Surgical best option**
• Hepatic resection
• After partial pericystectomy look for Biliary leak inside the cavity ( Communication)- we
must suture the leak site. Some surgeons also advise T tube insertion in CBD also.

Percutaneous Aspiration , Infusion of scolicidal agents and Reaspiraion ( PAIR)


 Given with prophylactic cover of albendazole.
 Scolicidal agents- 20% hypertonic saline ( Best) , 0.5% silver nitrate, 95% ethanol,
absolute alcohol, Mebendazole 2.4 microgram/ ml, 10% povidone Iodine**

Contraindications for PAIR:


1. Superficially located cyst (chance of rupture)
2. Honey combing of cysts ( multiple thick internal septae)- CE2
3. Communication with biliary tree.
4. Dead or inactive cysts
5. Lung/ Brain Cysts- CE4 and 5
6. Peritoneal rupture or Pleural Rupture

Also Know:
• PAIR: < 6 cm cyst ( < 100 ml)
• PAIR- Catheter: > 6 cm cyst ( > 100 ml)
• PEVAC ( percutaneous Evacuation of Cyst Content)

SURGERY SIXER APP BASED WORK-BOOK 2020 353


Chapter 3d- Tumors in Liver
• IOC for SOL liver- CECT abdomen
• IOC for hemangioma-MRI **

BENIGN LIVER TUMOUR


Characters Hemangioma Focal nodular Liver cell adenoma
hyperplasia
Incidence m/c Incidentaloma Second common benign Rare
m/c benign tumour tumour
Age group/sex About 45 years Middle aged females Young women
m/c in females 20-40 years
Associated ------- Female hormones and OCP and steroids
factors OCP
Complications Rupture / Rupture/ Rupture and malignant
hemorrhage hemorrhage transformation**

Giant
Hemangioma- > 5
cm
Kassabach merit
syndrome- Huge
hemangioma
consuming platelets
and causing
Thrombocytopenia+
CCF
Histology Small capillary type Characteristically central Benign histocytes
and large cavernous fibrous scar with containing glycogen or
types are there radiating septa fat

Imaging studies Slow contrast Characteristic Central CT- heterogenous mass


CECT abdomen- enhancement from stellate scar seen** with early enhancement
Triple phase periphery to centre
due to small vessel SULFUR COLLOID LIVER
uptake . Typical SCAN- kupffer cells in
nodular peripheral FNH only take up the
enhancement colloid and appear as
HOT Spot**
MRI – T2 weighted
image shows –
Light Bulb sign”

SURGERY SIXER APP BASED WORK-BOOK 2020 354


Treatment No treatment No treatment needed Laparotomy and
needed resection**

Extra edge points in Liver cell adenoma:


• MC they are single**
• Such single adenomas are MC in females
• 20% cases are multiple- these are not associated with OCP and no female
preponderance**- These cases need liver transplant**
• Beta Catenin Mutation+ ve cases- High risk of malignancy is seen- Early surgery needed

Malignant Tumors in liver


• MC malignant tumor is Secondary
• MC primary malignant tumor is HCC**

Hepatocellular cancers:
Etiological factors:
Infections Cirrhosis Environmental Metabolic diseases
HBV Alcohol Aflatoxins Hemochromatosis
HCV Autoimmune Pyrrozolidone Alpha 1 AT
Primary biliary alkaloids deficiency
Thorotrast Wilsons
Nitrosamines Glycogen storage
diseases
EHBA
• HCC usually arises from Cirrhotic liver background only except in HBV induced HCC**

Clinical Features:
• K/C/O Cirrhosis sudden decompensation of Liver with Pain, Mass and weight loss all
suggest malignancy**
• Paraneoplastic syndromes:
o MC is Hypercholesterolemia > Hypoglycemia**

SURGERY SIXER APP BASED WORK-BOOK 2020 355


o Hypercalcemia
o Polycythemia( Erythrocytosis)
( MC PNS in RCC- increased ESR**)

Types of HCC:

• Hanging type- Best prognosis


• Infiltrative type- Bad prognosis

Investigations:
Tumor markers:
• AFP > 20 ng/ml- 70% have elevation
• AFP > 400 ng/ml- Conclusive of HCC
• Glypical-3
• PIVKA- protein induced Vitamin K absence** - Descarboxy prothrombin**
• AFP-L3 Lectin 3 fraction

CECT abdomen with triple Phase is Gold standard:


• Rapid contrast enhancement of Tumor in arterial phase**
• Rapid wash out in Portal venous phase**
• Phases in Liver on CECT:
o Early arterial
o Late arterial
o Portal venous phase
o Delayed

SURGERY SIXER APP BASED WORK-BOOK 2020 356


Figure: See the phases of HCC in CECT

NCCN Diagnostic criteria


• > 2cm lesion: With Typical Finding in CECT explained – Diagnosis is confirmed
• 1-2 cm lesion: we need 2 imaging techniques needed like CECT+ MRI
• No biopsy in operable cases. Inoperable cases Biopsy may be taken

Various Staging systems in HCC:


OKUDA staging CLIP- Cancer of Liver Italian Chinese University Prognostic
Program index ( CUPI)
T- Tumor size
A- Albumin
A- Ascites
B- Bilirubin

Latest Staging system which helps in Diagnosis and treatment- Barcelona Clinic Liver Staging (
BCLC Staging )
• Very early stage- < 2cm size, Single lesion- Best treatment is Resection**
• Early stage- < 3 lesions , < 3cm size all lesion- Resection or Transplant advised.
• Intermediate stage
• Advanced Stage
• Terminal Stage

SURGERY SIXER APP BASED WORK-BOOK 2020 357


Modalities of treatment:
• Best is Surgical resection:
o In a cirrhotic liver – Future Liver remnant ( FLR ) must be atleast 40%
o In a normal liver- FLR must be atleast 25%
o If above FLR is not possible- Liver resection in contraindicated
• Liver transplantation: Milan Criteria** of Liver transplant in Cirrhotic liver
o < 5 cm single lesion
o < 3 lesions all 1-3 cm each
o No Extra hepatic spread seen

Ablative therapies:
• Percutaneous ethanol Injection ( PEI)
• Percutaneous Acetic acid injection
• Percutaneous Thermal Ablations- RFA, Cryoablation**
• Heat sink effect- is seen in patients who have tumors very close to blood vessels will have
the thermal effect dissolved by running blood and less effective*

Interventional Procedures:
• TACE – Transarterial chemo embolization: Into hepatic artery we can inject
Chemotherapy – Loaded with Lipoidal sodium, which carries chemotherapy to tumor

SURGERY SIXER APP BASED WORK-BOOK 2020 358


• TARE- Transarterial Radio embolization

Properties of HCC:
• Tumor can grow into HA or PV
• PV invasion is bad prognosis
• Only 10-15% cases are operable
• Inoperable cases- Latest therapy- SORAFENIB- Multikinase inhibitor** - Increases 6-9
months survival:
▪ Serine threonine inhibitor
▪ VEGF inhibitor
▪ Tyrosine Kinase inhibitor
▪ Platelet Derived Growth Factor inhibitor

Fibrolamellar HCC
• M:F= Equal
• MC in young age
• Liver is normal here, NO cirrhosis
• HBV and HCV are negative
• AFP is not elevated
• Tumor marker- Neurotensin B**
• CECT abdomen: Important DD- FNH as they both have central scar**
• Good prognosis
• Mostly resectable tumors

Metastasis to liver
• MC primary for Liver mets- Colorectal cancer
• MC primary from Extra abdominal- Breast
• Mets with good prognosis- NET mets**> CRC mets
• Resection is approved for CRC and NET mets only so far.

Hypodense/ Hypovascular mets Hyperdense/ Hypervascular mets


They are detected on Late Portal Venous They are detected on Late Arterial Phase after
Phase** 35 Seconds of contrast injection.

• Git primaries • Renal cancer


• Lung cancer • Insulinomas
• Breast Cancer • Carcinoids
• Head and Neck cancer • Sarcomas
• Melanoma
• Also Breast cancer

SURGERY SIXER APP BASED WORK-BOOK 2020 359


CECT- Hypodense mets
CECT- Hyperdense mets

Fong Score of Outcome for Colorectal Cancer Metastasis in Liver: ( NEET SS)
• 5 Predictive factors for outcome of CRC mets liver: all the below are bad points*
1. Size > 5cm
2. Disease free interval < 1 year
3. More than 1 tumour
4. Lymph node positive primary
5. CEA > 200 ng/ml
Each given 1 point and scoring made as shown below:
Based on these points Survival rate is mentioned.
• If score is 1- Median survival is 51 months
• If score is 5- Median survival is 22 months**

Other liver lesions- Images may be asked in exams

Simple cyst liver:


• Symptomatic cases are treated by
Laparoscopic Fenestration (
Deroofing)
• No risk of malignancy

SURGERY SIXER APP BASED WORK-BOOK 2020 360


Biliary cystadenoma:
• Septal wall enhancement seen on
CECT
• Can go for malignancy
• Resection of entire cyst advised.

Polycystic liver disease:


• Liver failure never happens inspite of
so many cysts
• Treatment is only for Symptomatic
reasons
• They will develop PCKD and renal
failure**

SURGERY SIXER APP BASED WORK-BOOK 2020 361


Chapter 4: Spleen

Anatomy of Spleen:
• Ligaments attached with Spleen:
- Gastrosplenic Ligament
- Spleno colic Ligament
- Spleno phrenic Ligament
- Lieno renal Ligament

• The ligament containing the Splenic Vessels – Lienorenal ligament**. Hence this is the
ligament which must be Ligated and cut in emergency Splenectomy of Trauma.
• Gastrosplenic ligament has short gastric vessels. We must cut the Gastrosplenic Ligament
1st in elective surgeries.
• The ligaments which are avascular- Spleno colic and spleno phrenic ligament**

Dimensions:
• 150 gm
• 12X7X3 cm size
• Lying along 10th Rib it lies.
• Notch seen in Anterior border of Spleen.

Splenic artery Is the largest artery from Coeliac trunk:


- Divides near Hilum known as Magistral type- 30%
- Divides very proximal into multiple branches – 70%

• Splenic artery runs on the superior border of pancreas


• Splenic Vein runs inferiorly on the posterior part of Pancreas.

Functions of Spleen:
• Immune Function: IgM produced*
• Filtering Function: Culling – removal of iron from RBC and bringing into
circulation**, Removal of senescent RBCs.

SURGERY SIXER APP BASED WORK-BOOK 2020 362


• Pitting- Removal of Particulate inclusions in RBC.
• Reservoir function- Stores 8% of RBC**- 2% Blood volume is inside spleen
• Haematopoiesis.

Peripheral smear after splenectomy:


• Howel Jolly Bodies- Non Functional nucleus
• Heinz Bodies- Denatured Haemoglobin
• Pappenheimer Bodies- Iron Particles in RBC
• Target Cells

Indications for Splenectomy:


Emergency Splenectomy Elective Splenectomy
MC indication for splenectomy- Blunt trauma Idiopathic Thrombocytopenic purpura is the
MC Elective indication
During Some other Surgery:
- D2 Lymphadenectomy
- Gastric cancer + Spleen infiltration
- Pancreatic Tail neoplasm
Haemolytic Anemias
- Sickle cell
- Auto immune
- Hereditary spherocytosis
Multiple Abscess in Spleen
Tumors in Spleen

Methods:
• Open method

SURGERY SIXER APP BASED WORK-BOOK 2020 363


• HALS- Hand Assisted Laparoscopic Surgery. Hand port used to remove the specimen
finally
• Laparoscopic method- Spleen Removed via a Plastic pouch with maceration. It can be
used in Tumors as there will be spillage.

Complications of Splenectomy:
Immediate complications Intermediate Complications Delayed Complications
o Haemorrhage o MC complication o Thromboembolic
o Gastric distension overall is Left Lung manifests
o Hematemesis Atelectasis** o OPSI
o Pancreatic fistula o PV thrombosis
o Gastric Fistula
o Subphrenic abscess

Opportunistic Post Splenectomy Infections:


o MC after splenectomy for Haematological causes > trauma causes
o Bacterial infection**-Inability to remove capsulated Bacterias- Streptococcal Pneumonia,
H.Influenza and meningococcal
o Parasitic Infection- Babesia and malaria
o Viral Infection- Not increased

Haematological Effects:
o Increased Platelet Counts- Can cause Thrombotic manifests ( if count > 8 Lakhs ;Anti
platelet drugs started)
o Increased WBC counts.

Prevention of OPSI:

SURGERY SIXER APP BASED WORK-BOOK 2020 364


On discharge: Post op antibiotics:
o Method 1: Children are advised for next 2 years after splenectomy
o Method 2: Some advocate for life-long Prophylaxis
o Method 3: Rational approach: To have antibiotics in hand and take when there is onset of
febrile illness and rigors by patient himself and later come to hospital**- MC followed.

Idiopathic Thrombocytopenic Purpura


o Autoimmune disease
o IgG Antibodies are produced from Spleen against the Platelets** and destroy the platelets.
o Hence platelet count falls, Mega thrombocytes seen.
o Bone marrow study is normal and all causes of Thrombocytopenia ruled out before
branding a patient as ITP
o Clinical Features:
▪ MC in Young women
▪ In children M=F
▪ C/B- Mucocutaneous Bleeding, Epistaxis, petechiae and Purpura**
▪ Massive Bleed or GI bleed or Hematuria or Intracerebral bleed are very
rare.
o Management depends on Platelet Counts:
▪ > 50000: No need to worry, Even major surgery can be done. Observation
▪ 30000-50000: Frequent Follow up is advised
▪ < 20000 + Mucous bleeding : Hospitalise the patient
▪ < 10000: Internal bleeding can happen.
o Treatment of ITP:
▪ < 50000 with Mucous bleeding only treatment started.
▪ Above 50000 with no symptoms no treatment needed.
▪ DOC: Prednisolone 1mg/ kg Body weight**
▪ Infusion of platelets: Only in significant haemorrhage
▪ Infusion of immunoglobin: Pregnancy or planning a major surgery/
Splenectomy
▪ Refractory ITP : Rituximab IV or Thromboplastin receptor antagonists like
Romiplastin or Eltrombag**

Indications for Splenectomy in ITP


• Severe thrombocytopenia of <10000 for >6 weeks*
• Refractory to steroids
• Requirement of toxic doses of steroids
• Incomplete response to steroids
• Pregnant women in 2nd trimester
• Engagement in High risk activities.

SURGERY SIXER APP BASED WORK-BOOK 2020 365


During Splenectomy:
• Platelets are given only after ligating the splenic pedicle**

Post op response:
• 72% Complete response
• Preop Indium 111 Scan shows Splenic Sequestration- Excellent
response, Liver sequestration will have bad response( NEETSS)
• 15% cases relapses. MC cause of relapse is missed out Ectopic spleen

Tumors in Spleen:
• Most common neoplasm of Spleen- Lymphoma (NHL)
• Most common primary benign tumor of spleen – Hemangioma*
• Most common primary malignant tumor of spleen – Hemangiosarcoma
Mets in spleen are very rare:
• “Types- Multi visceral (MC) and Isolated”
o Most common primary for Multi-visceral- Lung > Breast> Malignant melanoma**
o Most common primary for Isolated- Ovary> Colorectal**
o Most common primary with highest percentage (No of cases of splenic metastasis/
100 Primaries) – Malignant Melanoma**

Ectopic Spleen: (Splenenculi= Accessory Spleen)


• MC reason is due to embryological non fusion or incomplete fusion during 5 th Week of IUL
• Incidence= 10-30%
• It is seen along the Dorsal Mesogastrium:
▪ Splenic Hilum ( MC)
▪ Tail of Pancreas ( 2nd MC)
▪ Splenocolic ligament
▪ Presacral
▪ Renal
▪ Left side scrotum
• Properties of Ectopic spleen:
▪ Multiple usually
▪ Always have capsule
▪ Contains Red and white pulp like normal spleen.
▪ MC to go for Torsion**
Splenosis:
• Rupture of Spleen and its distribution all along the abdominal cavities
• Seen anywhere in abdominal cavity ( DD- Ectopic spleen in Dorsal mesogastrium only)

SURGERY SIXER APP BASED WORK-BOOK 2020 366


Splenic artery Aneurysm:
• Incidence < 1%
• Old age : MC in females
• Solitary
• MC cause- Arteriosclerosis
• Other aetiology: Sepsis and pancreatitis**
• Usually asymptomatic, May rupture suddenly**
• Rupture risk is increased in:
▪ Young
▪ Pregnancy ( 2nd and 3rd Trimester)
▪ Absence of Calcification
▪ Hypertension
▪ Size > 1.5cm
• Management:
- Upto 2cm (Asymptomatic cases)- Observation
- Indications for Splenectomy with aneurysm:
• Symptomatic
• > 2cm
• Pregnancy
• Young age
- Angioembolisation can be done for unfit cases.

Splenic Abscess:
• MC in immunocompromised, Polycythemia, IV drug abusers, Trauma.
• 70% due to Hematogenous route
• MC is Staphylococcus > Streptococcus
• Immunocompromised fellows may develop Fungal abscess- Candida
• Sickle cell anemia- Salmonella causes abscess
• IOC for Diagnosis: Hypodense lesion
• Unilocular abscess- Percutaneous drainage
• Multiple abscess- Splenectomy

Figure: Splenic abscess


Splenic Cyst:
• MC cyst in spleen- Hydatid Cyst
• MC non parasitic cyst- Pseudocyst following a Trauma**

SURGERY SIXER APP BASED WORK-BOOK 2020 367


• MC cause of true cyst- Congenital Epidermoid Cyst; Rare < 10% ( Lined by Squamous
epithelium). Fluid analysis shows CEA, CA 19-9 +ve. But they are benign**. Partial
splenectomy done and leave 25% of spleen to prevent OPSI.

Space for Extra Points:

SURGERY SIXER APP BASED WORK-BOOK 2020 368


Section: E: Surgical Specialty Topics

Chapter 1: Urology
Chapter 2: Vascular
Chapter 3: Plastic Surgery and Skin lesions
Chapter 4: Paediatric Surgery
Chapter 5: Oncosurgery
Chapter 6: Cardiothoracic surgery
Chapter 7: Endocrine Surgery
Chapter 8: Neurosurgery
Chapter 9: Transplantation

SURGERY SIXER APP BASED WORK-BOOK 2020 369


Chapter 1: Urology

Part : 1: Kidney, Ureter, Bladder and Urethra

BRODELS LINE-Avascular line on the greater curvature of the kidney

Narrow points in the ureter:

1. Pelvic uretric junction (PUJ)

2. Crossing of common iliac artery

3. Male - Vas deferns

4. At the point of entry to bladder

5. VUJ (vesicoureteric junction)**M/c site for ureteric obstruction

• Length of ureter - 20cm

KIDNEY INVESTIGATION

CYST IN KIDNEY

● IOC- CECT ABDOMEN

SURGERY SIXER APP BASED WORK-BOOK 2020 370


BOSNIAK CLASSIFICATION

Category I- Simple Cyst Clear cyst


No septum
Category II- Benign Cyst Hair Line Septations+
No soft tissue component or calcification
Category IIF- Complex Cyst Hair line septations++
No soft Tissue component
Septations have no enhancement
<5% risk of malignancy
Category III- Indeterminate Cyst Cyst wall thick enhancement+
Septations enhancements*
50% risk of malignancy
Category IV- Malignant Cystic mass Soft Tissue components are seen
100% risk of malignancy
• Category I, II - no risk of malignancy

• Category IIF- <5% risk of malignancy

• Category III- biopsy is done

• Category IV- Resection

SPECIAL SCANS IN KIDNEY

● DMSA(Dimercapto succinic Acid)- IOC- for accessing the Morphology of kidney

● DTPA (Diethylene Triamine Penta Acetate)-For accessing the Perfusion: Physiological

study of kidney. PUJ obstruction can be detected.

SURGERY SIXER APP BASED WORK-BOOK 2020 371


● MAG-3 (Mercapto Acetyl Triglycine Scan)- IOC FOR PUJ OBSTRUCTION**

RENAL STONES
CLINICAL FEATURES

● Mostly asymptomatic

● Based on the location of the stone the symptoms are :

■ Renal-back pain

■ PUJ- pain radiating from loin to testicles ( Genital branch of

Genitofemoral nerve)

■ Mid ureter-mimics pain like appendicitis

■ VUJ- pain radiating from loin to groin or mid thigh ( ilioinguinal

nerve>iliohypogastric )**

■ Intramural ureteric stone- strangury

Figure: Symptoms related to Location of Stone

DIETEL'S CRISIS- After an attack of acute renal pain following an alcoholic binge, a swelling in
loin is found, some hours later following the passage of stone large amount of urine is passed ,
pain is relieved and swelling disappears.

Clinical features-

● Hematuria

● Increase frequency

SURGERY SIXER APP BASED WORK-BOOK 2020 372


● Strangury

● Pain

**Worst pain experienced by a male in his life time- Ureteric Stone pain- DOC for him is Injection
Diclofenac**

Types of stones

Calcium Oxalate (Calcium oxalate Monohydrate and Dihydrate)

• Also known as Mulberry stone

• M/c type of stone overall

• M/c stone to cause haematuria

Triple Phosphate ( Calcium , magnesium ,NH4,PO4 / Struvite )

● Staghorn stone

● Jack Stone

● M/C IN INFECTION (associated organism- Proteus)

● M/c STONE IN ALKALINE URINE**

● Smooth stone

Cystine

● Hard stone

● Cystinuria

● SH=SH (disulphur bond makes it radiopaque); without calcium they are still radio opaque

due to the Bond**

Uric acid

● Only radiolucent stone

● Yellow colour

● Associated with hyperuricemia in Gout, cancer

Rare stones

● Indinavir (HIV patients)- Radiolucent

● Xanthine ( In Xantine Oxidase deficient patients)- Radiolucent

SURGERY SIXER APP BASED WORK-BOOK 2020 373


Urinary crystals

Image Description
Calcium Oxalate:
- Bipyramidal shape/ Envelope
shaped
- m/c type of crystals in urine

Calcium oxalate monohydrate:


• Dumbbell shaped or Hour glass
shaped crystals

SURGERY SIXER APP BASED WORK-BOOK 2020 374


Uric Acid Crystals:
• Multifaceted
• Irregular plates
• Rosettes like

Cystine crystals:
• Hexagonal
• Benzene ring like

Struvite:
• Coffin lid shape

Brushite:
• Needle shaped

By ESWL- Hardest stone to break- MBC ( Calcium oxalate Monohydrate> Brushite> Cystine)

SURGERY SIXER APP BASED WORK-BOOK 2020 375


Drug causing renal stone ( mnemonic- TIMES- CALL)

• T-Triamterine

• I-indinavir

• M-Mg trisilicate ( Antacids)**

• E-ephedrine

• S- Septran

• C-ciprofloxacin

• A- Acetazolamide

• L-loop diuretic

• L- laxatives containing Mg**

MANAGEMENT OF STONE

● < 5mm - No treatment needed

● Fluid intake :>2.5 l/urine

● Restrict : Protein, salt , oxalate diet

● Increase consumption - Calcium supplements**

● Juice - Cranberry juice** is used in management of renal stones**

BLADDER STONE

Primary Bladder stone Secondary Bladder stone


• Dropped from Above ( From Kidney) • Secondary to some pathologies like
BOO, BPH and Stenosis
• Formed in Bladder itself**
• Common type is Calcium oxalate • This is the most common Bladder
stone. stone**
• Ammonium Urate is the MC
secondary bladder stone.
• The calcium Apoplexy ( Coating of
Calcium over the ammonium urate
stone) results in radio opaque
MANAGEMENT

● ESWL-EXTRACORPOREAL SHOCK WAVE LITHOTRIPSY

● PCNL- PERCUTANEOUS NEPHROLITHOTOMY

● RIRS- RETROGRADE INTRA RENAL SURGERY ( Via Ureter the Stone removed from

kidney)

● Open Nephrolithotomy- Opening the kidney along the Brodel’s line and remove the Stone

SURGERY SIXER APP BASED WORK-BOOK 2020 376


● Open Pyelolithotomy- Opening the renal pelvis and removing the stone.

ESWL
• Passing Piezo electric waves and breaking the stones.

Indication

● <1.5cm size

● Mainly done for stone in kidney, pelvis, upper ureter ( Cannot use it for lower ureteric

stone (d/t pelvic brim bone hindrance))

Contraindications

● Absolute- pregnancy, bleeding diathesis

● Relative:

○ Obstruction / distally

○ Infection

○ Hematuria

○ Renal dysfunction

Complications

● Hematuria

● Incomplete clearance

● If stone >1.5 cm a rare complication known as stein strasse ( Stone Street)** happens. To

prevent this problem happening we must keep a 20 cm Double J stent before ESWL.

PCNL
Indication

● Stone >2cm

● Infected

● Obstructed

● Cysteine

● Failed ESWL

● Anatomical abnormality- Calyceal Diverticulum containing Stones**

SURGERY SIXER APP BASED WORK-BOOK 2020 377


Procedure:
• Trans costal approac
• Subcostal Approach
• We insert a guidewire under
fluoroscopic Guidance- we dilate the
tract and insert the Nephroscope via
the tract and remove the stone.

Complications

● Bleeding

● Sepsis

● Perforation of bowel

● Trans costal approach- Pleural injury and Hydrothorax can happen

● Subcostal Approach- Duodenal injury, Colon injury and Liver and spleen injury

● Injury to renal artery /vein/IVC**

Staghorn Stone:

• Sandwich technique- ESWL followed by PCNL

• If this technique is not mentioned – go with PCNL**

RIRS
● For renal pelvis stone

● For mobile stone

● By retrograde method we use ureteroscopy to remove intra renal stones.

Management of Ureteric stones:

● <5mm - it will pass

● >5mm/impacted somewhere- Removed by Dormia basket technique

SURGERY SIXER APP BASED WORK-BOOK 2020 378


Figure: Dormia Basket removal

• For upper ureteric stone- PUSH BANG technique** we push the upper ureteric stone

back into the renal pelvis by Ureteroscope and break the stone by ESWL**

Bladder stone

● Old technique- Open Cysto lithotomy

● Latest- Transurethral Cysto lithotripsy

● Lasers used now- HO:YAG(best) Nd:YAG

Image Based question:

• The image shown below is asked in AIIMS- please note the Doube J stent in right

Ureter**

• The left kidney is having a Percutaneous DJ stent- for percutaneous nephrostomy**

Image : Right Ureteric Stent and left Percutaneous stent**

SURGERY SIXER APP BASED WORK-BOOK 2020 379


RENAL CELL CARCINOMA @ Grawitz Tumor @ Hypernephroma
● a/k/a Clear cell adenocarcinoma

● M/c site- Upper pole

● Origin- Proximal convoluted tubule

● MC in 50-60yrs

● MC in men 2:1 ratio

● M/c malignant neoplasm

● MC arises from Proximal Convoluted tubule**

● M/c presentation – RCC Triad

o Hematuria (MC symptom)

o Pain

o Loin mass

Pathology

Clear cell type Papillary type Chromophobe type Medullary type

o Most common o Trisomy of o Best • Associated


type 7,16,17 prognosis with Sickle
o M/c type o Chromosome cell trait
o Von Hippel 7 mutation
Lindau Gene o M/c in PCKD
mutation patients ON
o 3p DIALYSIS
mutation**

Clinical Features-

● Earliest and m/c feature – HAEMATURIA**

● Mass can grow inside renal vein- it can grow up to Right atrium- it is still operable.

● Young male patient can present with left side varicocele which is irreducible on lying

down**

● Hematogenous metastasis- Pulsatile Skull secondaries and Cannon Ball Secondaries can

develop

Paraneoplastic syndrome in

• Increased ESR (M/C)**

SURGERY SIXER APP BASED WORK-BOOK 2020 380


● Hypertension (2nd m/c)

● Anemia (3rd m/c)

● Hypercalcemia (5%)

● Polycythemia (3%)

● Stauffer's syndrome- non metastatic liver enzyme elevation**

● Other features- Neuropathy, Amyloidosis, PUO and weight loss

GRADING and SCORING SYSTEMS from Bailey and Love:


• FUHRMANS - Nuclear grading system
• LEIBOVICH SCORE -Histological scoring

STAGING

● Stage I- mass <7 cm within the kidney

● Stage II- mass >7 cm within the kidney

● Stage III- Involving Renal Vein,,IVC or even upto Right atrium /nodes/ adrenal gland**.

Tumor not spread out of Gerota’s Fascia**

● Stage IV- Out of Gerotas fascia

Radical nephrectomy Partial nephrectomy (nephron sparing


procedure)

● Structures removed: ● Indication


○ Kidney ○ T1 - <7cm (even with normal
○ Adrenal gland opposite kidney)
○ Perinephric pad of fat ○ Single functioning kidney
○ Regional nodes ○ VHL mutation- B/L RCC**
○ Ureter cases
○ Gerota’s fascia ○ Renal failure (DM,HTN)

Renal cell cancer Wilm’s Tumor


Chromosome 3p mutation 8/11th Chromosome Mutation
Age: 50-60 years Age: 1- 5 years
MC tumors in Adults MC primary tumor in Children
C/F: C/F
Hematuria ( MC presentation) Hematuria
Mass Mass ( MC presentation)

SURGERY SIXER APP BASED WORK-BOOK 2020 381


Pain Pyrexia

Associated with
• Beckweith Wiedman syndrome-
Macroglossia+ Visceromegaly+
Hyperinsulinemia
• WAGR syndrome- Wilms, Aniridia, GU
anomalies and Renal anomalies
MC distant mets- Lung MC distant Mets- Lungs
No need of preop Biopsy Preop Biopsy is must
Surgery is TOC Surgery is TOC
Chemo and Radioresistant Chemo sensitive and Radio sensitive
Sunitinib** - ( monoclonal antibody) used for Chemo- Vicristine, Actinomycin
metastatic RCC

Bad prognosis Good Prognosis

2 protocols in Wilm’s Tumor management:

NWTSG- National Wilm’s tumor study group

• Advise surgery first followed by Chemotherapy

• Pre chemotherapy based staging**

SIOP- ( International Society of Paediatric Oncology)-

• Chemotherapy followed by surgery

• Post Chemotherapy Based staging system**

Overall survival is same for both.

Surgery- Radical nephroureterectomy

• For Bilateral Wilms - chemo is done first, then following the shrinkage of the tumor-

nephron sparing surgery is done

ANGIOMYOLIPOMA
● Hamartoma

● 50% of Tuberous sclerosis ( EPILOA- Epilepsy+ Low Intelligence+ Adenoma Sebaceum)

● This contains- Blood vessels+ Muscles+ Fat content inside**

● IOC- CT SCAN (Fat inside lesion)

● Tumor marker: HMB 45 +ve

SURGERY SIXER APP BASED WORK-BOOK 2020 382


● Clinical features:

○ Hematuria

○ Hypertension

○ Mass

○ Pain

○ Anemia

○ Wunderlich syndrome-spontaneous Retro peritoneal hemorrhage- self limiting

Treatment

● <4 cm- no treatment

● >4 cm- partial nephrectomy

● Selective embolization

BLADDER CANCER
● M/c type: Transitional cell cancer(90%)

● 2nd m/c- SCC

● 3rd m/c – Adenocarcinoma ( Arises from Urachal remnants)

Risk factors:

TCC Squamous cell cancer


● M/C risk factor- smoking • Stones
● Schistosomiasis • Schistosomiasis
● Occupational • Special diseases- Diverticula diseases,
○ Beta naphthylamine (coal) Long term catheters
○ Diaminobiphenyl (Aniline)
○ Aminobiphenyl
○ Nitrophenyl
• M/c presentation- painless terminal haematuria

• M/c site - Dome and Lateral wall of bladder**

• Cancer from anterior wall producing spread to posterior wall is known as KISSING

cancer**

Investigation

● Urine examination: Screening test for

○ NMP-2 Protein

SURGERY SIXER APP BASED WORK-BOOK 2020 383


○ MCM Protein

● IOC FOR STAGING- CECT ABDOMEN / PELVIS

● IOC FOR DIAGNOSIS-CYSTOSCOPIC BIOPSY

STAGING

● Ta- Non invasive papillary cancer

● Tis- Cancer in situ

● T1- Subepithelial connective tissue

● T2- Muscularis propria

● T3- Peri vesical tissue

● T4- adjacent organs - prostate, seminal vesicles, vagina , abdominal wall

Intravesical therapy agents:

● Mitomycin -C

● Doxorubicin

SURGERY SIXER APP BASED WORK-BOOK 2020 384


● Thiotepa

● BCG (BEST)

RADICAL CYSTECTOMY : STRUCTURES REMOVED

● Bladder+ fat

● Prostate + seminal vesicles

● Uterus+ cervix+ fallopian tube +anterior vaginal wall

● +/- Urethrectomy

● Pelvic lymphadenectomy

Ileal conduit Colon Conduit


○ M/c type ○ Drained into Sigmoid Colon
○ M/c complication- ○ M/c complication-infection
ureteroilial stricture ○ Risk of malignancy++

• Important pre-requisite for this


procedure- Patient must be able to hold
200 ml saline in Rectum with continence
• Neobladder can also can be created if urethra is not involved using ileum.

● Continent Urinary diversion procedure – Kock’s Ileostomy ( Ileum made like a valve)

● Mitranoff procedure- appendix can be used as a drainage of urine procedure from

Bladder.

SURGERY SIXER APP BASED WORK-BOOK 2020 385


Figure: Mitranoff procedure

Conduit associated metabolic problems:

● Ileal - metabolic acidosis+ hyperchloremia

● Jejunum- metabolic acidosis+ hypochloremia**

● Colon- metabolic acidosis+ hyperchloremia

● Stomach- Metabolic alkalosis** + hyperchloremia- Stomach conduit is used in Renal

failure patients**

Management of the above problems:


• IV SALINE + NaHCo3
• Oral { potassium citrate+ NaHCo3} – Three times daily

Chemotherapy
• Indicated in ≥T3a, LN positive, Margin +ve
• GC regimen- Gemcitabine+ Cisplatin
• MVAC regimen (Old)- Methotrexate+ Vinblastin + Actinomycin+ Cisplatin
• Post op RT is not useful

URINARY TRACT INFECTION


● M/c organism- E.coli

● Kass criteria

○ Men: >105 Colony forming units/ ml (mid steam urine)

○ Female: >103 CFU/ml (catheterised urine)

● MC cause of prostatitis- E.Coli

RENAL TB

● M/C - KIDNEY; other sites- URETER and BLADDER

● Route of entry- Hematogenous route

SURGERY SIXER APP BASED WORK-BOOK 2020 386


● Earliest and m/c features of renal TB - Increased frequency** ( DD for increased

frequency- DM, UTI, TB)

● O/E of urine in Renal TB- Sterile pyuria** (Bacteria is absent but WBC count is increased)

● **Early morning urine specimen for 3 days – Acid Fast Bacilli to diagnose TB

IOC to diagnose early TB:

• Earliest feature in TB kidney- Blunting of Calyx**

● OLD modality- IVP

● NEW modality- CT PYELOGRAM**

Complications of chronic TB:

• Cement or putty kidney

• Golf hole ureter

• Thimble small bladder

Figure: Golf hole ureter and Blunting of calyx

Treatment of chronic TB – Resection and reconstruction(optimum after 6-12 weeks after ATT
started)

Trauma to urinary system


Renal trauma

● M/C mechanism - BLUNT INJURY

● Haematuria (omnious sign)

SURGERY SIXER APP BASED WORK-BOOK 2020 387


● IOC- CECT ABDOMEN

Grades of Renal Trauma:

• Grade 1- Subcapsular; Non Expanding Perirenal hematoma

• Grade 2- Laceration <1cm, Non Expanding Perirenal hematoma confined to

retroperitoneum and Gerota fascia

• Grade 3- Laceration > 1cm, No urine extravasation

• Grade 4- Laceration with Urine Extravasation**, Injury to Main renal artery and Vein**

• Grade 5:

• Shattered kidney.

• Avulsion of renal hilum: Devascularisation of a kidney due to hilar injury.

• Uretero pelvic avulsions**

• Complete laceration or thrombus of the main renal artery or vein.

● IOC- CECT abdomen (stable pt)

● Laparotomy (unstable) , nephrectomy done if needed

● 90% conservatively treated

● 10% need surgery

● In a stable patient one Clinical Scenario:

▪ In CT abdomen ,if there is injury of kidney IVP is a must, if it is normal

patient can be discharged

▪ In IVP- If a kidney is not visualized suspect renal artery compression by a

perirenal hematoma, immediately explore the hilum and repair the renal

artery on the table and repeat the IVP on the table and confirm normal

flow into that kidney( AIIMS PG question)

Ureteric injury

● Usually during surgery

● Ureter can be identified by looking for the – Peristalsis**

● On table ureteric injury- Repair it end to end by using 3'0 vicryl**

Repaired by

o keeping a stent in ureter

o Boari’s flap using the urinary bladder as shown below

SURGERY SIXER APP BASED WORK-BOOK 2020 388


o Auto transplant of kidney or ureter- 1st done by Hardy**

Figure: Boari Flap operation (Image question)

Bladder injuries

Intraperitoneal rupture of bladder Extraperitoneal rupture of bladder


● M/c due to direct blow in Dome of ● Cause - fracture of pelvis**
bladder ● Superficial extravasation of urine with
● Cause uremic peritonitis urine extravasated from nipple level to
● Immediate surgery needed Holden’s line level.
● Catheter inserted immediately.
● Wait and watch- gradually the injury
heals

IOC for Bladder injury -Retro Grade Cystography**

SURGERY SIXER APP BASED WORK-BOOK 2020 389


Urethral injury:

4 parts of urethra:

• Most dilatable part- prostatic urethra

• Least dilatable part- Membranous urethra

• Anterior urethra- Penile and Bulbar

• Most narrowest part of urethra- External urethral meatus

Clinical feature of Urethral injuries:

• Blood at urethral meatus

• Retention of Bladder

Membranous Urethral injury Bulbar Urethral injury


• Mainly due to fracture pelvis ● Mainly due to straddle injury
• Per rectal examination: ● Typical presentation of urethral injury
Prostate: high lying and Floating seen.
(Vermooten sign)**
• The above examination is not
advised in latest guidelines of ATLS

SURGERY SIXER APP BASED WORK-BOOK 2020 390


• Deep extravasation +++ of Blood
seen
• Bladder shape - Inverted tear drop

Figure: Straddle injury causing Bulbar urethral injury

Steps to be done in Urethral injury:

• Never put Foley’s catheter as it will cause more damage of the urethra

• Put Suprapubic catheter ( SPC)

• Via SPC we can do Descending cystourethrogram / Micturating Cysto urethrogram

• Other Investigation-Retrograde urethrogram ( Via Foley’s inserted only upto penile

urethra and push contrast)

NEET PG 2020 Image based Question:

For understanding purpose in X ray- to say In MCU we have;


the image is RGU we will have: • Bladder will be full; and will be
• Cannulated penis appearing like below: with Bladder
• Incomplete bladder filling being filled with Contrast by Supra
pubic catheter.

SURGERY SIXER APP BASED WORK-BOOK 2020 391


Management:

o Wait for 6-8 weeks after SPC insertion

o Gradually the urethral injury goes for stricture

o If stricture is short and single- we can use Dilators

o If stricture is multiple and long- we plan Stricturoplasty using Buccal mucosa**

Figure: RGU shows Urethral stricture

Congenital anomalies of Urinary System

SURGERY SIXER APP BASED WORK-BOOK 2020 392


ECTOPIC URETER

● M/C anomaly of Urinary system-Duplication of renal pelvis and ureters**

● Unilateral > bilateral

● Ectopic ureter arises from upper pole, it runs lateral, outwards and joins distal to normal

ureter**

● Scenarios in Ectopic Ureter are:

WEIGERT - MAYER RULE: 2nd type: 3rd Type


- Upper pole ectopic it's
orifice inserts infero - Upper ectopic ureter - YoY o Reflex
medially to normal gets obstructed. Urine - Ureter draining
ureter . gets collected inside into the ureter
resulting in itself.
Ectopic ureter opens into: Ureterocele**
- Boys- Prostatic urethra - Lower ureter which is
- Girls- Distal to External normal results in VUR
urethral sphincter. in normal ureter

Hence girls will have Paradoxical


urinary incontinence and
continuous dribbling of urine
seen

SURGERY SIXER APP BASED WORK-BOOK 2020 393


• IVP- diagnostic of ectopic ureter

• Drooping Lilly appearance ( Ectopic ureter with ureterocoele)

POLYCYSTIC KIDNEY DISEASE

INFANTILE PKD

● <1yr

● Autosomal recessive

● Child won't survive more than 1 year

AUTOSOMAL DOMINANT Polycystic Kidney Disease Adult type ( ADPKD):

Criteria to diagnose ADPKD is based on usg

● On USG one kidney: 3 cysts

● B/L kidney : 2 in each

SURGERY SIXER APP BASED WORK-BOOK 2020 394


Clinical features

● PKD - Bilateral (95%)

● Incidence 1 in 400 to 1 in 1000

● Before 40 years - Presentation- hypertension (from 20yr onwards),

● kidney failure from 40year onwards

● The renal tissue is progressively replaced by Cysts and results in renal failure.

● Rare- Haematuria , kidney stone (20%)- MC type of Renal stone is URIC ACID STONE**

● End stage renal failure - M/c in male than female

IVP shows- Spider leg appearance**

Associated anomalies

Cysts seen in Berry aneurysm Mitral valve Colonic Liver fibrosis


● Liver - Sudden prolapse diverticulitis
● Spleen headache - Sudden
● Pancreas cardiac
● Ovary failure
● Not seen
in lungs

Treatment-

● Rovsing’s operation (olden days,not used now)

● Tolvapton ( NEET PG 2020)

○ Vasopressin receptor antagonist

○ 1st approved drug

SURGERY SIXER APP BASED WORK-BOOK 2020 395


○ Increased urine excretion

○ Increase sodium content in the body

● Treatment of choice- Renal Transplant

● Until transplant they survive by Dialysis

CONGENITAL PUJ OBSTRUCTION

● M/c cause of congenital hydronephrosis** is PUJ obstruction

● Cause unknown

● Aberrant renal vessels may be seen- but they are not the cause of obstruction**

● M/c in boys

● Unilateral mostly

● Bilateral in 40% cases

● Whitaker test - pressure measurement of renal pelvis and bladder. High pressure in Pelvis

and low in bladder

● MAG-3 (Mercapto Acetyl Triglycine Scan)- IOC FOR PUJ OBSTRUCTION**

● Treatment : Anderson Hynes Pyeloplasty operation**

VESICOURETERIC REFLUX

● Congenital - m/c in boys

● Acquired- m/c in girls

● Main problem is the longitudinal bladder muscle deficiency**

● The urine from the bladder refluxes into the ureter on micturition.

International classification of VUR

● I and II - Non dilated ureter

SURGERY SIXER APP BASED WORK-BOOK 2020 396


● III, IV and V – Dilated ureter seen

IOC – Micturating Cysto urethrogram**

Management:

● I and II - No surgery needed, Only medical treatment

SURGERY SIXER APP BASED WORK-BOOK 2020 397


● For UTI - antibiotics (*if not responding to antibiotics surgery recommended)

● III (Bilateral),IV (Bilateral) ,V (Unilateral) - Surgery is recommended

● Surgical Treatment- Reimplantation of ureter

HORSESHOE KIDNEY

● 1 in 1000

● M/c in men

● Lower pole fusion at L4

● Most of them are Asymptomatic

● The isthmus is cut only in one Surgery- Abdominal Aortic Aneurysm Surgery

● Blood Supply is from Major arteries nearby.

● Pregnancy is not a Contraindication- but Urinary infection is common for them**

● IVP- FLOWER VASE or HAND SHAKE APPEARANCE**

Figure: Flower vase appearance in Horse Shoe Kidney

ECTOPIA VESICA (EXSTROPHY of BLADDER)

• Embryology- the basic defect in Exstrophy bladder is abnormal over development of the

cloacal membrane and its rupture.

• Associated anomalies

Males Females
○ Umbilical hernia ○ Umbilical hernia
○ Inguinal hernia ○ Epispadias
○ Epispadias ○ Split Clitoris
○ Undescended testis ○ Wide open pelvis (duck like waddling
gait)
○ Wide labia
○ Urinary incontinence

SURGERY SIXER APP BASED WORK-BOOK 2020 398


○ Increased risk of bladder cancer

• C/F- total urinary incontinency with dribbling and risk of bladder cancer

• Treatment- Plastic surgery Reconstruction

Figure: Ectopia Vesica reconstructed

URETEROCELE

● M/C in females

● 10% bilateral

● M/c : Urinary tract infection due to stagnation of urine in Dilated part of ureter

● IVP- cobra head appearance

● Treatment- Ureteric Reimplant

Figure: Cobra Head deformity on IVP

SURGERY SIXER APP BASED WORK-BOOK 2020 399


Part 2: Prostate and Seminal Vesicles
Basic anatomy: Mc Neal’s 3 Surgical zones

● Transition zone

● Central zone

● Peripheral zone

In BPH - Transition zone is more involved*

In Cancer- Peripheral zone is more involved**

5 lobes in prostate:

• 2 lateral lobe

• 1 median lobe

• 1 posterior lobe

• 1 anterior Lobe

BPH arises from Median Lobes

Cancer arises from Posterior Lobe

(Remember : Cancer- P -P : Peripheral zone, Posterior lobe)

Prostatitis:

• MC organism causing – E.Coli

• C/F- Low grade fever+ perineal pain

• PR- Soft Boggy swelling of prostate due to conversion to abscess

SURGERY SIXER APP BASED WORK-BOOK 2020 400


• Treatment: Antibiotics for 4 weeks

Most common type of stones:


• MC stone in Prostate : Calcium Phosphate
• MC stone in Kidney- Calcium Oxalate
• MC stone in Bladder- Ammonium Urate
• MC stone in Salivary gland- Calcium carbonate
• MC stone in Gall bladder- Mixed Stones

Benign Prostatic Hyperplasia:

● Mc in Old age

● Symptoms of BPH is given by bladder outlet obstruction

● Symptoms associated with bladder outlet obstruction

Irritative symptoms Obstructive


o Increased frequency ( MC • Dribbling
symptom) • Thin stream urine
o Nocturia • Retention
o Urgency
o Hesitancy
o Haematuria

• IOC to detect bladder outlet obstruction- uroflometry**

• USG abdomen :

o Prostate enlargement

o Post void residual urine

o B/l hydroureteronephrosis

SURGERY SIXER APP BASED WORK-BOOK 2020 401


Medical treatment

Alpha 1 a Blockers: 5 Alpha reductase inhibitors


• Tamsulosin • Finasteride 5 mg
• Decreases smooth muscle tone • Slow action – needs 6 months
• Quick action • Decreases the volume (upto 40%)

Other drugs like Prazosin and terazosin will Remember Finasteride is used in 1 mg dosage.
cause postural hypotension

Indication of Surgery

● Retention

○ Acute

○ Chronic ( PVR >200 ml)

● Complications

○ Diverticula

○ B/L HUN

○ Bladder stone

○ Haematuria

○ UTI

● Uroflometry value

○ <10ml/s

○ >80 cm H20

SURGERY
TURP( gold standard ( 1)
Open
○ Retropubic – MILIN’s operation (2)
○ Trans vesical- Freyers operation
(3)
○ Perineal - Young operation (4)

SURGERY SIXER APP BASED WORK-BOOK 2020 402


TURP

● Distal limit of TURP -Verumontanum**

● IUS is under involuntary control

● EUS is under voluntary control

● In TURP – using energy device we remove the prostatic tissue via cystoscopy.

Modalities used for TURP:

● Diathermy

● HO: Laser (Best )

● ND: Yag

During dissection:

• If you go beyond Verumontanum we can injure the EUS- Results in Incontinency**

• If you injure the IUS ( most common Complication)**- Retrograde ejaculation happens in

>50% cases

• If you injure the bladder neck- Bladder neck stenosis happens and symptoms will be same

like BOO**

During surgery

● Fluid of choice for monopolar diathermy- 1.5% isotonic glycine**

● If not available distilled water

SURGERY SIXER APP BASED WORK-BOOK 2020 403


● Normal saline shouldn't be used as the current may get dissipated and wont cut and

coagulate properly if I use Monopolar diathermy (As NS is a good conductor of electricity)

● Bipolar diathermy or with LASERs- we can use normal saline

• HO laser - HOLEP -Holmium laser enucleation of prostate

• TULIP-. Transurethral USG laser induced prostatectomy

• Post op of fluid of choice for washing bladder and irrigation with Three lumen Foley’s

catheter 18 G catheter: Normal saline. We will get clear Pink urine during the irrigation.

Figure: 3 lumen Foley’s catheter for post op irrigation

Immediate post op complications-

● Hemorrhage- Usually arises from FLOCH arteries arising from Inferior Vesical arteries**

● TUR syndrome- patient confused, irrelevant words, if we use distilled water there is high

chance of dilutional hyponatremia and water intoxication

Delayed complication:

● Bladder neck stenosis

● Retrograde ejaculation (m/c complication overall) due to IUS injury

● Incontinence due to EUS injury

CANCER PROSTATE

● Clinical Presentation- bladder outlet obstruction + back pain

● Screening protocol : Digital rectal examination+ PSA

Prostate Specific Antigen

● Normal value: <4 ng/l

SURGERY SIXER APP BASED WORK-BOOK 2020 404


● Men 50-69 year :>4ng/l - Indication for biopsy

● >10ng/l : Localised Cancer+

● >35ng/l : metastatic Cancer prostate+

Other Values:

• PSA velocity

• PSA density

• Old – Acid phosphatase

Biopsy methods:

● TRUS guided biopsy ● Transperitoneal template


biopsy(TPTBP)
○ Traditional
○ Increased risk of infection, ○ Detect lesion in anterior
septicemia, Bleeding etc aspect of prostate
○ Prophylactic antibiotics ○ 36 biopsy possible**
required ○ No infection risk is there
○ Less areas of biopsy possible ○ Helps to evaluate TRUS guided
negative biopsy cases

• IOC to stage prostatic cancer- CECT ABDOMEN/ PELVIS

• 1st node involved in Cancer prostate: Obturator Nodes*

SURGERY SIXER APP BASED WORK-BOOK 2020 405


T-STAGING

● T1:
○ T1a - <5% of TURP
specimen shows cancer
○ T1b ->5 % of TURP
specimen shows cancer
○ T1c- PSA increase
● T2-
○ T2a- involved one lobe
○ T2b - involved both love
● T3- seminal vesicle; involved beyond
the capsule
● T4- spread to rectum, bladder,
Pelvic bone

• T1 and T2: Early disease

• T3 and T4: Locally advanced

Management:

● T1,T2- Radical prostatectomy

○ <70yr

○ Fit for surgery

● >70 years , unfit- T1/T2 Prostatic cancer- wait and watch

● T3,T4,M1- Androgen ablation (1st step)

Androgen Ablation methods:


• Gold std method for androgen ablation: Subscapular orchidectomy or Conventional
orchidectomy+ testicular prosthesis kept
• If not willing for orchidectomy: Medical Castration methods : LHRH Agonist like
Flutamide; Leuprolide, Goserelin

GLEASON SCORE

● Grade 1 - Small uniform glands

● Grade 2- More space between glands and increase in stroma

● Grade 3- Distinct infiltration of cell margin

● Grade 4- Irregular masses of neoplastic cells

SURGERY SIXER APP BASED WORK-BOOK 2020 406


● Grade 5 - Lack of glandular pattern

● Old score- grade 1(well differentiated)-grade 5(poorly differentiated)

● New Gleason score: Extends from grade 1 to grade 10

• MC site of mets- Lumbar vertebra; More of Osteoblastic secondaries**

MALE INFERTILITY
Figure: Travel of Sperm: Testis- Vas- Seminal vesicles gives fructose- Ejaculatory duct- Prostatic
urethra- Penis- Ejaculated

Semen analysis findings

Decreased Volume of Semen (Sperms+) Azoospermia ( No sperm)


o Retrograde ejaculation o Ejaculatory duct obstruction
o Obstruction of vas deferns or o Testicular failure
Ejaculatory duct, Seminal vesicle o Vas obstruction bilateral
(unilateral)

Scenario: Patient with sperm count normal, decreased fructose level** - suggestive of seminal
vesicle obstruction

● M/ c cause of obstruction- TB

● IOC of ED obstruction- TRUS > Vasography ( Invasive). TRUS is non invasive and superior

● Final test - Testicular biopsy

SURGERY SIXER APP BASED WORK-BOOK 2020 407


URETHRA AND PENIS

Hypospadias

● m/c congenital anomaly ( lower urinary)

● I in 250

● Features:

○ Dorsal hood

○ Ventral opening of urethra

○ Ventral chordae

● Associated Cryptorchidism

● Associated Indirect inguinal hernia

Types

● Glandular (m/c type)- No treatment needed

● Coronal

● Penile

● Perineal

● Age group for surgery- 6 months - 1 year

● Circumcision is contraindicated

● Old method of Surgery- Dennis brown 2 stage

● New and used procedure- MAGPI (MEATAL ADVANCEMENT GLANULOPLASTY

INCORPORATION)

● Order of reconstruction of MAGPI

○ Orthoplasty ( Penile straightening)

○ Urethroplasty

○ Glanuloplasty

SURGERY SIXER APP BASED WORK-BOOK 2020 408


○ Meatoplasty

○ Scrotoplasty

○ Skin reconstruction and Circumcision is done as last step**

Epispadias

● Dorsal opening of urethra

● Rare

● M/c associated with ectopia vesica

Phimosis

● Unable to retract the prepuce backwards

● Smegma produced underneath from Tyson's gland gets collected inside. Smegma is

carcinogenic. Hence phimosis is premalignant**

● Congenital phimosis ● Acquired phimosis


○ Physiological adhesions up to 6 ○ Inflammation
years ○ Diabetes
○ No need to operate before 6 ○ BXO
years ○ Cancer
○ Wait up to 5-6 years ○ Trauma
○ Surgery is done for all
acquired cases

● Less than 1 year child with severe Ballooning of prepuce will need surgery as this is

causing severe obstruction.

● Less than 1 year children can be operated by a HOLLISTER/ PLASTIBEL device- Suture

less method.

Figure: Plastibel device

SURGERY SIXER APP BASED WORK-BOOK 2020 409


Conventional operations

● Dorsal slit method- safer method

● Guillotine method- Religious circumcision by Muslims use this method.

Types of circumcision

● Neonatal- at birth practiced by Jews, 100% protective of cancer penis

● Infantile- <1yearr practiced by Muslims ,decreases incidence of cancer of penis

● Adults: no relation with cancer

Paraphimosis

● Perpucial skin get retracted back and does not goes to the original position

● Treatment

○ Keep ice packs

○ Multiple punctures

○ Try to bring back manually

○ Circumcision - Dorsal slit method

Peyronie’s Disease

● Associated with Dupyutrens , Retroperitoneal fibrosis, Riedels

● C/F- Painful, persistent erection with deformity in a middle aged man.

● Penis gets curved (dorsal curvature usually**)

● Xray - Calcified penis

● NESBITT Procedure is done electively to straighten the penis**

SURGERY SIXER APP BASED WORK-BOOK 2020 410


Priapism

● Painful erected penis following sexual activity

High Flow Priapism Low Flow Priapism


• Increased Arterial Flow • Venous return Blockage
• Spinal Cord injury • Sickle cell anemia
• RP injuries • Leukemia and Lymphomas
• Papaverine injections

High Oxygen content on Penile Blood gas Low O2 Content on Penile Blood
analysis It’s like Strangulation ( Venous gangrene)
Less dangerous Urgent Treatment Needed

Treatment:

Immediate treatment

● Inj phenyl ephrine/ adrenaline ( **should be injected within 36 hours)

● >36 hours-Shunt operations are done

o Winter- Distal corpora spongiosum Shunt

o Sacher-proximal corpora spongiosum shunt

o Gray hack- corpora cavernosa with saphenous vein

SURGERY SIXER APP BASED WORK-BOOK 2020 411


Figure: Winter Shunt operation for Priapism

Congenital PUV

● Cause of urinary outflow obstruction

● M/c boys

● Cystoscopy- normal

● IOC - MCU

Cancer penis

● MC in 4th decade of men.

● M/c type: SCC

● Rare types:

○ BCC

○ Adenocarcinoma ( Arises from Tyson glands)

○ Melanoma

○ Kaposi

● Common sites

○ Glans penis(50%)

○ Preputial skin (20%)

○ Other places- Shaft, Corona glandis etc

● Predisposing condition

SURGERY SIXER APP BASED WORK-BOOK 2020 412


○ M/c – Smegma**

○ Other risk factors:

■ Leukoplakia

■ BXO

■ Buschke Lowenstein tumor ( caused by Human Papilloma virus)

● Cancer in situ of Glans- Erythroplasia of Queyrat**

● Cancer in situ of Shaft of Penis- Bowen’s Disease

● Verrucous cancer:

o a variety of SCC

o Low malignant

o Aggressive growth on RT

Figure: Verrucous cancer penis

Clinical features

● Fungating growth

● Foul smelling odour (d/t secondary infection)

● Sentinel node of cancer penis- CABANA node** ( Superficial inguinal node)

Clinical staging (Jackson's staging)

● I-Glans and perpuce involvement

● II- shaft involved

● III- Node +

● IV - Mets +

TNM Staging

● T1- <2cm growth

● T2- 2-5cm

SURGERY SIXER APP BASED WORK-BOOK 2020 413


● T3- >5 cm can invade corpus spongiosum, cavernosum, urethra ( Usually urethra not

involved due to transitional epithelium inside)

● T4- adjacent structure

● N1 : Single LN +

● N2 : Multiple LN (unilateral/ bilateral)

● N3 :Deep inguinal / iliac node

One of the most common cause of death – Erosion of femoral artery by the nodes**

Management of primary:

Non surgical Surgical treatment is best


○ Radiotherapy ● 2cm margin is given and after
○ Interstitial brachytherapy resection atleast 2cm of stump must
○ 5FU Cream be available for Sexual activity and
○ Moh's micrographic passing urine in standing position.
○ Laser excision. ● Usually partial amputation of penis is
done for glans and distal growth
● Indication of total amputation:
○ <2cm shaft.
○ Urethral involvement++

● Young operation- partial penectomy+ Bilateral ilioinguinal dissection

● Piersey Gold operation-Total amputation + scrotum+ Testis ( EMASCULATION)- Not done

**

SURGERY SIXER APP BASED WORK-BOOK 2020 414


Secondary nodal management:

● Node positive cases : lazy "S" incision done and Ilioinguinal block dissection done

● Node negative cases:

○ Sentinel node biopsy of Cabana done in olden days**

○ Superficial inguinal dissection of DASSLER- all superficial nodes removed and sent

for Frozen section: If positive ilioinguinal block is done

● Latest protocol for node negative cases

○ Modified ilioinguinal dissection of Catalona ( superficial + deep nodes removal in a

rectangular area) Send for Frozen section -if nodes are positive ilioinguinal block is

done.

Testicular pathologies
Testicular cancer

Pathological types:

Germ cell tumors Sex cord Tumors


Seminomas Leydig cell tumors
Non seminomas: Sertoli cell tumors
o Choriocarcinoma
o Yolk sac
o Teratoma
o Embryonal cell

Seminomas Non-Seminomas
o 4th decade • Seen in 30’s
o Highly radiosensitive • Radioresistant
o LN mets > hematogenous mets • Choriocarcinoma- Hematogenous
o LDH increased mets+
o Good prognosis • LN mets also seen
• Tumor markers- HCG, Alpha FP, LDH
• Bad prognosis
Clinical features

● 3rd decade- non seminomatous

● 4th decade-Seminomatous

● >5th decade - Lymphoma testis.

● Predisposing factors

SURGERY SIXER APP BASED WORK-BOOK 2020 415


○ Undescended testis(seminoma)- Please remember even by doing orchidopexy the

risk is not reduced; but helps in early diagnosis**

○ Klinefelter’s

○ Testicular feminisation syndrome

○ Maternal DES

○ Siblings history - h/o testicular cancer

● Painless lump+

● Testicular sensation lost ( Syphilis and Tumors have this clinical feature)

● Abdominal lump

○ Sentinel node on Right side- inter aortocaval node

○ Sentinel node on Left side- Preaortic node

● Hurricane tumour- aggressive choriocarcinoma**

● Sertoli cell tumors-Feminisation and Gynaecomastia

● Leydig cell tumors- Precocious puberty

Investigations for testicular tumors:

● IOC

○ To diagnose- Biopsy** ( Inguinal method- Chevasu manuever)

○ To stage- MRI**

○ Tumor markers-HCG, AFP, LDH

● For Biopsy testing-CHEVASU MANUEVER BIOPSY

○ Don’t do Trans scrotal biopsy (Violation of Tumor Principle will happen and you

may disseminate the tumor via Superficial inguinal nodes which is not it’s regular

course)

○ Put an Inguinal incision, Pull the testis and examine the tumor and take biopsy

from tumor.

○ Do frozen section

○ If positive- High inguinal orchidectomy

○ If negative- keep testis back in scrotum

STAGING

● T1-Tumour confined to testis, no lympho vascular invasion

● T2-Tumour confined to testis with lympho vascular invasion(+) and Tunica albuginea(+)

● T3-Tumour goes to spermatic cord

● T4-Tumour goes to scrotal wall

SURGERY SIXER APP BASED WORK-BOOK 2020 416


● N1 - <2 cm nodal mass

● N2- 2-5 cm nodal mass

● N3 - >5cm nodal mass

● M 1 - Mets

Treatment

● After High Inguinal orchidectomy: Nodal secondaries must be managed.

● Seminoma is Radiosensitive and hence RT given to nodes

● Non-Seminoma is not Radiosensitive and hence we do Retro peritoneal Lymphnode

dissection (RPLND)

● N1,N2 cases:

○ Seminomatous- Radiotherapy to Lymph nodes

○ Non seminomatous-RPLND+ CHEMO

● N3 cases and M1

○ Seminoma-chemotherapy

○ Non seminoma- chemotherapy

● Radiotherapy in Seminomatous tumors:

○ Inverted "Y" field RT

● Chemotherapy

○ B-bleomycin

○ E-Etoposide

○ P - Cisplatin

VARICOCELE TESTIS

● MC in young tall thin males

● C/F - Bag of worms on palpation

● Cough impulse (+)

● Disappears on lying down

● M/c in left side**

SURGERY SIXER APP BASED WORK-BOOK 2020 417


Why varicocele is mc on Left side?

❑ Left testicular vein drains at right angle

❑ Left testicular vein is long

❑ Left adrenal vein is near left testicular vein and hence can cause increased spasm

❑ Sigmoid colon compresses Left testicular vein

❑ RCC can invade left renal vein and causes obstruction of left testicular vein

• The left varicocele not disappears on lying down- LEFT SIDE RCC**

● M/c cause of infertility in men**

● Paloma operation -Retroperitoneal ligation of Testicular veins

● Inguinal approach- Invanissevich procedure

● Transcortical approach

● After ligating vessels testis will drain through cremasteric vessel

Painful testis Causes: Torsion Vs Orchitis:

● 2 causes

○ Torsion- Emergency**

○ Orchitis

● Orchitis

○ Chlamydia (mc organism)

○ Follow urinary tract infection

● Torsion

○ Bell clapper deformity (m/c deformity associated with torsion)- Bilateral

pathology

○ Other causes

- UDT

SURGERY SIXER APP BASED WORK-BOOK 2020 418


- Testicular inversion

○ Prehn’s sign-

▪ To differentiate between torsion and orchitis

▪ On lifting the testis

● If pain increases- suggestive of torsion

● If pain decrease - orchitis

○ Angel sign - Normal testis lying horizontally**

○ Deming sign- Affected testis lying higher up**

Management of torsion testis:

○ 1st hour- manual detorsion

○ <4 hours- save testis ( 4 hours is Golden period)

○ >4 hours- testis would be Gangrenous

● IOC - Colour Doppler scan**

● Bilateral exposure is done for Torsion testis always:

○ Affected testis removed

○ Unaffected testis may have pathologies like Bell clapper deformity and hence do

orchidopexy

SURGERY SIXER APP BASED WORK-BOOK 2020 419


Figure: Bilateral exposure for torsion testis

HYDROCELE

● Collection of fluid between tunica vaginalis

● Amber colour fluid

Clinical features:

● Transilluminant +

● Get above swelling +

● Fluctuation +

Types of Vaginal Hydrocele:

● Primary hydrocele - testis not palpable

● Secondary hydrocele- Testis palpable separately

○ Primary - cause unknown

○ Secondary:

■ Infection- Filariasis, Syphilis etc

■ Inflammation

■ Tumours

■ Trauma

Types of hydrocele based on closure of Tunica Vaginalis:

● Vaginal

○ Confined only to scrotum

○ Get above the swelling+

SURGERY SIXER APP BASED WORK-BOOK 2020 420


● Infantile hydrocele

○ Fluid collection seen up to superficial ring.

○ Get above swelling not possible

● Congenital hydrocele

○ Communicating with the peritoneum

○ Reduced on lying down overnight

○ INVERTED INK BOTTLE EFFECT is seen**

● Encysted hydrocele cord.

○ Decreased movement on traction

Figure: Vaginal- Infantile- Congenital- Encysted hydrocele types in order

Epididymal Cyst Spermatocele


❑ Congenital and derived from ❑ Acquired condition
embryonic remnant around the ❑ Due to retention of portion of sperm
epididymis conducting duct of epididymis
❑ MC in Middle age ❑ Filled with fluid of barley water
❑ Clear fluid colour** and contains spermatozoa
❑ Multilocular ❑ Any age, Unilateral
❑ Bilateral rarely ❑ Occasionally the swelling may enlarge
❑ Cyst located behind testis and appear as three testicles**
❑ Chinese Lantern Transillumination** ❑ Transillumination may be negative or
positive.

Management Of Hydrocele

● Jaboulay’s Eversion of sac- bigger hydrocele

● Lords plication of sac - small hydrocele with thin sac

SURGERY SIXER APP BASED WORK-BOOK 2020 421


● Excision of thick sac

Complications of Hydrocele:

● Pyocele

● Hematocoele

● Rupture is not seen

UNDESCENDED TESTIS.

Testis is an organ of lumbar region

Course

● 2nd month - lumbar region

● 3rd month- iliac fossa

● 7th month- Deep ring.

● 7-8 month -travels in the canal

● 8th month - superficial ring

● 9th month - scrotum

Gubernaculum has 5 tails

● Public tail

● Perineal tail

● Femoral tail

● Inguinal tail- if inguinal tail- is taking the dominance it will reach superficial inguinal

pouch ( most common site of ectopic testis)

● Gubernaculum tail - normal one reaching scrotum

SURGERY SIXER APP BASED WORK-BOOK 2020 422


Undescended testis Ectopic testis
● Arrest in normal path ● Deviation from normal path
● Underdeveloped ● Normal development
● Can cause ● Can cause trauma as only
○ T- torsion complication.
○ E-Epididymo orchitis ● Scrotum normal
○ S-sterility
○ T- trauma
○ I- indirect inguinal hernia
○ S-seminoma
● Scrotum not developed**

Retractile testis

● Cremasteric muscle overaction

● Testis will be retracted and lying in Superficial ring and we can manually pull it back to

scrotum which is impossible in ectopic testis.

● Scrotum normal

Clinical features:

● M/c location - inguinal canal

● 70% of it resolve by 2 months

● M/c preterm babies

● Right side

● Leydig cell normal, Sertoli cell affected- so secondary sexual characters normal**

SURGERY SIXER APP BASED WORK-BOOK 2020 423


Investigation

● Gold std - diagnostic laparoscopy

● Inguinal UDT- USG is enough ( Indirect inguinal hernia is also noted)

● On Diagnostic Laparoscopy: ( NEET PG 2020)

- Blind Ending testicular vessels means the testis has gone for atrophy and

no use of further exploration.

- If vessels are seen entering deep ring- we will explore the inguinal canal**

Management:

● BILATERAL UDT- Measure Beta HCG Level ( May be hypertrophied clitoris looking like

penis and the patient may be a female )

● Testosterone value

● Testis should be brought to scrotum back : Uses

○ To prevent trauma

○ To detect malignancy.

○ Normal spermatogenesis returns if done early.

● Orchidopexy + fixation

○ Sub-dartus pouch (m/c)

○ Ombredanne procedure (kept in opposite side with normal tail)

○ Ladd and gross technique- 2 sutures applied ( Testis and Thigh)

○ Keetley torek- temporarily keep it in thigh- after 3 months bring it back

SURGERY SIXER APP BASED WORK-BOOK 2020 424


● Testis unable to bring it to scrotum

○ Silbar – Disconnect the testicular artery keep it in scrotum and do microvascular

anastomosis

○ Fowler Stephen – 2 stage procedure

SURGERY SIXER APP BASED WORK-BOOK 2020 425


Chapter 2: Vascular Surgery

Topic: 2a- Arterial System

Aneurysm:
• Def: blood vessel dilatation >50% is Aneurysm.
• Important DD is ECTASIA (def: blood vessel dilatation <50%).

One liners:
• m/c cause of aneurysm : Atherosclerosis
• m/c site of aneurysm : Circle of Willis
• m/c extra cranial site of aneurysm : Abdominal Aorta (Infra Renal AA is MC site)
• m/c peripheral artery aneurysm : Popliteal Artery
• m/c site of visceral artery aneurysm : Splenic Artery
• m/c site of Mycotic aneurysm : Femoral Artery
(Mycotic aneurysm is a misnomer it is caused by Bacteria)

Classification of aneurysm:
• True aneurysm: contains all the layers of blood vessel
• False aneurysm: lacks 1 or 2 layers of the blood vessel
• Fusiform aneurysm (Symmetric dilatation of Blood vessel)
• Saccular aneurysm (Asymmetric dilatation of Blood vessel; only one part of Blood vessel
is dilated; high chances of Rupture**)
• Dissecting aneurysm (Dissection happens in between the layers of B.V)

Abdominal Aortic aneurysm:


• m/c location : Infra renal A.A
• m/c presentation: Back pain
• Major issue with this aneurysm: it ruptures
• Anterior rupture (20%) Die on spot due to haemorrhage into abdominal cavity
• Postero-lateral (80%) presentation Back pain and Shock; high mortality

IOC for AAA: CT Angiography**


For follow up: USG
Indication of treatment:
• Symptomatic patient
• Asymptomatic patient: AP diameter >55mm (until 5.5cm if patient is asymptomatic
follow up)

Conventional Treatment:
• Resect the aneurysm out

SURGERY SIXER APP BASED WORK-BOOK 2020 426


• Replace it with DACRON PANT Type GRAFT

Figure: DACRON GRAFT

Complications:
• m/c cause of death : MI
• Renal failure
• Leak from graft
• Colonic ischemia (inferior mesenteric artery if not re implanted properly)
• Injury of artery of Adamkieklicz
- It arises from Posterior intercostal Artery
- Supplies : anterior part of spinal cord
- Injury leads to: Anterior Spinal Artery Syndrome (Paraplegia)
• Mortality
- Elective:2%
- Emergency: 50%

Endo Vascular Aneurysm Repair (EVAR)- Latest treatment


• A Stent is placed inside the blood vessel at the site of aneurysm.
• Blood flows through the stent and the aneurysm collapses.

SURGERY SIXER APP BASED WORK-BOOK 2020 427


Figure: EVAR

HUNTERIAN LIGATURE OPERATION:


• Leaving the blood vessel above and below, the aneurysm is ligated.
• The aneurysm collapses after ligation
• Not done in recent day because of various ischemias developing.

Material used in aneurysm repair: DACRON and Polytetrafluoroethylene


Suture material: 3’0 or 4’0 Prolene

AORTIC DISSECTING ANEURYSM:


• Aortic dissection is caused by a circumferential or less frequently, transverse tear of the
intima
• Most common site: Right lateral wall of the ascending aorta**
• Another common site: Descending thoracic aorta (just below the ligamentum arteriosum)
• Peak incidence: 6th and 7th decades**
• Men are more commonly affected**
• m/c Risk factor: HTN**
• other risk factors:
o Marfan’s syndrome
o EDS
o Takayasu arteritis
o Coarctation of Aorta
• Presentation: Severe pain; Hypotension; M.I ( if coronary ostium is involved in dissection)
• X-ray chest: Widened mediastinum

• Investigation of Choice

o Stable patients: CECT thorax

SURGERY SIXER APP BASED WORK-BOOK 2020 428


o Unstable patient: TEE (Trans Esophageal Echo)

De Bakey Classification STANFORD Classification


• Type I: Ascending, Arch & Descending • STANFORD A: De Bakey I &II (A –
aorta involved Ascending aorta)
• Type II: Ascending aorta alone • STANFORD B: De Bakey III
involved
• Type III: Descending aorta alone
involved

Classification of Dissection Aneurysm

Clinical Features
• Pain
• Shock
• BP: disproportionate BP between Upper limbs or disproportionate BP between UL & LL
Treatment: Emergency DACRON GRAFT repair**

CRAWFORD CLASSIFICATION OF THORACO-ABDOMINAL AORTIC ANEURYSM


• Type I: from Lt subclavian artery
up to renal artery.
• Type II: from Lt subclavian artery
up to bifurcation of aorta
(largest**)
• Type III: from middle of descending
aorta up to bifurcation of aorta.
• Type IV: it involves supra renal
aorta +/- renal artery. ( Not
present inside Thorax)

Popliteal artery aneurysm:


• It is the m/c peripheral artery aneurysm
• 2/3rd of times bilateral

SURGERY SIXER APP BASED WORK-BOOK 2020 429


• Can wait up to 25mm (>25mm surgery)
• Distal occlusion ( Due to thrombus or embolus formation)

Mycotic aneurysm:
• Caused by Staphylococcus
• Site : Femoral Artery
• False aneurysm

Acute Limb Ischemia:


• M/C cause: embolus that comes from Lt atrial thrombus
• Clinical features: 6P
• Pain
• Paresthesia
• Pulselessness
• Poikilothermia
• Paralysis of limb
• Pallor
• It is an emergency: do embolectomy in 4-6hrs

Seldinger technique:
• Cannulate the artery in retrograde way using seldinger needle
• Pass the guide wire distal to the block
• Through guide wire a balloon (Fogarty balloon catheter) is passed distal to block, balloon is
inflated & withdrawn
• Through a small incision the embolus is removed

CHRONIC LIMB ISCHEMIA:


Peripheral Vascular Disease- Atherosclerosis of LL TAO
Causes: Cause
• DM • Smoking
• HTN

SURGERY SIXER APP BASED WORK-BOOK 2020 430


• Hyper cholesterolemia
• Obesity....
Thrombus formed in vessel occludes the vessel Vasospasm occludes the vessel
Involves : Involves
Major vessels • Infra popliteal arteries
• Aorta
• Common iliac artery
• External iliac artery
• Femoral artery
It is U/L or B/L It is BL Symmetrical
Common in Old age Common in Young age <50yrs**
Males > Females Seen only in males
UL not affected UL also effected
Adjacent Veins & Nerves not affected Adjacent Veins & Nerves are affected
C/F: Intermittent claudication C/F:
• Pain not present at rest • Young male with bilateral leg claudication
• Pain stops on walking pain.
• Pain not present on standing • On CT angiography: symmetric lesion is
• Pain decreases on rest seen
(Neurogenic claudication pain increases on
standing)

• m/c site of claudication pain : Calf**


• m/c site of occlusion:
Superficial femoral artery

Q. Claudication pain in Buttock, both thigh, both calf. Site of occlusion? (NEET 2020 pattern)
• Ans. Aortoiliac occlusion.
Q. Claudication pain in Buttock(rare), one thigh & one calf. Site of occlusion?
• Ans. Unilateral Common Iliac artery occlusion.
Q. Claudication pain in thigh & calf. Site of occlusion?
• Ans. Externa Iliac Artry occlusion.
Q. Claudication pain in calf. Site of occlusion?
• Ans. Superficial femoral artery/ popliteal artery occlusion.

LERICH SYNDROME:
• Saddle shaped thrombus at bifurcation of aorta.
• Clinical Features:
- Buttock claudication
- Impotence

SURGERY SIXER APP BASED WORK-BOOK 2020 431


BOYDS CLINICAL CLASSIFICATION OF CLAUDICATION PAIN:
• Grade I: Pain relieved on continued walking
• Grade II: Walks in pain with limp
• Grade III: Pain compels to sit
• Grade IV: Rest pain

Other classification systems


• Fontaine
• RUTHERFORD

Ankle Brachial Pressure Index:


• ABPI= AP/BP = 0.9-1.1 (Normal)
• If Ankle pressure less than Brachial pressure: there is stenosis of vessel seen:
o <0.9 : Claudication
o <0.5 : Rest pain
o <0.3: Imminent necrosis
• False high AP/BP >1.3 seen in calcified arteries in DM & CKD

Investigations of vascular disease:


• 1st done: Colour Doppler (Duplex scan)
o Non invasive
o Repetitive
o Gold standard: CT Angiography (Road map)

TREATMENT:
Olden days:
• BYPASS OPERATION is done using
• DACRON (for Aortic bypass & Aortic- femoral bypass. Suture material 3’0 or 4’0
Prolene)
• PTFE (for Ileo femoral artery bypass. Suture material 5’0 Prolene)
• SAPHENOUS VEIN (for infra femoral bypass)
▪ INSITU do valvotome of the LSV valves. (Or)
▪ REVERSE LSV**

SURGERY SIXER APP BASED WORK-BOOK 2020 432


Figure: Bypass operations

ANGIOPLASTY: Gold standard treatment now


• Seldinger needle is passed beyond the block and an Intra-arterial stent is placed

Thrombo angitis Obliteras ( TAO)- Buerger’s disease


Shionaya Criteria:
• History of smoking
• Onset should be before 50yrs
• Infra popliteal occlusion only
• UL involvement present
• Superficial phlebitis present
• No other risk factors Except Smoking

Pathophysiology:
• Occlusion is due to sympathetic over activity leads to vaso-spasm in infra popliteal
arteries, it is bilateral
Angiography demonstrates various collaterals
• Cork screw collaterals
• Tree root collaterals
• Spider leg collaterals

SURGERY SIXER APP BASED WORK-BOOK 2020 433


Figure: Angiography shows TAO

• TAO Patient: Young male with bilateral leg claudication pain.


• On CT angiography: symmetric lesion is seen

Treatment :
• Stop smoking
• Buerger’s exercises: collaterals in lower limbs increases
• Lifestyle modifications

Surgery: Lumbar sympathectomy


• On one side L1, L2&L3 are cut on the other side L2&L3 are cut.
• L1 on one side is preserved to prevent retrograde ejaculations

Indications of lumbar sympathectomy**


• Rest pain
• Ulcer
• Gangrene
Contraindications for lumbar sympathectomy
• Intermittent claudication
• ABPI > 0.3
• Diabetic patients

A-V FISTULA
• Abnormal communication between artery and vein is A-V fistula
• M/C cause: Iatrogenic
• CIMINO FISTULA: Surgically created fistula between Cephalic vein and Radial artery for
Dialysis purposes
• M/C acquired cause: penetrating trauma

SURGERY SIXER APP BASED WORK-BOOK 2020 434


• Genetic syndromes
o Sturge weber
o Beckwith weidmann
o Klippel trenanauy
Patho physiology:
o Arterialisation of vein
o Limb lengthening
o Temperature high on that limb
o Pulse rate high on that limb
o O/E: Machinery murmur (continuous thrill)

Figure: AV Fistula
NICOLDANI / BRANHAM’S SIGN: on compressing proximal to fistula
o Murmur disappears
o Swelling decreases
o Thrill absent
o Pulse rate decrease
RAYNAUD DISEASE
• Due to unknown cause
• If it happen due to Scleroderma, SLE it is called Raynaud’s phenomenon
• Vasospasm is due to
• Cold exposure
• Continuous Vibrations

Phenomenon seen:
W-B-C : initially limb colour is White later
Blue and then Crimson red
• White: due to spasm of both arteries
and veins
• Blue (cyanotic phase) : here arteries
are still in spasm and veins &
capillaries opens
• Crimson red: Arteries opens in an
extensive way, veins already in
opened state

SURGERY SIXER APP BASED WORK-BOOK 2020 435


Treatment:
• Calcium Channel Blockers
• Avoid cold
• Cervico dorsal sympathectomy (benefit is doubt full)

Image Based Questions:


Image 1: Seldinger Needle for Retrograde cannulation of vessels

Image 2: Miller cuff: interposition of vein between graft and Artery during anastomosis

SURGERY SIXER APP BASED WORK-BOOK 2020 436


Image: 3: Fogarty Balloon Catheter for Embolectomy

Topic: 2b – VENOUS SYSTEM

Anatomy of Venous system:


• Deep venous system: femoral vein (FV) that continuous as popliteal vein (PV) and that divides
into anterior tibial vein (AT) and posterior tibial vein (PT)
• Superficial venous system: medial side great saphenous vein (GSV) that enters deep system at
sapheno-femoral junction (SFJ). At lateral side short saphenous vein (SSV) drains into deep
system at popliteal vein
• Perforators: vessels that connect superficial and deep venous system

Perforators on medial side:


• Adductor canal
perforator or
Hunterian
perforator : At mid
thigh: Connects
GSV to FV

• DODD: Above knee


perforator:
Connects GSV to FV

• BOYD: Below knee


perforator:
Connects GSV to PT

• COCKETT: Lower
leg perforator:
Connects Posterior
arch vein(a branch
of GSV) to Posterior
Tibial vein

SURGERY SIXER APP BASED WORK-BOOK 2020 437


• MAY/KUSTER:
Ankle small
perforator

Varicose veins:
Causes:
• DVT
• SFJ incompetence
• Perforator incompetence
C/F:
• Dilated tortuous veins of > 3mm
• Skin stretched
• Eczema, pigmentation
• Ulceration
• Loss of hair in LL
• Brittle nails
• Lipo Dermato Sclerosis: is skin pigmentation due to rupture of RBC releasing Hemosiderin

Types of Dilated veins:


• Telangiectasia: Vein size < 1 mm diameter
• Reticular veins: Vein size 1 – 3 mm diameter
• Varicose veins: vein dilatation > 3 mm diameter

TESTS
SF incompetence: Perforator Incompetence DVT:

• Trendelenburg I • Trendelenburg II • Perthes test


• Morrissey’s cough • Multiple tourniquet • Modified Perthes test
impulse test test • Homan’s test
• Pratt’s test • Moses test
• Fegan test • Pratt’s sign

Trendelenburg I test:
• The patient is made to lie down flat and superficial veins are emptied. With the thumb
compress SFJ (SFJ present 4 cm below & lateral to pubic tubercle) and make the patient
stand on taking the thumb out we can see the veins filling from above to below. It is due
to SFJ incompetence**
Trendelenburg II test:
• The patient is made to lie down flat and superficial veins are emptied. With the thumb
compress SFJ and make the patient stand and without removing the thumb we can see

SURGERY SIXER APP BASED WORK-BOOK 2020 438


veins filling from below to above it is due to perforator incompetence** ( THUMB Not
taken out)

Multiple tourniquet test:


• The patient is made to lie down flat and superficial veins are empties. Multiple tourniquets
are tied 1st is below SFJ, tourniquets are tied in such a way that a tourniquet is present
above and below each perforator. Vein dilatation between two tourniquets indicates that
particular perforator incompetence

Perthes test: For DVT:


• Compress the superficial veins with tight elastic crape and ask the patient to walk. If he
complains pain it indicates DVT. It is a subjective test

Modified Perthes test:


• A tourniquet is tied below SFJ and patient is asked to walk if the veins become more
dilated and patient complains of pain it indicates DVT.
• Prerequisite to do this test: perforator incompetence should not be present**

PRATT’S TEST:
• Patient is made to lie down flat, superficial veins are drained and a tight elastic crape is
rolled from below upwards, another elastic crape is rolled from above downwards
simultaneously first band is removed loop by loop.
• Quick filling of veins between the bandages and a blowout is seen indicate that particular
perforator is incompetent. On removing the crape and on palpation of the blow out the
deep fascia will have a pit this is FEGAN’S TEST

SCHWARTZ TEST:
• Valve incompetency can be detected.
• Tap at lower end of the varicose vein and you can feel the impulse at the upper level

MORRISEY’s COUGH IMPULSE TEST:


• Patient is made to lie down flat and the superficial veins are drained and on coughing an
expansile impulse is felt at SFJ.
• Saphena varix: bulge in the SFJ before coughing

Homan’s sign: forceful dorsiflexion of the ankle (C/I due to chances of PE)
Moses sign: forceful compression at mid calf level (C/I due to chances of PE)
Pratt’s sign: lateral compression of calf

SURGERY SIXER APP BASED WORK-BOOK 2020 439


CEAP classification of varicose veins
Clinical classification:
• C1: Telangiectasia / reticular veins
• C2: Varicose veins
• C3: Edema in legs
• C4a: Pigmentation, eczema
• C4b: LipoDermato Sclerosis or Atrophic Blanche
• C5: Healed venous ulcer
• C6: Active venous ulcer
Etiological classification:
• Ep: primary
• Ec: congenital
• Es: Secondary

Anatomical classification:
• As: superficial veins
• Ap: Perforators
• Ad: Deep veins

Pathological classification:
• Pr: Reflux
• Po: Obstructive
• Pro: Reflux + obstruction

Risk factors
• Family H/O
• Long standing job
• DVT
• Defective valves
• Mass in abdomen
• Pregnancy
• Congenital syndrome: KLIPPEL TRENANAY SYNDROME
o Port wine stain
o Lateral varicose veins
o Limb lengthening

SURGERY SIXER APP BASED WORK-BOOK 2020 440


Figures: Showing Klippel Trenanay Syndrome

Complications:
Skin complications:
• Eczema
• Pigmentation
• LDS
• Atrophic blanche
• Inverted champagne bottle appearance**
• Skin ulcer
• Bleeding from Varicose veins

Figure: LDS with inverted Champagne Bottle appearance

Skin ulcer: on medial malleolus the venous pressure is high that lead to venous ulcer
• Gaiter area
• Sloping edge
• Floor covered by granulation tissue

SURGERY SIXER APP BASED WORK-BOOK 2020 441


• Base formed by tibia

Figure: venous ulcer and Marjolin’s

Complications of Venous ulcer:


• Ulcer can bleed
• Ulcer can cause periostitis tibia
• Equinus deformity
• On long standing can go for Marjolins ulcer- Squamous cell cancer
Management: Bisgaard method of management of Venous ulcers.

Investigations:
• Hand Held Doppler:
o Obstruction: sound+
o Woosh sound on compression & Release
• IOC: color Doppler (Duplex scan)
o Normal flow Blue color
o Reflex Red color: incompetence
o MICKEY MOUSE SIGN: Face FEMORAL VEIN, Lateral ear: FEMORAL ARTERY,
Medial ear: GSA
o Duplex scan is done to
R/O DVT & To mark the perforators

SURGERY SIXER APP BASED WORK-BOOK 2020 442


Figure: Mickey mouse sign

Management:
Compression stockings
BRITISH CLASSIFICATIONOF STOCKINGS
o Class 1: give pressure of 14 -17 mm Hg (used prophylactically)
o Class2: gives pressure of 17 – 24mm Hg
o Class 3: gives pressure of 25 – 35 mm Hg (used for venous ulcer & varicose veins)

VENOUS ULCER:
BISGAARD METHOD:
• Leg lifting is advised
• Clean the ulcer with water / NS
• Pus C/S – Antibiotics
• Bleeding: lift the leg
• 4 layer bandage should be applied to seal the venous ulcer** ( NEET SS)
1. Cotton wool
2. Cotton Elastocrepe bandage
3. Elastic bandage
4. Cohesive bandage

• If ABPI is normal i.e 0.9 – 1.1 apply pressure of 35 – 40 mm Hg


• If ABPI is less: 0.5 - 0.8 apply pressure of 30mm Hg
• If ABPI is < 0.5: no compression, 1st revascularize the artery

• Clean the ulcer and put a split skin graft.

SURGICAL MANAGEMENT OF VARICOSE VEINS


Conventional treatment : TRENDLENBERG OPERATION:
• Identify the sapheno-femoral junction, flush ligation of the GSV close to SFJ & ligate all
the superficial veins at SFJ to prevent recurrence

SURGERY SIXER APP BASED WORK-BOOK 2020 443


• Insert a stripper (OESCH PIN STRIPPER) from midcalf level and tie the vein at SFJ and
withdraw the stripper the entire vein will come out

Figure: Stripped Veins

Note:
• If inserted below the mid calf level, Saphenous nerve will be injured medially and sural
nerve will be injured laterally
• Ensure there is no DVT before performing this surgery
• Surgery is contraindicated if DVT is present in the same limb

Perforator incompetence:
• Supra facial ligation of LINTON (in Lipodermatosclerosis Linton operation is not possible)
• Sub facial ligation of COCKETT & DOD (can be done in LDS)
• Sub Facial Endoscopic Perforator Surgery (SEPS):
- Perforators are clipped below the fascia on visualizing through a endoscope
• TRIVEX SYSTEM:
- Subcutaneous illuminator is used
- Veins are visualized
- By a minimally invasive method visible veins are Hooked and ligated

FOAM SCLEROTERAPY
• By using USG dilated veins are sclerosed by TESSARI METHOD

Foam Sclerosants used are:


• Sodium Tetra Decyl sulphate (M/C used)
• Sodium morrhuate
• Polidocanol
• Ethanolamine oleate

SURGERY SIXER APP BASED WORK-BOOK 2020 444


Figure: Tessari method of FOAM sclerotherapy

Recent Advances:
Endovenous laser ablation (EVLA) Radio Frequency Ablation

• A laser catheter is introduced into the A coil is inserted into the dilated vein, Electro
dilated vein, 1470nm is applied veins magnetic current is used to produce thermal
are totally destroyed energy and burn out all the vessels
Preset cycle of current application
Advantages
• Any size vein can be ablated Advantages:
• Can be used for Perforator** • Set standard protocol is available
incompetence • Easy for the beginners
• Cheaper than Radio Frequency • Less procedure time
Ablation • Less painful
• Laser protocol is not needed

Disadvantages:
• RFA fiber is costly
• RFA is not possible for tortuous veins
• RFA is not possible for perforators

DEEP VEIN THROMBOSIS


Clinical features:
• M/C pain and swelling in calf (Usually Unilateral)
• Low grade fever
• M/C site of DVT: calf veins
• M/C site of DVT causing PE is: Femoropopliteal veins
Signs of DVT
• Pratt’s sign can be done
• Perthes test
• Modified Perthes test

SURGERY SIXER APP BASED WORK-BOOK 2020 445


IOC: Duplex scan (Color Doppler)

Causes of DVT:
Patient factors Diseases:
• Old age • Trauma surgery
• Obesity • Surgery to orthopaedic fractures
• Immobilization • Malignancy
• Pregnancy / Puerperium / High dose • Heart failure / MI
of estrogen therapy • IBD
• Previous H/O of DVT • Nephrotic Syndrome / MI
• Thrombophilia • CVA
• PNH, Homocystinemia

Virchow’s triad: result in DVT


• Stasis
• Hypercoagulable stage
• Endothelial damage

Thrombophilia: group of diseases


Congenital Acquired
Anti Thrombin III deficiency Anti phospholipid antibody
Protein c / s deficiency Lupus anticoagulant present
Factor V leiden deficiency
Lupus anticoagulant present
Dysfibrinogenemia

Risk of DVT:
High risk Surgeries for DVT:
• Urology surgeries 40>Yrs
• Abdomen surgeries
• Pelvic surgery
• Orthopedic surgery
Moderate risk surgeries
• General surgery > 40Yrs
• Patients on OCP undergoing surgeries
• Surgery lasting for > 1 hour
Low risk
• Uncomplicated surgery < 40yrs
• Surgery duration < 30 min

SURGERY SIXER APP BASED WORK-BOOK 2020 446


Modified Well’s criteria to predict DVT (NEET DVT going to Pulmonary embolism can be
2020 TOPIC) predicted by another Modified Well’s criteria
• -2 to 0: low probability • >4: suggestive of PE
• 1 to 2: Moderate probability • <4: less chances of PE
• >2: High probability of developing DVT
Factors:
Factors 3 points :
• Lower limb injury / surgery / • C/E: signs & symptoms of DVT: 3
immobilization: 1 • Alternative diagnosis less likely then
• Bed ridden > 3days / any surgery <4 DVT: 3
weeks: 1 1.5 points each:
• Clinical examination • HR > 100/min: 1.5
o Tenderness:1 • Hemoptysis: 1.5
o Swollen limb: 1 • H/O DVT / PE: 1.5 points
o Calf diameter> 3cm: 1
o Pitting edema: 1 1 point each:
o Dilated superficial vein: 1 • H/O immobilization >3days / surgery
• History points: in < 4 weeks- 1 point
o H/O:DVT: 1 • H/O Malignancy ( Treated or
o H/o Malignancy: 1 palliation in 6 months)- 1 point
o H/o Drug abuse: 3
• Alternate diagnosis more likely: -2

Prevention of DVT:
Mechanical methods
• Early mobilization of patient
• Pneumatic compression devise
Pharmacological method (Best to prevent DVT)
• Inj Heparin (aPTT monitoring)
• Low molecular weight heparin(prophylaxis) (Warfarin not used in prophylaxis)

Treatment of DVT:
• First 5 days: Heparin + warfarin
• After 5 days: only Warfarin (monitor PT/INR)
• 1st episode: 3 months
• Recurrent: life long
• Thrombolytics are used for iliac vein involvement

International Normalised Ratio : INR


• INR = PT of patient / PT of controller usually 1 to 1.1
• Surgery is done only if INR < 1.4

SURGERY SIXER APP BASED WORK-BOOK 2020 447


• INR > 1.4: No surgery advise, give FFP before surgery
• DVT : on warfarin target INR: 2 – 3
• Mechanical heart valves on Warfarin target INR: 2.5 – 3.5

Surgical Treatment:
• Thrombectomy and place a IVC filter (Greenfield filter)
• Bypass operation: Palma operation
• Green field filter indication
o Anticoagulants contra indicated: Cerebral hemorrhage
o DVT + anticoagulants developing recurring PE
Palma Operation:
• In cases of DVT of the external
iliac vein on one side the
contralateral saphenous vein is
anastomosed to the femoral
vein on the affected site below
the level of occlusion or stenosis.

SYNDROMES RELATED TO DVT


May Thurner and cockett syndrome:

• Right common iliac


artery is running on Lt
common iliac vein and
compressing it: resulting
in clinical feature like
DVT

SURGERY SIXER APP BASED WORK-BOOK 2020 448


Phlegmasia Alba Dolens (White leg/
Milk Leg)
• Seen in pregnancy &
underlying malignancy
• Deep veins are thrombosed
• Superficial veins are patent
• Not much dangerous as the
blood is flowing

Phlegmasia cerulean Dolens (Blue


leg)
• M/C seen in Malignancy
patients
• Deep veins thrombosed
• Superficial veins
thrombosed
• Leg is in a gangrenous
stage: Venous gangrenous
stage
• Immediate surgery is
needed

TOPIC 2C: LYMPHATIC SYSTEM


LYMPHEDEMA:
• M/C cause worldwide: FILARIASIS
• M/C cause of upper limb Lymphedema: Post mastectomy

Classification:
Primary Lymphedema Secondary Lymphedema
Cause: Congenital • Filariasis(M/C)
• Fungal
• RT
• Malignancy
• Surgery (MRM)
• Trauma
• Superficial thrombophlebitis
• DVT
• Silica Exposure

SURGERY SIXER APP BASED WORK-BOOK 2020 449


PRIMARY LYMPHEDEMA:
Congenital Lymphedema praecox Lymphedema tarda

• Onset < 1 year • Onset 1-35 years • Onset > 35 years


• M/c Bilateral • M/c primary lymph edema
• Involve whole leg • m/c in women
• Milroy’s disease. • m/c unilateral
• Mostly involve below knees only
• Also known as Meig’s disease

Figure: Milroy’s disease

Malignancies associated with Lymphedema:


• Lymphangiosarcoma (Stewart Treves Syndrome)
• Kaposi sarcoma (HIV)
• SCC
• Liposarcoma
• Malignant Melanoma
• Malignant fibrous histiocytoma
• BCC
• Lymphoma

Clinical features of LYMPHEDEMA


• More confined to foot, toes and ankle
• Dorsum of foot looks like buffalo hump, toes look like square foot
• Skin is not pinchable: STEMMER’s SIGN
• Eczema
• Fungal infection: of skin: Dermatophytosis
• Fungal infection of nail: Onychomycosis
• Fissures, verrucae, wart
• Ulceration is unusual if present rule out venous insufficiency
• May get a problem: LYMPHANGIOMA

SURGERY SIXER APP BASED WORK-BOOK 2020 450


LYMPHANGIOMA
• Dilated Dermal Lymphatics- Forming Blisters
• Clear fluid present( Rarely Blood stained)
• Long term- Lymphangiomas- Thrombose and form nodules.
o < 5cm = Lymphangioma Circumscriptum
o More widespread- Lymphangioma Diffusum
o Forming Reticulate pattern of Ridges- Lymphedema ab igne.
• Weeping Lymphangioma- Lymphorrhea/ Chylorrohoea
• Protein-losing diarrhoea, chylous ascites, chylothorax, chyluria and discharge from
lymphangiomas suggest lymphangectasia (megalymphatics) and chylous reflux

Figure: Lymphangioma Circumscriptum

Figure: Lymphangioma Ab Igne

BRUNNER’s CLINICAL GRADING OF LYMPHEDEMA:


o Grade I: pitting edema, edema decreases on lying over nigh
o Grade II: STEMMER’s SIGN +ve: non pitting edema, not subsided on lying over night
o Grade III: Irreversible skin changes: Fibrosis & papillae

Management:
Lymphedema usually treated conservatively:
CONSERVATIVE management
o Pain management
o Infection (Lymphangitis in Lymphedema is due to Streptococcal) treated with antibiotics
o Skin management: Moisturisers, antifungal ointment
o Multilayer compression stockings are used > 35 mm Hg:
- at the level of ankle 100% pressure,
- at knee 70% pressure,

SURGERY SIXER APP BASED WORK-BOOK 2020 451


- at mid thigh 50% pressure and
- at groin 40% pressure is applied
Exercises:
o Vigorous exercise contraindicated
o Mild exercise is prescribed like swimming
o Rhythmic exercise is advised
DRUGS: not much available
Tab DEC is given in acute cases and acute exacerbation

Surgical option in Lymphedema:


Bypass procedures: not much successful
o Omental pedicle
o Skin bridge (GILLIES procedure)
o Ileal mucosa (KINMONTH surgery)
o Lymphatico venular anastomosis
o Lymph node to saphenal anastomosis (KINMONTH)

Limb reduction procedure: successful & M/C used


o Charles
o Sistrunk
o Thompson
o Homans

Charles procedure:
o Remove entire skin and subcutaneous tissue up to the muscles or fascia, place a split skin
graft. Continuous lymph ooze is present

Figure: Charles procedure

SISTRUNK operation:
o Wedge excision of skin and subcutaneous tissue + Reapproximation. No lymph ooze
ThomPson:
o Buried dermal flap operation, Pilonidal sinus is a complication
HOMAN’s Operation:

SURGERY SIXER APP BASED WORK-BOOK 2020 452


o Most satisfactory surgery, skin flap is raised all the subcutaneous tissues is removed and
the skin flap is closed. Skin flap necrosis is a complication

CHYLURIA:
• Filariasis is the MC cause of Chyluria.
• 1-2% cases of filariasis will get this complication after 20 years.
• Painless passage of milky White Urine, after a fatty meal
• Chyle may Clot leading to Renal Colicky and Hypoproteinemia may result.
• Chyluria may be caused by Ascariasis, Malaria, TB and Tumour also.
• IV urography or Lymphagiography may show the fistula ( Lympho Urinary)
• Low fat, High Protein diet, Plenty of liquids to avoid clots of chyle are advised.
• Laparotomy and ligation of Dilated lymphatics are advised.
• Sclerotherapy of lymphatics are attempted.

SURGERY SIXER APP BASED WORK-BOOK 2020 453


Chapter 3 – Plastic surgery and Skin lesions

Part 1: Skin lesions, Ulcers, Swellings etc

Topic 3a: Pressure sores / Bed sores


• Pathophysiology: as patient stays in one position capillary pressure becomes >30mm Hg
that results in ischemia. Ischemia leads to loss of skin subcutaneous tissue
Classification:
• Stage I: Erythema
• Stage II: Partial thickness (Epidermis & Part of dermis lost)
• Stage III: Full thickness (Epidermis & Dermis lost; raw area extends up to subcutaneous
tissue
• Stage IV: involves Fascia, Muscles & Tendon

M/C sites of bed sore in sequence:


• Indra - m/c : ischium
• Gandhi- Greater trochanter
• Statue- Sacrum
• Has - Heel
• More- Malleoli(lateral> medial)
• Ornaments- Occiput

Figure: Bed sore- MC site- Ischium

Prevention:
• Bed ridden patient :Change the position - 2 hrs once
• Wheel chair patient : lift him up for every 10 minutes once for 10 second
• BED- Aqua bed for quadriplegic patient

Management:

SURGERY SIXER APP BASED WORK-BOOK 2020 454


• Grade I & II: Moist dressings
• Grade III & IV: Debridement and use VAC devices at -120mm Hg negative suction.
• TOC : Vacuum Assisted Closure Device

Topic 3b: NECROTISING FASCITIS


• Definition: It is a rapidly progressing bacterial infection causing necrosis of skin,
subcutaneous tissue, fascia and underlying muscles.
• M/C organism: Group A Beta Hemolytic Streptococci**
• It is usually polymicrobial infection
• Seen in immune compromised patients like
o DM
o Cancer patients
o Patients on chemo-therapy
• Necrotising fasciitis involving scrotum : Fournier’s Gangrene** . Remove necrotic tissue on
debridement results in Shameful exposure of testis
• Necrotising fasciitis involving Abdominal wall: MELENEY’S Gangrene**
• Necrotising fasciitis involving Lower limb: M/C site of Necrotising Fascitis**
• Rarely involves head & neck

Figure: Fournier’s Gangrene


Treatment:
• Hyperbaric Oxygen therapy
• Immediate Debridement
• Higher antibiotics

Topic 3c: CELLUTLITIS VS ERYSIPELAS


CELLULITIS ERYSIPELAS
Caused by : Streptococcal pyogenes Caused by : Streptococcal pyogenes
Subcutaneous tissue (mainly) + Deeper tissues Skin + subcutaneous tissue involved
Poor demarcation between involved tissue and Well demarcation
normal tissue
Bacteremia Due to loss of skin multiple organisms attack
Tenderness & the patient will be in severe TOXAEMIA
Subcutaneous tissue absent in ear:

SURGERY SIXER APP BASED WORK-BOOK 2020 455


Hence Ear not affected Ear is affected(MILIANS EAR SIGN)**
Butterfly shaped lesion seen in face

Figure: Cellulitis and Erysipelas

Topic 3d: ABSCESS


PYOGENIC ABSCESS COLD ABSCESS
Caused by Bacterial infection Caused by
Staph aureus TB(M/C)
Leprosy
Syphilis

Signs of inflammation present No signs of inflammation


Treatment
• Abscess drained by putting a Cold abscess drained by non-dependent
CRUCIATE incision method to prevent sinus formation
• If abscess present in parotid region,
neck & axilla or any place where
neuro vascular structure is present use
HILTONS METHOD to drain the
abscess (no cruciate incision use
LISTER’S SINUS FORCEPS)

Topic 3e: HIADRENITIS SUPPURATIVA


• It is apocrine sweat gland infection
• Caused by Staphylococcus aureus
• M/C site : Axilla > Pubis
• M/C in DM patients
• To prevent spread of infection advice not to shave axilla just cut the hair

SURGERY SIXER APP BASED WORK-BOOK 2020 456


• Extensive conditions may need split skin grafting

Figure: Hiadrenitis Suppurativa

CARBUNCLE
• Rapidly spreading cellulitis in diabetics
• Caused by S.aureus
• M/C site nape of neck

POTT’S PUFFY TUMOR:


• It is frontal bone osteomyelitis
• infection spreads from frontal sinusitis
• it can extend deeper and can cause subdural empyema
• I&D

Topic 3f: ULCER


Parts of ulcer:
• Margin
• Edge
• Floor : is seen on inspection (Slough or granulation tissue is seen)
• Base : is felt on palpating the ulcer

TYPES OF ULCER:
• Sloping edge: seen healing ulcer or venous ulcer
• Punched out edge: seen in Syphilis, trophic ulcer & arterial ulcer
• Undermined edge: seen in TB
• Rolled out edge: seen in BCC(rodent ulcer)
• Everted edge: seen in SCC

SURGERY SIXER APP BASED WORK-BOOK 2020 457


Venous ulcer:
• Seen in varicose veins
• Seen on medial malleolus
• A long standing venous ulcer converted into malignancy MARJOLINS ULCER(SCC)

MARJOLINS ULCER
• Arise from Burns scar, venous ulcer & keloid.
• Usually no LN mets
• If it crosses normal skin LN mets are seen
• Radiotherapy is contraindicated. Only Surgery Advised

Figure: Marjolin’s Ulcer

Topic 3g: SWELLINGS


LIPOMA SEBACEOUS CYST DERMOID CYST
• Also known as universal • Also known as Sequestration Dermoid:
tumor epidermoid cyst • Due to epithelium entering
• M/C skin swelling • Formed due to at the line of fusion
• DERCUM’S Disease: obstruction of
multiple painful lipomas sebaceous duct .It is a Seen in
retention cyst • Scalp (M/C)
Emergency presentation: • 70% punctum present • External angular dermoid

SURGERY SIXER APP BASED WORK-BOOK 2020 458


• GIT(Intussusception) • Content : bad • Post auricular dermoid
• Respiratory smelling pultaceous
Tract(Obstruction) material
• M/C site : Head &
Malignant conversion Neck**
(Liposarcoma) • It is absent in palms
• Thigh & soles
• Retroperitonium
FORDYCE DISEASE:
• Presence of ectopic
sebaceous tissue in lips
Clinical Examination: Clinical Examination: Clinical Examination:
• Slip sign + • Skin is not • Indentation + of Bone may
• Pseudo fluctuation + pinchable** be present
• Trans illumination+/- • Putty consistency

Treatment: Treatment: Treatment:


• Enucleation • Elliptical incision Excision
• Excision made along with • Always take a CT scan
punctum sebaceous before surgery to rule out
cyst excised intra cranial extension
• Infected sebaceous
cyst: cruciate I&D
Complications of sebaceous Rare types of dermoid:
cyst: • Implantation dermoid m/c
• Cock’s peculiar in finger tips
tumor- ulcerated • Tubulo dermoid: present at
sebaceous cyst in midlines Eg: Thyroglossal
scalp looks like a cyst, post anal dermoid
tumor • Teratomatous dermoid:
• Sebaceous horn- contains all 3 layers usually
dried up sebum seen in ovaries(contains
• M/C complication: skin, hair, teeth& muscle)
Infection

SYNDROMES ASSOCIATED
• FAP + sebaceous cyst
= GARDNER’S
• HNPCC+ sebaceous
cyst = MUIR TORRE

One liner’s
• DERCUM DISEASE multiple painful lipomas
• COCK’S PECULIAR tumour- ulcerated sebaceous cyst in scalp
• WEN: multiple sebaceous cyst in scrotum
• DIMODEX FOLLICULARUM: Worm inside sebaceous cyst
• SEBACEOUS HORN: Widow Dimanche is the Bailey and Love Model**

SURGERY SIXER APP BASED WORK-BOOK 2020 459


Figure: Cock’s Peculiar Tumors

Figure: Sebaceous Horns

Topic 3h: SCARS


HYPERTROPHIC SCAR KELOIDS
Is due to proliferation of Arises from
• mature fibroblasts • immature fibroblasts
• mature blood vessels • immature blood vessels
Limited only to previous scar It may happen from previous scar or
anywhere without scar
It extends to normal skin
More seen in flexor surfaces More seen in extensor surfaces
Eg: sternum, face
C/F : Itching+++++
No family Genetic: Negroes & young females
No genetic
Treatment: Treatment:
• Excision • No excision
• Intra lesional steroid (Triamcinolone
acetate) for preventing the growth of
keloid & to decrease itching.
Long standing keloid leads to Marjolin’s ulcer

SURGERY SIXER APP BASED WORK-BOOK 2020 460


Topic 3i: HEMANGIOMA OF SKIN
Classification of Hemangioma based on Involution
NICH RICH
• Non involuting congenital hemangioma • Rapidly involuting congenital
• Happens at 4-6 months hemangioma
• Doesn’t disappear • Happens at 4-6 months
• Disappears in 5-7 yrs

Classification based on depth of invasion


CAPILLARY HEMANGIOMA CAVERNOUS HEMANGIOMA
Superficial hemangioma of skin Deeper or in bigger blood vessel

Capillary hemangiomas:
SALMON PATCH STRAWBERRY (RICH) PORT WINE STAIN
(NICH)
Appears: at Birth Appears: After birth Appear:at Birth
Disappears: by 1 yr Disappears: by 5-7 yrs And never disappears
• M/C site : NECK in
New Bailey ( Errata )
Treatment not necessary Treatment not necessary Treatment needed:
pulsed wave laser
therapy

SURGERY SIXER APP BASED WORK-BOOK 2020 461


Part 2: Plastic Surgery _ Reconstructive surgery

SKIN GRAFT FLAPS


Graft is a piece of tissue without its own blood Flap is a tissue carrying its own blood vessel
supply • Pedicle flap:
• Free flap: micro vascular anastomosis
needed
Partial thickness graft Full thickness graft Axial pattern flap Random pattern flap
• Involves • Involves • Contains • Leash of
epidermis + epidermis+ full named blood unnamed
part of dermis vessel blood vessel
dermis supply

Skin graft: survival


• First 48hrs: Plasma Imbibition

SURGERY SIXER APP BASED WORK-BOOK 2020 462


• 48hrs – 5 days: Inosculation (alignment of Blood vessels)
• >5 days : Revascularisation
Absolute contra indication for skin grafting- Beta haemolytic streptococci present on wound.

Partial thickness graft Full thickness graft


Also called as: Thiersch graft Also known as Wolf graft
Donor site: Donor site:
• Thigh • Behind ear.
• Back
Advantage: Advantages:
• No need to cover donor area • No contracture
• Wider areas can be taken • Cosmetically good
• Repeated grafting is possible from
same site
Disadvantage: Disadvantage:
• Cosmetically ugly • Wider areas not possible
• Contractures present • Donor area needs coverage
Procedure: Procedure:
• Using Humby knife or Electrotome • Manually done
At donor site:
• Punctuate spots & absence of fat
indicates a proper partial thickness
graft is taken

Figure: Humby Knife for SSG

Partial thickness graft


Thiersch graft: 0.006 – 0.012 inches ( Thin Partial Thickness graft)
• Most commonly used
• Thin graft
• High take up

SURGERY SIXER APP BASED WORK-BOOK 2020 463


• Low primary contraction
• Increased secondary contraction
Blair Brown graft: Intermediate 0.012- 0.018 inches
Padgett graft: 0.018 – 0.024 inches ( Thick Partial Thickness graft)
• Thick graft
• Poor uptake
• High primary contracture
• Low secondary contracture

Meshing: Split skin graft


• Size of the graft can be increased
• It helps to drain any fluid

Figure: Partial Thickness Split Skin Graft

Indications of Flap:
Flap is used instead of graft in the following condition
• Exposed Bone (graft can be placed on periostium but not on exposed bone)
• Exposed tendon (graft can be placed on tendon sheath but not on exposed tendon)
• Exposed joint
• Exposed prosthesis or metal plates
• Complex defects
• Radiation necrosis

Axial pattern flap (Based on a named Blood vessel)


• PMMC flap based on Thoraco acromian artery
• DP (Delto pectoral) flap based on Internal Mammary artery
• TRAM flap based on Superior Epigastric Artery or Inferior Epigastric Artery
• DIEP flap (Deep inferior epigastric artery free flap) based on Inferior Epigastric Artery
• PMMC & DP flap used in Oral cavity reconstruction
• TRAM & DIEP flap is used in Breast reconstruction

SURGERY SIXER APP BASED WORK-BOOK 2020 464


Figure: DIEP flap ( Free Flap)

Random pattern flap


• It is based on dermal & subdermal plexus
• Accepted length: Width ratio = 3:1

Types
Types of Random pattern Flaps Figures corresponding
Transposition flap:
• Flap rotated about a pivot
point into adjacent defect

Z plasty:
• A type of transposition flap,
1.7 times elongation of wound
happens in 60 degree
angulation. Used mainly for
burns contracture.

Rhomboidal flap (Limberg flap):


• a type of transposition flap, it
can be used in any place. Very
commonly used in pilonidal
sinus surgery.

SURGERY SIXER APP BASED WORK-BOOK 2020 465


Rotational flap: similar to
transposition flap but differ in that
they are semi circular

Advancement flap:
• Burrow’s triangle is made and
the flap slides forward or
backward.
• Common variants include
Rectangular advancement flap,
V-Y advancement flap &
Abbes flap ( to reconstruct lip
cancer after resection)

Interpolation flap:
• Like rotation flap they rotate
about a pivot point but they
insert into defects nearby area
but not into adjacent donor
site (Eg: thenar flap for finger
tip)

SURGERY SIXER APP BASED WORK-BOOK 2020 466


Classification based on tissue included
• Cutaneous flap
• Fascio cutaneous flap
• Myocutaneous flap
• Osteo cutaneous flap
• Muscle flap + skin graft
• Fascia- fat flap

Types of MUSCULAR FLAPS


Mathes- Nahai classification ( NEET SS)**
• I - 1 vascular pedicle (Eg: Gastrocnemius)
• II - 1 dominant + minor pedicles (Eg: Gracilis muscle )
• III - 2 dominant pedicles (Eg: Rectus abdominus (TRAM flap))
• IV- Segmental pedicle (Eg: Sartorius muscle)
• V- Dominant pedicle + Segmental pedicles (Eg: Pectoralis muscle(PMMC flap))

SURGERY SIXER APP BASED WORK-BOOK 2020 467


Part 2: Plastic Surgery - Cosmetic surgery
Reduction mammoplasty: Principles
• Volume of breast reduced
• Ptosis is decreased
• Anatomical location corrected
• Reposition on nipple/ areola is done maintain blood & nerve supply
• Minimal scar
M/C technique used: inferior pedicle based technique
Other techniques:
• Superior pedicle
• Lateral pedicle
• Central mound
• Superomedial pedicle

Figure: Reduction mammoplasty- Inferior Pedicle Technique


ISSUES in reduction mammography:
• Can be done at any age (breast tissue should not grow for 12 months before planning
surgery)
• >35 yrs mammography is needed before surgery
• Resected tissue should be sent for HPE
• Only one time the procedure allowed

Contraindications:
• Smoking
• H/O recent radiotherapy – surgery is delayed

Complication:
• Lactation breastfeeding affected
• Nipple will have increased sensation
• Wound breakdown
• Hematoma
• Fat necrosis

SURGERY SIXER APP BASED WORK-BOOK 2020 468


Re-Implantation of cut digit / limb
Scenario:
• Cut limb case to M.B.B.S doctor:
• Wash in clear water
• Wrap it in a sterile gauge
• Place it in a plastic bag
• Place the plastic bag in ice box
• Send him to a plastic surgeon within 6hrs
• Reimplantation by plastic surgeon

Order of anastomosis of cut limb / digit:


• B- Bone
• E- Extensor tendon
• F- Flexor tendon
• A- Artery followed by Vein
• N- Nerve
• S- Skin

EXTRA EDGE POINTS


Langer’s lines : Kraissl lines: Borges lines:
• Cosmetic lines • Lines perpendicular to • Lines formed
identified in underlying muscle. Better along the furrows
cadavers cosmetic than Langer’s line in when skin is
living patients relaxed in face

SURGERY SIXER APP BASED WORK-BOOK 2020 469


Dressing materials in surgery
• Alginate dressings: absorptive used for high drainage wounds with heavy exudates
• Foam dressing: also used for high drainage wound, not to be used in non draining wound
• Hydrocolloid dressing: used for bedsores, facilitates autolysis debridement and remove the
dead tissue, not to use in high drainage wounds

Figure: Hydrocolloid Dressings

• Hydrogel dressing: Rehydrates wound bed, not to use in high drainage wounds
• Transparent film dressing: Autolytic debridement, not to use in high drainage wounds
• VAC dressing: Negative suction of -120mm Hg, TOC for bedsores.

Figure: VAC devices

SURGERY SIXER APP BASED WORK-BOOK 2020 470


Chapter: 4 : Paediatric surgery

ABDOMINAL WALL DEFECTS

OMPHALOCELE or EXAMPHALOS GASTROSCHISIS


It is a congenital defect in anterior It is a congenital defect in the abdominal wall
abdominal wall with bowel protruding out with bowel protruding out in angry looking
wards with covering membrane manner with no covering membrane
(peritoneal covering)
Umbilical cord attached to the summit of Umbilical cord attaches to left of the defect**
the membrane
CVS anomalies : Septal defects seen in It can be associated with Intestinal atresia (M/C
20-40% cause for short bowel syndrome in children)
Major Minor • Size of defect <4cm
• Defect <5cm • Defect • Immediate closure
• Small bowel >5cm
protrudes • Small
• Can wait bowel,
and watch Liver
• Good • Difficult to
prognosis close
• Strapping abdomen
wall
• poor
prognosis
• Surgical
closure

INTESTINAL ATRESIA
• Duodenal atresia is M/C Intestinal type of atresia

SURGERY SIXER APP BASED WORK-BOOK 2020 471


DUODENAL ATRESIA
Associated with
• Down’s syndrome
• Annular pancreas
• Malrotation of gut
Types
• Type I: M/C : complete atresia
• Type II: Fibrous cord separating two ends
• Type III: In complete or partial obstruction
• WINDSOCK DEFORMITY: Is a type of duodenal atresia with mucosal web and an intact
muscle wall

Figure: Duodenal atresia types

Presentation
• 80%: Bilious vomiting
• 20%: Non Bilious vomiting, atresia is present proximal to ampulla
Treatment of choice duodeno duodenostomy

JEJUNOILEAL ATRESIA:
• 1 IN 2000 Intra uterine mesenteric ischemia
• Associated with cystic fibrosis
Types
• Type I: Mucosal web
• Type II: Fibrous cord + Intact
mesentry
• Type III: Blind end with V shaped
mesenteric defect
• Type IIIB: Distal bowel with Apple
peel or Christmas tree appearance**.
Distal bowel receives blood supply
from ileo colic artery in distal way
• Type IV: Multiple Sausage shaped
atresias

SURGERY SIXER APP BASED WORK-BOOK 2020 472


Figure: type 3b- Apple peel appearance

Treatment: Resect the atresia and anastomose the normal bowel

CONGENITAL DIAPHRAGMATIC HERNIA

BOCHDALEK HERNIA: MORGAGNI HERNIA


Herniation through Bochdalek space Herniation through larey’s space
M/C on Lt postero lateral side Seen M/C on Rt side antero medially
M/C content stomach M/C content is Transverse colon

• C/F: Of congenital diaphragmatic hernia


Difficulty in breathing (bag and mask ventilation C.I)
Intubate the baby immediately
• X-ray chest: Bowel contents present in the chest
• Ryles tube: present in the chest
• Prognosis depends on: pulmonary hypoplasia
• In 24-48hrs lung expands Honey moon phase: Trans abdominal repair of the hernia

SURGERY SIXER APP BASED WORK-BOOK 2020 473


CLEFT LIP & PALATE
• Isolated cleft lip 15% (rare)
• Isolated cleft palate 40%; common in females.
• Combined cleft lip & palate 45% (m/c seen 1 in 600 births); common in male**

Figure: Types of Cleft lip and palate

Antenatal USG can detect: Cleft lip; cannot detect isolated cleft palate
Cause: Maternal exposure to drugs like
• Phenytoin
• Steroids
• Diazepam
Syndromes associated:
• M/C: PIERRE ROBIN SYNDROME** : Glossoptosis, Retrognathia &Respiratory difficulties
• Downs’s syndrome
• Treacher Collin
• Sprintzner
• Sticker’s

C/F:
• Cosmetic
• Speech problems
• Feeding problems
• Middle ear infection

Complete cleft: cleft is not attached to vomer bone

SURGERY SIXER APP BASED WORK-BOOK 2020 474


Incomplete cleft: cleft attached to vomer bone

LAHSHAL OR THALLWITZ CLASSIFICATION


LAHSHAL ( all Capital )stands for Complete cleft in order of
• Rt Lips
• Rt Alveolus
• Rt Hard palate
• Soft palate
• Lt Hard palate
• Lt Alveolus
• Lt Lips

lahSH phrase means


• Rt Side partial cleft lip, alveoli & hard palate
• Lt Side complete soft palate & Hard palate

KERNAHAN’S CLASSIFICATION: (Given numbers from 1-9 each representing a structure)

SURGERY SIXER APP BASED WORK-BOOK 2020 475


OPERATION
Timing of operation
Cleft lip and Soft palate only:
• Unilateral: 6 Months (Single surgery)
• Bilateral: 4-5Months 1st surgery; 6 Months 2nd surgery
• Only Soft palate defect: 6 Months only one surgery

Combined Soft palate & Hard palate:


• 2 operations: Soft palate operated 6Months; Hard palate 15-18Months

Combined Cleft lip + cleft soft palate + cleft hard palate:


• 2 operations:
• 1st Cleft lip & cleft soft palate operated 6Months;
• Hard palate 15-18Months

Name of surgery
Cleft lip operation Cleft soft palate Cleft hard palate
Millard Furrow’s technique Wardill & Langenbeck
Mulliken
Manchester

Rule of 10: (Old)


• 10 Wks age
• 10 lbs weight
• 10 gm% Hb

Complication after surgery


• M/C: Velopharyngeal incompetency
• Speech problem
• Hearing problem
• Oroantral fistula
• Cupid bow
Secondary problems:
• Facial growth
• Dental problems
• Hearing problems
• Speech problems

SURGERY SIXER APP BASED WORK-BOOK 2020 476


Time of operation
Urology:
• Congenital hernia : As early as possible
• Congenital hydrocele: 2 years
• Hypospadias: 6Months – 1Yr
• Undescended testis: 6 Months – 1Yr
• Phimosis: 1 – 2Yrs

Gastro surgeries
• Congenital diaphragmatic hernia: 24-48hrs
• Extra hepatic biliary atresia: <6 weeks (Kasai procedure)
• CHPS: 4 Weeks (Ramstaed procedures)
• Midgut volvulus: earlies (Ladd’s procedure)
• Umbilical hernia: 5Yrs

Head & Neck


• Cleft lip: 6 Months
• Cleft soft palate: 6 Months
• Cleft hard palate: 16 – 18 Months
• Cystic hygroma: after birth

PAEDIATRIC ONCOLOGY
• M/C cancer in childhood Leukaemia
• M/C solid tumor or 2nd M/C cancer of childhood : Brain tumor (Astrocytoma grade I or
Pilocytic Astrocytoma)
• M/C abdominal tumor or M/C extra cranial solid tumor : Neuroblastoma > Wilms tumor
• M/C malignancy in <1Yr child: Neuroblastoma
• M/C soft tissue sarcoma in children : Rhabdomyosarcoma
• M/C Renal tumor in childhood: Congenital mesoblastic nephroma > Wilms tumor
• M/C paediatric tumor producing mets: Neuroblastoma (Mets to Long bones)

NEUROBLASTOMA
Arises from sympathetic nervous system
M/C from
• Adrenal medulla (38%)
• Paravertebral 30%
• Chest 20%
• Neck
• pelvis

Neuroblastoma in abdomen: ( From adrenal medulla)


• Seen in 1-5yrs of age

SURGERY SIXER APP BASED WORK-BOOK 2020 477


• Clinical presentation as Abdominal mass or Bony metastases, Skin mets, Paralysis
• 70% will have metastases on presentation

Abdominal mass: Neuroblastoma vs Wilms tumor


Favouring Neuroblastoma:
• M/C
• Calcification
• Cross midline
• Invade spinal cord
• Extensive mets
• Elevated VMA

IVP: kidney drooping Lilly appearance

Investigation:
• Urinary VMA elevated
• Urinary Homovanillic acid elevate
• Urinary Dopamine elevated
• Urinary Nor adrenalin elevated
• Accurate diagnosis is possible with CT/MRI, take biopsy
International Neuroblastoma staging system
Treatment
• Low risk patients : Surgery alone
• Intermediate risk: Surgery +Chemo therapy
• High risk : CT + Stem cell therapy (3 Yr survival is < 30% )

SURGERY SIXER APP BASED WORK-BOOK 2020 478


SURGERY SIXER APP BASED WORK-BOOK 2020 479
Chapter 5: Oncosurgery

Part A- General aspects

Bone metastasis
• Most common site of bone involved is: Vertebra**
• After vertebra, 2nd most common site is femur > pelvis > ribs > sternum > humerus >
skull
• Skull is the last common site

Tumor Lysis Syndrome


• Rapid destruction of large number of rapidly proliferating cells**
• MC associated with Burkitt’s Lymphoma, other high-grade lymphomas and Leukemias.
Pathophysiology:
• Rapid turn over of Nucleic acid – releases excess uric acid- Hyperuricemia*
• Increased release of intracellular potassium- Hyperkalemia and arrythmias
• Release of Intracellular phosphate- Hyperphosphatemia
• Phosphate combines with Calcium- Hypocalcemia
• Calcium Phosphate is deposited in renal tubules- Renal failure
• Deranged Oxidative metabolism- Lactic acidosis

Tumor Lysis Syndrome Hypercalcemia of Malignancy


Due to rapid destruction of neoplastic cells Due to Increased calcium release from bone or
due to rapid absorption by Renal tubules
Underlying Abnormalities: Underlying Problem:
• Hyperuricemia • Increased secretion of PTH related
• Hyperkalemia peptide
• Hyperphosphatemia • There will be hypercalcemia on Serum
• Lactic acidosis measurement

SURGERY SIXER APP BASED WORK-BOOK 2020 480


• Hypocalcemia**
• Acute renal failure develops**
MC seen in treatment of Lymphomas and MC seen in Osteolytic secondaries**
leukemias.
Rare with Solid tumors*
Treatment: Treatment:
• Aggressive Hydration • Hydration and Dialysis
• Sodium Bicarbonate for lactic acidosis • Diuresis with Frusemide
• Allopurinol to reduce the uric acid by • Bisphosphonates ( DOC – Zolendronic
(Blocking on Hypoxanthine→ Acid)
Xanthine→ Uric Acid pathway ) • Calcitonin
• RASBURICASE- Recombinant uric • Mithramycin
oxidase – Ex vivo enzymatic • Gallium nitrate
dehydration happens • Steroids
• Hemodialysis- Renal failure

Typhilitis
• @ Neutropenic Colitis @ Necrotising Colitis @ Cecitis @ Ileocaecal syndrome
• Classically seen in Neutropenic patients after chemotherapy with cytotoxic agents.
• MC in children especially having ALL and AML
• Clinical features:
o Patient is immunosuppressed
o Fever+
o Right Lower abdominal tenderness+
o Diarrhea which is Bloody
• Diagnosis made by USG or CECT abdomen which shows thickened wall of cecum

CECT abdomen Shows Typhilitis


• Treatment:
o Most patients improve with medical treatment i.e Antibiotics
• Surgical indication:
o No improvement in 24 hours
o Perforation

SURGERY SIXER APP BASED WORK-BOOK 2020 481


Superior Vena Cava Syndrome:
• Presents with SVC obstruction features+ Severe reduction in Venous return from head
and neck and Upper limbs
• MC cause- SMALL CELL Cancer Lung and SCC lungs
• MC cause in young adults- Lymphoma**
Clinical Features:
• Neck and facial swelling
• Dyspnea
• Cough
• Hoarseness
• Tongue swelling
• Headache
• Nasal stiffness- Epistaxis
• Dizziness and syncope
• Dilated veins over chest wall and neck
• Increased Collaterals over Anterior chest wall
• Cyanosis
• Edema of face, arm and chest
Investigations:
• CECT
• Xray chest- widened Superior mediastinum and more common in right side
Treatment:
• Life threatening Complication – Tracheal obstruction
• Emergency- Surgery for superior mediastinal mass
• Treatment depends on underlying cause:
o Radiotherapy- NSCLC and Metastatic Solid Tumors
o Chemotherapy - Small cell cancer and Lymphoma

SURGERY SIXER APP BASED WORK-BOOK 2020 482


Image Based Question:

Recent AIIMS Question


CHEMOPORT DEVICE

Part B- Soft Tissue Sarcoma ( STS)


• Sarcomas are a heterogeneous group of tumors that arise predominantly from the
EMBRYONIC MESODERM, but also can originate as does the peripheral nervous system
from the ectoderm.
• Incidence:1% in adult cancers ( 7% in children’s cancers)

Most Commons in STS:


• MC Type- Liposarcoma ( Ref.Sabiston Page 762)> Leiomyosarcoma
• MC site- Extremities ( MC in Lower limb > Upper limb) > Trunk> Retroperitoneum> head
and Neck.
• MC Type of Paediatric STS- Rhabdomyosarcoma ( MC in Head and Neck)
• MC Retroperitoneal Tumor- Liposarcoma
• Most common type of STS in Extremities- MFH
• Most common type of STS in Retroperitoneum- Liposarcoma
• Most common type of STS in Viscera- GIST

Tumor Spread:
• STS grows along Facial planes and compresses the soft tissues- and hence results in
Pseudo capsules**
• MC route of spread- Hematogenous
• MC site of Distant mets- Lungs
• MC site of Distant mets in Retroperitoneal sarcoma- Liver> lungs
• Lymphatic mets is very rare
• Lymph node mets is seen in 2-10% cases only

SURGERY SIXER APP BASED WORK-BOOK 2020 483


• LN dissection is performed only in such patients with LN mets.

Histologic types with propensity for lymph nodal metastasis are (SCREAM)
• S - Synovial Sarcoma
• C – Clear cell sarcoma
• E – Ewing’s sarcoma
• R – Rhabdomyosarcoma
• E – Epitheliod Sarcoma
• A – Angio sarcoma

Etiological Factors:
• Germline mutations:
• NF- 1 ( Chromosome 17q)
• Li Fraumeni Syndrome ( P53 mutation @ Chr. 17p)
• FAP- Desmoid Tumors ( APC gene)
• Radiation induced
• Carcinogens- Thorotrast, polyvinyl chloride, and arsenic- Especially Hepatic
Angiosarcoma**
Cahan’s Criteria for Radiation induced Sarcoma:
• Different histology
• Must occur within the irradiated field
• Latency period >4 years
• Second malignancy must histo pathologically be a sarcoma

Example of Radiation induced Sarcoma is Stewart Treves syndrome- But it happens in area
not subjected to Radiotherapy as a complication of Lymphedema after RT to Cancer breast
patients.

Clinical Features of STS:


• MC presentation- Painless mass**
• Size at presentation depends on the location:
o Smaller- In distal extremities
o Larger- in Proximal extremities and Retroperitoneal

Investigations:
• IOC for diagnosing- Core cut Biopsy
• IOC to assess the extent of Soft tissue Sarcoma- MRI**
• IOC to assess the extent of RP sarcoma- CECT**
STS present as Heterogenous mass in MRI and helps to delineate muscle groups, Bone,
Vascular invasion of tumor.

SURGERY SIXER APP BASED WORK-BOOK 2020 484


Treatment of STS:
• Surgery- Wide Local Excision with 2cm – we may even go beyond the Compartment
Boundaries to achieve clear margins.
• Radiotherapy- EBRT or Intensity Modulated Radiotherapy
• Chemotherapy- Doxorubicin+ Ifosfamide

Prognostic factors:
• Most important prognostic factor- GRADE**
• Most important predictor of metastasis- GRADE**
• MC cause of death in STS- Mets**

Factors predicting Better survival:


• Younger age
• Selected histologic type ( fibrosarcoma)
• Lower grade
• Smaller tumour size <5 cm
• Extremity site (** Best prognosis- Extremity STS)

Discussion of Each Type of STS

1. Rhabdomyosarcoma: ( RMS)
• MC type of Paediatric STS
• MC site- Head and Neck ( Parameningeal)> Extremity> GU tract> Trunk
• PM sites (nasopharynx/nasal cavity, the middle ear, the paranasal sinuses, and the
infratemporal fossa/pterygopalatine space) are considered as un favorable factor in RMS
• Types of Rhabdomyosarcoma:
o Embryonal Rhabdo myosarcoma (M/C type overall) 70%
o Alveolar Rhabdo myosarcoma
o Pleomorphic Rhabdo myosarcoma ( MC type in adults)

Clinical Features:
• MC presentation- Mass**

SURGERY SIXER APP BASED WORK-BOOK 2020 485


• Bimodal- 1st Peak 2-5 years; 2nd peak- 15-19 years
• MC route of spread- Hematogenous
• MC site of mets- Lungs**

Figure: Parameningeal RMS**


• Diagnostic cell- Rhabdomyoblasts ( Strap cells)
• Tumor also contains- Tadpole cells** ( White arrow in below image)

• Embryonal Type RMS:


o Embryonal type- Spindle cell variant and Sarcoma Botryoides
o In Sarcoma Botryoides- Tumor cells on HPE resemble like Tennis Racquet

Management and Prognosis:


• Unresectable tumors- Neoadjuvant Chemotherapy is given
• Positive margins on resection- Post op radiotherapy
• Embryonal variant- Favourable
• Alveolar type- Unfavourable
• Favourable primary sites: Non parameningeal( head and neck), Para testis, Orbit and
Vagina
• Unfavourable sites: Extremity, Parameningeal**

2. Dermato fibro sarcoma protuberans ( DFSP)


• Low grade sarcoma
• Recurs locally
• Rarely metastasis
• MC site- Trunk( 50%) > Extremity ( 30%) > head and Neck ( 20%)
• Important DD is Keloid** or Hypertrophic Scar**

SURGERY SIXER APP BASED WORK-BOOK 2020 486


Figure: DFSP
Clinical Features:
• MC presentation- Nodular cutaneous mass
• Pattern of growth – Slow and persistent
• There is radial extension of tumor through Subcutaneous tissue
• The tumor involves both Dermis and Subcutis without penetration into EPIDERMIS**
• Lesion enlarges over many years- Protuberans name given
• Tumor associated with Satellite nodules
• CD34+
• Majority of DFSP displays the t(17;22)(q22;q13) translocation
• IOC- Incisional biopsy
• Treatment- 2-4 cm margin+ underlying Fascia
• First line treatment for advanced and unresectable tumor- Neoadjuvant therapy with
Imatinib**

3. Liposarcoma:
• M/C sites: Nape of neck, thigh, Retro peritoneum
• In the Retro peritoneum it arises from the perinephric pad of fat
• Macroscopically- Huge, soft, well circumscribed
• Histological Types- Well differentiated, Myxoid, Round cell pleomorphic Liposarcoma

Cect shows Retroperitoneal Liposarcoma

4. Malignant Fibrous Histiocytoma ( MFH):


• Old Books used to say this is Most common – Now the MC is Liposarcoma**
• Arises from fibroblastic differentiation
• M/C in Lower limbs followed by Retroperitoneum
• Intermuscular involving muscles
• Firm consistency.
• Around 5 subtypes there.

SURGERY SIXER APP BASED WORK-BOOK 2020 487


5. Synovial Sarcoma:
• Histologic hallmark of soft tissue synovial sarcoma is ‘BIPHASIC MORPHOLOGY’ of tumor
cells (Epithelial like & spindle cell)
• They do not have synoviocytes (misnomer).
• M/C around young age
• M/C arises around Knee joint (85%) and hip but <10 % are intra articular.
• Lymph nodal spread is common

6. Leiomyosarcoma:
• M/C in females
• M/C site: Uterus
• Retro peritoneum
• In Retro peritoneum it arises from the wall of Inferior vena cava.
• In intestine known as GIST* . ( Refer GIT section for GIST)

7. Angiosarcoma:
• Can occur anywhere in body
• Arises from mesenchyme of blood vessels
• H/O chronic lymphedema
• H/O chronic exposure to Arsenic, Thorotrast
• Short duration.
• Rapid increase in size
• Systemic metastasis in common

8. Kaposi Sarcoma:
• Rubbery Bluish nodules seen primarily of the extremities, but may appear anywhere on
skin and Viscera**
• MC in people of East and Africa.
• Usually multifocal rather than metastatic*
• Histologically lesions are composed of capillaries lined by atypical endothelial cells*
• Lesions are locally aggressive with intermittent periods of remission.
• Treatment: Radiation** or Combination Chemotherapy
• Surgical treatment is reserved for Intestinal or Airway obstructions only

SURGERY SIXER APP BASED WORK-BOOK 2020 488


Figure: Kaposi Sarcoma ( Multifocal)

9. AIDS related Kaposi Sarcoma:


• Seen primarily in Male homosexuals and not in IV drug abusers or Hemophiliacs*
• Lesions spread rapidly to nodes and GI and Respiratory tract are also often involved*
• AIDS- KS is associated with Herpes like virus-8 ( HHV-8) infection concurrently.

Radiotherapy and STS:


• Highly sensitive: Synovial sarcoma, Fibro sarcomas, Malignant Fibrous Histiocytomas
• Intermediate: Liposarcoma, Myxoid fibro sarcoma
• Resistant: GIST, Leiomyosarcoma, Chondrosarcoma

SURGERY SIXER APP BASED WORK-BOOK 2020 489


Part 3: Skin Cancers
• Basal cell Ca – Most common
• Squamous cell Ca- 2nd common
• Melanoma

Layers of Skin:
(Mnemonic – Cute Looking Girl Sangeetha Bijilani)

Basal Cell Cancer:


• MC in Men
• MC in Age > 40 Years
• MC site- Face ( inner canthus, Nose and upper lip along tear flow area- Tear Cancer)
• MC in White Skinned individual
• Spreads:
- Direct spread ( MC and only Mode of spread)- Rolled out edge and Rodent ulcer is
the other name.
- LN spread- Not seen as the Cells are large to enter lymphatics
- Hematogenous- Rare ( < 0.01%)
• Wedge Edge Biopsy Taken from the Tumor.
• Biopsy shows: Peripheral Palisade pattern of Cells **

SURGERY SIXER APP BASED WORK-BOOK 2020 490


Figure: Peripheral Palisading cells

• Predisposed by- UV light( Strongest), Arsenic , Coal tar and Aromatic Hydrocarbon
• Types of BCC:
- Nodular ( nodulo cystic): 90%- Usually produces ulceration**
- Superficial spreading type : 10%- Other name- Field Fire type ( Peripheral lesions
with central clearance like a burnt field)- Scaling ++ ( DD: Actinic Keratosis and
Eczema)
• Clinical Features:
- Nodular or Ulcerated lesion in Tear flow area

Figure: BCC in face

Squamous Cell cancer:


• 2nd MC
• M> F
• Arises from Malphigian/ Prickle/ Spinosum layer of skin
• Premalignant lesions:
- Bowen’s Disease: Insitu SCC of Skin/ Shaft of Penis
- Erythroplasia of Queyrat: insitu SCC of Glans Penis
- Paget’s Disease
- Leukoplakia
- Erythroplakia
- Senile keratosis

Figure Shows- Bowen and Erythroplasia of Queyrat

• Risk Factors:

SURGERY SIXER APP BASED WORK-BOOK 2020 491


- Radiotherapy
- Long standing scars or ulcers: Marjolin’s ulcer
- Sunlight
- Xeroderma pigmentosa
- Lupus vulgaris
- HPV 5 and 16
- Prolonged exposure to Irritants**
• Irritants causing cancer:
o Kang cancer- Abdominal wall cancer in Kashmiris keeping hot pot on abdominal
wall
o Kangri Cancer- Buttock Cancer in Tibetans sitting on hot pot
o Chimney Sweeper Cancer- Scrotal cancer

• Clinical features:
o Excavated edge with proliferative ulcer++
o Ulcerative or Ulceroproliferative
o Verrucous type ( Where RT is contraindicated)

• On HPE- Keratin Pearls seen++ ( Onion skin appearance in epidermis)

• Epithelial keratin pearl formation not seen in:


o Rapidly growing tumor
o Esophagus SCC
o Bladder SCC
• The grading system used is BRODER’S Grading system
• Tumor Marker for SCC- Cytokeratin CK 1 and CK 10

Treatment of BCC and SCC:


• Excision is the treatment of Choice**
• Margin given 3 dimensionally is 4-6 mm Margin for > 2cm size and 4mm margin for <
2cm size.
• Larger tumors are given 1cm margin
• Cosmetic areas like face, Penis etc- MOH’S Micrographic surgery: serial excision by
Shaving until the tissue becomes negative.

SURGERY SIXER APP BASED WORK-BOOK 2020 492


Other modalities:
• 5FU cream
• Electro dissection
• Cryosurgery
• C02 Lasers
• Radiotherapy

Malignant Melanoma:
• 4% of skin cancer
• 79% of skin cancer deaths
• 1.4% of all cancer deaths.
Incidence in India
• Male : 0.5 / lakh
• Female : 0.2 / lakh
• Melanoma is the leading cause of death in women 20 – 40 yrs.
• m/c cancer in age group from 20-39 years- malignant melanoma.

Risk Factors:
• Sunlight
• Xeroderma pigmentosa
• Past/family history of melanoma
• Dysplatic naevi( 10% life time risk)
• Immunocompromised- HIV, cyclosporin A therapy, Hodgkin’s disease.

Classical features of Malignant Melanoma: ( ABCDE)


• A Asymmetry
• B Border irregularity
• C Color variation
• D Diameter > 6 mm
• E Evolution, Elevation, Enlargement
• F Funny looking

Discussion of Each types:


• Superficial spreading – Most common
• Acral lentiginous – Most common in dark skin
• Nodular – Worst prognosis
• Lentigo maligna – Best prognosis

Superficial Spreading Melanoma:


• M/C type occurring in white population
• Melanomas arising in preexisting Dysplastic naevus are usually SSM.
• M/C Sites:

SURGERY SIXER APP BASED WORK-BOOK 2020 493


Males: Trunk
Females: Lower legs and back

Nodular Melanoma:
• Second most common
• Can occur on any surface of the body, 50 – 70 yrs
• M / C site: Head, neck and trunk
• Only vertical growth, no radial spread.
• High Risk group and POOR PROGNOSIS**

Acral Lentiginous Melanoma:


• MC in Black people
• MC in white areas – Palms and Soles
• Bad prognosis

Figure: Acral Lentiginous Melanoma

Lentigo Maligna Melanoma:


• Older age individuals.
• Previously called as Hutchinson’s melanotic freckle.
• Common in face**
• Most commonly affects women than men
• Insitu variant is called lentigo maligna.
• Lowest metastatic potential
• Least common
• Least malignant
• Good prognosis

Subungual Melanoma:
• MC below the nail bed
• Hutchinson’s Sign- Pigmentation extending to the nail lateral fold from the nail bed

SURGERY SIXER APP BASED WORK-BOOK 2020 494


Amelanotic Melanoma:
• No pigmentation is seen and hence present late

Tumor markers of Melanoma:


• MELAN – A
• HMB – 45 ( Hydroxy Methyl Bromide)
• S – 100

Mode of Spread
Local spread Lymphatic spread Hematogenous spread
Vertical spread is more Nodal spread is common Brain
dangerous Nodules may be seen Lungs
MC with Nodular type • Nodules along the way of Liver
spread: Small intestine
• < 2cm from primary Skin
lesion: Satellite Nodules** Bones
• >2cm from primary lesion: MC cause of death is
In transit nodules** metastasis

Figure: Satellite and In-transit nodules

Prognostic factors:
• Depth ( most important)
• Ulceration
• Lymph node status
• Satellite lesions
• Distant mets
Based on
• Site- Lower extremities ( good)
• Sex- female( good)
• Histology- lentigo maligna(good)

SURGERY SIXER APP BASED WORK-BOOK 2020 495


Breslow Thickness Clark’s level
Based on thickness of invasion by optical Based on layer of invasion
micrometer
• Level I -Only in Epidermis
• I : Less than 0.75 mm • Level II-Into papillary dermis
• II : Between 0.76 to 1.5mm • Level III- Up to the junction.
• III : 1.51 mm to 4 mm • Level IV - Into reticular dermis
• IV : Greater than 4 mm • Level V -Extension into subcutaneous tissue

Treatment of Malignant Melanoma:


• Excision biopsy with preferred margins
• Sentinel node biopsy
• Block dissection if sentinel node is positive
• Reconstruction of defects.

Isolated Hyperthermic Limb Perfusion therapy

• For Extremity Melanoma and


Sarcomas
• The artery and Vein of the lower limb
containing the Tumor with satellite /
intransit nodules is isolated by
clamping.
• The chemotherapy ( Usually
Melphalan) is given with high
temperature as infusion and the blood
is removed and reinfused after
oxygenation only into that limb.
• Systemic toxicity is thereby avoided

Margin Excisions:
• Lesion 0-1 mm thick ( Depth)- 1cm margin
• Lesion 1-2 mm Thick- 1-2 cm margin
• Lesion 2-4 mm Thick- 2cm margin
• Lesion > 4 mm thick – 2cm margin

SURGERY SIXER APP BASED WORK-BOOK 2020 496


Chapter: 6 Cardiothoracic surgery

Topic 6a Thorax

Mediastinum parts
• Superior mediastinum (Thyroid mass, parathyroid mass, thymoma)
• Inferior mediastinum
• Anterior mediastinum
• Middle mediastinum
• Posterior mediastinum

M/C mediastinal mass Thymoma (Actual correct answer is neurogenic tumor)**

Anterior mediastinal masses


• Thymoma (M/C)
• Lymphoma
• Germ cell tumor
• Thyroid mass
• Parathyroid tumor
• Mesenchymal tumors

Middle mediastinum masses


• M/C lesion CYST (pericardial cyst, bronchogenic cyst)
• Aneurysms
• Lymph Node masses
• Ectopic pheochromocytoma

Posterior mediastinum masses


• Neurogenic tumors (M/C)
• Meningocele

SURGERY SIXER APP BASED WORK-BOOK 2020 497


• Mesenchymal tumors
• Pheochromocytoma
• Lymphoma

THYMOMA
• M/C neoplasm of thymus
• M/C location : Anterior mediastinum
• Seen in 40-60yrs age group
• It is surrounded by well defined fibrous capsule
Types of thymoma
• Lymphocytic
• Epithelial
• Mixed (M/C)
C/F:
• Usually a symptomatic, Usually detected on X-ray chest as a an incidental finding
• Dysplasia
• Dyspnoea
• SVC Syndrome
• Paraneoplastic syndrome (M/C: Myasthenia gravis)

MUSCLE related PNS Blood related PNS Orthopaedic related PNS


• Myasthenia gravis • Cytopenia • RA
• Neuro muscular • Red cell aplasia • SLE
dystrophy • Hypogammaglobulinemia • Polymyositis
• Myotonic dystrophy • Erythrocytosis • Sarcoidosis

Investigations:
• Tissue biopsy is needed to confirm thymoma (D.D Lymphoma)
• Elevated Cytokeratin is the best marker

IOC : CT scan
• On CT : Lymphoma – associated lymph node enlargement is present along with the mass
• On CT: Thymoma – well defined solitary lesion

Masoaka staging system


o I: Macroscopic well encapsulated lesion, no microscopic invasion
o II: Well defined capsule, infiltrated into fat & pleura, microscopic invasion present
o III: Macroscopic invasion into lungs, pericardium & great vessels
o IV a: Pleural & Pericardial mets

SURGERY SIXER APP BASED WORK-BOOK 2020 498


o IV b: Lympho / Hematogenous mets present

Management
• Total thymectomy performed through sternotomy for symptomatic cases
• Large thymoma >5cm Thymectomy + chemo
• Myasthenia gravis : Thymectomy + Anti cholinesterase drugs
• VATS thymectomy (Recent)

BRONCHOGENIC CYST
• Congenital lung lesion due to anomalous development of foregut or trachea bronchial tree
• It can arise in two places
• Mediastinum (M/C 85%)
• Pulmonary parenchyma (15%)
Mediastinum Pulmonary parenchyma
No trachea bronchial communication Tracheobronchial communication +
M/C location Rt lung lower lobe**

• It can rupture resulting in pneumothorax


• It can compress the lung tissue ATELECTASIS
• It can go for infection
• Risk of malignant transformation
• HEMOPTYSIS is seen If communicates with tracheobronchial tree
Treatment
• Excision of cyst with VATS method

LUNG SEQUESTRATION:
• It is a normally developed lung tissue containing bronchioles, alveoli and receiving blood
supply from aorta**
• M/C in lower lobes, Left more common than Right.
• 2 types
Intra lobar sequestration Extra lobar sequestration
Sequestration within the lung parenchyma Sequestration outside the lung
parenchyma
75% (M/C) 25%
Also known as Rokitansky lobe**
Has communication with tracheobronchial No communication with tracheobronchial
tree tree
It can get infection Associated with congenital diaphragmatic
Abscess hernia
haemoptysis
M/C in Lt lung

SURGERY SIXER APP BASED WORK-BOOK 2020 499


Drains 100% of its blood into pulmonary Drains blood into
vein** • Azygous vein 80%
• Pulmonary vein 20%

CHYLOTHORAX
• TG level in fluid >110mg/dL (1.2mmol/L)
• It is a complication of surgery or malignancy
Management:
• Initially managed by conservative means
• Fat free diet
• Medium chain TG
• Protein diet
• Fat free TPN

Indication for surgery:


• No improvement after 5days of conservative management
• Chyle loss > 1500ml
• Non heeling for >2 Weeks

Surgery: Ligation of thoracic duct in POIRER TRIANGLE

Boundaries of POIRIER’s TRIANGLE:


• Vertebral column
• Arch of aorta
• Lt subclavian Artery

SURGERY SIXER APP BASED WORK-BOOK 2020 500


In this triangle the thoracic duct drain into the junction of Lt Subclavian vein +Lt Internal jugular
vein

LUNG TUMORS
BENIGN MALIGNANT
• M/C : Pulmonary Hamartoma • Bronchogenic cancers
• Pulmonary Hamartoma: • Malignant mesothelioma (it arises
• M/C in males from pleura)
• X-ray: Popcorn calcification
• M/C located peripherally

BRONCHOGENIC CANCER:
• M/C primary malignant tumor in mediastinum
• M/C seen in Lung Hilum
• M/C visceral cancer
• M/C cause of death due to cancer
Risk factors:
• Smoking
• Air pollution
• Asbestos , nickel exposure
• Scars in lungs – can lead to adeno cancer
• Radiation
Mutations:
• K ras (M/C in Adeno cancer)
• Bcl 2
• Cmyc

Pathological types:
SMAL CELL CANCER (OAT CELL) NON SMALL CELL CANCER
• Adeno cancer
• SCC
• Large cell cancer

SURGERY SIXER APP BASED WORK-BOOK 2020 501


• Broncho alveolar cancer (it spreads by
air , satellite lesions are present)
• High risk of metastasis present • Less risk of metastasis
• Poor prognosis • Good prognosis
• Response to chemotherapy is excellent • No response to chemotherapy
• M/C type Adenocarcinoma
• M/C type among smokers is SCC
• Undifferentiated type is large cell
cancer
Increased Paraneoplastic syndromes are Low risk of producing Paraneoplastic
present** syndromes
Usually centrally located ** Peripherally located (SCC is centrally
located)**
High chances of metastasis Less risk of metastasis
• Brain
• Bone
• Liver
• Adrenal

M/C location of tumor is Right lung upper lobe**


C/F of Lung cancer:
• M/C is Cough
• Dyspnoea
• Chest pain
• Hemoptysis (Source of Hemoptysis is BRONCHIAL ARTERY)**
• Malignant pleural effusion
• Clubbing is present (Resolves after tumor excision)
• Hoarseness + (RLN involvement)
• SVC obstruction (SCC)
• Eaton lambert syndrome Seen in small cell tumors

SURGERY SIXER APP BASED WORK-BOOK 2020 502


PANCOAST TUMOR
A tumor located at the apex of lung
Causes destruction of 1st & 2nd ribs
Causes atrophy of hand muscles
Causes Horner’s syndrome
• Ptosis
• Miosis
• Anhidrosis
It is usually SCC**

SQUAMOUS CELL CANCER of lung


• M/C cancer in India : SCC
• M/C among smoking males: SCC
• Pancoast tumor is: SCC
• Usually centrally located**
• It can form cavity inside the tumor
• It has the best prognosis
• PNS: It produces ectopic PTH – Hypercalcemia, Hypophosphatemia & Clubbing**
• Treatment: surgical excision of tumor

ADENOCANCER:
• M/C type world wide
• M/C in females
• M/C peripherally located
• It produces lung to lung mets
• Good prognosis (not as good as SCC)
• NOGUCHI CLASSIFICATION is used in olden days (now TNM is used)
• PNS: DIC, Hypertrophic pulmonary osteo arthropathy & migratory
thrombophlebitis(Trousseau’s syndrome)

SURGERY SIXER APP BASED WORK-BOOK 2020 503


SMALL CELL/ OAT CELL CANCER
• Most aggressive cancer
• Worst prognosis
• Responds to CT & RT
• M/C type causing extra thoracic metastasis
• PNS M/C: produces
• Ectopic ADH – SIADH - Hyponatremia
• Ectopic ACTH – Hypocalcemia & Cushings sundrome
• Myasthenic Eaton Lamber syndrome
• Cerebellar degeneration

LARGE CELL CANCER


• PNS: Galactorrhea in females & Gynecomastia in males

Paraneoplastic syndrome (Non Metastatic Complication)


Small cell cancer Squamous cell cancer Large cell cancer Adeno carcinoma
• Excess ADH • Ectopic PTH • Galactorrhea • Hypertrophic
secretion causes production in females pulmonary
SIADH and causes: • Gynecomastia osteo
hyponatremia Hypercalcemia in males arthropathy
• Ectopic ACTH & hypo • DIC
causes hypo phosphatemia • Migratory
kalemia and • Clubbing in all thrombophlebit
Cushing syndrome. non small cell is(Trousseau’s
• Retinal blindness cancer syndrome)
• Myasthenic Eaton-
Lambert syndrome
• Cerebellar
degeneration

T staging of bronchogenic cancer (NEET 2020 TOPIC)


T1 Tumor size </= 3 cm
Involving lobar bronchus
Not involves main bronchus

• T1a: < 1 cm
• T1b: 1-2 cm
• T1c: 2-3 cm
T2 Tumor size 3-5 cm
• Main bronchus involved (but Carina not involved)
• Atelectasis, obstructive pneumonitis present
• It involves pleura (Visceral)

SURGERY SIXER APP BASED WORK-BOOK 2020 504


T3 Tumor size 5-7 cm
Carina not involved
It involves
• Parietal pleura
• Chest wall
• Phrenic nerve
• Parietal pericardium
T4 Tumor size >7 cm
• Satellite tumor nodules in IL lobe
• Infiltration into esophagus, trachea, heart, vertebra
Carina is involved

Non invasive investigations for lung cancer Invasive investigations

• Chest X-ray • Bronchoscopy transbronchial needle


• CT thorax: 1st investigation to be done biopsy
for suspects • Endobronchial USG and biopsy
• Assess for lung Resectability • CT guided biopsy
• Any node >2cm in mediastinum is • Chances of pneumothorax
metastatic node • Done for peripherally located tumors
• PET- CT
• Sputum cytology

Surgical methods of taking Biopsy


Mediastinoscopy: Mediastinotomy: VATS:
• Under GA a transverse incision is • Incision is made in • Latest
made over supra sterna notch 2nd ICS • Staging is
• A scope is passed through the incision • Mediastinal nodes done
• Biopsy is taken from Sub carinal biopsy can be based on
nodes & para tracheal nodes taken (Lt Para VATS
• Pneumothorax & Bleeding can aortic nodes biopsy
happen in this procedure can be taken)

Management of Lung Cancer


• Surgical excision (Best)
• Chemotherapy given only for small cell cancer (90% regression is seen)
• N2 cases: Neoadjuvant CT is advised
• Adenocancer: Oral Tyrosin Kinase Inhibitor : Geftinib**

SURGERY SIXER APP BASED WORK-BOOK 2020 505


Types of resection:
Segmentectomy and Wedge resection
• For small tumors
• If each segment is removed anatomically ligating the segmental Pulmonary artery and
Vein and bronchus it is segmental resection.
• Non anatomical resection is wedge resection

Lobectomy:
• Treatment of choice in Early Stage Lung cancer**
• Can be performed via Thoracotomy or VATS.
• Bronchus is stapled or sewn.
• One or two ICD drain left and removed later.

Pneumonectomy:
• Removal of whole lung and has highest mortality- 5-8%
• Procedure reserved for centrally placed tumor* or tumors involving Main bronchus.
• Fixation of tumour to aorta, heart or Esophagus means unresectable.
• Ligate the main Pulmonary artery, Superior and inferior Pulmonary veins and Main
bronchus divided without Stump.
• Drainage of space is important- No suction is applied. Only ICD drain kept and leave it
unclamped or unclamp it for 1 minute for every 1 hour, Slowly the fluid level within
the space rises and fills the pneumonectomy space.

Prognosis: 5 yrs survival


• SCC: 35-50%
• Adeno cancer: 25-45%
• Small cell: 5%

CHEST WALL ANOMALIES


Pectus excavatum (Figure A) Pectus carinatum (Figure B)
Disc shaped deformity Pigeon shaped deformity @ Pigeon chest
Open operation: RAVICH procedure
Minimally invasive Surgery: NUSS
procedure

SURGERY SIXER APP BASED WORK-BOOK 2020 506


THORACOSCORE: (NEET 2020 TOPIC)
• It is the most widely used model to assess risk of operative mortality in thoracic surgery
• It is done preoperatively
It is based on 9 variables

Mnemonic “ASAP- Plan Doctor CME”


• Age
• Sex
• American society of anaesthesiologist score
• Performance status
• Priority of surgery
• Dyspnoea score
• Co morbidity score
• Malignant diagnosis
• Extent or duration of surgery

One liners from Thoracic Surgery:


• MC histological type overall- Adenocarcinoma
• MC type in Non-smokers- Adenocarcinoma
• MC type in females- Adenocarcinoma
• MC type in smokers-Squamous cell carcinoma
• MC type in India- Squamous Cell carcinoma
• MC type associated with SVC syndrome Small cell carcinoma
• Peripherally located tumor-Adenocarcinoma
• Noguchi Classification -Adenocarcinoma
• Central Located- Small cell and Squamous cell
• Pancoast tumor - Squamous cell type
• Paraneoplastic Syndromes are MC with Small cell cancer
• Worst prognosis- Small cell cancer
• M/C site of metastasis is Liver.
• M/C endocrine organ involved is –adrenal
• Lung Ca is M/C tumor metastasizing to heart

SURGERY SIXER APP BASED WORK-BOOK 2020 507


• Brain metastasis may present with neurological deficit.
• CA lung is M/C primary for metastasis to Kidney, Esophagus, Pancreas, Spleen, Liver,
Adrenal, Brain, Skin

Image Based Questions:


Image 1: Nuss Procedure done for Pectus excavatum

Image 2: Heimlich Valve used in Pneumothorax

Topic 6b: Cardiac surgery

Cardio Pulmonary Bypass Machine:


• It oxygenates blood, Remove particles, maintains temperature and circulation (Except to
Heart)

Figure: Cardio pulmonary Bypass machine*

SURGERY SIXER APP BASED WORK-BOOK 2020 508


Procedure to connect Cardio pulmonary bypass machine
• Sternotomy
• Expose
• Arterial cannulation : Ascending aorta
• Venous cannulation: purse string in Rt atrium

Machine is going to take care of entire circulation of body except myocardium.


To prevent myocardial damage:
1. Infuse cardioplegic solution into Coronary artery (potassium containing fluid) it arrests the
heart in diastole and hence commonly used
2. Intermittent cross clamping is performed usually heart develops Ventricular fibrillation
3. In paediatric cases total circulatory arrest: myocardium can with stand ischemia for 15-
20min

CPB working complications


• Bleeding disorders
• Infections
• Air embolism
• GIT: mesenteric ischemia
• Myocardial ischemia
• Neurologic dysfunction
• Pancreatitis
• Vascular injury
• MODS

ACUTE CORONARY ISCHEMIA (M.I)


Blood supply of heart
• Lt coronary artery: it gives 2 branches Lt anterior descending artery that runs in the
anterior interventricular groove and Lt circumflex artery that runs in A.V Septum
• Rt coronary artery: it gives Rt circumflex artery that ends as Posterior descending
coronary artery in 90% and in 10% LCA gives posterior descending artery

SURGERY SIXER APP BASED WORK-BOOK 2020 509


Three vessels that supply heart:
• Lt anterior descending artery (LAD)
• Lt circumflex artery
• Posterior descending coronary artery

• M/C diseased artery: Lt Anterior Descending artery


• M/C bypassed artery: Lt Anterior Descending artery

IOC for coronary occlusion: Selective coronary angiography ( See the video in app)

CABG indications
• >50% stenosis of Lt Coronary Artery
• >70% stenosis of LAD (anterior interventricular artery)
• Triple vessel disease
• Poor ventricular function

Chronic stable angina:


• PTCA (Percutaneous Transluminal Coronary Angioplasty) > CABG
Acute coronary syndrome:
• Unstable angina: becomes asymptomatic with antianginal drugs if recurrent unstable
angina PTCA / Thrombolysis.
• CABG done after 6 weeks

Other indications:

SURGERY SIXER APP BASED WORK-BOOK 2020 510


Myocyte necrosis (VSD):
• Patient presents in 3-7days with pulmonary edema+ pansystolic murmur+ hypotension.
• Treatment : patch closure

Papillary muscle necrosis: (Mitral Regurgitation)


• Presents with pulmonary edema + pansystolic murmur.
• Treatment: Mitral valve replacement

Ventricular aneurysm:
• Can happen due to necrosis, wall of ventricle bulges out.
• Repaired after connecting the heart to Bypass machine

BYPASS OPERATION
Commonly used vessels
• Long saphenous vein
• LIMA (Lt internal mammary artery)

Long saphenous vein LIMA ( Lt internal mammary artery)


M/C used vein For LCA & LAD bypass we use only LIMA
10 years patency 60% 10 years patency is 90%
Other veins: Other arteries
• Short saphenous vein • Rt internal mammary artery
• Cephalic vein • Radial artery (Allen test is done before
taking radial artery)
Wound infection in diabetic patients

SURGERY SIXER APP BASED WORK-BOOK 2020 511


Complications of bypass operation:
• Bleeding
• Arrhythmia (M/C Sinus tachycardia)
• Neurological dysfunction
• Poor cardiac output
• Mortality 2-3%
• Wound infection
Prognosis:
• 95% living at 1 year after operation
• 90% after 5 years
• 75% after 10 years

OFF PUMP CORONARY ARTERY SURGERY: OCTUPUS DEVICE is used without Bypass machine
usage

VALVE REPLACEMENTS:
TYPES OF VALVES AVAILABLE
MECHANICAL VALVE BIOLOGIAL VALVE
Can be used in any age group for any valve • Allograft from Cadavers
Available in many sizes • Autograft fom same person
• Hetero(Xeno) graft: M/C used
Porcine valves
Advantage: long durability Advantage: Turbulence is less no
anticoagulant needed
Disadvantage: Thrombogenic, hence lifelong Disadvantage: Less life expectancy
anticoagulants needed

SURGERY SIXER APP BASED WORK-BOOK 2020 512


• M/C complication using a valve is thrombus formation**
• Thrombus formation M/C in mechanical valve
• Thrombus formation M/C in Mitral valve
• Valve leaks
• Prosthetic valve endocarditis- (MC organism- Staph epidermidis) Mortality 50%

TUMORS:
• M/C cardiac tumor is metastasis**
• M/C primary tumor of heart Atrial Myxoma
• M/C organ causing metastasis is lung cancer
• The primary which has highest propensity of metastasis to heart is from Malignant
Melanoma skin**

ATRIAL MYXOMA:
It is common in 3rd to 6th decade among females
SPORADIC FAMILIAL
As a part of Carney’s complex
M/C in Lt atrium M/C found in Ventricle
Usually solitary Usually Multiple

• Clinical feature is most commonly like MS or MR


• Ventricular forms present as Obstructive features
• Tumor plop sound heard – Mid diastolic
Complication:
• Emboli like manifestation can happen if it disconnected
Investigation: 2D Echo or TEE(Trans Esophageal Echo)
Treatment: Surgical excision of myxoma

SURGERY SIXER APP BASED WORK-BOOK 2020 513


SURGERY SIXER APP BASED WORK-BOOK 2020 514
Chapter 7: Endocrine Surgery

Topic 7A- MEN syndromes

MEN-1 ( WERMER Syndrome)


• Truncated menin protein mutation**
• Autosomal Dominant
• Male and Female= Equally affected
• Manifests from age of 5 years**
3 P’s affected
- Parathyroid Hyperplasia
- Pituitary adenoma ( Prolactinoma MC)
- Pancreas NET( Gastrinomas MC)

Clinical features:
• PHPT- 90% ( Multiglandular hyperplasia)
• 2nd MC- Pituitary NET (Gastrinoma MC in Duodenum > Pancreas)- 30-80%
• Prolactinomas ( 15-30%)
• Carcinoids in Thymus and Bronchus**
• MC cause of death in MEN 1 is from Duodenal/ Pancreatic NET or Intra thoracic
Carcinoids

Hyperparathyroidism in MEN-1
• MC manifest in MEN -1
• Due to multiglandular hyperplasia**
• MC in 2nd decade*
• 1st Biochemical abnormality noted is Hypercalcemia*
• Enlargement is asymmetrical**

Treatment : 2 options:
• Total Parathyroidectomy with heterotopic implantation of one parathyroid in
Brachioradialis muscle
• Subtotal ( removal of 3 and half glands)

SURGERY SIXER APP BASED WORK-BOOK 2020 515


MEN 2 Syndrome
Hall mark – C cell Hyperplasia and Medullary cancer is always seen ( 100%)
Autosomal Dominant
RET oncogene mutation :
❑ Loss of Function: Hirschsprung disease**
❑ Gain of Function: MEN -2 Syndromes
Somatic Mutation: Thyroid Cancers

MEN 2A (Sipple Syndrome) MEN 2B


• Medullary thyroid cancer- 100% • Medullary carcinoma thyroid- 100%
• Pheochromocytoma- 40-50% • Pheochromocytoma- 40%
• Parathyroid adenoma/ hyperplasia- • Intestinal ganglioneuroma
20% • Mucosal neuromas in Lips- 100%
• Hirschsprung disease** • Megacolon- 100%
• Cutaneous lichen amyloidosis • Marfanoid features
• Ganglioneuromas in intestines
• Esophageal Dysmotility

Other Tumors:
• Medullary cancer- 100%
• Phaeo chromocytoma- 40-50%
• MEN 2A develops PHPT – in 20% cases- Multiglandular more common
• MEN 2B never develops PHPT
• MEN- 2B develops in very young age, at infancy**
• MEN- 2B never develops Hirschsprung disease**

SURGERY SIXER APP BASED WORK-BOOK 2020 516


Familial Medullary Thyroid MEN2A MEN 2B
Cancer( FMTC) (Sipple Syndrome) (Also known as MEN 3
Syndrome)
RET oncogene RET oncogene. RET Oncogene
Exon 618 mutation* Exon 634 mutation Exon 918 mutation

Medullary cancer in MEN-2 Syndrome:


• Medullary cancer is sporadic in 75% cases and occurs in MEN syndrome as Hereditary
case*
• Hereditary MTC is Multifocal, Bilateral**
• MTC will not take up RAI**
• Survival is 70% at 10 years.
• 75% have node involvement if tumor is palpable.

Phaeochromocytoma in MEN-2 Syndrome:


• Neoplasms arising from Chromaffin cells in Adrenal medulla*
• 40-50% of MEN 2A and 2B develop Pheochromocytoma.
• Age at diagnosis is 30-40 years
• Unlike Sporadic cases which may be 10% malignant and 10% Extra adrenal- these cases
associated with MEN-2 Syndrome is Benign always and will be limited to adrenal always.
• MEN-2 related cases will be Bilateral and Multifocal**
• Surgery advised recently is Adrenalectomy for the involved side only and follow up.
• Previously bilateral adrenalectomy was done which resulted in adrenal insufficiency and
hence not done nowadays.

MEN 2 Syndrome Screening and prophylactic thyroidectomy:


• 1st Degree relatives should be counselled and screened for RET oncogene mutation.
• If 918 mutation ( MEN-2B) is seen- High risk- Prophylactic thyroidectomy < 1 year
• If 634 mutation ( MEN- 2A)is seen- Low risk, Prophylactic thyroidectomy @5-6 years

MEN -4 Syndrome:
• Mutation of CDKN1B( Cyclin Dependent kinase Inhibitor) gene on Chromosome 12.
• Parathyroid adenomas
• Pituitary adenomas
• Reproductive organ tumors (Testis, Cervix)
• Renal Tumors

SURGERY SIXER APP BASED WORK-BOOK 2020 517


Topic 7B- Parathyroid

Anatomy of Parathyroid Gland:


• Superior Parathyroid – From 4th Pharyngeal pouch ( Constant in position)
• Superior Parathyroid present in a Fixed position behind RLN near Zuckerkandl
• Inferior Parathyroid- From 3rd Pharyngeal pouch ( Variable in position)
• Inferior Parathyroid present anterior to RLN in inferior pole MC ; But can be highly
variable from Carotid to mediastinum
• Weight of gland- 30 gm ( Portland Brick Yellow brown color)
• PTH is 84 amino acid peptide
• Half life – 3-5 minutes
• PTH acts on Bone, Kidneys and GIT.
• Normal Calcium level: 2.2 – 2.7 mmol/litre (or) 8.5 to 10.5 mg/dl.
• Ionized calcium- 4.4 to 5.5 mg/dl

Calcium and Phosphate Homeostasis:


❑ PTH-Increased Calcium absorption & Increased Phosphate excretion from Kidney**
❑ Calcitonin- Inhibits absorption of Both Calcium and Phosphate**in kidney
❑ 1,25 Dihydroxy Vitamin D ( Calcitriol) – Absorption of Calcium and Phosphate from
Kidney, Intestines and Bones.

Hyperparathyroidism
• Primary- Due to intrinsic abnormality
• Secondary- Physiological increased secretion in response to lowered calcium
• Tertiary – Advanced form of secondary HPT in which in which autonomous secretion of
PTH occurs despite correcting underlying abnormality like renal failure and also seen in
Renal transplant cases**

Primary Hyperparathyroidism

• 5th – 6th decade, MC in females ( 3:1 ratio).


• Most are asymptomatic.
• MC presentation is Kidney stones. ( 15-20%)
• 40% patients will have Hypercalciuria**

Etiology:
• MC cause is Solitary adenoma**
• Multi gland disease (Hyperplasia)- 15% - Associated with MEN syndromes
Sporadic
Familial:
❑ MEN 1
❑ MEN 2A

SURGERY SIXER APP BASED WORK-BOOK 2020 518


❑ HPT- Jaw Tumor Syndrome

Clinical features:
• Salt and pepper Skull appearance**

Figure: Salt and pepper Skull

• Brown Tumor:
❑ Brown tumour, also known as osteitis fibrosa cystica and rarely as osteoclastoma
❑ Excess Osteoclast activity resulting in Subperiosteal bone resorption.

Figure: Brown Tumors

Bones- Stones- Abdominal Groans- Psychic Moans


❖ Others: Hypertension, LVH
❖ Muscle weakness, Fatigue
❖ Soft tissue calcifications

SURGERY SIXER APP BASED WORK-BOOK 2020 519


Hypercalcemic crisis can also happen** (When Serum Calcium> 14 mg/dl)- IVF with NS
at rate 200 ml/hour+ Start Steroids, Calcitonin, Bisphosphonates ; Do
parathyroidectomy as early possible

Diagnosis:
• Elevated Total and Ionised Calcium levels
• Increased PTH levels.
• Low serum Phosphate ( in the presence of Normal Vitamin D and Creatinine Levels)
• 24 Hour urinary calcium excretion may be normal or increased.
• Elevated alkaline phosphatase.

Localisation studies:
• Sestamibi Nuclear scan is the most accurate and reliable method to localise the
adenoma**
• Parathyroid adenomas have high concentration of Oxyphilic cells with high mitochondria
content**.
• False positive can happen with Hurthle cell nodules which also have high oxyphil content.
• This can be reduced by Tc99 Pertechnetate – Subtraction images.

Other investigations:
• MIBI scan with USG showing concordant image is obtained.
• 4D CT Scanning
• MRI- Not commonly used.
• Selective Parathyroid Angiography and Venous sampling.

SURGERY SIXER APP BASED WORK-BOOK 2020 520


Surgical Management:
Guidelines for Surgical management:
Age • < 50 years
Serum Calcium • 0.25 mmol/L ( 1.0 mg/dl) above upper limit
Skeletal • BMD (Bone marrow density) by DEXA scan,
• T score -2.5 at Lumbar spine, Total Hip, Femoral
Neck or Distal 1/3rd Radius, Vertebral Fracture
Renal • Creatinine clearance- < 60 ml/min
• 24 hour Urinary calcium> 10 mmol/ dl( > 400
mL/day)
• Increased risk of stone formation by risk analysis
method

Surgical options:
• Minimally invasive Focussed Parathyroidectomy (MIP)- by an incision less than 3 cm in
neck (or)
• Bilateral Neck Exploration operation with removal of all 4 glands or 3 and half glands

Figure: Minimally invasive Parathyroidectomy

SURGERY SIXER APP BASED WORK-BOOK 2020 521


Indications for Bilateral Neck Exploration:
MIAMI Criteria
• Drop in PTH to normal range and to less than half of the maximum per operative PTH at
10 minutes appears to accurately predict the Single Gland Disease.
• If no such drop is detected suspect multiglandular disease or Hyperplasia and plan
Bilateral Cervical Neck Exploration.

Bilateral cervical Exploration:


Indications:
• Discordant images
• MEN 1
• MEN 2 Syndromes
• Lithium induced Hyperparathyroidism
• Miami criteria failed
• Multi gland disease

Procedure:
• 3 and Half glands removed with leaving back of half gland in left inferior or normal
looking gland.
Mark it with non-absorbable suture. Preferred gland to be left is Inferior Parathyroid as
RLN is posterior to it and marked with TITANIUM Clips**
• Alternately all 4 glands are removed and small pieces of parathyroid are sutured into
pockets of Brachioradialis muscle.
• Thymectomy is also advised in MEN 1 associated HPT as there is chances of Supernumery
glands on Thymus.

Newer techniques:
• MIVAP- minimally invasive Video assisted Parathyroidectomy- Cervical, Axillary, Post
auricular and Transthoracic approaches
• Robotic Technique

Post op complications:
• Permanent Hypocalcemia (Need for Vit.D and Calcium > 1 year postoperatively)- <0.5%
• Recurrent Laryngeal Nerve damage- <1%
• Persistent Hyperparathyroidism – Elevated Se. Calcium within 6 weeks of surgery
• Recurrent Hyperparathyroidism- Increase in Calcium levels after 6 months with a
intervening period of Normal Calcemia.

SURGERY SIXER APP BASED WORK-BOOK 2020 522


Medical treatment:
• Only for unfit and patients for whom Surgical treatment is contraindicated.
• Bisphosphonates
• Hormone replacement therapy (Better than SERM)
• Calcimimetics (Cinacalcet) – Amplify the sensitivity of calcium sensing to extracellular
calcium, altering the set point and thereby decreasing PTH production.

Special Cases of Hyperparathyroidism:


• Lithium induced HPT
Familial Syndromes:
• Familial Isolated HPT
• MEN 1
• MEN 2 Syndrome
• HPT – JT syndrome ( Jaw Tumor Syndrome)- Cystic Nature of Adenomas, Risk of
Parathyroid cancer is there.
• Familial Hypocalciuric Hypercalcemia ( Mutation of Calcium Receptor gene – CASR gene
on Chromosome 3)- No need of Surgical Intervention in this case. Low urinary calcium /
creatinine clearance ratio is used to diagnose.

Secondary Hyperparathyroidism
Derangement in Calcium Homeostasis with compensatory increased PTH:
• Chronic kidney Disease ( Renal HPT)
• GI malabsorption
• Vitamin D deficiencies
• Liver disease
• Lithium usage

Pathogenesis:
• Abnormalities in renal Tubular absorption of phosphate leads to Hyperphosphatemia,
which acts on Parathyroid cells and stimulates PTH secretion.
• This increased Phosphate leads to reduction of 1,25 Dihydroxy Vitamin D which acts by
reducing the intestinal absorption of calcium, also acts to increase secretion of PTH.
• Lab values shows:
o Increased Phosphate,
o Increased PTH,
o Decreased or Normal Calcium level** and
o low Vitamin D**

Investigations:
• Traditional Ostetitis Fibrosa Cystica not seen in Plain Xray
• Bone densitometry ( DEXA Scan)- Osteopenia and Osteoporosis.
• Localisation studies not needed as there is no plan of Minimally invasive surgery.

SURGERY SIXER APP BASED WORK-BOOK 2020 523


Casanova Test:
• In cases of recurrent disease after bilateral cervical exploration, when there is no active
evidence of disease in neck and a previous allograft has been kept in forearm, a selective
venous sampling for PTH from side of allografted neck or Brachial vein is useful.
• To prove there is graft hyperplasia in forearm the ratio must be greater than 20: 1

Calciphylaxis:
• Calcific uremic arteriolopathy seen in 4% patients undergoing surgery for Secondary HPT.
• There is vascular calcification and skin necrosis
• Cutaneous purpuric lesions+, Ischemic necrosis, Gangrene, Sepsis and death.
• Poor prognosis

Management of Secondary HPT:


• Renal Transplant is TOC.

Other therapies are Bridge to this:


• Replacement of Calcium and Vit D
• Phosphate Binders
• Calcimimetic drugs ( Cinacalcet)- make a set point and inhibit the stimulation of PTH

Surgical indications:
Essential components Clinical findings
• Persistently High serum level of Intact One of the below symptoms:
PTH > 500 pg/ml • Ostetitis Fibrosa Cystica
• Hyperphosphatemia (Phosphate > • Subjective symptoms like Bone pain,
6mg/dl) or Hypercalcemia ( Ca>2.5 arthralgia, muscle weakness,
mmol/L or 10 mg/dl) which is irritability , pruritus and Depression
refractory to medical treatment • Ectopic calcifications
• Estimated Volume of largest gland > • Calciphylaxis
300-500 mm3 or Longitudinal axis > • Progressive reduction in bone mineral
1 cm content.
• Anemia resistant to Erythropoietin
stimulating agents
• Dilated cardiomyopathy and cardiac
failure

SURGERY SIXER APP BASED WORK-BOOK 2020 524


Surgery:
❑ Subtotal Parathyroidectomy – procedure of Choice
With 3 and half removal and half Remnant left in situ with marked non absorbable
suture
Or
Parathyroid morcellated and inserted in Brachioradialis pocket.
Dramatic response is seen to surgery

Tertiary Hyperparathyroidism
Persistent Autonomous Nodule causing Hyper calcemic Hyperparathyroidism after Kidney
Transplant

Biochemical analysis:
• Elevated Ionised and total Calcium
• Elevated PTH
• Reduced Phosphate
Occurs after 1 year of Transplant…

Management:
• No indication of calcimimetics.
• Definitive treatment: Subtotal parathyroidectomy or Total Parathyroidectomy with Auto
Transplantation are acceptable surgical options.

Indications of Surgery in tertiary HPT:


• Subacute Severe Hypercalcemia ( > 3 mmol/l)
• Impaired Graft function
• Nodular Hyperplasia of Parathyroid glands
• Progressive symptoms ( >2 years following Transplantation)
- Worsening bone disease ( pain/fractures/ Bone loss)
- Renal stones
- Soft tissue / vascular calcifications

Recurrent Hyperparathyroidism:
MC cause is Ectopic Parathyroid
MC site- Paraesophageal> mediastinal> Intra Thymic

Parathyroid Cancers
Incidence:
• 1% if PHPT cases.
• 0.005% of all cancers.
• H/o neck irradiation is only risk factor.
Genetic factors:

SURGERY SIXER APP BASED WORK-BOOK 2020 525


• HPT- JT syndrome.
• HRPT 2 mutation- Parafibromin inactivation.

Differential Diagnosis from PHPT adenoma


• Diagnosis one decade earlier
• Equal gender ( M:F=1:1)
• Symptomatic at presentation.
• Palpable neck mass is seen in 36-52% cases.
• Increased Calcium level
• PTH increased (5-10 times of normal range)

Immunohistochemistry:
• Downregulation of Parafibromin +ve
• Protein Gene Product ( PGP 9.5)+ve

Treatment:
• Complete removal of tumour + avoiding spillage is the Treatment of choice

Topic 7C- Adrenal Gland


Anatomy:
• Adrenal Gland Pyramidal Shape Right Side, Crescent shaped on Left Side
• 4 gm each
• Tumors arising from Chromaffin cells of Adrenal Medulla – Pheochromocytoma
• Tumors arising from Chromaffin cells Extra adrenal- Paraganglioma
Substances produced:
• Adrenal Medulla secretes- Adrenaline > NA, DA
• Zona Glomerulosa secretes Aldosterone
• Zone Fasciculata/ Reticularis secretes Cortisol and DHEA (Precursor of Androgen)

SURGERY SIXER APP BASED WORK-BOOK 2020 526


Cushing Syndrome

Clinical Features:

• Hypertension
• Diabetes
• Hyper lipidemia
• Depression
• Amenorrhea
• Impotence
• Lemon on stick appearance
• Buffalo Hump
• Stria++
• Moon Facies
• Increased Hair Growth

Diurnal Variation of Cortisol:

SURGERY SIXER APP BASED WORK-BOOK 2020 527


• ACTH peaks in Early morning at 4am to 10 am… Hence Cortisol values will be high at
this time normally.
• We have to see for Diurnal Variations.
• It is not altered in “Pituitary Source of Cushing’s Disease” and lost in Adrenal Source.

High Dose Dexamethasone Test:


• It’s a routine when Steroids are injected immediately the Levels of ACTH will decrease.
• Such Decrease of ACTH by High Dose Dexamethasone is seen in “ Pituitary source of
Cushing disease” only.

SURGERY SIXER APP BASED WORK-BOOK 2020 528


Post op management:
• After unilateral adrenalectomy – the contralateral adrenal gland will be suppressed and
hence supplemental Cortisol is given.
• Opposite gland takes 1 year to recover fully.
• Before stopping Cortisol Supplements- Synacthen Test done ( ACTH Stimulation test,
Synacthen a, drug that acts like ACTH ) is given** This test confirms the normal secretion
of Cortisol by the adrenal gland.

• Conn’s Syndrome- Primary Hyperaldosteronism; Hypertension due to increased


Aldosterone**, Hypokalemia Associated. MC cause is solitary adenoma*
• Waterhouse Friderichsen Syndrome- Bilateral Adrenal Infarction seen in Meningococcal
sepsis- Rapidly fatal complication.

Adrenocortical Carcinoma:
• Rare
• Bimodal- Children and 4th decade
• F:M= 1.5:1
• MC functional
• Adrenocortical carcinoma- 60% present with Cushing Syndrome**
• Imaging: MRI >CT
• Mc Farlane Classification is used**
Treatment- ENBLOC RESECTION
• Post op- Etoposide, Doxorubicin, Cisplatin
• Hormone Excess- Ketoconazole, Mitotane are used.

Phaeochromocytoma
Catecholamine producing tumor from Adrenal medulla and Sympathetic chain.

Extra – adrenal Phaeo chromocytomas:


• MC site is Para Aortic region (75%)
• Thoracic
• Bladder
• Neck and Pelvis

“10% TUMOUR” is the other name of Pheochromocytoma:


▪ 10% extra-adrenal (closer to 15%)
▪ 10% occur in children
▪ 10% familial (closer to 20%)
▪ 10% bilateral or multiple (more if familial)
▪ 10% recur (more if extra-adrenal)
▪ 10% malignant

SURGERY SIXER APP BASED WORK-BOOK 2020 529


▪ 10% discovered incidentally
Syndromes associated with Pheochromocytoma:
o MEN 2A and 2B
o Familial Paraganglioma Syndrome
o Von Hippel Lindau Syndrome
o Sturge weber syndrome
o Neurofibromatosis 1

Features suggestive of malignancy


PASS Score- Phaeochromocytoma Adrenal Glans Scale score
• Increased Pheochromocytoma Adrenal Scale Score
• High ki- 67 Positive cells
• Vascular Invasion
• Breached Capsule
• Metastasis

Clinical features:
The five P’s:
◦ Pressure (HTN) 90% - MC manifestation
◦ Pain (Headache) 80%
◦ Perspiration 71%
◦ Palpitation 64%
◦ Pallor 42%
◦ Paroxysms (the sixth P!)-
The Classical Triad seen in >50% Cases : Pain (Headache), Perspiration, Palpitations

Hormones produced by Each site:


• Most Pheochromocytoma produce NA and Adrenaline with NA> Adrenaline*
• Extra adrenal Pheochromocytomas will secrete only Noradrenaline*
• Pheochromocytoma associated with MEN secrete only Adrenaline
• Malignant Pheochromocytoma secretes- Dopamine and Homovanillic acid(HVA)

Paroxysmal Spells:
• 10-60 minute duration
• Daily to monthly once
• Spontaneous
• Precipitated by various factors like Contrast injection, Drugs, Exercises, Micturition etc

Carney’s Triad:
* Functioning Paraganglioma+ GIST+ Pulmonary Chondromas

SURGERY SIXER APP BASED WORK-BOOK 2020 530


Investigations for Phaeochromocytoma:
• Screening Investigation is 24 hours urinary VMA, Urinary Catecholamines like meta
nephrines, normetanephrines.
• Plasma metanephrines is IOC to diagnose.
• IOC to Localise the Tumor is MRI**for Adrenal, Extra Adrenal Phaeochromocytoma in
Pregnancy. (EVEN for ECTOPIC IOC is MRI)
• Iodine 123 MIBG ( met Iodo Benzyl Guanidine) scan is used only for metastatic and
multiple Extra adrenal tumors.
• But now FDG PET and DOPA PET has more sensitivity to detect mets**

Figure: Swiss Cheese Pattern on MRI in Phaeochromocytoma

Preop preparation:
• Phenoxybenzamine ( Alpha Blocker)- 20 mg to 160 mg daily until patient develops
Postural Hypotension.
• Propranolol ( Beta blockers)- for patients with tachycardia and arrythmias ( Must be
started after complete Alpha Blockade happened)
• Surgical management of Choice is Laparoscopic Adrenalectomy**
• Anaesthesia of Choice- Enflurane or Isoflurane** (Don’t use Halothane**)

Malignant Phaeochromocytoma:
• Hypertension controlled first
• Mitotane- Palliative treatment for Mets cases
• Chemotherapy-Vincristine, Cyclophosphamide and Dacarbazine

SURGERY SIXER APP BASED WORK-BOOK 2020 531


Chapter 8 – Neuro Surgery ( ELECTIVE)

Topic 8a-Basic Neuro Anatomy and Benign Topics in CNS

CSF PATHWAY
❖ CSF secreted by choroid plexus in 2 lateral ventricles
❖ CSF flows through MONRO foramen into the 3rd ventricle
❖ CSF flows down through AQUEDUCT OF SYLVIUS into the 4th ventricle
❖ CSF from 4th ventricle enters the sub arachnoid space through
o Medial opening: FORAMEN MAGENDIE
o Lateral opening: FORAMEN LUSCHKA
❖ CSF from sub arachnoid space absorbed into dural venous sinuses
❖ From dural venous sinuses into systemic circulation.

HYDROCEPHALUS

• High pressure in CSF: C/F: Headache that increases on coughing & bending. Associated
with Vomiting, Nausea+ vision problem
• Low pressure in CSF: C/F: Headache that increased on standing**

SURGERY SIXER APP BASED WORK-BOOK 2020 532


Infants: High pressure CSF
C/F:
• Bulging Fontanelles
• Papilledema
• PARINAUD SYNDROME: Dorsal midbrain compression (due to increased pressure)-
Upward gaze affected
• Results in Sunset sign

Figure: Sun Set Sign

Types of Hydrocephalus
OBSTRUCTIVE HYDROCEPHALUS COMMUNICATING HYDROCEPHALUS
Is due to compression by lesion • Is due to failure of absorption of CSF
• Inside • Post hemorrhagic
• outside • CSF infection
• Increased protein content
LP is contra indicated LP can be done
• 10-30ml removal of CSF decreases
the symptoms

Normal pressure Hydrocephalus Idiopathic Intracranial Hypertension (IIH)


• Important cause of dementia, it is seen • Also know as Benign Intracranial
in old men. Hypertension & Pseudotumor
• Triad in Normal pressure cerebri**
Hydrocephalus • It rapidly progress to Blindness
o Gait disturbance • Common in young over weight females
o Incontinence Characterized by
o Cognitive decline • Head ache
Etiology : • Visual disturbance
o Denovo • Papilledema
o SAH • Cranial nerve dysfunction

SURGERY SIXER APP BASED WORK-BOOK 2020 533


o Meningitis LP : pressure >25mm Hg (High pressure)
o Head injury Reasons for IIH:
o Tumors • Sinus thrombosis
CT scan: Ventriculomegaly • Increased intra abdominal pressure in
LP: Normal Pressure obese people
Treatment:
• Weight loss
• Stop OCP usage
• Acetazolamide
• Shunt operations: prevents visual loss
o Ventriculo peritoneal shunt
o Lumbar peritoneal shunt
• Optic nerve sheath fenestration: done
to prevent visual loss

Management of Hydrocephalus:
• Hydrocephalic attack is an emergency- Rapid coma & death can happen
• Immediate external ventricular drain should be placed: through coronal suture, catheter
tip is placed near foramen of monro.
• Lumbar drain can be done for communicating hydrocephalus

Figure: External Ventricular Drain

Types of Internal
Shunts
• Ventriculo
Peritoneal
shunt
• Ventriculo
plural shunt
• Ventriculo
atrial shunt

SURGERY SIXER APP BASED WORK-BOOK 2020 534


• Ventriculo
atrial and
Ventriculo
pleural Shunts
• Lumbar
Peritoneal
Shunts is done
in
Communicating
Hydrocephalus
and IIH

Ventriculo peritoneal Shunt:


• Starts from lateral ventricle to
abdominal cavity

• Anti siphon system is present


• Shunt valve -low/medium/high
• CSF reservoir is present in
shunt(sampling of CSF can be done)

Complications of shunt:
• Over drainage of CSF : patient develops low pressure headache on standing
• Lateral or 3rd Ventricles can collapse, leads to bleeding in ventricles, sub dural hematoma
& subdural hygroma can happen
• SLIT VENTRICLE SYNDROME:
o Seen in Children with ventricles and subarachnoid spaces are underdeveloped,
resulting in poor brain compliance.
o In these patients normal fluctuations in ICP are exaggerated so that coughing and
straining may cause symptoms of raised ICP.

SURGERY SIXER APP BASED WORK-BOOK 2020 535


o Any shunt blockage may not be evident on scan, as the ventricles fail to enlarge.

• Obstruction
• Disconnection
• Infection: Caused by Staph epidermidis
Risk factors for Infection:
o M/C seen in young patients
o Open meningo-myelocele
o Long operation time
o Increased movement of persons in and out of OT
o 75% develop in 1st month
o 15-20% needs shunt replacement in 3yrs
Treatment of Infection:
o Start IV antibiotics
o Remove the shunt & do External CSF drainage
o LP done- If protein content is Normal it indicates infection is subsided then
place a new shunt

ENDOSCOPIC VENTRICULOSTOMY
• Aqueduct stenosis: a Shunt is passed through neuro endoscope from 3rd to 4th ventricle
• Serious complication: Basilar artery/Fornix can be injured that results in permanent
memory impairment

BRAIN ABSCESS
Causes
• Infected from direct exposure (Trauma to head)
• Hematogenous infection
• 25% No cause is found

Causative organism
• If infected from Mastoiditis: Streptococci & Bacteroids
• If infected from Hematogenous route: Streptococci & Bacteroids
• Open infection (Trauma): Staph aureus & Clostridia
• Food born : Toxoplasma & Neurocysticercosis

Infections causing brain abscess in HIV patients


• Toxoplasma
• Cryptococcal
• JC virus – Multifocal Leuco Encephalopathy
• Mycobacterial

C/F:

SURGERY SIXER APP BASED WORK-BOOK 2020 536


• Low grade fever
• Seizers
• Confusion
• FND

INVESTIGATIONS:
• CRP elevated(all inflammatory markers elevated)
• Blood culture +
• IOC :CECT Brain

CECT of evolving abscess (Cerebritis stage): CECT of classical abscess: Smooth walled well

Hypodense lesion with edematous brain defined hypodense ring enhancing lesion

• Important DD is Tumor in brain


• Do an MRI to confirm the tumor
• T2 weighted on MRI Hyper intense
lesion

• Mortality is 4% in brain abscess


• If abscess ruptures into ventricles that leads Ventriculitis with mortality of 80%

Treatment: of brain abscess


• TOC: Surgical drainage of abscess with 6 weeks antibiotics
• Prophylactic anticonvulsants

Subdural abscess:
• Sinusitis, mastoiditis & meningitis can cause subdural abscess
• Frontal bone osteomyelitis (Pott’s puffy tumor) can cause subdural abscess

SURGERY SIXER APP BASED WORK-BOOK 2020 537


• Subdural abscess is present in parafalcine region that can lead to cortical veins thrombosis
with 8-12% mortality
Treatment of subdural abscess
• Drainage of abscess by + IV antibiotics
• Burr hole
• Craniectomy
• Craniotomy
• LP is contraindicated

INTRACEREBRAL HEMORRHAGE (ICH)

SUBARACHNOID HEMORRHAGE
• M/C cause : Trauma
• Spontaneous SAH
o 80% Berry aneurysm rupture
o 20% AV malformation

Perimesencephalic SAH:
• A distinct subgroup of SAH patients suffer bleeds confined to the basal cisterns anterior to
the midbrain and pons, without an underlying lesion evident on angiogram.
• Represents venous bleeding**
• Excellent prognosis

Berry aneurysm:
• Berry aneurysm that leads to SAH is seen in 10-15/lakh population
• C/F: Thunder clap headache + LOC (sudden)

Predisposing factors
• Females
• Hypertension patients
• Smokers
• Cocaine
• Family H/O berry aneurysm
• ADPKD
• Marfan syndrome
• Ehler danlos syndrome
• Fibro muscular dysplasia
Site of berry aneurysm
• M/C Anterior Communicating Artery
• 2nd common Middle Cerebral Artery
• 3rd common Posterior Cerebral Artery
• Least common : Basilar tip

SURGERY SIXER APP BASED WORK-BOOK 2020 538


Clinical features:
• 10-15% die on spot
• 1/3rd hospitalized people die
• 1/3rd develop neurological deficit
• 1/3rd Recover

Complications:
• Most common cause of death after reaching hospital- Re bleeding
• Hydrocephalus- External ventricular drain must be done.
• Vasospasm ( To be prevented by Nimodipine, Induced Hypertension or Hypervolemia)
• Terson Syndrome- Combination of SAH and Vitreous Hemorrhage ( in 10-20% cases)
World federation of neurological surgeons grading
Grade Glasgow coma scale Focal deficits
I 15 -
II 13-14 -
III 13-14 +
IV 7-12 +/-
V 3-9 +/-
• Focal deficit = dysplasia or limb weakness

CLINICAL GRADING: HUNT & HEES SCALE- Gives idea on outcome


Grade Features Outcome
1 Minimal headache+ 70% recover
Nuchal rigidity+
2 Moderate to severe headache + 70% recover
Nuchal rigidity ++
Cranial Nerve palsy +/-

SURGERY SIXER APP BASED WORK-BOOK 2020 539


3 Drowsy + Confused +FND 15% recover
4 Hemiparesis** 15% recover
+ Stupor +Decerebrate rigidity
5 Coma**+ 0% recover
Moribund

Fisher grading on SAH based on CT scan


• IOC: CT Scan : Hyper dense signal is present in sub-arachnoid space

• Xanthochromic CSF on taping (Not done in recent times)


• Cerebral angiography can be done in a stable patient
• Digital Subtraction Angiography is Gold standard investigation for aneurysm.

Management :
• Medical: Delayed Ischemic neurological deficit(3-10 Days) should be prevented ( due to
vasospasm)
• Oral Nimodipine
• TRIPLE H Therapy**
H- Hypervolemia
H-Hypertension
H-Hemodilution
• Surgical Clipping & Coiling of aneurysms

SURGERY SIXER APP BASED WORK-BOOK 2020 540


AV MALFORMATION
• Can bleed and cause SAH
• Seen in 10-20% cases of SAH
• Congenital in origin
• Risk of bleeding - 4% / Year
• AVM should be intervened if size is >2.5cm
• By any one technique :
o Embolisation of AVM
o Surgical resection of AVM
o Stereotactic surgical resection

SPETLER MARTIN FRADING System is used in the management of AVM**


Factors Score
Size <3 cm 1
3-6cm 2
>6cm 3
Location Eloquent 1
Non-Eloquent 0
Vein Superficial 0
Deep 1

Other causes of SAH


• HTN
• Ischemic stroke

SURGERY SIXER APP BASED WORK-BOOK 2020 541


Image Based Questions:
Moya Moya Disease:
• Auto immune disease
• Internal carotid artery is obliterated
• Multiple collaterals are formed from
External carotid artery and give that
puff of smoke appearance
Treatment: Superficial temporal artery to
middle cerebral artery bypass.

Intracerebral hemorrhage:
C/F: Contra lateral hemiplegic
Mortality : 40% in 1 year
• Cause
Mainly due to HTN
Patients on Anticoagulants
Amyloid angiopathy patients
Treatment:
1.Stop anticoagulants
2.Craniotomy and evacuation of hemorrhage
can be done in fit patients

Topic 8 B- Brain Tumors


Presentation of brain tumor: TRAID
• Seizure
• Increase ICP
• FND

Syndromes associated
• Neuro Fibroma 1 : cause Astrocytoma, Schwannoma, Optic Nerve Glioma
• Neuro Fibroma 2 : causes Meningioma, Astrocytoma
• P53 mutation: Glioma
• TURCOT syndrome: Medulloblastoma
• HNPCC(Lynch 2 Syndrome): Astrocytoma
• MEN1: Pituitary adenoma
• VHL gene: Hemangioblastoma of brain
Based on location of the Tumors in various lobes: Presentation
• Pituitary tumors: Bitemporal Hemianopia

SURGERY SIXER APP BASED WORK-BOOK 2020 542


• Occipital lobe tumors: Homonymous Hemianopia
• Anterior skull base tumors (Olfactory group tumors):
o FOSTER KENNEDY SYNDROME**: Ipsilateral optic atrophy & Contralateral
Papilledema + Anosmia
• Parietal lobe tumors:
o GERSTMANN SYNDROME: Acalculia, Agraphia, Lt/Rt disorientation, Finger
agnosia
• Frontal lobe tumor: Gait changes; personality changes; urinary incontinency

METASTATIC BRAIN TUMORS:


• M/C brain tumor: Mets from Lungs 40% > Breast 15% > Melanoma10% > Renal 10%
• M/C brain tumor in females Mets from Breast cancer**
• M/C brain tumor in Males/over all : Mets from Lungs cancer (Small Cell Cancer)
• Lung mets usually go to cerebral hemisphere
• Breast mets usually go to leptomeninges
• IOC for brain mets : MRI with Gadolinium Contrast

SURGERY SIXER APP BASED WORK-BOOK 2020 543


Management of brain mets:
• DOC: STEROIDS
• Single met (Up to 3) : operated
• Whole Brain RT can be given
• Stereotactic Radiosurgery can be done with Gamma knife or cyber knife

PRIMARY BRAIN TUMORS

M/C primary tumor of brain


• Children: Grade I Astrocytoma (Pilocytic)
• Adults: Grade IV Astrocytoma(Glioblastoma Multiforme)
Adults:
• 80% of tumors Supratentorial
• 20% of tumors Infratentorial
Children:
• 40% of tumors Supratentorial
• 60% of tumors Infratentorial

SURGERY SIXER APP BASED WORK-BOOK 2020 544


Brain tumors arising from Glial cells:
• Astrocytes: Astrocytoma
• Oligodendrocyte: Oligodendroglioma
• Ependymal cell: Ependymoma

Other brain tumors:


• Meningioma:
o Tumor arising from meninges
o 2nd M/C tumor
o It is the M/C extra axila tumor (Vestibular schwannoma is other axial
tumor)

Other Malformations seen in brain


• Craniopharyngioma
• Dermoid cyst

Pituitary tumors
• Neuro Endocrine Tumor : Medulloblastoma

ASTROCYTOMA: (Mnemonic -NINE)


Grading based on 4 criteria:
• Nuclear atypia
• Increased cellularity (mitoses)
• Necrosis
• Endothelial proliferation

4 grades
• Grade 1: Pilocytic Astrocytoma
• Grade 2: Diffuse Astrocytoma – only one criteria(Nuclear atypia)
• Grade 3: Anaplastic Astrocytoma- two criteria (Nuclear atypia+ Mitoses)
• Grade 4: Glioblastoma Multiforme – three criteria (Nuclear atypia+ Mitoses + Endothelial
proliferation or necrosis)
Grade 1 & 2 Low grade tumors / Benign tumors;
Grade 3 & 4 High grade tumors/ Malignant

SURGERY SIXER APP BASED WORK-BOOK 2020 545


ASTROCYTOMAS
Grade 1 : Pilocytic Astrocytoma
• Characterised by Mural Nodule
• No infiltration
• MC in Children.
• Best prognosis among the 4**
• MRI- IOC
• Discrete Contrast Enhancing Cystic
lesion with a MURAL NODULE**
• It is curable

Grade 2 : Diffuse Astrocytoma


• MC in Children and young adults.
• Look for P16 and CDK N2A
mutation- If present it may convert
them to high grade tumors**
• IOC is MRI
• Treatment- Surgery followed by
Whole brain RT as it diffuse

Grade 3: Anaplastic Astrocytoma


• High Grade Astrocytomas.
• Presents with Seizures, personality
changes if Frontal lobe lesion.
• MRI- irregular enhancing lesion
• Treatment- Surgery and Whole brain
Radiotherapy.
• It has infiltrating margin and hence
RT is mandatory.
• Only Cytoreductive surgery is done

Grade 4: Glioblastoma Multiforme


• MC in elderly, MC presentation is
seizures.
• On HPE- Shows Tumor Necrosis,
Pseudo palisading pattern seen .
• MRI shows Ring enhancing lesion with
edema and mass effect and central
necrosis.

SURGERY SIXER APP BASED WORK-BOOK 2020 546


• Surgery + Post op RT +
Temozolomide is the Chemo-therapy
of Choice**

Grade 4 is high grade and can extend into opposite hemisphere also and hence known as
BUTTERFLY tumors**

Figure: GBM- Butterfly shaped

OLIGODENDROGLIOMA
• Chr 19q mutation
• Calcifications + in the tumor

Figure: Calcifications in Oligodendroglioma

Histopathological examination:
• Fried egg appearance
• Chicken wire Blood vessels
• Micro calcifications

SURGERY SIXER APP BASED WORK-BOOK 2020 547


EPENDYMOMA
• C/F hydrocephalus
• Spreads via CSF
HPE:
• Cells arranged as Pseudo rosette appearance

• Treatment: Surgery + Craniospinal irradiation

MEDULLOBLASTOMA
• Neuro endocrine tumor of brain
• Associated with TURCOT Syndrome
• 3 – 4 Years children
• M/C location Infratentorial or Cerebellum
• Drop mets into the CSF is seen
• Tumor also spreads to LN, Liver & Bones
• Most radio sensitive tumor among the brain tumors
• Since it has Drop mets into CSF : Craniospinal irradiation is advised
• Chemo agents: CARMUSTINE+ VINCRYSTINE
• CHANG STAGING
o M1: CSF mets
o M2: Intracranial tumor beyond primary
o M3: tumor spreading subarachnoid space
o M4: mets outside CSF (distant metastasis)

SURGERY SIXER APP BASED WORK-BOOK 2020 548


MENINGIOMA
• Tumor arises from meninges
• Calcifications present
• Meningiomas are associated with NF1 & NF2
• M/C in females
• Progesterone receptor +ve: increased growth during pregnancy
C/F:
• Headache
• Seizures
• FND

Findings in CECT
• Calcifications +
• Dural tail +
• Sunray spicules +
• Increased vascular markings

HPE: Psammoma Bodies seen (Due to dystrophic calcification)


Good prognosis
Recurrence

Psammoma Bodies:
- Papillary cancer of Thyroid
- Papillary variant of RCC
- Serous cystadenoma Ovary
- Meningioma
- Prolactinoma

SURGERY SIXER APP BASED WORK-BOOK 2020 549


Figure: Meningioma resection

SIMPSON GRADING SYSTEM: to know the recurrence percentage


• Grade I: Dura mater and abnormal bone Removed; Recurrence < 10%
• Grade II: Dura mater is cauterized; Recurrence 15%
• Grade III : Dura mater is left as such; Recurrence 30%
• Grade IV: incomplete resection; Recurrence 80 – 100 %
• Grade V: Biopsy; Recurrence 80 – 100 %

CRANIOPHARYNGIOMA
• It arises from Rathke’s pouch
• Classical finding: Suprasellar calcification + Polyuria
• Seen in children

Features:
• Hypo pituitarism
• Diabetes insipidus
• Growth retardation
• Visual abnormality: Bitemporal Hemianopia
• Sleep alteration

Figure: Suprasellar calcification


Treatment trans sphenoidal resection
PITUITARY ADENOMA
• M/C arise in anterior lobe
• M/C associated with MEN I syndrome
• M/C cause of Hyper pituitarism is Pituitary Adenoma**

SURGERY SIXER APP BASED WORK-BOOK 2020 550


• M/C Presentation is Bitemporal Hemianopia
• They present in 3rd &4th decade of life
• Both sexes equally affected
• M/C functional pituitary adenoma Prolactinoma**- Galactorrhea and Amenorrhea in
females; Gynecomastia in Males
• IOC: MRI
• Treatment: trans sphenoidal excision**

M/C suprasellar mass


• Children Craniopharyngioma
• Adult: Pituitary adenoma
SPINAL TUMORS
• M/C tumor Mets (Intradural & extra medullary)
• M/C primary spinal tumor: Nerve sheath tumor
• M/C intra medullary tumor: Astrocytoma

NEURAL TUBE DEFECT


• It can be due to Genetic or folic acid deficiency or drugs (Trimethoprim, Phenytoin &
Phenobarbitone, Sodium valproate)
Types of defects:
• Spina bifida occulta
• Spina Bifida Apperta:
o Meningocele: Sac of meninges (Dura + Arachnoid)
o Meningomyelocele: Sac of meninges (Dura + Arachnoid) containing Spinal cord or
Nerves

Spina bifida occulta


• Congenital absence of spinous process without exposure of meninges or neural tissue
• Tuft of hair
• No neurological abnormality
• IOC: X ray

SURGERY SIXER APP BASED WORK-BOOK 2020 551


Meningocele
• Sac of meninges (Dura + Arachnoid) covered by skin
• If not covered by skin continuous CSF leak is present
• M/C location Lumbar region
• Tethering of spinal cord
• C/E Transillumination +ve, Fluctuation +ve
• X ray: Opened vertebra

Figure: Meningocele

Meningomyelocele
• Sac of meninges (Dura + Arachnoid) containing Spinal cord or Nerves
• M/C seen in lumbo sacral region
If present in lumbo sacral region :
• Bladder incontinence
• Bowel incontinence
• Paraplegia

SURGERY SIXER APP BASED WORK-BOOK 2020 552


If present in thoracic region
• Hydrocephalus +
• FND +
Associated problems in meningo myelocele:
• Hydrocephalus is 80% ( MC in Thoracic defect)
• Chiari 2 Malformation
• Bladder and Bowel Dysfunction

Treatment:
• Repair in 24 – 48 hrs (if delayed they develops CNS infection),
• Myofacial closure is advised.
• Don’t use latex gloves
• Post op 25% mortality
• Low IQ & bladder problems inspite of recovery

Chiari malformation:
Cerebellar Tonsil Herniation through Foramen magnum >5m
is Chiari malformation

Type I: Tonsil descent alone; occipital headache aggravated


on coughing & Syringomyelia, Numbness +

Type II: Tonsil descent + Meningomyelocele; Associated with


Hydrocephalus & Syringomyelia

Dandy Walker malformation


• Cystic expansion of 4th ventricle
• Results in Cerebellar hypoplasia

SURGERY SIXER APP BASED WORK-BOOK 2020 553


C/F:
• Microcephaly
• Hydrocephalus
• Ataxia
Triad is
• Enlarged posterior fossa
• Rotation of vermis
• Hypoplasia of vermis

HERNIATION OF BRAIN
• Uncus of temporal lobe will be compressed that causes ipsilateral oculomotor nerve
compression = Fixed dilated pupil; Contralateral hemiplegia
• Kernohan notch phenomenon: Ipsilateral hemiplegia** due to corticospinal tract of
opposite side is compressed

Craniosynostosis
Craniosynostosis: is premature fusion of one or more cranial sutures, preventing growth
perpendicular to fusion

Types of Craniosynostosis:
• Scaphocephaly: Sagittal suture fuses early; Boat shaped head
• Brachycephaly : coronal suture is fuses early; Broad flat forehead
• Plagiocephaly : Unilateral lambdoid and Unilateral coronal suture fuses early; Asymmetric
head
• Trigonocephaly: Metopic suture fuses early; Pointed forehead

SURGERY SIXER APP BASED WORK-BOOK 2020 554


Epilepsy surgery:
• M/C performed surgery for epilepsy is anterior temporal lobectomy
• Complex partial seizures usually starts from temporal lobe
• Intra op grid electrode is placed area of high activity is removed (Temporal lobectomy)
• WADA test is done before performing temporal lobectomy.
• WADA TEST: used to locate which temporal lobe is dominant, inject sodium amytal into
each ICA one side at a time & check for speech and memory if impairment is seen that
indicates that side of temporal lobe is dominant

SURGERY SIXER APP BASED WORK-BOOK 2020 555


Chapter 9 - Transplantation

Topic 9a – Introduction

• August 13th – organ donation day every year

The 1st
• Kidney transplantation is done by MURRAY in 1954
• Liver transplantation is done by STARLZ in 1963
• Heart transplantation is done by BARNARD
• Pancreas transplantation is done by KELLY & LILLEHEI

Organ donation: is the process of surgical removing of organ or tissue (Graft) from one person
(Donor) and placing it into another person (Recipient) legally. It is performed by Transplant
surgeon.

GRAFT (NEET 2019 TOPIC)


• Autograft: transplantation within same individual
• Isograft: transplantation within Identical twins
• Allograft: transplantation within same species
• Xenograft: transplantation between different species (Pigs are usually used for humans)

Based on Placement of Graft


• Orthotopic graft placement: Graft is placed in same anatomical location Eg: Liver, Heart
• Heterotopic graft placement: Graft is placed in a non-anatomical location Eg: Kidney,
Pancreas

Before transplanting the graft HLA Matching is done ( Major Histocompatibility Complex (MHC))

SURGERY SIXER APP BASED WORK-BOOK 2020 556


• Class I MHC: we look for HLA A, B
• Class II MHC: we look for HLA DR
• HLA matching is done for antigen recognition
• HLA matching not needed for Liver and cornea**
• HLA matching is not possible for Heart because Cold ischemic time is: 3hrs**

DONOR
Types of donor
Liver donor Brain dead donor Donation after Xenograft donor
Cardiac Death ( DCD)
• Kidney They are the donor’s They are the donor’s Pigs are the ideal
• Heart who have heart beat who have no heart Heart valves are used.
and circulation+ beat and cardiac
arrest happened.

C.I for organ donation: the following persons should not donate organ
• HIV Patient
• CJ disease
• Malignancy within 5 years (Exclude SCC, BCC, Cancer in situ uterus & brain tumors)
• HBV patients
• Death due to Sepsis

Donor Matching of Blood group:


• O Group can donate to: O, A, B, AB
• A group can donate to: A, AB
• B group can donate to: B, AB
• AB group can donate to: AB
• O group is universal donor
• AB group is universal recipient
• Rh testing is not needed for organ transplantation. Only ABO and HLA matching is
done.**

Brain dead donor:


• Brain not working
• Heart is still beating
In UK 2 Doctors should certify to declare as brain dead
- one should be a Consultant
- None should be a Transplant involved surgeon in the 2 doctors

Two doctors should separately examine the following in two separate occasions:
To condemn them as brain dead

SURGERY SIXER APP BASED WORK-BOOK 2020 557


• Cranial reflexes – CNS reflexes absent ( Pharyngeal, Gag reflexes etc)
• Motor response – NO Response
• No Spontaneous breathing on raising PaCO2 > 60mmHg

Donation after Cardiac Death (DCD) Donor:


Maastricht classification of DCD donor**
• Cat 1: dead on arrival at hospital
• Cat 2: dead even after attempt of resuscitation
• Cat 3: Awaited cardiac arrest after withdrawal of supports (who is not brain dead)
• Cat 4: Cardiac arrest while Brain dead
• Cat 5: Cardiac arrest in Inpatient

• Cat 3 & 4 are good donors because they have circulation in organs until death they
are controlled donors
• Cat 1, 2 & 5: Uncontrolled Donors

Donor Age group


• KIDNEY: 2yrs – no upper limit
• LIVER: No age limit
• HEART: 1 – 65 Years
• LUNGS: 5 – 65Years
• PANCREAS: 10 – 60 Years

The crisis in organ supply has required the use of extended donor criteria
Extended donor criteria for kidney: Extended donor criteria for liver:
• Age > 60 yrs • Steatosis
• Age 50 -59 yrs with history of 2 or • Age > 65 years
more of the following • HCV +ve patients
• Death due to CVA • HBV core Ag +ve patients
• Impaired kidney function (S.Creatinine
> 15mg/dl)
• H/O of HTN

Process of organ extraction from Donor


On Operating Table procedure:
• The brain dead patient is taken to the OT
• Anesthetist disconnects the supports
• The moment the heart beat stops
• All the organs needed are extracted by respective surgeons simultaneously

SURGERY SIXER APP BASED WORK-BOOK 2020 558


Back Bench Procedure:
• Immediately cold ischemic solution is infused into the organ
• The organ is kept in a ice bag and taken to the recipient

Warm ischemic time is the time between the heart stops and infusion of ischemic cold solution
into the organ. It should be as less as possible <30 min

Cold Ischemic time is the time of infusion of Cold preservative solution to transplantation to the
recipient is completed and there is reperfusion of Recepient blood

Cold Ischemic Solution: m/c used is University of Wisconsin Solution. Eurocollin is another solution
University of Wisconsin Solution components
• Lactobionate: prevents cell swelling
• AdENosine: provides ENergy
• Allopurinol: it is a free radical scavenger
• Glutathione: Antioxidant

ORGAN Optimum cold ischemic time Maximum cold ischemic time


Kidney < 18 Hrs 36 Hrs
Liver < 12 Hrs 18Hrs
Heart < 3 Hrs 6 Hrs
Lungs < 3 Hrs 8Hrs
Pancreas < 10 Hrs 18 Hrs
Small intestine < 4Hrs 6 Hrs

Hyperacute rejection:
• Happens in minutes to days
• It is due to pre formed antibodies.
• It is either due to ABO incompatibility or HLA mismatching.
• Immediate Graft thrombosis followed by Gangrene of organ

Acute rejection:
• Usually happens in first 6 months
• Usually due to T cell mediated rejection
• Can be prevented by giving immune suppressants.
• Biopsy shows lymphocytic infiltration

Chronic rejection:
• Occurs after 6 months
• M/C cause of Graft failure.
• Fibrosis happening in the organ.

SURGERY SIXER APP BASED WORK-BOOK 2020 559


• It can be due to T & B lymphocytes or it can be no immune also.

Chronic rejection in
• Kidney: Glomerular sclerosis
• Liver: vanishing bile duct syndrome
• Heart: accelerated coronary artery disease
• Pancreas: acinar loss
• Lungs: obliterative bronchiolitis

Risk factors for chronic rejection in kidney (NEET SS)


• Previous episodes of acute rejection
• Poor HLA matching
• Prolonged cold ischemic time
• CMV infection
• Increased blood lipids
• Inadequate immunosuppression

Immunosuppression is given to prevent graft getting rejection


INDUCTION PHASE:
• Is started immediately after surgery
MAINTENANCE PHASE:
• Is continued for life time in many patients

Category of drugs for immune suppression drugs


• Calcineurin inhibitors: Cyclosporine, Tacrolimus
• Anti proliferative agents: Azathioprine, Mercaptopurine, Mycophenolate mofetil
• Steroids
• m ToR inhibitors (mammalian Target of Rapamycin inhibitor): Sirolimus, Everolimus

Monoclonal antibodies
• Muromonab: anti CD3 antibody
• Basiliximab: anti CD25 antibody
• Rituximab: anti CD20 antibody

M/C complication on immunosuppression- Infections:


• Infections: Recipient infection reactivation is M/C:
• CMV reactivation in 6 – 12 weeks patient presents with CMV pneumonitis

Post transplant Malignancy:


• M/C SCC > PTLD (Post Transplant Lympho proliferative disorder M/C caused EBV)
• HHV 8 Patients: Kaposi’s sarcoma

SURGERY SIXER APP BASED WORK-BOOK 2020 560


• HBV & HCV patients: HCC
• HPV : Ca cervix

Non immune mediated:


Calcineurin inhibitors:
• 1.Chronic allograft Nephropathy
• 2. NoDAT: New Onset DM after transplant happening in 30% < 2yrs)

COMPLICATION RELATED MCQS


• M/C infection : CMV
• M/C parasitic infection: Pneumocystis carinii
• Polyoma BK virus: ureteric stricture**
• M/C malignancy is SCC

Topic 9B: Organ Transplant- Individual Focus

LIVER TRANSPLANT
• M/C indication of liver transplant in adults: CIRRHOSIS (HCV>HBV)**
• M/C indication of liver transplant in children: EHBA (Extra hepatic Biliary Atresia)**
• M/C metabolic disorder: Alpha 1 antitrypsin deficiency
• M/C indication for emergency transplantation: Acute Fulminant Liver Failure (PCM
toxicity)

History:
• 1st liver transplant was done by: STARZL -1963
• 1st split liver transplant was done by: PICHLMAYR
• 1st live donor liver transplant was done by: TANAKA

CRITERIAS for liver transplant:


MODIFIED CHILD PUGH SCORE: Child C > B
• Bilirubin
• Albumin
• Ascites
• PT
• Encephalopathy

MELD (Model for End Liver Disease)


Logarthmic score of : C B I
• Creatinine
• Bilirubin
• INR

SURGERY SIXER APP BASED WORK-BOOK 2020 561


MELD Score>35Priority
MELD Score<15 put under waiting list

MELD EXCEPTION: in these patients we can do transplant in a lesser score also


• HCC
• Amyloidosis
• Hepato pulmonary syndrome
• Porto pulmonary HTN

KING’S COLLEGE CRITERIA (NEET 2020 TOPIC)


Liver transplant in acute fulminant liver failure
Paracetamol induced liver failure:
• pH < 7.30 (irrespective of encephalopathy)
OR
• PT > 100sec + Serum Creatinine > 300micro mol/L + Grade 3 or 4 encephalopathy

Non Paracetamol induced liver failure:


• PT >100sec
OR
• Any 3 of the below
1. Age < 10 >40 Years
2. Aetiology: Non A/ Non B liver damage may be due to halothane or idiosyncronatic
drugs
3. 7 days of jaundice before encephalopathy developed
4. S. Bilirubin > 300 micro mol/L
5. PT > 50Sec

MILAN CRITERIA:
• Followed for HCC patients with cirrhotic liver
• Single Tumor < 5cm
• Less than 3 lesions (all 1-3 cm diameter)
• No Extra hepatic spread.

ORTHOTOPIC GRAFT HETEROTOPIC GRAFT


Diseased liver is removed & new liver is placed New liver is placed in adjacent to original liver
in the same place
M/C done Is usually followed for acute fulminant liver
failure

SURGERY SIXER APP BASED WORK-BOOK 2020 562


TYPES OF LIVER TRANSPLANT
AUXILIARY TRANSPLANT:
• Indicated for acute fulminant liver failure,
• Stop immunosuppressant drugs when the original liver recovers or can remove the
transplanted liver
• Auxiliary partial orthotopic liver transplant: part of diseased liver is removed and part of
new liver is placed in the same position
• Auxiliary heterotopic liver transplant: new liver is placed just beneath the original liver

DOMINO LIVER TRANSPLANT:


• Amyloidotic liver is removed and given to someone in need of liver and new liver is given
to amyloidotic patient. 2 patients are saved with 1 liver

SURGERY SIXER APP BASED WORK-BOOK 2020 563


SPLIT LIVER TRANSPLANT:
• Segments 5 6 7 8 will be given to adult and segment 2 3 alone is enough for children.

LIVE DONOR (can donate only to relatives):


• Segment 5 6 7 8 is taken if donating to adult;
• Segment 2 3 is taken if donation is for the paediatric patient

REDUCED LIVER TRANSPLANT:


• The retrieved liver is trimmed to suit for the body weight of the recipient (mostly to
children)

STEPS IN ORTHOTOPIC LIVER TRANSPLANT


• Remove cadaveric liver along with supra hepatic IVC & Portal Triad (Portal Vein, Bile
duct & Hepatic artery)
• Recipient operation: Remove liver along with hepatic vein & part of IVC and small
amount of portal vein
Place the cadaveric liver in the donor and anastomose the structures in the following order
• 1st: IVC (supra hepatic followed by infra hepatic)
2nd: Portal vein anastomosis (May develop reperfusion syndrome: Hypotension &
Bradycardia)
• 3rd: Hepatic artery
• 4th: Bile duct (End to End anastomosis)

** Bile duct anastomosis is C/I in Primary sclerosing cholangitis & Extra Hepatic Biliary Atresia
For the above 2 conditions do a Roux en Y jejuna anastomosis ( NEET SS )

Piggy Back Transplant:


• If liver alone is removed in Recipient and donor liver is removed along with IVC do PIGGY
BACK anastomosis. Advantage is blood flows throughout the process of surgery

SURGERY SIXER APP BASED WORK-BOOK 2020 564


Rejection

Hyper acute rejection Acute rejection Chronic rejection


Rare or not happens in liver Manifests as Hepatic Artery Manifests as Vanishing bile
transplant thrombosis : immediately duct (Rare)
need Re transplant

Portal vein thrombosis no


Re transplant advised

• ROTEM: Rotational thrombo


elastometry used to measure PT/INR
dynamically while transplantation is
going on.

RENAL TRANSPLANT
• M/C cause in adults: Renal failure due to Diabetic nephropathy
• M/C cause in children: Chronic glomerulonephritis
• It is always a heterotopic graft (Rt iliac fossa > Lt iliac fossa)

Types of graft:
CADAVERIC LIVE DONOR
Kidney preferred is Lt kidney b/c Lt renal vein is
longer

Donor criteria: C/I for donor:


• Age 18 – 70 Years • BMI > 40
• BMI < 35 • DM

SURGERY SIXER APP BASED WORK-BOOK 2020 565


• No caner • Malignancy
• No active sepsis • HIV +ve, sepsis
• Kidney has normal function • HTN
• ABO compatibility / HLA matching • GFR < 70, Albuminuria
• Horse shoe kidney
• Kidney stones

PROCEDURE:
• In cadaveric kidney along with the renal artery small portion of the aorta is also removed
known as Carrel patch
Ureter is anastomosed to the bladder by
• Short tunnel technique @ Lich Gregoir technique M/C done
• Long tunnel technique @ Lead Better Politano technique

Renal vein anastomosed to common iliac vein


Renal artery anastomosis: ( NEET SS)
• In cadaveric donor carrel patch is taken: do end to side wide anastomosis of carrel patch
with external iliac artery
• In live donor carrel patch is not possible: do end to end anastomosis of renal artery with
internal iliac artery. Renal artery stenosis is M/C with live donor kidney transplant**

Figure: a- Cadaveric Donor and b is Live donor

Complications of kidney transplant


Complications to live donor:
• M/C complication is urinary retention and paralytic ileus

Complications to recipient:
• Haemorrhage

SURGERY SIXER APP BASED WORK-BOOK 2020 566


• Renal vein thrombosis (1–5%)
• Renal artery thrombosis (1%)
• Renal artery stenosis (1-23%)
• Viral infections
• Bacterial infections’
• Lymphocele
• Polyoma BK virus produce multiple ureteric strictures**

Allograft dysfunction of transplanted kidney:


• BANFF classification

Risk factors for chronic rejection:


• HLA mismatch
• H/O acute rejection
• Long cold ischemic time
• CMV infection
• Increased blood lipids
• Poor compliance to immune suppression

INTESTINE TRANSPLANT
• M/C indication in adult / children: Short bowel syndrome
• M/C cause of Short bowel syndrome in adults: Acute superior mesenteric artery ischemia
• M/C cause of Short bowel syndrome in children: Gastroschisis > Midgut volvulus

Short bowel syndrome patients will be on TPN. Indications of SI transplant:


• TPN failure is the m/c indication of Small Intestine transplant
• Patient on TPN develops liver failure
• Patient on TPN develops central vein thrombosis
• Patient on TPN develops > 2 episodes of infection
• Patient on TPN develops frequent dehydration
• Single time fungal infection in central line

Small bowel can be taken from


Cadaveric donor: Live donor: Xenograft
• Isolated SB graft Last 150 – 200cm of ileum From Pig
• Combined SB and Liver transplant is taken; it is based on ileo
• Multi visceral transplant: SB + colic artery or ileo colic vein
Stomach +/- Colon
• Modified multi visceral transplant:
Liver is not included**

• Body weight of donor should be 50-75% of the recipient

SURGERY SIXER APP BASED WORK-BOOK 2020 567


• Avoid prolonged cold ischemic time

CADAVERIC SBT
• Small bowel is anastomosed with duodenum and ileum is kept as ileostomy only.
• Superior mesenteric vein is anastomosed with portal vein of the recipient
• Superior mesenteric artery is anastomosed to the infra renal aorta

Complications of SB transplant:
Rejection
• Highest chance of rejection: 70-80%
• C/F: Fever, abdomen pain and increased stool
• Do ileoscopy and biopsy to look for rejection
Graft vs Host Disease is M/C in SB transplant of all: 0-14%
New Markers for rejection: Stool Calprotectin & Stool Citrulline**
PTLD also M/C 10-20%
M/C complication overall is Bacterial infection 70-90%**

Extra edge
• Combined Liver + SB transplant has better prognosis than isolated SB transplantation
alone
• M/C cause of death is infection (sepsis)
• M/C cause of graft loss is Sepsis > graft rejection

PANCREAS TRANSPLANT
• 1st done by Kelly and Lillehei
• Donor age 10 – 45 years
• Only cadaveric transplant is possible
• M/C complication following pancreas transplant is Graft Thrombosis

Types of pancreas transplant


SPK (80%) simultaneous pancreas and kidney transplant
• Has 85% 1yr graft survival rate (highest among the 3 types)
• Done in Type I (IDDM) with severe C peptide deficiency

SURGERY SIXER APP BASED WORK-BOOK 2020 568


• Type II with C peptide levels < 2ng/ml
PTA (15%): Pancreas transplant alone
PAK (5%): pancreas after kidney transplant

Contraindication for donor:


• Type I DM
• Pancreatic surgery
• Chronic pancreatic disorders
• IPMN
Procedure:
• Pancreas is retrieved including C shaped duodenum, 1 vein Superior mesenteric vein and
2 arteries superior mesenteric artery (SMA) and Splenic artery (SA).
• Y shaped graft is also taken from the cadaver (common ileac artery with internal and
external iliac arteries) anastomose the IIA with SA and EIA to SMA.
• Anastomosis of the graft can be done in 2 ways
o Enteric drainage
o Bladder drainage

Enteric drainage Bladder drainage


M/C performed
Advantage: Advantage:
it is physiological drainage Graft failure can be identified by looking
No increased infection at urinary Amylase
No UTI Early rejection can be diagnosed
Leak rate is less
Disadvantage: Disadvantage:
Leak is M/C Urethritis, UTI
Metabolic acidosis

• M/C cause of graft failure is Graft thrombus (SMV> artery); Pancreas is to removed
• M/C cause of death post transplant: CVS cause

SURGERY SIXER APP BASED WORK-BOOK 2020 569


Complication:
• Rejection
• Leak: Debridement and re anastomosis
• Bleeding (Rx Octreotide)
• Infection
• Necrosis
• Abscess

ISLETS CELL TRANSPLANT


• 8000 – 10000 islets / kg body weight is needed to make him insulin free, to get this we
need 4 donor pancreas**
• These islets is injected into the portal vein
• They live in the liver
• Complication: Bleeding and PV thrombosis
• EDMONTON Protocol** ( NEET SS)
o Steroids not to used
o IL 2 Abs are used

HEART TRANSPLANT
• Orthotopic transplant
• M/C cause for transplant is Ischemic Dilated Cardiomyopathy
• Do sternotomy and remove the heart preserve the Lt atrial Cuff, Aorta, SVC, IVC &
pulmonary arteries. Insert the donor heart and anastomose in the following order
o Left atrial cuff ( 1st)
o SVC & IVC ( 2nd)
o Aorta
o Pulmonary artery

SURGERY SIXER APP BASED WORK-BOOK 2020 570


Space for Additional Points:

SURGERY SIXER APP BASED WORK-BOOK 2020 571


Back Cover :

About the author:

Dr. Rajamahendran MS, MRCS ( Edinburgh), FMAS, MCh ( Surgical Gastro) is a renowned author/ Surgery
faculty in South India since 2007. By his Illustrative method of Teaching skills and Surgical Skills he is now a
Renowned and most opted Surgery faculty in the Country. His experiences and encounters of various patients in his
practice and his future guidance for the PG aspirants has been an eye opener for a lots of Medicos.

He finished his MBBS and MS General Surgery from Kilpauk Medical College, Chennai with best academic score. He
joined MCh Surgical gastroenterology in Madras Medical College, Chennai by acquiring the Top rank in State entrance
examination. He acquired Gold Medal in Esophagus. He completed MRCS from Royal College of Edinburgh. Other
fellowships of his credit are- FMAS, Dip. Lap, FMGE. He has published about nearly 8 articles in International Journals
as prime author and co author. He has presented more than 30 -40 papers in various Conferences and Workshops.

He is the Founder/ faculty/ Director of Koncpt Pg Medical Coaching centre in Tamil nadu started in the year 2009 and
running around 8 branches all over the state. He is a leading Gastro Intestinal and Laparoscopic Surgeon in
Villupuram District and the founder/ Director of RRM Gastro Super speciality clinic and Endoscopy centre,
Villupuram. He is presently working as Senior Assistant Professor in Surgical gastroenterology Department in
Government Medical College, Villupuram, Tamilnadu.

While most other faculties were only involved in Classes, Dr. Rajamahendran is a multifaceted personality as
Faculty, Author, Motivator , Clinician and apart all he carries Enormous and Astonishing Surgical Skills in
Advanced Laparoscopy, Pancreatic and Biliary Surgeries. His record on Pancreatic surgeries and researches on
outcomes of the Pancreatic operations has made his Hospital as one of the main referral centre for Pancreatic
Surgeries.

He is very famous for his authorship since 2009. Some books of his credit are Long cases in Surgery, Short cases in
Surgery, Clinical Cases in Paediatrics, Clinical cases in Paediatrics, Nutshell series for FMGE, Koncpt 20 author for
TNPG, JIPMER 20 author, AIPG 20 author, Complete review of Surgery etc, He is also a leading faculty in various
coaching centres and Surgical Educator in UNACADEMY Platform . He is the Pioneer to start MCh Surgical
Gastroenterology Entrance coaching in INDIA- presently most Surgical post graduates are trained from his Koncpt
NEET SS -Institute in Chennai and Delhi.

❖ Follow the Faculty on : “ Surgery Sixer for NEXT” Facebook Group


❖ Follow the Faculty on Telegram group : t.me/surgerysixer
❖ Follow the Faculty on Twitter, Instagram and Facebook Page : “Rajamahendran”
❖ Follow and subscribe my You tube channel for regular update videos : “ Rajamahendran”

SURGERY SIXER APP BASED WORK-BOOK 2020 572

You might also like